Sie sind auf Seite 1von 374

A Simple and Alternative Approach

to Pure Mathematics (P1, P2 and P3)

by

D K R Babajee (BSc, PhD) & A R Appadu


(BSc, MSc)
c
Copyright 2016
Dr Diyashvir Kreetee Rajiv Babajee
Atish Appadu
Tel : 00(230) 57527479
Email: dkrbabajee@gmail.com

ALL RIGHTS RESERVED

No part of this publication may be reproduced, stored in a retrieval system, or


transmitted in any form or by any means without prior permission of the author.

i
Preface and Acknowledgements

This book has been written to cover the Pure Mathematics Syllabus (P1, P2, P3) for
A level Principal. The chapters are arranged with the first 9 chapters dealing with
P1 Syllabus and the next 6 chapters with P2 and P3 Syllabus and last 3 chapters
with P3 Syllabus.

In Mathematics, it is known that there are different methods to arrive to a solution.


In this book, we have included some simple and quick methods to arrive at the
solution.
The alternative methods are:

1. Complete to the square using formula, Alternative Use of Quadratic formula


(Quadratics)

2. Find the equation of the line with known gradient using determinants and
finding the equation of the perpendicular bisector using equidistant formula
(Coordinates)

3. Quick expansion of polynomials using conventional multiplication of numbers


(Algebra)

4. Find the perpendicular distance of a line from a point, find a vector which is
perpendicular to two other vectors using cross-product, find the perpendicular
distance of a point from a plane (Vectors 2)

5. Find the two square roots of a complex number (Complex Numbers)

We have also included 10 miscellaneous exercises of which most are exam-typed


questions, for each chapter. This will help students to become confident to tackle
such questions. Also, the challenging questions are denoted by *. The author is
grateful to Mr Girish Gansam for preparing some notes and exercises.

All diagrams are generated using the Geogebra software.

Dr D K R Babajee wrote most notes with explanations and prepared the exercises.

ii
Contents

Preface and Acknowledgements ii

Contents iii
1 Quadratics 1
1.1 Objective . . . . . . . . . . . . . . . . . . . . . . . . . . . . . . . . . . . . 1
1.2 Polynomials . . . . . . . . . . . . . . . . . . . . . . . . . . . . . . . . . . 1
1.3 Quadratic Polynomials . . . . . . . . . . . . . . . . . . . . . . . . . . . . 2
1.4 Completion to the square . . . . . . . . . . . . . . . . . . . . . . . . . . . 2
1.5 Graphs of quadratic polynomials . . . . . . . . . . . . . . . . . . . . . . 4
1.6 Roots of Quadratic Equations . . . . . . . . . . . . . . . . . . . . . . . . 7
1.7 Disguised Quadratic Equations . . . . . . . . . . . . . . . . . . . . . . . 10
1.8 Simultaneous equations: one linear and the other nonlinear . . . . . . 13
1.9 More on graphical representation . . . . . . . . . . . . . . . . . . . . . . 15
1.10 Quadratic Inequalities . . . . . . . . . . . . . . . . . . . . . . . . . . . . 15
1.11 Miscellaneous Exercises . . . . . . . . . . . . . . . . . . . . . . . . . . . 18
2 Functions 20
2.1 Objective . . . . . . . . . . . . . . . . . . . . . . . . . . . . . . . . . . . . 20
2.2 Basic Definitions . . . . . . . . . . . . . . . . . . . . . . . . . . . . . . . 20
2.3 Geometrical Interpretation: Vertical Line Test . . . . . . . . . . . . . . 21
2.4 Representation of functions . . . . . . . . . . . . . . . . . . . . . . . . . 22
2.5 Finding the range of a function . . . . . . . . . . . . . . . . . . . . . . . 22
2.6 Composite functions . . . . . . . . . . . . . . . . . . . . . . . . . . . . . . 25
2.7 Inverse function . . . . . . . . . . . . . . . . . . . . . . . . . . . . . . . . 28
2.8 Miscellaneous Exercises . . . . . . . . . . . . . . . . . . . . . . . . . . . 35
3 Coordinate Geometry 39
3.1 Objective . . . . . . . . . . . . . . . . . . . . . . . . . . . . . . . . . . . . 39
3.2 Coordinates . . . . . . . . . . . . . . . . . . . . . . . . . . . . . . . . . . 39
3.3 Distance between two points (Distance’s formula) . . . . . . . . . . . . 40
3.4 Midpoint of a line joining two points . . . . . . . . . . . . . . . . . . . . 40
3.5 Gradient of a line joining two points . . . . . . . . . . . . . . . . . . . . 42
3.6 Collinear points . . . . . . . . . . . . . . . . . . . . . . . . . . . . . . . . 43
3.7 Parallel Lines . . . . . . . . . . . . . . . . . . . . . . . . . . . . . . . . . 43

iii
CONTENTS

3.8 Perpendicular Lines . . . . . . . . . . . . . . . . . . . . . . . . . . . . . . 44


3.9 Equation of a line . . . . . . . . . . . . . . . . . . . . . . . . . . . . . . . 44
3.10 Perpendicular bisector . . . . . . . . . . . . . . . . . . . . . . . . . . . . 47
3.11 Angle between two straight lines . . . . . . . . . . . . . . . . . . . . . . 51
3.12 Miscellaneous Exercises . . . . . . . . . . . . . . . . . . . . . . . . . . . 52
4 Circular Measure 57
4.1 Objective . . . . . . . . . . . . . . . . . . . . . . . . . . . . . . . . . . . . 57
4.2 Definition of a radian . . . . . . . . . . . . . . . . . . . . . . . . . . . . . 57
4.3 Conversion between degrees and radians . . . . . . . . . . . . . . . . . 57
4.4 Sector . . . . . . . . . . . . . . . . . . . . . . . . . . . . . . . . . . . . . . 58
4.5 Problems involving length of arc and area of sector. . . . . . . . . . . . 62
4.6 Miscellaneous Examples . . . . . . . . . . . . . . . . . . . . . . . . . . . 66

5 Trigonometry 1 70
5.1 Objective . . . . . . . . . . . . . . . . . . . . . . . . . . . . . . . . . . . . 70
5.2 Graphs of Trigonometric functions . . . . . . . . . . . . . . . . . . . . . 71
5.3 Trigonometric ratios . . . . . . . . . . . . . . . . . . . . . . . . . . . . . 75
5.4 Solving Trigonometrical Equations . . . . . . . . . . . . . . . . . . . . . 77
5.5 Trigonometric Identities . . . . . . . . . . . . . . . . . . . . . . . . . . . 80
5.6 Miscellaneous Exercises . . . . . . . . . . . . . . . . . . . . . . . . . . . 85
6 Vectors 1 89
6.1 Objective . . . . . . . . . . . . . . . . . . . . . . . . . . . . . . . . . . . . 89
6.2 Standard Notation of vectors . . . . . . . . . . . . . . . . . . . . . . . . 89
6.3 Vector Operations . . . . . . . . . . . . . . . . . . . . . . . . . . . . . . . 90
6.4 Parallel Vectors . . . . . . . . . . . . . . . . . . . . . . . . . . . . . . . . 91
6.5 Dot product of vectors . . . . . . . . . . . . . . . . . . . . . . . . . . . . . 93
6.6 Perpendicular Vectors . . . . . . . . . . . . . . . . . . . . . . . . . . . . . 94
6.7 Magnitude of a vector . . . . . . . . . . . . . . . . . . . . . . . . . . . . . 95
6.8 Unit vector . . . . . . . . . . . . . . . . . . . . . . . . . . . . . . . . . . . 95
6.9 Angle between two vectors . . . . . . . . . . . . . . . . . . . . . . . . . . 95
6.10 Position Vectors . . . . . . . . . . . . . . . . . . . . . . . . . . . . . . . . 97
6.11 Miscellaneous Exercises . . . . . . . . . . . . . . . . . . . . . . . . . . . 106
7 Series 109
7.1 Objective . . . . . . . . . . . . . . . . . . . . . . . . . . . . . . . . . . . . 109
7.2 Binomial expansion . . . . . . . . . . . . . . . . . . . . . . . . . . . . . . 109
7.3 Arithmetic Progression (A. P.) . . . . . . . . . . . . . . . . . . . . . . . . 114
7.4 Geometric Progression (G. P.) . . . . . . . . . . . . . . . . . . . . . . . . 119
7.5 Miscellaneous Exercises . . . . . . . . . . . . . . . . . . . . . . . . . . . 125

iv
CONTENTS

8 Differentiation 1 128
8.1 Objective . . . . . . . . . . . . . . . . . . . . . . . . . . . . . . . . . . . . 128
8.2 Gradient of a curve at a point . . . . . . . . . . . . . . . . . . . . . . . . 129
8.3 Differentiation of xn together with constant multiples, sums and differences129
8.4 Chain Rule . . . . . . . . . . . . . . . . . . . . . . . . . . . . . . . . . . . 131
8.5 Tangents and Normal . . . . . . . . . . . . . . . . . . . . . . . . . . . . . 132
8.6 Increasing and decreasing functions . . . . . . . . . . . . . . . . . . . . 138
8.7 Stationary Points . . . . . . . . . . . . . . . . . . . . . . . . . . . . . . . 140
8.8 Rate of Change . . . . . . . . . . . . . . . . . . . . . . . . . . . . . . . . 146
8.9 Connected Variables . . . . . . . . . . . . . . . . . . . . . . . . . . . . . 146
8.10 Miscellaneous Exercises . . . . . . . . . . . . . . . . . . . . . . . . . . . 147
9 Integration 1 151
9.1 Objective . . . . . . . . . . . . . . . . . . . . . . . . . . . . . . . . . . . . 151
9.2 Integration as a reverse process of differentiation . . . . . . . . . . . . 151
9.3 Finding constant of integration and equation of a curve . . . . . . . . . 154
9.4 Definite Integrals . . . . . . . . . . . . . . . . . . . . . . . . . . . . . . . 157
9.5 Area under a curve . . . . . . . . . . . . . . . . . . . . . . . . . . . . . . 160
9.6 Volume of solids of revolution . . . . . . . . . . . . . . . . . . . . . . . . 165
9.7 Miscellaneous Exercises . . . . . . . . . . . . . . . . . . . . . . . . . . . 169
10 Algebra 173
10.1 Objective . . . . . . . . . . . . . . . . . . . . . . . . . . . . . . . . . . . . 173
10.2 Modulus Function . . . . . . . . . . . . . . . . . . . . . . . . . . . . . . 173
10.3 Polynomials . . . . . . . . . . . . . . . . . . . . . . . . . . . . . . . . . . 177
10.4 Partial Fractions (P3 only) . . . . . . . . . . . . . . . . . . . . . . . . . . 183
10.5 Binomial Expansion of (a + b)n , n is a negative integer or a rational (P3
only) . . . . . . . . . . . . . . . . . . . . . . . . . . . . . . . . . . . . . . . 188
10.6 Miscellaneous Exercises . . . . . . . . . . . . . . . . . . . . . . . . . . . 192
11 Logarithms and exponential functions 195
11.1 Objective . . . . . . . . . . . . . . . . . . . . . . . . . . . . . . . . . . . . 195
11.2 Indices and Logarithms . . . . . . . . . . . . . . . . . . . . . . . . . . . 195
11.3 Laws of Logarithms . . . . . . . . . . . . . . . . . . . . . . . . . . . . . . 196
11.4 The graphs of y = ex and y = ln x and their relation . . . . . . . . . . . 198
11.5 Solving equations and inequalities involving logarithms . . . . . . . . 200
11.6 Use of logarithms to transform a given relationship to linear form . . . 202
11.7 Miscellaneous Exercises . . . . . . . . . . . . . . . . . . . . . . . . . . . 204
12 Trigonometry 2 207
12.1 Objective . . . . . . . . . . . . . . . . . . . . . . . . . . . . . . . . . . . . 207
12.2 Secant, cosecant and cotangent functions . . . . . . . . . . . . . . . . . 207
12.3 Trigonometric Identities . . . . . . . . . . . . . . . . . . . . . . . . . . . 209
12.4 Compound angles . . . . . . . . . . . . . . . . . . . . . . . . . . . . . . . 211

v
CONTENTS

12.5 Double and Triple Angles . . . . . . . . . . . . . . . . . . . . . . . . . . 214


12.6 Express a cos θ ± b sin θ as R cos (θ ± α) or R sin (θ ± α). . . . . . . . . . . 217
12.7 Miscellaneous Exercises . . . . . . . . . . . . . . . . . . . . . . . . . . . 222
13 Differentiation 2 225
13.1 Objective . . . . . . . . . . . . . . . . . . . . . . . . . . . . . . . . . . . . 225
13.2 Derivatives of exponential and trigonometric functions . . . . . . . . . 225
13.3 Product and Quotient Rules . . . . . . . . . . . . . . . . . . . . . . . . . 226
13.4 Implicit Differentiation . . . . . . . . . . . . . . . . . . . . . . . . . . . . 229
13.5 Parametric Equations . . . . . . . . . . . . . . . . . . . . . . . . . . . . . 232
13.6 Miscellaneous Exercises . . . . . . . . . . . . . . . . . . . . . . . . . . . 235
14 Integration 2 238
14.1 Objective . . . . . . . . . . . . . . . . . . . . . . . . . . . . . . . . . . . . 238
14.2 Integration of exponential and trigonometric functions . . . . . . . . . 238
14.3 Integration using exact derivatives (P3 only) . . . . . . . . . . . . . . . 241
14.4 Integration using Partial Fractions (P3 only) . . . . . . . . . . . . . . . 242
14.5 Integration by parts (P3 only) . . . . . . . . . . . . . . . . . . . . . . . . 245
14.6 Trapezium Rule . . . . . . . . . . . . . . . . . . . . . . . . . . . . . . . . 251
14.7 Miscellaneous Exercises . . . . . . . . . . . . . . . . . . . . . . . . . . . 255
15 Numerical Solution of equations 259
15.1 Objective . . . . . . . . . . . . . . . . . . . . . . . . . . . . . . . . . . . . 259
15.2 Root of an equation . . . . . . . . . . . . . . . . . . . . . . . . . . . . . . 259
15.3 Number of roots of an equation . . . . . . . . . . . . . . . . . . . . . . . 259
15.4 Location of the root . . . . . . . . . . . . . . . . . . . . . . . . . . . . . . 260
15.5 Determination of the root by Iterative Method . . . . . . . . . . . . . . 260
15.6 Application of Fixed iteration Method . . . . . . . . . . . . . . . . . . . 263
15.7 Miscellaneous Exercises . . . . . . . . . . . . . . . . . . . . . . . . . . . 269
16 Vectors 2 274
16.1 Objective . . . . . . . . . . . . . . . . . . . . . . . . . . . . . . . . . . . . 274
16.2 Line . . . . . . . . . . . . . . . . . . . . . . . . . . . . . . . . . . . . . . . 275
16.3 Point on a line . . . . . . . . . . . . . . . . . . . . . . . . . . . . . . . . . 276
16.4 Pairs of lines . . . . . . . . . . . . . . . . . . . . . . . . . . . . . . . . . . 276
16.5 Angle between two lines . . . . . . . . . . . . . . . . . . . . . . . . . . . 279
16.6 Perpendicular distance of a line from a point . . . . . . . . . . . . . . . 280
16.7 Finding a vector v perpendicular to both vectors v1 and v2 . . . . . . . 282
16.8 Planes . . . . . . . . . . . . . . . . . . . . . . . . . . . . . . . . . . . . . . 284
16.9 Miscellaneous Exercises . . . . . . . . . . . . . . . . . . . . . . . . . . . 295
17 Differential Equations 301
17.1 Objective . . . . . . . . . . . . . . . . . . . . . . . . . . . . . . . . . . . . 301
17.2 Understanding Differential equations . . . . . . . . . . . . . . . . . . . 301
17.3 Solving Differential Equations: Separation of Variables . . . . . . . . . 302

vi
CONTENTS

17.4 Formulation of Differential Equation . . . . . . . . . . . . . . . . . . . . 306


17.5 Miscellaneous Exercises . . . . . . . . . . . . . . . . . . . . . . . . . . . 314
18 Complex Numbers 317
18.1 Objective . . . . . . . . . . . . . . . . . . . . . . . . . . . . . . . . . . . . 317
18.2 Idea of complex number . . . . . . . . . . . . . . . . . . . . . . . . . . . 318
18.3 Operations involving complex numbers . . . . . . . . . . . . . . . . . . 318
18.4 Polynomial with real coefficients . . . . . . . . . . . . . . . . . . . . . . 320
18.5 Simultaneous equations involving complex number . . . . . . . . . . . 322
18.6 Square roots of a complex number . . . . . . . . . . . . . . . . . . . . . 323
18.7 Modulus and Argument . . . . . . . . . . . . . . . . . . . . . . . . . . . 324
18.8 Geometrical Interpretation . . . . . . . . . . . . . . . . . . . . . . . . . . 327
18.9 Loci . . . . . . . . . . . . . . . . . . . . . . . . . . . . . . . . . . . . . . . 333
18.10Miscellaneous Exercises . . . . . . . . . . . . . . . . . . . . . . . . . . . 341
Answers 345

vii
C HAPTER 1

Quadratics

1.1 Objective
At the end of this chapter, students should be able to understand:

1. carry out the process of completing the square for a quadratic polynomial
ax2 + bx + c, and use this form to locate the vertex of the graph of y = ax2 + bx + c
and to sketch the graph;

2. solve quadratic equations by factorisation and by using the quadratic formula;

3. find the discriminant of a quadratic polynomial ax2 +bx+c and use the discriminant
to determine the number of real roots of the equation ax2 + bx + c = 0;

4. recognise and solve equations in x which are quadratic in some function of x


(disguised quadratic equations);

5. solve by substitution a pair of simultaneous equations of which one is linear


and one is quadratic;

6. solve quadratic equations, and linear and quadratic inequalities, in one unknown;

7. understand the relationship between a graph and its associated algebraic equation,
and use the relationship between points of intersection of graphs and solutions
of equations (including, in simple cases, the correspondence between a line
being tangent to a curve and a repeated root of an equation).

1.2 Polynomials
A polynomial of degree n is an algebraic expression written in the form
P (x) = a0 + a1 x + a2 x2 + a3 x3 + . . . + an xn , where a0 , a1 , a2 ,..., an are real constants.
These constants are also known as coefficients. For example,

a0 = constant term, a1 = coefficient of x, a2 = coefficient of x2 ,


..., an = coefficient of xn .

1 dkrbabajee@gmail.com
c
CHAPTER 1. QUADRATICS

1.3 Quadratic Polynomials


A quadratic polynomial is an algebraic expression of degree 2 i.e. an expression in
which the highest power of x is 2.
The quadratic polynomial can be written in the standard form of ax2 + bx + c where
a is the coefficient of x2 , b is the coefficient of x and c is the constant term.

1.4 Completion to the square


Any quadratic expression can be expressed in the vertex form a(x + p)2 + q, where a,
p and q are constants.
2
∴ ax
| +{zbx + }c ≡ a(x + p)2 + q .
| {z }
Standard form Vertex form

The process by which the standard form of the quadratic expression is transformed
to its vertex form is called completing to the square. We note that the vertex form
allows us to identify the turning point (or vertex point) of a quadratic curve.

1.4.1 Method for completing the square


Consider the quadratic expression ax2 + bx + c.
Method 1
2
 of x become 1,
Step 1: Makethe coefficient
b c
i.e. a x2 + x + .
a a
Step 2: We
"simplifybetweensquare brackets as follows:
2 2 #
b c b
a x+ + −
2a a 2a
b
i.e. divide the coefficient of x by 2 and subtract the square of .
2a
Step 3: Expand and simplify to obtain
 2  2 !
b b
a x+ + c−a
2a 2a

Method 2

ax2 + bx + c = a(x + p)2 + q


ax2 + bx + c = ax2 + 2apx + ap2 + q (1.1)

2 dkrbabajee@gmail.com
c
CHAPTER 1. QUADRATICS

b
Comparing coefficient of x in eq. (1.1), we have 2ap = b ⇒ p = .
2a
Comparing constant term in eq. (1.1), we have ap2 + q = c ⇒ q = c − ap2 .
The standard form of ax2 + bx + c can be transformed in the form a(x + p)2 + q using
b
p= and q = c − ap2 .
2a
Example 1.1 Transform the standard form of x2 + 6x + 11 in the form
a(x + p)2 + q.

Solution
Method 1
   2
2 6 6
x + 6x + 11 = x + + 11 −
2 2
= (x + 3) + 11 − 9 = (x + 3)2 + 2.
2

Method 2
a = 1, b = 6, c = 11
b 6
p= = = 3, q = c − ap2 = 11 − (1 × 32 ) = 2
2a 2(1)

Example 1.2 Transform the standard form of 3x2 + 12x + 7 in the form
a(x + p)2 + q.

Solution
Method 1
 
2 2 7
3x + 12x + 7 = 3 x + 4x +
3
 
2 7 2
= 3 (x + 2) + − 2
3
 
2 5
= 3 (x + 2) − = 3(x + 2)2 − 5.
3

Method 2
a = 3, b = 12, c = 7
b 12
p= = = 2, q = c − ap2 = 7 − (3 × 22 ) = −5
2a 2(3)

3 dkrbabajee@gmail.com
c
CHAPTER 1. QUADRATICS

Example 1.3 Transform the standard form of −13+12x−2x2 in the form


a(x − p)2 + q.

Solution
Method 1
 
2 2 13
−13 + 12x − 2x = −2 x − 6x +
2
 
2 13 2
= −2 (x − 3) + −3
2
 
2 5
= −2 (x − 3) − = −2(x − 3)2 + 5.
2

Method 2
a = −2, b = 12, c = −13
b 12
p= = = −3, q = c − ap2 = −13 − (−2 × (−3)2 ) = 5
2a 2(−2)

1.5 Graphs of quadratic polynomials


The graph of a quadratic polynomial ax2 + bx + c is called a parabola.
The point at which its direction changes is called its turning point or the vertex of
the parabola.

The diagram shows for a > 0, the graph


of the quadratic polynomial is concave
upwards. The vertex is a minimum point.

The diagram shows for a < 0, the graph


of the quadratic polynomial is concave
downwards. The vertex is a maximum
point.

4 dkrbabajee@gmail.com
c
CHAPTER 1. QUADRATICS

Example 1.4 For each of the following quadratic polynomials, write


down the coordinates of the turning point, stating whether it is minimum
or maximum. Determine the line of symmetry and hence sketch the curve.
(i) y = 2x2 − 12x + 7 (ii) y = 3 − 4x − 2x2 (iii) y = (x − 1)(x − 2)

Solution
(i) By completing the square,
 
2 2 7
2x − 12x + 7 = 2 x − 6x +
2
 
2 7 2
= 2 (x − 3) + − 3
2
 
2 11
= 2 (x − 3) −
2
= 2(x − 3)2 − 11.

Since a = 2 > 0, we have a minimum point


which occurs when x − 3 = 0 ⇒ x = 3 and
y = −11.
Minimum point (3, −11).
The line of symmetry is x = 3.
The curve cuts the y-axis at (0, 7).
(ii) By completing the square,
 
2 2 3
3 − 4x − 2x = −2 x + 2x −
2
 
2 3 2
= −2 (x + 1) − − (−1)
2
 
2 5
= −2 (x + 1) −
2
2
= 5 − 2(x + 1) .

Since a = −2 < 0, we have a maximum


point which occurs when
x + 1 = 0 ⇒ x = −1 and y = 5.
Maximum point (−1, 5).
The line of symmetry is x = −1.
The curve cuts the y-axis at (0, 3).

5 dkrbabajee@gmail.com
c
CHAPTER 1. QUADRATICS

(iii) By completing the square,

(x − 1)(x − 2) = x2 − 3x + 2
 2  2
3 3
= x− +2−
2 2
 2
3 1
= x− −
2 4

Since a = 1 > 0, we have a minimum point


3 3
which occurs when x − = 0 ⇒ x =
2 2
which is the average of the x−coordiantes
of the points where the curve cuts the
x-axis.
 
1 3 1
y = − . Minimum point ,− .
4 2 4
3
The line of symmetry is x = .
2
The curve cuts the y-axis at (0, 2).
m+n
In general, if y = (x − m)(x − n), then y = (x − h)2 + k where h = .
2

Example
 1.5 The curve y = 2x2 + bx + c has a minimum point at

1 11
− ,− . Find the values of b and c.
2 2
Solution
1 1 11
a = 2, −h = − ⇒ h = k = −
2 2 2
b 1 b
h= ⇒ = ⇒b=2
2a 2 2(2)
 2
2 11 1 11 1 11 1
k = c − ah ⇒ − = c − 2 ⇒ − = c − ⇒ c = − + = −5
2 2 2 2 2 2
Exercise 1.1. [Complete to the square, graphs of quadratic polynomials]
1. Express each of the following quadratic polynomial in the form a(x + p)2 + q:
(i) x2 − 8x + 15 (ii) x2 + 3x + 4 (iii) 3x2 + 2x − 5
x2 2
(iv) (3x + 1)(2x − 1) (v) + 4x + (vi) 4 + 2x − x2
2 3
4 3
(vii) 5 − 3x − x2 (viii) 3x − 7x2 (ix) − x2 − 2x +
3 4
6 dkrbabajee@gmail.com
c
CHAPTER 1. QUADRATICS

2. For each of the following quadratic polynomial in question 1, write down the
coordinates of the turning point, stating whether it is minimum or maximum.
Determine the line of symmetry and hence sketch the curve.
3. The curve y = x2 + bx + c can be expressed in the form y = (x − 2)2 + 7. Find the
values of b and c.
4. Find the values of p and q for which p + 4x − 2x2 = 3 − 2(x − q)2 .
 
2 7 2209
5.* The minimum point of the curve y = ax −7x+c is ,− . Find the values
30 60
of a and c.
6.* The curve y = ax2 + bx + c cuts the y-axis at (0, 3) and has a minimum point at
(−3, −15). Find the values of a, b and c.

1.6 Roots of Quadratic Equations


1.6.1 Factorisation
Example 1.6 Solve by factorisation:
(i) x2 − 7x + 6 = 0 (ii) 12x2 − 7x − 10 = 0
Solution
(i) x2 − 7x + 6 = 0 ⇒ x2 − x − 6x + 6 = 0 ⇒ x(x − 1) − 6(x − 1) = 0
⇒ (x − 1)(x − 6) = 0 ⇒ x = 1 or 6
(ii) 12x2 − 7x − 10 = 0 ⇒ 12x2 + 15x − 8x − 10 = 0 ⇒ 3x(4x + 5) − 2(4x + 5) = 0
5 2
⇒ (4x + 5)(3x − 2) = 0 ⇒ x = − or
4 3
If a quadratic equation has roots r1 and r2 , then

Sum of roots, S = r1 + r2
Product of roots, P = r1 × r2
Quadratic equation is given by x2 − Sx + P = 0

Example 1.7 If a quadratic equation has roots 1 and 2, find the sum
and products of the roots and deduce the quadratic equation in the form
ax2 + bx + c = 0.
Solution
Sum of roots, S = 1 + 2 = 3 and product of roots, P = 1 × 2 = 2.
The quadratic equation is given by x2 − Sx + P = 0 ⇒ x2 − 3x + 2 = 0.

7 dkrbabajee@gmail.com
c
CHAPTER 1. QUADRATICS

1.6.2 Quadratic Formula


The roots of the quadratic equation ax2 + bx + c = 0 are given by the quadratic
formula:

−b ± b2 − 4ac
x= .
2a
Proof.

ax2 + bx + c = 0
 
2 b c
a x + x+ =0
a a
" 2  2 #
b c b
a x+ + − =0
2a a 2a
" 2 #
2
b 4ac − b
a x+ + =0
2a 4a2
 2
b b2 − 4ac
x+ =
2a 4a2

b ± b2 − 4ac
x+ =
2a √ 2a √
b b2 − 4ac −b ± b2 − 4ac
x=− ± = .
2a 2a 2a
b2 − 4ac is called the discriminant of the quadratic equation.

Example 1.8 Solve 2x2 − 3x − 1 = 0.

Solution

a = 2, b = −3, c = −1,
b2 − 4ac = (−3)2 − 4(2)(−1) = 17

−(−3) ± 17
x=
2(2)

3 ± 17
x= = 1.78 or − 0.28.
4

8 dkrbabajee@gmail.com
c
CHAPTER 1. QUADRATICS

1.6.3 Type of roots


The discriminant helps us to determine the nature of the roots.

Discriminant Type of Roots Geometrical Interpretation


2
b − 4ac > 0 Real and Distinct Curve meets the x-axis at two different points
b2 − 4ac = 0 Real and Equal x-axis is a tangent to curve
(coincident) or x-axis touches the curve
2
b − 4ac < 0 Not Real Curve does not cross the x-axis
(complex) or curve is either above or below the x-axis
b2 − 4ac ≥ 0 Real −

Example 1.9 Determine the nature of the roots for the following
quadratic equations:
(i) 3x2 − x + 4 = 0 (ii) 2x2 + x − 7 = 0 (iii) x2 + 2x + 1 = 0

Solution
(i) 3x2 − x + 4 = 0 (ii) 2x2 + x − 7 = 0 (iii) x2 + 2x + 1 = 0
a = 3, b = −1, c = 4 a = 2, b = 1, c = −7 a = 1, b = 2, c = 1
b2 − 4ac b2 − 4ac b2 − 4ac
= (−1)2 − 4(3)(4) = −47 < 0 = (1)2 − 4(2)(−7) = 43 > 0 = (2)2 − 4(1)(1) = 0
No real roots. Roots are real and distinct. Roots are real and equal.

Example 1.10 Find the values of p for which the quadratic equation
x2 + (2 + p)x + (13 − p) = 0 has equal roots. For these values of p, find the
roots.

Solution

x2 + (2 + p)x + (13 − p) = 0
a = 1, b = 2 + p, c = 13 − p
For equal roots, b2 − 4ac = 0
(2 + p)2 − 4(1)(13 − p) = 0
4 + 4p + p2 − 52 + 4p = 0
p2 + 8p − 48 = 0
(p + 12)(p − 4) = 0 ⇒ p = −12 or 4.
When p = −12, x2 − 10x + 25 = 0 ⇒ (x − 5)2 = 0 ⇒ x = 5.
When p = 4, x2 + 6x + 9 = 0 ⇒ (x + 3)2 = 0 ⇒ x = −3.

9 dkrbabajee@gmail.com
c
CHAPTER 1. QUADRATICS

Exercise 1.2. [Roots of Quadratic Equations, Quadratic formula, Type of roots]


1. Solve the equations by factorisation
(i) x2 − 5x + 6 = 0 (ii) x2 + 2x − 3 = 0 (iii) 5 + 4x − x2 = 0
2 2
(iv) x + 17x + 72 = 0 (v) 3x + 2x − 1 = 0 (vi) 3x2 − 10x + 3 = 0
(vii) 5 − 3x − 2x2 = 0 (viii) 6x2 + 13x + 6 = 0 (ix) 12x2 + 11x − 15 = 0
2. Solve the equations by the quadratic formula giving your answers in exact form:
(i) x2 − 4x + 2 = 0 (ii) x2 + 4x + 1 = 0 (iii) 2 + 2x − x2 = 0
(iv) x2 + 3x − 7 = 0 (v) 5x2 − 9x + 2 = 0 (vi) 1 − 7x − 4x2 = 0
(vii) 3x − 2x − 4 = 0 (viii) 2x + 6x + 3 = 0 (ix) 12x2 − 20x − 5 = 0
2 2

3. For the following roots r1 and r2 of a quadratic equation, find the sum and
products of the roots and deduce the quadratic equation in the form
ax2 + bx + c = 0.
(i) r1 = 2, r2 = 3 (ii) r1 = −1, r2 = −3 (iii) r1 = −2, r2 = 5
1 3 2
(iv) r1 = 3, r2 = −4 (v) r1 = , r2 = 1 (vi) r1 = − , r2 = −
2 2 5
√ √
4. A quadratic equation has roots −1 + 2 and −1 − 2, deduce the quadratic
equation.
5. Determine the nature of the roots of each of the following quadratic equations:
(i) 2x2 + 2x − 1 = 0 (ii) 3x2 − x + 1 = 0 (iii) x2 − 4x + 4 = 0 (iv) x2 + 5x + 5 = 0
6. Determine the set of values of p for which the quadratic equation px2 + 4x + 1 = 0
has real roots.
7. Determine the value of k for which the quadratic equation 4x2 − 3kx + 1 = 0 has
equal roots. For these values of k, find the roots.
8. Find the values of k for which the quadratic equation x2 +7(3+2k)−2x(1+3k) = 0
has equal roots.
9. If the equation (1 + m2 )x2 + 2mcx + c2 − a2 = 0 has equal roots, show that
c2 = a2 (1 + m2 ).
10. Show that the equation kx2 + (k − 1)x − 1 = 0 has real roots for all values of k.

1.7 Disguised Quadratic Equations


Sometimes, we come across equations which at the outset are not quadratic equations
but by proper substitution or after simplification, can be transformed into quadratic
equations. Such equations are called equations reducible to quadratic equations or
disguised quadratic equations.
Let us consider some examples to illustrate the solutions of such equations.

10 dkrbabajee@gmail.com
c
CHAPTER 1. QUADRATICS

1.7.1 Type I: Equation of the form ax4 + bx2 + c = 0


Method: Use the substitution y = x2 , then the given equation becomes ay 2 +by +c = 0
which is quadratic in y.
Example 1.11 Solve the equation x4 + 3x2 − 4 = 0.
Solution

Put y = x2
(x2 )2 + 3x2 − 4 = 0 ⇒ y 2 + 3y − 4 = 0
(y + 4)(y − 1) = 0 ⇒ y = −4 or 1
x2 = −4 or x2 = 1
2
No solution since x ≥ 0 or x = ±1

q
1.7.2 Type II: Equation of the form px + =r
x
q
Method: Multiply px + = r by x throughout, we get px2 + q = rx which is quadratic
x
in x.
2
Example 1.12 Solve the equation x − = 2.
x
Solution

Multiply x throughout
x2 − 2 = 2x ⇒ x2 − 2x − 2 = 0

x = 1 ± 12 − −2, using alternative quadratic formula

x = 1 ± 3.
p
1.7.3 Type III: Equation of the form a − x2 = bx + c

Method: Since the given equation involves one radical a − x2 , we must seek those
2 2
√ for which a − x ≥ 0 i.e. x ≤ a.
solutions
Since a − x2 ≥ 0 ⇒ bx + c ≥ 0. Hence we need solutions for which

x2 ≤ a and bx + c ≥ 0. (1.2)

The next step when squaring on both sides, we have



( a − x2 )2 = (bx + c)2
a − x2 = b2 x2 + 2bcx + c2
(b2 + 1)x2 + 2bcx + c2 − a = 0

11 dkrbabajee@gmail.com
c
CHAPTER 1. QUADRATICS

which is quadratic in x.
The solutions of this equation which satisfy eq. (1.2) are the solutions of the given
equation.

Example 1.13 Solve the equation x − 25 − x2 = 1.

Solution
Separating the radical

x − 1 = 25 − x2 . (1.3)

We must seek solutions for which 25 − x2 ≥ 0 i.e. x2 ≤ 25 and x − 1 ≥ 0 i.e. x ≥ 1. We


look for solutions which satisfy

x2 ≤ 25 and x ≥ 1. (1.4)

Squaring on both sides of (1.3), we have

(x − 1)2 = 25 − x2
x2 − 2x + 1 = 25 − x2
h i
2x2 − 2x − 24 = 0 ÷ 2
x2 − x − 12 = 0
(x − 4)(x + 3) = 0 ⇒ x = 4 or − 3.

Only x = 4 satisfies eq. (1.4). Hence the required solution is x = 4.

1.7.4 Type IV: Equation of the form a(p2x ) + bpx + c = 0


Method: Use the substitution y = px , then the given equation becomes ay 2 +by +c = 0
which is quadratic in y.

Example 1.14 Solve the equation 22x − 3(2x+2) + 32 = 0.

Solution

Put y = 2x
(2x )2 − 3(2x × 22 ) + 32 = 0 ⇒ y 2 − 12y + 32 = 0
(y − 4)(y − 8) = 0 ⇒ y = 4 or 8
2x = 4 or 2x = 8
2x = 22 or 2x = 23
x=2 or x = 3

12 dkrbabajee@gmail.com
c
CHAPTER 1. QUADRATICS

Exercise 1.3. [Disguised Quadratic Equations]

1. Solve the following equations by reducing them to quadratic equations:


3 1
(i) x4 − 26x2 + 25 = 0 (ii) z 4 + 8z 2 − 9 = 0 (iii) y − =
y 2
5 4
(iv) 3y + =2 (v) 2x + = 9 (vi) 8x6 + 9x3 + 1 = 0
16y x
2. Solve the following equations:
(i) 32x + 9 − 10(3x ) = 0 (ii) 22x + 2x+1 = 4 − 2x (iii) 4x − 3(2x+3 ) + 128 = 0

(iv) 3x+2 + 3−x = 10 (v) 9x+2 − 6(3x+1 ) + 1 = 0 (vi) 51+x + 51−x = 26

3. Solve√ the following equations:



(i) √x + 2x = 1 (ii) √217 − x =
√x − 7
(iii) 2x + 9 + x = 13 (iv) x + 1 + x + 3 = 4

4. Solve the following equations:


1 13 1 1 3 3
(i) 4 = 2 − 36 (ii) 5x 2 − 6x− 2 + 1 = 0 (iii) x 2 = 9 − 8x− 2
x x

5.* Solve the equation 5x4 = 2(x2 + 2) giving your answers in exact form.

1.8 Simultaneous equations: one linear and the other


nonlinear
1.8.1 Algebraic Method
Procedure:

Step 1: From the linear equation, express one of the unknown in terms of the other.

Step 2: Substitute this expression in the given nonlinear equation and obtain a quadratic
equation in a single unknown.

Step 3: Solve the resulting quadratic equation to obtain the solutions of the single
unknown.

Step 4: Replace these solutions in Step 1 to obtain the solutions of the second unknown.

13 dkrbabajee@gmail.com
c
CHAPTER 1. QUADRATICS

Example 1.15 Solve x + y = 5 and x2 + y 2 = 13.


Solution

x+y =5 ...... Linear (1.5)


2 2
x + y = 13 ...... Nonlinear (1.6)
from (1.5), y = 5 − x (1.7)
replace (1.7) in (1.6) we get
x2 + (5 − x)2 = 13
x2 + 25 − 10x + x2 = 13
h i
2x2 − 10x + 12 = 0 ÷ 2 ⇒ x2 − 5x + 6 = 0
(x − 2)(x − 3) = 0 ⇒ x = 2 or x = 3
From (1.7), when x = 2, y = 5 − 2 = 3
From (1.7), when x = 3, y = 5 − 3 = 2
(2, 3) and (3, 2) are the solutions.

1.8.2 Graphical Method


When solving simultaneous equations with one linear and one nonlinear equation,
the solutions give the coordinates of the points intersection of the line with the curve.

Example 1.15 can be interpreted


graphically, that is the line x + y = 5
meets the curve x2 + y 2 = 13 at the points
(2, 3) and (3, 2).

Example 1.16 The straight line 2x+y = 7 meets the circle x2 −xy+y 2 = 7
at the points A and B. Find the coordinates of A and B.
Solution

2x + y = 7 ...... Linear (1.8)


2 2
x − xy + y = 7 ...... Nonlinear (1.9)
from (1.8), y = 7 − 2x (1.10)
replace (1.10) in (1.9) we get
x2 − x(7 − 2x) + (7 − 2x)2 = 7
x2 − 7x + 2x2 + 49 − 28x + 4x2 = 7

14 dkrbabajee@gmail.com
c
CHAPTER 1. QUADRATICS

Solution
h i
7x2 − 35x + 42 = 0 ÷ 7 ⇒ x2 − 5x + 6 = 0
(x − 2)(x − 3) = 0 ⇒ x = 2 or x = 3
From (1.10), when x = 2, y = 7 − 2(2) = 3
From (1.10), when x = 3, y = 7 − 2(3) = 1
A(2, 3) and B(3, 1) are the points of intersection.

1.9 More on graphical representation

When a line is a tangent to the


curve then b2 − 4ac = 0. In this
case, we have equal roots.

When a line cuts a curve at two


distinct points A and B then
b2 − 4ac > 0. In this case, we have
real and distinct roots.

When a line does not cross a


curve, then b2 − 4ac < 0. In this
case, we have no real roots. The
curve is either above or below the
line.

1.10 Quadratic Inequalities


Suppose that a < b, consider the quadratic inequality (x − a)(x − b) < 0.

15 dkrbabajee@gmail.com
c
CHAPTER 1. QUADRATICS

The diagram shows a number line. The left part (pink) represents the inequality
x < a. The middle part (green) represents the inequality a < x < b and the right part
(blue) represents the inequality x > b.

1. Graphical Method:

The diagram shows the curve


y = (x − a)(x − b). When y > 0,
the brown region is represented
by x < a or x > b. When y < 0,
the blue region is represented by
a < x < b.

2. Numerical Method
The critical values occur when (x − a)(x − b) = 0 ⇒ x = a or x = b.

(x − a) (x − b) y = (x − a)(x − b)
x<a −ve −ve +ve (> 0)
a < x < b +ve −ve −ve (< 0)
x>b +ve +ve +ve (> 0)

In both cases,

(x − a)(x − b) > 0 ⇒ x < a or x > b


(x − a)(x − b) < 0 ⇒ a < x < b.

Example 1.17 Solve the inequality (x + 1)2 + 2x ≥ 13

Solution

(x + 1)2 + 2x ≥ 13
x2 + 2x + 1 + 2x ≥ 13
x2 + 4x − 12 ≥ 0
(x + 6)(x − 2) ≥ 0
Critical values of x are -6, 2

(x + 6) (x − 2) (x + 6)(x − 2)
x ≤ −6 −ve −ve +ve (≥ 0) X
−6 ≤ x ≤ 2 +ve −ve −ve (≤ 0) ×
x≥2 +ve +ve +ve (≥ 0) X
∴ x ≤ −6 or x ≥ 2

16 dkrbabajee@gmail.com
c
CHAPTER 1. QUADRATICS

Example 1.18 Find the range of values of p for which the equation
px2 + 4x + (p + 3) = 0 has real roots.

Solution
a = p, b = 4, c = p + 3
For real roots, b2 − 4ac ≥ 0
42 − 4(p)(p + 3) ≥ 0 ⇒ 16 − 4p2 − 12p ≥ 0 ⇒ [4p2 + 12p − 16 ≤ 0] ÷ 4
p2 + 3p − 4 ≤ 0 ⇒ (p + 4)(p − 1) ≤ 0
Critical values of p are -4, 1
(p + 4) (p − 1) (p + 4)(p − 1)
p ≤ −4 −ve −ve +ve (≥ 0) ×
−4 ≤ p ≤ 1 +ve −ve −ve (≤ 0) X
p≥1 +ve +ve +ve (≥ 0) ×
∴ −4 ≤ p ≤ 1

Exercise 1.4. [Simultaneous equations and Quadratic Inequalities]

1. Solve the following simultaneous equations:


x+y = 5 y = 2x + 1 x+y+3 =0
(i) (ii) 2 (iii)
2x2 = y 2 − 1 x = 3y − 11 xy = 2
3x − 2y = 7 2x + 3y = 5 5x + 5y + 1 = 0
(iv) (v) (vi)
x2 − xy = 3 x2 + y 2 = 13 x2 − 2x + y 2 + 2y + 1 = 0

2. Solve the inequalities:


(i) (x + 1)(x + 2) < 0 (ii) (2x − 1)(3x − 1) > 0 (iii) (x + 2)(5x − 2) ≤ 0
(iv) (x − 3)(x + 4) ≥ 0 (v) x2 + 4x + 3 < 0 (vi) 5x2 − 4x − 1 ≥ 0
(vii) (x + 2)2 < 4 (viii) (x − 1)2 + 2x ≤ 10 (ix) (1 − 2x) ≤ (2 + x)2

3. Find the range of values of p for which the given equation has real roots:
(i) x2 + (p − 3)x − 4p = 0 (ii) (2x + 1)2 = px

4. Find the range of values of a for which the given equation has no real roots:
(i) 2x2 − ax + (a + 6) = 0 (ii) a(x2 + 2) = 2x2 + 6x + 1

5. Find the range of values of k for which the line y = kx − 3 does not intersect the
curve y = x2 + 2x − 2.

17 dkrbabajee@gmail.com
c
CHAPTER 1. QUADRATICS

1.11 Miscellaneous Exercises


Example 1.19 A curve has equation x2 + xy + 2 = 0 and a line has
equation y = x + p, where p is a constant.

(i) Find the set of values of p for which the curve and the line have no
common points.

(ii) State the values of p for which the line is a tangent to the curve and
find the coordinates of the points where the line touches the curve.

Solution

(i) y = x + p ...... Linear (1.11)


x2 + xy + 2 = 0 ...... Nonlinear (1.12)
replace (1.11) in (1.12) we get
x2 + x(x + p) + 2 = 0
2x2 + px + 2 = 0 (1.13)
a = 2, b = p, c = 2
For no common points (no roots), b2 − 4ac < 0
p2 − 4(2)(2) < 0 ⇒ p2 − 16 < 0
(p − 4)(p + 4) < 0
Critical values of p are -4, 4
(p + 4) (p − 4) (p + 4)(p − 4)
p < −4 −ve −ve +ve (> 0) ×
−4 < p < 4 +ve −ve −ve (< 0) X
p>4 +ve +ve +ve (> 0) ×
∴ −4 < p < 4

(ii) For equal roots, b2 − 4ac = 0 ⇒ p2 − 16 = 0 ⇒ p = ±4


From (1.13), when p = 4, 2x2 + 4x + 2 = 0 ⇒ x2 + 2x + 1 = 0
(x + 1)2 = 0 ⇒ x = −1 ⇒ y = x + p = −1 + 4 = 3
From (1.13), when p = −4, 2x2 − 4x + 2 = 0 ⇒ x2 − 2x + 1 = 0
(x − 1)2 = 0 ⇒ x = 1 ⇒ y = x + p = 1 − 4 = −3
(−1, 3) and (1, 3) are the points where the line touches the curve.

Miscellaneous Exercise 1.
5 1
1. Solve the equation 4 − 2
+ 4 = 0.
x x

18 dkrbabajee@gmail.com
c
CHAPTER 1. QUADRATICS

2. Solve the equation (x + 1)4 = 13(x + 1)2 − 36.


3. Find the range of values of k for which the curve y = x2 + 3kx + k is above the
x-axis.
4. Find the range of values of k for which the equation 4x2 + 8x − 8 < k(4x − 3).
5. (i) Find the set of values
√ of the constant c for which the line y = x + c intersects
the curve y = 2 x at two distinct points.

(ii) Verify whether or not the line y = x intersects the curve y = 2 x at two
distinct points. If yes, find the coordinates of the two points.
6. The equation x2 + bx + c = 0 has roots 3 and 7.
(i) Find the values of b and c.
(ii) Using the values of b and c, find the range of values of d for the equation
x2 + bx + c + d = 0 has no real roots.
7. Find the values of r for which the line x + y = r is a tangent to the curve
x2 + 2y 2 = 6. In these cases, find the coordinates of the points where the line
touches the curve.
8. The equation of a curve is given by y = (x − 1)(x − 2)
(i) Express y in the form y = (x − p)2 + q.
(ii) Find the turning point of the curve and determine its nature. Sketch the
curve.
(iii) Find the greatest and least values of y when 0 < x < 2.
9. A circle has equation x2 + y 2 = 8 and a line has equation y = x + p, where p is a
constant.
(i) Find the set of values of p for which the line intersect the circle at two
distinct points.
(ii) State the values of p for which the line is a tangent to the circle and find the
coordinates of the points where the line touches the curve.
10.* The equation of a curve is given by y = px2 + qx + r. The line y = x + 1 is a
tangent to the curve.
(i) Show that (q − 1)2 = 4p(r − 1).

3 1
(ii) Given further that the curve has a minimum point at − , − , show that
2 4
q = 3p and 4r = 9p − 1. Hence, find the values of p, q and r.

19 dkrbabajee@gmail.com
c
C HAPTER 2

Functions

2.1 Objective
At the end of this chapter, students should be able to:

1. understand the terms function, domain, range;

2. use the vertical line test whether a relation is a function;

3. identify the range of a given function in simple cases;

4. find the composition of two given functions;

5. determine whether or not a given function is one-one using the horizontal line
test, and find the inverse of a one-one function in simple cases;

6. illustrate in graphical terms the relation between a one-one function and its
inverse.

2.2 Basic Definitions


A function is a special relation between values: Each of its input values (object) gives
back exactly one output value (image).

The diagram shows that f is a function because the object x1 is related to image y1 ,
the object x2 is related to image y2 , the objects x3 and x4 are related to image y3 and

20 dkrbabajee@gmail.com
c
CHAPTER 2. FUNCTIONS

the image y4 is not related to any objects.


What can go into a function is called the Domain.
Domain = {x1 , x2 , x3 , x4 }.
What may possibly come out of a function is called the Codomain.
Codomain = {y1 , y2 , y3 , y4 }.
What actually comes out of a function is called the Range.
Range = {y1 , y2 , y3 }.

The diagram shows that f is not a function because the object x1 has two images y1
and y2 .

2.3 Geometrical Interpretation: Vertical Line Test


If the graph of y = f (x) intersects at only one point with every vertical line drawn,
then f (x) is a function.
When vertical lines are drawn and the graph of y = f (x) intersects at more than one
point, f (x) is not a function.

The diagram shows that f is a function because the graph of y = f (x) intersects at
only one point with every vertical line drawn. It also shows that g is not a function
because the graph of y = g(x) intersects at two points with every vertical line drawn.

21 dkrbabajee@gmail.com
c
CHAPTER 2. FUNCTIONS


For example, f (x) = x + 1 is function but g(x) = x is not a function.

2.4 Representation of functions


A function is a rule that maps a given value onto another.

f maps x onto 2x ⇒ f : x 7→ 2x ⇒ f (x) = 2x.

So f (0) = 2(0) = 0, f (1) = 2(1) = 2, f (−1) = 2(−1) = −2, etc. The function f maps 0
onto 0, 1 onto 2, -1 onto -2.

2.5 Finding the range of a function


1 If the function f (x) is linear on the interval a ≤ x ≤ b, its range is given by
min(f (a), f (b)) ≤ f (x) ≤ max(f (a), f (b)).

2 If the function f (x) is quadratic and has a minimum point (h, f (h)) on the
interval a ≤ x ≤ b, its range is given by f (h) ≤ f (x) ≤ max(f (a), f (b)).

3 If the function f (x) is quadratic and has a maximum point (h, f (h)) on the
interval a ≤ x ≤ b, its range is given by min(f (a), f (b)) ≤ f (x) ≤ f (h).

Example 2.1 Sketch the graphs of the following functions and find
their range:
(i) f (x) = x + 1, −3 ≤ x ≤ 3 (ii) f (x) = x2 − 4x + 3, −3 ≤ x ≤ 3

(iii) f (x) = 3 − 4x − 2x2 , −3 ≤ x ≤ 3 (iv) f (x) = x3 , −2 ≤ x ≤ 2


1
(v) f (x) = ,x≥1
x

Solution

The graph is a straight line.


(i) f (−3) = −2 and f (3) = 4.
The range is −2 ≤ f (x) ≤ 4.

(ii) By completing the square, f (x) = x2 − 4x + 3 = (x − 2)2 + 3 − 4 = (x − 2)2 − 1.

22 dkrbabajee@gmail.com
c
CHAPTER 2. FUNCTIONS

Since x = 2 is found the interval −3 ≤ x ≤ 3, the graph of y = f (x) contains the


minimum point (2,-1). f (−3) = 24 and f (3) = 0. The range is −1 ≤ f (x) ≤ 24.

(iii) By completing the square, f (x) = 3 − 4x − 2x2 = 5 − 2(x + 1)2 .


Since x = −1 is found the interval −3 ≤ x ≤ 3, the graph of y = f (x) contains the
minimum point (-1,5). f (−3) = −3 and f (3) = −27. The range is −27 ≤ f (x) ≤ 5.

(iv) The graph of f (x) contains the point of inflexion at the origin.
f (−2) = −8 and f (2) = 8. The range is −8 ≤ f (x) ≤ 8.

23 dkrbabajee@gmail.com
c
CHAPTER 2. FUNCTIONS

(v) The graph of f (x) has the asymptotes x = 0 and y = 0.


As x tends to infinity, f (x) tends to zero. f (1) = 1. The range is 0 < f (x) ≤ 1.

Example 2.2 Given that the function f : x 7→= ax2 + b, x ∈ R such that
f (0) = 1 and f (1) = −2, find

(i) the values of a and b,

(ii) the range of f corresponding to the domain −2 ≤ x ≤ 2.

(iii) the values of p for which f (p) + f (−p) + 22 = 0.


Solution
(i) f (0) = a(0)2 + b = 1 ⇒ b = 1 and f (1) = a + b = −2 ⇒ a = −2 − b = −2 − 1 = −3.
(ii) f (x) has a maximum point at (0, 1) and x = 0 lies in the domain −2 ≤ x ≤ 2.
f (−2) = f (2) = −11. The range is −11 ≤ f (x) ≤ 1.
(iii) f (p) + f (−p) = (1 − 3p2 ) + (1 − 3(−p)2 ) = 2 − 6p2 .√
So 2 − 6p2 + 22 = 0 ⇒ 6p2 = 24 ⇒ p2 = 4 ⇒ p = ± 4 = ±2.
Exercise 2.1. [Vertical line test and range of a function]
1. Using the vertical line test, determine for x ∈ R which of the following mappings
are functions or not:
(i) f : x 7→ 3x + 1 (ii) f : x 7→ x3 + 1
√ √
(iii) f : x 7→ x + 1 (iv) f : x 7→ x − 1

2. Sketch the graphs of the following functions and find their range:
(i) f (x) = 4x + 1, x ≥ 0 (ii) f (x) = 1 − 3x, x ≥ 0

(iii) f (x) = 2x − 3, −1 ≤ x ≤ 2 (iv) f (x) = 1 − x, −5 ≤ x ≤ 5

(v) f (x) = 2(x − 1)2 + 3, −2 ≤ x ≤ 2 (vi) f (x) = 3 − 2(x − 1)2 , −2 ≤ x ≤ 2

24 dkrbabajee@gmail.com
c
CHAPTER 2. FUNCTIONS

(vii) f (x) = x2 + 6x − 1, −5 ≤ x ≤ 5 (viii) f (x) = 1 + 3x − 2x2 , 0 ≤ x ≤ 1


1
(ix) f (x) = (x + 1)(x − 3), −1 ≤ x ≤ 3 (x) f (x) = ,x≥0
x+1
1
(xi) f (x) = (x − 1)3 , −2 ≤ x ≤ 2 (xii) f (x) = − 2 , x < 0
x

3. Given that the function f : x 7→= ax + b, x ∈ R such that f (0) = −2 and f (2) = 2,
find

(i) the values of a and b,


(ii) the value of p for which f (p) + f (−p) = f (2p).
ax + b 1
4. Given that the function f : x 7→ , x 6= 6 such that f (0) = and f (−1) = 1,
x−6 2
find

(i) the values of a and b,


(ii) the values of p for which 4f (2p) = 25f (p).

5. A function f (x) is defined by f (x) = (x − 4)2 + 1.

(i) Find the range of f corresponding to the domain 0 ≤ x ≤ 5.


(ii) Find the domain for x corresponding to the range 2 ≤ f (x) ≤ 5.
(iii) Solve the inequality f (x) ≤ 17.
(iv) Find the value of p for which f (p) = f (−p) − 32.

2.6 Composite functions

The diagram shows the composite functions g ◦ f and f ◦ g.

25 dkrbabajee@gmail.com
c
CHAPTER 2. FUNCTIONS

Thus g ◦ f (x) = g(f (x)) and f ◦ g(x) = f (g(x)).


Furthermore, f 2 (x) = f (f (x)).

Example 2.3 Find f 2 , g 2 , f g and gf for the following functions:

(i) f (x) = 3x − 1, g(x) = x − 3,

(ii) f (x) = 2x + 1, g(x) = x2 + 1,


x−1
(iii) f (x) = 3x + 2, g(x) = , x 6= −1,
x+1
x−5
(iv) f (x) = 4x + 5, g(x) = .
4
Solution
(i) f 2 (x) = f (f (x)) = 3(f (x)) − 1 = 3(3x − 1) − 1 = 9x − 3 − 1 = 9x − 4,
g 2(x) = g(g(x)) = g(x) − 3 = x − 3 − 3 = x − 6,
f (g(x)) = 3g(x) − 1 = 3(x − 3) − 1 = 3x − 10,
g(f (x)) = f (x) − 3 = (3x − 1) − 3 = 3x − 4.

(ii) f 2 (x) = f (f (x)) = 2(f (x)) + 1 = 2(2x + 1) + 1 = 4x + 2 + 1 = 4x + 3,


g 2 (x) = g(g(x)) = g(x)2 + 1 = (x2 + 1)2 + 1 = x4 + 2x2 + 2,
f (g(x)) = 2(g(x)) + 1 = 2(x2 + 1) + 1 = 2x2 + 3,
g(f (x)) = (f (x))2 + 1 = (2x + 1)2 + 1 = 4x2 + 4x + 2.
(iii) f 2 (x) = f (f (x)) = 3(f (x)) + 2 = 3(3x + 2) + 2 = 9x + 6 + 2 = 9x + 8,
x−1 x − 1 − (x + 1)
g(x) − 1 −1 −2 1
g 2(x) = g(g(x)) = = x+1 = x+1 = = − , x 6= 0,
g(x) + 1 x−1 x − 1 + (x + 1) 2x x
+1
x+1 x+1
 
x−1 3(x − 1) + 2(x + 1) 5x − 1
f (g(x)) = 3(g(x)) + 2 = 3 +2= = , x 6= −1,
x+1 x+1 x+1
f (x) − 1 3x + 2 − 1 3x + 1
g(f (x)) = = = , x 6= −1.
f (x) + 1 3x + 2 + 1 3x + 3
(iv) f 2 (x) = f (f (x)) = 4(f (x)) + 5 = 4(4x + 5) + 5 = 16x + 20 + 5 = 16x + 25,
x−5 x − 5 − 20
g(x) − 5 −5 x − 25
g 2 (x) = g(g(x)) = = 4 = 4 = ,
4   4 4 16
x−5
f (g(x)) = 4g(x) + 5 = 4 + 5 = x − 5 + 5 = x,
4
f (x) − 5 4x + 5 − 5
g(f (x)) = = = x.
4 4
26 dkrbabajee@gmail.com
c
CHAPTER 2. FUNCTIONS

For parts (i)-(iii), we note that f g 6= gf .


For part (iv), we note that f g(x) = gf (x) = x. This leads us to the definition of inverse
function in section 2.7.
Example 2.4 Given that the functions f : x 7→ 2x + a and
x 1
g : x 7→ , x 6= −b such that f g(0) = −1 and gf (1) = , find
x+b 2
(i) the values of a and b,

(ii) the value of p for which f g(p) = gf (p).

Solution
2x (a + 2)x + ab
(i) f (g(x)) = 2g(x) + a = +a=
x+b x+b
0 + ab
f (g(0)) = −1 ⇒ = −1 ⇒ a = −1
0+b
f (x) 2x + a
g(f (x)) = =
f (x) + b 2x + a + b
1 2+a 1 2−1 1
g(f (1)) = ⇒ = ⇒ =
2 2+a+b 2 2−1+b 2
1 1
⇒ = ⇒ 2= 1+b⇒b= 2−1 =1
1+b 2
x−1 2x − 1
(ii) f g(x) = , gf (x) =
x+1 2x
p−1 2p − 1
f g(p) = gf (p) ⇒ = ⇒ 2p(p − 1) = (2p − 1)(p + 1)
p+1 2p
1
2p2 − 2p = 2p2 + p − 1 ⇒ 3p = 1 ⇒ p =
3
Exercise 2.2. [Composite functions]

1. Find f 2 , g 2, f g and gf for the following functions:

(i) f (x) = x + 3, g(x) = 2x + 1,


x−4
(ii) f (x) = 5x + 4, g(x) = ,
5
(iii) f (x) = 2x − 5, g(x) = x2 − 1,
x+1
(iv) f (x) = 2x − 3, g(x) = , x 6= 1.
x−1
2. Given that f : x 7→ 2x, g : x 7→ 1 + x and h : x 7→ x2 , write a function composition
in terms of f , g or h for the following functions:
(i) 2x2 (ii) 2 + 2x (iii) 1 + 2x (iv) 4x2 (v) (1 + x)2 (vi) 1 + x2

27 dkrbabajee@gmail.com
c
CHAPTER 2. FUNCTIONS

1
3. Given that f : x 7→ 2x + 1 and g : x 7→ , x 6= 0 for the domain −2 ≤ x ≤ 2,
x
(i) find f 2 in a similar form and state the range of f 2 ,
(ii) solve the equation f (x)=g(x),
(iii) show that the equation f g(x) = gf (x) has only one root and find it.

4. Given that the functions f : x 7→ 3x + 4 and g : x 7→ x2 + p find the range of


values of p for which the equation f g(x) = gf (x) has no real roots.
x+b
5.* Given that the functions f : x 7→ x + a and g : x 7→ , x 6= 2 such that
x−2
3
f g(0) = and gf (−1) = −2, find
2
(i) the values of a and b,
(ii) the value of p for which f g(p) = 3 gf (p).

2.7 Inverse function


g(x) is called the inverse of a function f (x) if f (g(x)) = x. Then g(x) = f −1 (x).

2.7.1 Geometrical Interpretation: Horizontal Line Test

The condition for a function f to have an inverse is that f must be one-one.


The diagram shows that f is one-one because the graph of y = f (x) intersects at only
one point with every horizontal line drawn.
It also shows that g is not one-one (two-one) because the graph of y = g(x) intersects
at two points with every horizontal line drawn.
For example, f (x) = x + 1 is one but g(x) = x2 is not one-one.

28 dkrbabajee@gmail.com
c
CHAPTER 2. FUNCTIONS

2.7.2 Finding the inverse of function


To find the inverse of a function,

Step 1: We let y = f (x).

Step 2: We make x subject of formula in the equation y = f (x) so that x = f −1 (y).

Step 3: We replace each y by x to get an expression for f −1 (x).

Step 4: The domain of f −1 = Range of f .

Example 2.5 Given that f : x 7→ 3x − 2, x ≥ 0, find f −1 (x) stating its


domain.

Solution
y+2 x+2
let y = f (x) ⇒ y = 3x − 2 ⇒ 3x = y + 2 ⇒ x = ⇒ f −1 (x) =
3 3
f (0) = −2, Range of f = {f (x) ≥ −2}
Domain of f −1 = Range of f = {x ≥ −2}.

2.7.3 Graphical Relation between a Function and its Inverse


1. The graph of y = f −1 (x)
is obtained by reflecting the
graph of y = f (x) in the line
y = x.

2. Any point (a, b) on the graph


of y = f (x) becomes (b, a)
(interchange) on the graph
of y = f −1 (x). That is, if
b = f (a) then a = f −1 (b).

2.7.4 Inverse of a quadratic function on an interval


Let f (x) = ax2 +bx+c, A ≤ x ≤ B. After completing to the square, f (x) = a(x−h)2 +k.
Suppose that A ≤ h ≤ B, f (x) has an inverse (1-1) if

1. a > 0 (minimum) and f (x) is defined on the interval A ≤ x ≤ h,


Range of f = {k ≤ f (x) ≤ f (A)} and
r
−1 x−k
f (x) = h − , k ≤ x ≤ f (A)
a

29 dkrbabajee@gmail.com
c
CHAPTER 2. FUNCTIONS

2. a > 0 (minimum) and f (x) is defined on the interval h ≤ x ≤ B,


Range of f = {k ≤ f (x) ≤ f (B)} and
r
−1 x−k
f (x) = h + , k ≤ x ≤ f (B)
a

3. a < 0 (maximum) and f (x) is defined on the interval A ≤ x ≤ h,


Range of f = {f (A) ≤ f (x) ≤ k} and
r
x−k
f −1 (x) = h − , f (A) ≤ x ≤ k
a

4. a < 0 (maximum) and f (x) is defined on the interval h ≤ x ≤ B,


Range of f = {f (B) ≤ f (x) ≤ k} and
r
−1 x−k
f (x) = h + , f (B) ≤ x ≤ k
a

5. Any point (p, q) on the graph of y = f (x) becomes (q, p) (interchange) on the
graph of y = f −1 (x). That is, if q = f (p) then p = f −1 (q).

Example 2.6 For the following functions f , find its inverse stating its
domain and sketch on a single diagram the graphs of y = f (x) and
y = f −1 (x), making clear the relationship between these two graphs:

(i) f (x) = x − 1, −2 ≤ x ≤ 2,

(ii) f (x) = x2 − 2x + 2, 1 ≤ x ≤ 3,

(iii) f (x) = x2 − 2x + 2, −2 ≤ x ≤ 1,

(iv) f (x) = 1 − 4x − x2 , −4 ≤ x ≤ −2,

(v) f (x) = 1 − 4x − x2 , −2 ≤ x ≤ 2.

Solution
(i) let y = f (x) ⇒ y = x − 1 ⇒ x = y + 1 ⇒ f −1 (x) = x + 1
f (−2) = −3, f (2) = 1, Range of f = {−3 ≤ f (x) ≤ 1}
Domain of f −1 = Range of f = {−3 ≤ x ≤ 1}.

30 dkrbabajee@gmail.com
c
CHAPTER 2. FUNCTIONS

The diagram shows the graphs of


y = f (x) and y = f −1 (x). The
graph of y = f −1 (x) is obtained by
reflecting the graph of
y = f (x) in the line y = x. Also
the coordinates (2, 1) → (1, 2) and
(−2, −3) → (−3, −2) under this
transformation.
p
(ii) let y = f (x) ⇒ y = x2 − 2x + 2 ⇒ y = (x − 1)2 + 1 ⇒ x = 1 ± y − 1

f −1 (x) = 1 ± x − 1
f (x) has a minimum point at (1, 1)
f (1) = 1, f (3) = 5, Range of f = {1 ≤ f (x) ≤ 5}
Domain of f −1 = Range of f = {1 ≤ x ≤ 5}
we have two expressions, we choose the point (5, 3)
to check which expression is correct.

f −1 (x) = 1 + x − 1 ⇒ f −1 (5) = 1 + 2 = 3 which is correct.

f −1 (x) = 1 − x − 1 ⇒ f −1 (5) = 1 − 2 = −1 which is incorrect.

f −1 (x) = 1 + x − 1, 1 ≤ x ≤ 5

The diagram shows the graphs of


y = f (x) and y = f −1 (x). The
graph of y = f −1 (x) is obtained by
reflecting the graph of
y = f (x) in the line y = x.
Also the coordinates (3, 5) → (5, 3)
under this transformation. (1, 1)
is invariant.
p
(iii) let y = f (x) ⇒ y = x2 − 2x + 2 ⇒ y = (x − 1)2 + 1 ⇒ x = 1 ± y − 1

f −1 (x) = 1 ± x − 1
f (x) has a minimum point at (1, 1)
f (1) = 1, f (−2) = 10, Range of f = {1 ≤ f (x) ≤ 10}
Domain of f −1 = Range of f = {1 ≤ x ≤ 10}
we have two expressions, we choose the point (10, −2).
to check which expression is correct.

f −1 (x) = 1 + x − 1 ⇒ f −1 (10) = 1 + 3 = 4 which is incorrect.

f −1 (x) = 1 − x − 1 ⇒ f −1 (10) = 1 − 3 = −2 which is correct.

f −1 (x) = 1 − x − 1, 1 ≤ x ≤ 10

31 dkrbabajee@gmail.com
c
CHAPTER 2. FUNCTIONS

The diagram shows the graphs of


y = f (x) and y = f −1 (x). The
graph of y = f −1 (x) is obtained by
reflecting the graph of
y = f (x) in the line y = x. Also the
coordinates (−2, 10) → (10, −2)
under this transformation. (1, 1)
is invariant.
p
(iv) let y = f (x) ⇒ y = 1 − 4x − x2 ⇒ y = 5 − (x + 2)2 ⇒ x = −2 ± 5 − y

f −1 (x) = −2 ± 5 − x
f (x) has a maximum point at (−2, 5)
f (−2) = 5, f (−4) = 1, Range of f = {1 ≤ f (x) ≤ 5}
Domain of f −1 = Range of f = {1 ≤ x ≤ 5}
we have two expressions, we choose the point (1, −4).
to check which expression is correct.

f −1 (x) = −2 + 5 − x ⇒ f −1 (1) = −2 + 2 = 0 which is incorrect.

f −1 (x) = −2 − 5 − x ⇒ f −1 (1) = −2 − 2 = −4 which is correct.

f −1 (x) = −2 − 5 − x, 1 ≤ x ≤ 5

The diagram shows the graphs of


y = f (x) and y = f −1 (x). The
graph of y = f −1 (x) is obtained by
reflecting the graph of
y = f (x) in the line y = x.
Also the coordinates (−4, 1) →
(1, −4) under this transformation.
(−2, 5) is invariant.
p
(v) let y = f (x) ⇒ y = 1 − 4x − x2 ⇒ y = 5 − (x + 2)2 ⇒ x = −2 ± 5 − y

f −1 (x) = −2 ± 5 − x
f (x) has a maximum point at (−2, 5)
f (−2) = 5, f (2) = −11, Range of f = {−11 ≤ f (x) ≤ 5}
Domain of f −1 = Range of f = {−11 ≤ x ≤ 5}
we have two expressions, we choose the point (−11, 2).
to check which expression is correct.

f −1 (x) = −2 + 5 − x ⇒ f −1 (−11) = −2 + 4 = 2 which is correct.

f −1 (x) = −2 − 5 − x ⇒ f −1 (−11) = −2 − 4 = −6 which is incorrect.

f −1 (x) = −2 + 5 − x, −11 ≤ x ≤ 5

32 dkrbabajee@gmail.com
c
CHAPTER 2. FUNCTIONS

The diagram shows the graphs


of y = f (x) and y = f −1 (x). The
graph of y = f −1 (x) is obtained by
reflecting the graph of
y = f (x) in the line y = x. Also the
coordinates (2, −11) → (−11, 2)
under this transformation.
(−2, 5) is invariant.

Example 2.7 The functions f and g are defined by f : x 7→ 2x − 3, x ∈ R


1 2
and g : x 7→ , x ∈ R, x 6= , respectively.
3x − 2 3
(i) Find expressions for f −1 (x) and g −1 (x) and find the values of x for
which f −1 (x) = g −1 (x).

(ii) Sketch on a single diagram the graphs of y = f (x) and y = f −1 (x),


making clear the relationship between these two graphs.

(iii) Sketch on a single diagram the graphs of y = g(x) and y = g −1(x),


making clear the relationship between these two graphs.

Solution
y+3 x+3
(i) let y = f (x) ⇒ 2x − 3 = y ⇒ x = ⇒ f −1 (x) =
2 2
1 1 1
let y = g(x) ⇒ = y ⇒ 3x − 2 = ⇒ 3x = 2 +
3x − 2 y y
2y + 1 2y + 1 2x + 1
3x = ⇒x= ⇒ g −1(x) = , x 6= 0
y 3y 3x
x+3 2x + 1
f −1 (x) = g −1(x) ⇒ = ⇒ 3x(x + 3) = 2(2x + 1)
2 3x
1
3x2 + 9x = 2 + 4x ⇒ 3x2 + 5x − 2 = 0 ⇒ (3x − 1)(x + 2) = 0 ⇒ x = or x = −2.
3

(ii) The diagram shows the


graphs of y = f (x) and y = f −1 (x).
The graph of y = f −1 (x) is
obtained by reflecting the graph
of y = f (x) in the line y = x. Also
the coordinates (0, −3) → (−3, 0)
and (1.5, 0) → (0, 1.5) under this
transformation.

33 dkrbabajee@gmail.com
c
CHAPTER 2. FUNCTIONS

(iii) The diagram shows the


graphs of y = g(x) (red) and
y = g −1(x) (blue). We note that
the asymptotes for y = g(x) are
2
the lines y = 0 and x = . The
3
graph of y = g −1(x) is obtained by
reflecting the graph of y = g(x) in
the line y = x. The asymptotes
for y = g −1 (x) are the lines x = 0
2
and y = .
3
Exercise 2.3. [Inverse functions]

1. Using the horizontal line test, determine which of the following functions are
one-one or not:
(i) f : x 7→ x − 1, 0 ≤ x ≤ 1 (ii) f : x 7→ x2 + 1, x ∈ R

(iii) f : x 7→ x2 + 1, x ≥ 0 (iv) f : x 7→ 4 − x2 , −2 ≤ x ≤ 2
1
(v) f : x 7→ 4 − x2 , −2 ≤ x ≤ 0 (iv) f : x 7→ ,x∈R
x2

2. For the following functions f , find its inverse stating its domain and sketch
on a single diagram the graphs of y = f (x) and y = f −1 (x), making clear the
relationship between these two graphs:
(i) f (x) = x + 1, −2 ≤ x ≤ 2 (ii) f (x) = 5 − 3x, 0 ≤ x ≤ 1

(iii) f (x) = (x − 1)2 + 3, 0 ≤ x ≤ 1 (iv) f (x) = 2x2 + 4x − 1, −1 ≤ x ≤ 0

(v) f (x) = 2 − (x − 2)2 , −2 ≤ x ≤ 2 (vi) f (x) = 1 − 3x − 12x2 , −1 ≤ x ≤ 1

3. The functions f and g are defined by


1 3
f : x 7→ 3x − 4, x ∈ R and g : x 7→ , x ∈ R, x 6= , respectively.
2x − 3 2
(i) Find expressions for f −1 (x) and g −1 (x) and find the values of x for which
f −1 (x) = g −1 (x).
(ii) Sketch on a single diagram the graphs of y = f (x) and y = f −1 (x), making
clear the relationship between these two graphs.
(iii) Sketch on a single diagram the graphs of y = g(x) and y = g −1 (x), making
clear the relationship between these two graphs.

34 dkrbabajee@gmail.com
c
CHAPTER 2. FUNCTIONS

2
4. The function f is given by f (x) = 1 + , x 6= 0. Find
x
(i) an expression for f −1 (x),
 
−1 1
(ii) f ,
2
(iii) the value of x for which f −1 (x) = 2,
(iv) the values of x for which f −1 (x) = x.
3x − 1
5.* The functions f and g are defined by f : x 7→ , x 6= 2 and
x−2
2x − 1
g : x 7→ , x 6= 3, respectively.
x−3
(i) Show that gf (x) = x and hence find an expression for g −1 (x).
(ii) Find the exact values of x for which f (x) = g(x).

2.8 Miscellaneous Exercises


Example 2.8 (a) The function f is defined
  by f : x 7→ 4x + a, x ∈ R,
1
where a is a constant. Given that f 2 = −1, find the value of a.
4
(b) The function g is defined by g : x 7→ x2 − 2x, x ∈ R.
Find the range of values of x for which g(x) < 3.

(c) The function h is defined by h : x 7→ x2 − 2x, x ≤ 1.

(i) Express h(x) in the form (x − p)2 − q.


(ii) Sketch the graph of y = h(x), stating its range. Explain why h
has an inverse.
(iii) Find an expression for h−1 (x) and state its domain.

Solution
   
1 1
(a) f =4 +a=1+a
4 4
    
2 1 1
f =f f = f (1 + a) = 4(1 + a) + a = 4 + 5a
4 4
 
1
f2 = −1 ⇒ 4 + 5a = −1 ⇒ 5a = −5 ⇒ a = −1
4

35 dkrbabajee@gmail.com
c
CHAPTER 2. FUNCTIONS

(b) x2 − 2x < 3
x2 − 2x − 3 < 0
(x + 1)(x − 3) < 0
Critical values of x are -1, 3
(x + 1) (x − 3) (x + 1)(x − 3)
x < −1 −ve −ve +ve (> 0) ×
−1 < x < 3 +ve −ve −ve (< 0) X
x>3 +ve +ve +ve (> 0) ×
∴ −1 < x < 3

(c) (i) x2 − 2x = (x − 1)2 − (−1)2 = (x − 1)2 − 1


(ii) The graph of y = h(x) has a
minimum point at (1, −1).
The range is h(x) ≥ −1. For the
domain x ≤ 1, if horizontal lines
are drawn, there is only one point
of intersection with the curve.
Therefore h is one-one and it has
an inverse.

p √
(iii) let y = h(x) ⇒ (x − 1)2 − 1 = y ⇒ x = 1 ± y + 1 ⇒ h−1 (x) = 1 ± x + 1
we have two expressions, let us choose the point (0, 0)
to check which expression is correct.

h−1 (x) = 1 + x + 1 ⇒ h−1 (0) = 1 + 1 = 2 which is incorrect.

h−1 (x) = 1 − x + 1 ⇒ h−1 (0) = 1 − 1 = 0 which is correct.
since the domain of h−1 = range of h,

h−1 (x) = 1 − x + 1, x ≥ −1.

Miscellaneous Exercise 2.

1. Given that the function f : x 7→= ax + b, x ∈ R such that f 2 (−1) = 9 and


f −1 (−3) = −2, find the values of a and b.
x+3
2. The functions f and g are defined by f : x 7→ , x ∈ R, x 6= 3 and
x−3
g : x 7→ mx + c, x ∈ R, respectively.

(i) Find an expression for f −1 (x).


(ii) Given that f (4) = g −1 (5) and g(f −1(−2)) = −1, find the values of m and c.

36 dkrbabajee@gmail.com
c
CHAPTER 2. FUNCTIONS

3. The functions f and g are defined by f : x 7→ 2x + 3, x ∈ R and


1 2
g : x 7→ , x ∈ R, x 6= − , respectively.
3x + 2 3
(i) Solve the equation f g(x) = 1.
(ii) Find expressions for gf (x), (gf )−1 (x), f −1 (x) and g −1 (x) and show that
(gf )−1(x) = f −1 g −1(x).

4. Given that the functions f : x 7→ 2x + 1 and g : x 7→ x2 + px − 1, find the range of


values of p for which the equation f g(x) = gf (x) has real roots.

5. The functions f and g are defined by f : x 7→ 3x + k, where k is a constant and


12
g : x 7→ , x 6= 3, respectively.
3−x

(i) Find the values of k for which the equation f g(x) = x has two equal roots.
(ii) Determine the roots of the equation f g(x) = x for the values of k found in
part (i).

6. The functions f and g are defined by f : x 7→ 4x + p, where k is a constant and


1
g : x 7→ − , x 6= −3, respectively.
x+3

(i) Find the values of p for which the equation gf (x) = x has two equal roots.
(ii) Determine the roots of the equation gf (x) = x for the values of p found in
part (i).

7. The functions f is defined by f : x 7→ x2 − 6x, for 2 ≤ x ≤ A.

(a) In case f (x) ≤ −8, find the value of A.


(b) (i) Express f (x) in the form (x − p)2 − q.
(ii) State the largest value of A for which f has an inverse.
(iii) When A has this value, find an expression for f −1 (x), stating its domain.

8. (a) The functions f and g are defined by f : x 7→ 2x + a, where a is a constant


and g : x 7→ x2 + 4x, x ≥ −2, respectively.
(i) Given that f 2 (a) = −7, find the value of a.
(ii) Using the value of a obtained in part (i), show that the equation
f (x) = g(x) has only one root and find it.

37 dkrbabajee@gmail.com
c
CHAPTER 2. FUNCTIONS


(b) The function h is defined by h : x 7→ −2 + x + 4, x ≥ −4.
(i) State the range of h(x).
(ii) Find an expression for h−1 (x) and show that h−1 (x) = g(x).
(iii) Sketch on a single diagram the graphs of y = h(x) and y = g(x), making
clear the relationship between these two graphs.

9.* (a) The function f is defined by f : x 7→ ax + b, x ∈ R, where a and b are positive


constants. Given that f 2 (1) = 17 and f −1 (−4) = −2, find the values of a and
of b.
(b) The function g is defined by g : x 7→ x2 + 2x, x ∈ R.
Solve the equation f (x) = g(x) using the values of a and b obtained in part
(a).
(c) The function h is defined by h : x 7→ x2 + 2x, x ≤ −1.
Express h(x) in the form (x+p)2 −q and find an expression for h−1 (x), stating
its domain.

10.* The function f is defined by f : x 7→ 3 + 8x − 2x2 for x ∈ R.

(i) Express f (x) in the form a(x + b)2 + c, where a, b and c are constants.
(ii) State the range of f .
(iii) Sketch the graph of y = f (x) and explain why f does not have an inverse.
The function g is defined by g : x 7→ 3 + 8x − 2x2 for x ≥ B, where B is a
constant.
(iv) State the smallest value of B for which g has an inverse.
(v) When B has this value, obtain an expression, in terms of x, for g −1 (x) and
state the range of g −1.

38 dkrbabajee@gmail.com
c
C HAPTER 3

Coordinate Geometry

3.1 Objective
At the end of this chapter, students should be able to:
1. find the length, gradient and mid-point of a line segment, given the coordinates
of the end-points;
2. find the equation of a straight line given sufficient information (e.g. the coordinates
of two points on it, or one point on it and its gradient);
3. understand collinear points as points lying on the same straight line;
4. understand and use the relationships between the gradients of parallel and
perpendicular lines;
5. interpret and use linear equations, particularly the forms y = mx + c and
y − y1 = m(x − x1 );
6. find the perpendicular bisector of the line joining two points;
7. find the acute angles between two lines.

3.2 Coordinates
Coordinate geometry is the study of the geometric properties of points, straight lines
and curves using algebraic method.
Consider the xy− plane, i.e. the plane containing the coordinates axes Ox and Oy
where O is the origin.
The coordinates of a general point P is written as P(x, y).

In vector form, the


 position
 vector of P
−→ x −→
written as OP = or OP = xi + yj
y
where i and j are the unit vectors along
the x and y axes respectively.

39 dkrbabajee@gmail.com
c
CHAPTER 3. COORDINATE GEOMETRY

3.3 Distance between two points (Distance’s formula)


Using Pythagoras Theorem

AB 2 = AC 2 + CB 2
= (x2 − x1 )2 + (y2 − y1 )2

pof the line segment AB is given


The length
by AB = (x2 − x1 )2 + (y2 − y1 )2 .
Example 3.1 Find the length of the straight line joining the points
A(1, 3) and B(4, 7).
Solution
p √ √
Length of AB = (4 − 1)2 + (7 − 3)2 = 9 + 16 = 25 = 5

Example 3.2 The points P, Q and R have coordinates (1, 11), (5, 7) and
(9, a), respectively.
If P Q and QR are of equal length, find the possible values of a.
Solution

P Q = QR
p p
(5 − 1)2 + (7 − 11)2 = (9 − 5)2 + (a − 7)2
√ p
32 = 16 + (a − 7)2
32 = 16 + (a − 7)2
(a − 7)2 = 16

a = 7 ± 16 = 11 or 3.

3.4 Midpoint of a line joining two points


Given A(x1 , y1 ) and B(x2 , y2 ), the coordinates of the line joining A and B are given by
 
x1 + x2 y1 + y2
M= ,
2 2

Example 3.3 Find the coordinates of the midpoint of the line joining
A(1, 8) and B(7, 4).
Solution
 
1+7 8+4
M= , = (4, 6)
2 2

40 dkrbabajee@gmail.com
c
CHAPTER 3. COORDINATE GEOMETRY

Example 3.4 L(−1, −2) is the midpoint of the line joining points P(a, −5)
and Q(3, b). Find the values of a and b.

Solution

Midpoint of PQ = L
 
a + 3 −5 + b
, = (−1, −2)
2 2
a+3
= −1 ⇒ a + 3 = −2 ⇒ a = −5
2
−5 + b
= −2 ⇒ −5 + b = −4 ⇒ b = 1
2

Example 3.5 P(−1, 5), Q(8, 10), R(7, 5) and S are the vertices of the
parallelogram PQRS. Calculate the coordinates of S.

Solution

Let S(a,b)
Midpoint of PQ = Midpoint of RS
   
−1 + 8 5 + 10 7+a 5+b
, = ,
2 2 2 2
7 7+a
= ⇒a+7=7⇒a=0
2 2
5 + 10 5+b
= ⇒ 5 + b = 15 ⇒ b = 10
2 2
∴ S(0, 10)
Exercise 3.1. [Distance and midpoint formula]

1. Find the length of the straight lines joining the given points, giving your answer
in exact form:
(i) A(1, 1), B(3, 1) (ii) A(−4, −2), B(−4, −4) (iii) A(1, 1), B(5, 4)
(iv) A(1, −2), B(8, 10) (v) A(2, 1), B(4, 5) (vi) A(−3, 2), B(3, 4)
(vii) A(2, −3), B(−1, 1) (viii) A(−5, −6), B(4, −2) (ix) A(6, 0), B(0, −8)

2. The points P, Q and R have coordinates (2, −3), (5, 1) and (1, a), respectively.
If P Q and QR are of equal length, find the possible values of a.

3. The points A, B and C have coordinates (0, 5), (b, 7) and (−4, 9), respectively.
If AB = BC, find the value of b.

4. The vertices of a triangle are A(-1,2), B(-4,-2) and C(2,-1). Find the perimeter of
the triangle correct to 1 decimal place.

41 dkrbabajee@gmail.com
c
CHAPTER 3. COORDINATE GEOMETRY

5. Find the midpoint of the line joining the given points:


(i) A(1, 1), B(3, 1) (ii) A(−4, −2), B(−4, −4) (iii) A(1, 1), B(5, 4)
(iv) A(1, −2), B(8, 10) (v) A(2, 1), B(4, 5) (vi) A(−3, 2), B(3, 4)
   
9 1 5 8
(vii) A(2, −3), B(−1, 1) (viii) A , − , B − , (ix) A(−6, 0), B(0, −8)
2 3 2 3
6. L(4, −1) is the midpoint of the line joining points P(5, a) and Q(b, 1). Find the
values of a and b.

7. P(−3, 2), Q(−2, 4), R(3, 4) and S are the vertices of the parallelogram PQRS.
Calculate the coordinates of S.

8. A, B and C are the points (0, −2), (−1, 2) and (3, 3), respectively.

(i) Show that △ ABC is isosceles.


(ii) Find the midpoint of BC.

3.5 Gradient of a line joining two points


The gradient of the line joining A(x1 , y1) and B(x2 , y2 ) is given by
y2 − y1
m=
x2 − x1

Example 3.6 Find the gradient of the line joining A(2, 7) and B(5, 11).
Another line PQ with P(a, −3) and Q(−1, 5) have same gradient as the
line AB. Calculate the value of a.

Solution
11 − 7 4
mAB = =
5−2 3
5 − −3 4
mP Q = =
−1 − a 3
8 4
= ⇒ 24 = 4(−1 − a)
−1 − a 3
6 = −1 − a ⇒ a = −1 − 6 = −7

42 dkrbabajee@gmail.com
c
CHAPTER 3. COORDINATE GEOMETRY

3.6 Collinear points

Collinear points are points which lie on


the same straight line.
If A, B, C and D are collinear points then
gradient of AB = gradient of BC = gradient
of CD.

Example 3.7 Three points A(0, −5), B(x, −9) and C(3, 1) are collinear.
Find the value of x.

Solution

gradient of AB = gradient of BC
−9 − −5 1 − −9
=
x−0 3−x
−4 10
= ⇒ −4(3 − x) = 10x
x 3−x
− 12 + 4x = 10x ⇒ 6x = −12 ⇒ x = −2

3.7 Parallel Lines

Parallel lines are lines which does not


intersect.
If two lines are parallel then their
gradients are equal.

Example 3.8 Using the points A(2, 4), B(8, 7), C(5, −2) and D(19, 5),
show that AB is parallel to CD.

Solution
7−4 1
Gradient of AB = =
8−2 2
5 − −2 1
Gradient of CD = =
19 − 5 2
Gradient of AB = Gradient of CD

43 dkrbabajee@gmail.com
c
CHAPTER 3. COORDINATE GEOMETRY

3.8 Perpendicular Lines


Two lines are perpendicular if they
intersect at an angle of 900 .
Consider two straight lines L1 and L2
which intersect at right angles at the
point A and cut the x-axis at the points
B and C respectively.
b = θ then D AC
If ABD b = θ. AD is the
perpendicular from A to x-axis.
AD
Gradient of line L1 = = tan θ
BD
AD 1 1
Gradient of line L2 = − =− =−
CD CD tan θ
AD
1
Gradient of L1 × Gradient of L2 = tan θ × − = −1.
tan θ

If two lines are perpendicular, the product of their gradients is -1.

Example 3.9 Using the points A(−3, 8), B(1, 4), C(2, 3) and D(5, 8), show
that AC is perpendicular to BD.

Solution
8−3
Gradient of AC = = −1
−3 − 2
8−4
Gradient of BD = =1
5−1
Gradient of AB × Gradient of CD = −1 × 1 = −1

3.9 Equation of a line


Consider a general point P(x, y) which lies on the straight line which cuts the y-axis
at the point R(0, c) and the gradient of the line is m.
y−c
= m =⇒ y − c = mx =⇒ y = mx + c
x−0
is called the cartesian equation of the straight line where m is the gradient and c is
called the y-intercept.

44 dkrbabajee@gmail.com
c
CHAPTER 3. COORDINATE GEOMETRY

Example 3.10 Write down the gradient and y-intercept of each of the
following lines.
(i) y = 2x + 7 (ii) y = 4 − 3x (iii) 2y = 5x + 1 (iv) 4x − 3y + 2 = 0
Solution
(i) m = 2, c = 7
(ii) m = −3, c = 4
5 1 5 1
(iii)y = x+ ⇒m= , c=
2 2 2 2
h i 4 2 4 2
(iv) − 3y = −4x − 2 ÷ −3 ⇒ y = x + ⇒ m = , c =
3 3 3 3
We note that the vertical line x = c1 has gradient undefined and the horizontal line
y = k1 has gradient 0. The lines x = c1 and x = c2 are parallel. Similarly the lines
y = k1 and y = k2 are parallel but the lines x = c1 and y = k1 are perpendicular.

To find the equation of the straight line


passing through the points A(x1 , y1 ) and
B(x2 , y2 ), we let P(x, y) be any point on the
line.
Since A, B and P are collinear,
gradient of PB = gradient of AB,
y − y2 y2 − y1
= is the required equation
x − x2 x2 − x1
of line.
Otherwise we find the gradient m of the
line AB and the equation of the line at
y − y1
A(x1 , y1 ) is given by =m.
x − x1

Example 3.11 (a) Find the equation of the line that passes through
4
(3, 7) and has gradient .
5
(b) Find the equation of the line passing through the points A(−4, 7)
and B(3, −5).
Solution

(a) Equation of line at (3, 7)


y−7 4
=
x−3 5
5(y − 7) = 4(x − 3) ⇒ 5y − 35 = 4x − 12 ⇒ 5y = 4x + 23.

45 dkrbabajee@gmail.com
c
CHAPTER 3. COORDINATE GEOMETRY

−5 − 7 −12
(b) mAB = =
3 − −4 7
Equation of line AB at A(−4, 7)
y−7 −12
=
x − −4 7
7(y − 7) = −12(x + 4) ⇒ 7y − 49 = −12x − 48 ⇒ 12x + 7y = 1.

Example 3.12 (a) Find the equation of the line that passes through
(3, −2) and parallel to the line 3x + 2y = 6.

(b) Find the equation of the line passing through the point A(4, 4) and
perpendicular to the line 4x + 5y = 11.

Solution

(a) Any line parallel to the line 3x + 2y = 6 has equation 3x + 2y = c. Since A(3, −2)
lies on the line, we have 3(3)+2(−2) = c ⇒ c = 5. So the equation of the parallel
line is 3x + 2y = 5.
4
(b) The line 4x + 5y = 11 has gradient − and the perpendicular line has
5
5
gradient . Equation of the perpendicular line at A(4, 4) is
4
y−4 5
= ⇒ 4(y − 4) = 5(x − 4) ⇒ 4y − 16 = 5x − 20 ⇒ 4y = 5x − 4
x−4 4

Exercise 3.2. [Equation of parallel and perpendicular lines]

1. Find the gradient of the line joining the given points:


(i) A(1, 1), B(3, 1) (ii) A(−4, −2), B(−4, −4) (iii) A(1, 1), B(5, 4)
(iv) A(1, −2), B(8, 10) (v) A(2, 1), B(4, 5) (vi) A(−3, 2), B(3, 4)
(vii) A(2, −3), B(−1, 1) (viii) A(−5, −6), B(4, −2) (ix) A(6, 0), B(0, −8)

2. Determine whether the given points are collinear:


(i) A(2, 1), B(5, 3), C(8, 5) (ii) A(−4, 0), B(−1, 2), C(4, 5)
(iii) A(−3, −3), B(−1, 0), C(1, 3) (iv) A(−3, 3), B(1, 1), C(7, −2)

3. (i) Find the gradient of the line joining A(3, 5) and B(−4, 2). Another line PQ
with P(−8, −3) and Q(6, a) have same gradient as the line AB. Calculate
the value of a.
(ii) Three points A(1, 3), B(x, −1) and C(8, −4) are collinear. Find the value of
x.

46 dkrbabajee@gmail.com
c
CHAPTER 3. COORDINATE GEOMETRY

4. Determine the lines AB and CD are parallel or perpendicular:


(i) A(1, 1), B(4, 3), C(5, −2), D(8, 0)
(ii) A(−2, −1), B(−1, 3), C(4, −2), D(−4, 0)
(iii) A(1, 2), B(−2, −2), C(1, −1), D(−3, 2)
(iv) A(1, −2), B(−1, 3), C(−3, −2), D(−5, 3)

5. Write down the gradient and y-intercept of each of the following lines.
(i) y = x + 7 (ii) y = −3x + 4 (iii) 6y = 3x − 5 (iv) 4x + 3y = 9

6. Find the equation of the line that passes through the point A and has gradient m:
3 7
(i) A(1, −2), m = 2 (ii) A(3, 1), m = −3 (iii) A(4, 3), m = (iv) A(−2, 5), m = −
4 3
7. Find the equation of the line joining the given points:
(i) A(1, 1), B(3, 1) (ii) A(−4, −2), B(−4, −4) (iii) A(1, 1), B(5, 4)
(iv) A(1, −2), B(8, 10) (v) A(2, 1), B(4, 5) (vi) A(−3, 2), B(3, 4)
(vii) A(2, −3), B(−1, 1) (viii) A(−5, −6), B(4, −2) (ix) A(6, 0), B(0, −8)

8. Determine whether the given lines are parallel or perpendicular:


(i) x + y = 1, y = x + 6 (ii) 3x + 4y = 5, 8x − 6y = 7
(iii) y = 2x + 4, 2y = 4x + 5 (iv) x + 3y + 1 = 0, x + 3y = 17

9. Find the equation of the line passing through the point A and parallel to the line
L:
(i) A(2, 3), L : y = 2x − 7 (ii) A(4, −3), L : 4y = 3x + 5
(iii) A(6, 6), L : 5x + 4y = 5 (iv) A(1, 0), L : 2y = −6x + 5

10. Find the equation of the line passing through the point A and perpendicular to
the line L:
(i) A(3, 4), L : y = x (ii) A(2, −1), L : 2y = x + 1
(iii) A(−5, 3), L : 2x + 4y = 3 (iv) A(1, 2), L : 3x + 8y = 10

3.10 Perpendicular bisector


3.10.1 Finding equation of perpendicular bisector of the line
AB
A perpendicular bisector is a line which intersect another line at right angles and
divide the line into 2 equal parts.

47 dkrbabajee@gmail.com
c
CHAPTER 3. COORDINATE GEOMETRY

The perpendicular bisector passes through the midpoint of two given points.
1. Find the gradient of AB, mAB .

2. Find the gradient of the


1
perpendicular bisector e.g. −
mAB
3. Find the midpoint of AB, say M

4. Find the equation of the


perpendicular bisector using its
gradient and the midpoint M.

Example 3.13 Find the equation of the perpendicular bisector of the line
joining the points A(4, −5) and B(2, 9).

Solution
 
4 + 2 −5 + 9
Midpoint of AB, M = , = (3, 2)
2 2
−5 − 9 −14
mAB = = = −7
4−2 2
1 1
mPerpendicular = − =
mAB 7
Equation of perpendicular bisector at M(3,2)
y−2 1
= ⇒ 7y − 14 = x − 3
x−3 7
7y = x + 11

Example 3.14 The line y = x + 5 cuts the curve y = x2 − 3x at the two


points A and B.

(a) Find the coordinates of A and B.

(b) Find the equation of the perpendicular bisector of the line joining
the two points A and B.

(c) Given that the perpendicular bisector of AB meets the y-axis at the
point (0, h), find the value of h.
Solution

(a) y =x+5 ...... Linear (3.1)


y = x2 − 3x ...... Nonlinear (3.2)

48 dkrbabajee@gmail.com
c
CHAPTER 3. COORDINATE GEOMETRY

replace (3.1) in (3.2) we get


x + 5 = x2 − 3x
x2 − 4x − 5 = 0
(x − 5)(x + 1) = 0 ⇒ x = 5 or x = −1
From (3.1), when x = 5, y = 5 + 5 = 10
From (3.1), when x = −1, y = −1 + 5 = 4
A(−1, 4) and B(5, 10) are the coordinates .

 
−1 + 5 4 + 10
(b) Midpoint of AB, M = , = (2, 7)
2 2
10 − 4 6
mAB = = =1
5 − −1 6
1
mPerpendicular = − = −1
mAB
Equation of perpendicular bisector at M(2,7)
y−7
= −1 ⇒ y − 7 = −x + 2
x−2
y = −x + 9

(c) At (0, h), y = −x + 9 ⇒ h = 0 + 9 ⇒ h = 9.

Example 3.15 Given that the perpendicular bisector of the line joining
A(-1,2) and B is y = −3x + 9, find the coordinates of B.

Solution

Step 1: We obtain the gradient of the perpendicular bisector from its equation and find
the gradient of AB.
1 1
y = −3x + 9 ⇒ m = −3, mAB = − =
m 3

Step 2: We find the equation of AB at A and solve the two equations to obtain the
midpoint of AB.

49 dkrbabajee@gmail.com
c
CHAPTER 3. COORDINATE GEOMETRY

Equation of AB at A(−1, 2)
y−2 1
=
x − −1 3
3y − 6 = x + 1 ⇒ 3y = x + 7 (3.3)
y = −3x + 9 (3.4)
replace (3.4) in (3.3) we get
3(−3x + 9) = x + 7 ⇒ −9x + 27 = x + 7 ⇒ 10x = 20 ⇒ x = 2
From (3.4), when x = 2, y = −3(2) + 9 = 3
Midpoint of AB = (2, 3)

Step 3: We use the formula of the midpoint of AB to recover B.

Let B(a, b)
Midpoint of AB = (2, −1)
 
a−1 b+2
, = (2, 3)
2 2
a−1
=2⇒ a−1=4⇒a=5
2
b+2
=3⇒b+2=6⇒b=4
2
Coordinates of B = (5, 4)

Example 3.16 The point A (3, 5) is reflected in the line y = x + 1 to get a


point B. Find the coordinates of B.

Solution The line y = x + 1 is the perpendicular bisector of AB. The two lines
intersect at the midpoint of AB.

Step 1: We find the equation of AB:

y =x+1⇒m=1
1
mAB = − = −1
m
Equation of AB at A(3,5)
y−5
= −1 ⇒ y − 5 = −x + 3 ⇒ y = −x + 8
x−3

Step 2: We find the midpoint of AB which is the intersection of the line AB and the line
y = x + 1:

50 dkrbabajee@gmail.com
c
CHAPTER 3. COORDINATE GEOMETRY

y = x + 1 .........(1), y = −x + 8 .......(2)
x + 1 = −x + 8 ⇒ 2x = 7 ⇒ x = 3.5 ⇒ y = 3.5 + 1 = 4.5
midpoint of AB = (3.5, 4.5)

Step 3: Let B(p, q).

Midpoint of AB = (3.5, 4.5)


 
3+p 5+q
, = (3.5, 4.5)
2 2
3+p
= 3.5 ⇒ 3 + p = 7 ⇒ p = 4
2
5+q
= 4.5 ⇒ q = 4
2
Coordinates of B = (4, 4)

3.11 Angle between two straight lines


Consider the two lines L1 : y = m1 x + c1
and L2 : y = m2 x + c2 .
Let θ1 and θ2 be the angles which these
lines make with the x-axis.
The angle between these two lines is given
by θ = θ2 − θ1 .
By Trigonometry Identities,

tan θ2 − tan θ1 m2 − m1
tan θ = tan (θ2 − θ1 ) = =
1 + tan θ2 tan θ1 1 + m2 m1

m1 − m2
So the acute angle between the two lines is given by θ = tan −1
1 + m1 m2 .

Example 3.17 Find the acute angle between the lines y = 2x + 4 and
y = x − 5.

Solution
m1 = 2, m2 = 1

m 1 − m2
θ = tan−1
1 + m1 m2
 
−1
2−1 1
θ = tan = tan−1
= 18.40
1 + (2 × 1) 3

51 dkrbabajee@gmail.com
c
CHAPTER 3. COORDINATE GEOMETRY

Exercise 3.3. [Perpendicular bisector and acute angle between two lines]
1. Find the perpendicular bisector of the line joining the two given points:
(i) A(1, 2) B(4, −2) (ii) A(7, −1) B(−3, 5) (iii) A(−3, −5) B(−2, −4)
(iv) A(2, 0) B(0, 3) (v) A(7, −2) B(7, −4) (vi) A(−1, 3) B(4, 3)

2. Given that the perpendicular bisector of the line joining A(4,3) and B is
5y = x − 2, find the coordinates of B.
3. The point A (2, 2) is reflected in the line x + y = 5 to get a point B. Find the
coordinates of B.
4. Show that the origin lies on the the perpendicular bisector of the line joining
A(2,4) and B(4,2).
5. The line y = x − 2 cuts the curve y = 2x2 − 2x − 1 at the two points A and B.
(i) Find the coordinates of A and B.
(ii) Find the equation of the perpendicular bisector of the line joining the two
points A and B.
(iii) Given that the perpendicular bisector of AB meets the x-axis at the point
(h, 0), find the value of h.
6. Find the acute angle between the two given lines:
(i) y = x + 1, 2x − y = 1 (ii) 3y = 2x + 5, 5x + 2y = 11 (iii) y = 3x + 1, 9y = 6x + 4

3.12 Miscellaneous Exercises

Example 3.18
The diagram shows a rhombus ABCD. The points B and D have
coordinates (2, 10) and (6, 2) respectively and A lies on the x-axis. The
midpoint of BD is M. Find by calculation the coordinates of M, A and C.

52 dkrbabajee@gmail.com
c
CHAPTER 3. COORDINATE GEOMETRY

Solution
Step 1: The midpoint of BD is obtained using the formula
 
2 + 6 10 + 2
M= , = (4, 6) .
2 2

Step 2: Let A(a, 0). We form a equation in a using the fact that

length of AB = length of AD
AB 2 = AD 2
(a − 2)2 + 102 = (a − 6)2 + 22
a2 − 4a + 4 + 100 = a2 − 12a + 36 + 4
8a = −64 ⇒ a = −8 ⇒ A(−8, 0)

Step 3: M is also the midpoint of AC. Let C(p, q).

Midpoint of AC = (4, 6)
 
−8 + p 0 + q
, = (4, 6)
2 2
−8 + p
= 4 ⇒ −8 + p = 8 ⇒ p = 16
2
0+q
= 6 ⇒ q = 12
2
Coordinates of C = (16, 12)

Example 3.19
The diagram shows a trapezium PQRS in which QR is parallel to PS
and QR is perpendicular to RS. The coordinates of P, Q and R are (2, 0),
(4, 6) and (10, 9) respectively.

(a) Find the equations of PS and RS.

(b) Find the coordinates of S.

53 dkrbabajee@gmail.com
c
CHAPTER 3. COORDINATE GEOMETRY

Solution
9−6 3 1
(a) mQR = = =
10 − 4 6 2
1
QR is parallel to PS ⇒ mP S = mQR =
2
Equation of PS at P(2,0)
y−0 1
=
x−2 2
2y = x − 2
1
QR is perpendicular to RS ⇒ mRS = − = −2
mQR
Equation of RS at R(10,9)
y−9
= −2
x − 10
y − 9 = −2x + 20 ⇒ y = −2x + 29

(b) P S : 2y = x − 2 (3.5)
RS : y = −2x + 29 (3.6)
replace (3.6) in (3.5) we get
2(−2x + 29) = x − 2 ⇒ −4x + 58 = x − 2 ⇒ 5x = 60 ⇒ x = 12
From (3.6), when x = 12, y = −2(12) + 29 = 5
Coordinates of S = (12, 5)

Miscellaneous Exercise 3.
1. The points A and B have coordinates (1, 1) and (3, 2). The line L1 passes through
B and is parallel to OA. The line L2 passes through A and is perpendicular to
AB.
(i) Find the equations of line L1 and L2 .
The lines L1 and L2 meet at C.
(ii) Find the coordinates of C.
1
2. The diagram shows the curve y = intersecting the line y = 2x + 3 at the
x+1
points X and Y.
(i) Find the coordinates of X and Y.

(ii) The point P lies between X and Y


such that 2XP = 3P Y . Find the
coordinates of P.

54 dkrbabajee@gmail.com
c
CHAPTER 3. COORDINATE GEOMETRY

3.* The coordinates of P, Q and R are (3, 5), (−3, 2) and (6, 1) respectively.

(i) Find the equation of the perpendicular bisector of the line joining the two
points P and Q.
(ii) Find the equation of the perpendicular bisector of the line joining the two
points Q and R.
(iii) Hence, find the point which is equidistant from P, Q and R and find this
distance.

4. The line x + y = 3 cuts the curve y = x2 + x at the two points A and B.

(i) Find the coordinates of A and B.


(ii) Find the equation of the perpendicular bisector of the line joining the two
points A and B.
(iii) Given that the perpendicular bisector of AB meets the x-axis at the point P
and the y-axis at the point Q, find the distance PQ.

5. The points A and B have coordinates (−1, −2) and (4, 1), respectively. The
perpendicular bisector of AB meets the line y = x + 2 at C.

(i) Find the perpendicular bisector of AB and the coordinates of C.


(ii) Show that △ ABC is isosceles.

6. The diagram shows a right-angled triangle ABC with ABC b = 900 .


The coordinates of A and B are (−2, 1) and (1, 5), respectively. The equation of
AC is 7y = x + 9.

(i) Find the equation of BC and the


coordinates of C.

(ii) Find the perimeter of △ ABC correct


to 1 decimal place.

7. The diagram shows a rectangle ABCD. The coordinates of A and B are (−2, 3)
and (3, 5) respectively. The equation of line CD is 5y = 2x − 18.

(i) Find the equations of BD and AC.

(ii) Find the coordinates of D and C.

55 dkrbabajee@gmail.com
c
CHAPTER 3. COORDINATE GEOMETRY

8.* The diagram shows a rhombus ABCD. The equation of line AB is y = x + 8 and
the point C has coordinates (16, 12) and A lies on the x-axis. The midpoint of AC
is M.

Find by calculation the coordinates of A,


M, B and D.

9. The diagram shows a trapezium PQRS in which QR is parallel to PS and QR is


perpendicular to RS. The coordinates of P, Q and S are (2, −4), (4, 2) and (12, 1)
respectively.

(i) Find the equations of QR and RS.

(ii) Find the coordinates of R.

10.* A kite has vertices A(−4, 6), B, C(2, 4) and D. The midpoint of AC is M and the
perpendicular bisector of AC passes through B and D. The line CD meets the
x-axis at E(−6, 0) and AB is parallel to EM.
(i) Find the coordinates of M and the
equation of BD.

(ii) Find the equations of AB and CD.

(iii) Find the coordinates of B and D.

56 dkrbabajee@gmail.com
c
C HAPTER 4

Circular Measure

4.1 Objective
At the end of this chapter, students should be able to:

1. understand the definition of a radian, and use the relationship between radians
and degrees;
1
2. use the formulae s = rθ and A = r 2 θ in solving problems concerning the arc
2
length and sector area of a circle.

4.2 Definition of a radian


Angles can be measured in either degrees (0 ) or radians (c ).
The radian is a unit of angle, equal to an angle θ at the center of a circle whose arc
is equal in length to the radius r.
r
In short θ = = 1. θ = 2 is the angle at the center of a circle whose arc is twice in
r
length to the radius.
s
In general, θ = , where s is the length of arc.
r
Since the magnitude in radians of one complete revolution (3600 ) is the length of the
2πr
entire circumference divided by the radius, or , or 2π. Thus 2π radians is equal to
r
3600 so that the relation between degrees and radians is given by π radians = 1800 .

4.3 Conversion between degrees and radians


Let θc be the angle in radians and α0 be the angle converted in degrees. Then
α0
θc = ×π .
1800

Example 4.1 Convert into radians:


(i) 900 (ii) 450 (iii) 1200

57 dkrbabajee@gmail.com
c
CHAPTER 4. CIRCULAR MEASURE

Solution
π π π π π 2π
(i) 900 = 0
× 900 = (ii) 450 = 0
× 450 = (iii) 1200 = 0
× 1200 =
180 2 180 4 180 3
0
We note that 1 revolution or 1 complete cycle = 360 = 2π radians.
Exercise 4.1. [Conversion between degrees and radians]
1. Convert into radians, giving your answers as a fraction of π:
(i) 300 (ii) 600 (iii) 700 (iv) 1100 (v) 1350 (vi) 1500

2. Convert into radians, giving your answers correct to 3 decimal places:


(i) 300 (ii) 600 (iii) 700 (iv) 1100 (v) 1350 (vi) 1500

3. Convert into degrees:


π 2π π π 7π 2π
(i) (ii) (iii) (iv) (v) (vi)
4 3 2 9 12 5

4.4 Sector

Consider a sector with radius r and


subtended at an angle θ.

4.4.1 Length of Arc


s
From θ = , the length of minor arc AB is given by
r

Length of Minor Arc, s = rθ

4.4.2 Area of Sector


If θ is in degrees, then the area of sector OAB is given by
θ
Area of sector = 0
× πr 2 .
360
58 dkrbabajee@gmail.com
c
CHAPTER 4. CIRCULAR MEASURE

If θ is in radians, then the area of sector OAB is given by


θ
Area of sector, A = × πr 2 , since 3600 = 2θ.

1
A = r2θ
2
1
A = r(rθ)
2
1
A = rs
2

4.4.3 Perimeter of a sector


Perimeter of sector, P = r + r + Length of Arc
P = 2r + rθ

4.4.4 Area of segment


Area of segment, A = Area of Sector − Area of Triangle
1 1
A = r 2 θ − (r)(r) sin θ
2 2
1 2
A = r (θ − sin θ)
2

4.4.5 Perimeter of segment


Perimeter of segment, = Chord AB + Arc AB
p
= 2r 2 (1 − cos θ) + rθ,
where the chord AB is obtained using cosine rule: AB 2 = r 2 + r 2 − 2(r)(r) cos θ.

59 dkrbabajee@gmail.com
c
CHAPTER 4. CIRCULAR MEASURE

Example 4.2 In a sector OAB with centre O, the radius is 5 cm and the
b is 1200 .
angle AOB
Find

(i) the length of arc AB,

(ii) the perimeter of the sector


AOB,

(iii) the area of the sector AOB,

(iv) the area of the shaded segment,

(v) the perimeter of the shaded


segment.

Solution

(i)
1200 2π
r = 5, θ = 0
×π =
180 3

length of arc s = rθ = 5 × = 10.5 cm
3

(ii)

perimeter of the sector = 2r + s = 2(5) + 10.5 = 20.5 cm

(iii)
 
1 1 2π
area of the sector AOB = r 2 θ = (5)2 = 26.2 cm2
2 2 3

(iv)
  
1 2π 2π
area of the shaded segment = (5)2 − sin = 15.4 cm2
2 3 3

(v)
s   

perimeter of the shaded segment = 2(5)2 1 − cos + 10.5 = 19.2 cm
3

60 dkrbabajee@gmail.com
c
CHAPTER 4. CIRCULAR MEASURE

Exercise 4.2. [Sector and segment]

1. Consider a sector AOB with centre O, radius r and angle at centre, θ.


For the following values of r and θ,
(a) r = 10 cm, θ = 1350 , (b) r = 8 cm, θ = 2 rad, find

(i) the length of arc AB,


(ii) the perimeter of the sector AOB,
(iii) the area of the sector AOB,
(iv) the area of the segment,
(v) the perimeter of the segment.

2. In a sector AOB with centre O, the radius is r cm, the angle at centre is θ in
radians, the length of arc AB is s cm and the area of sector is A cm2 . Find

(i) the value of s and of A when r = 10 and θ = 0.8.


(ii) the value of r and of A when θ = 1.2 and s = 6.
(iii) the value of θ and of A when r = 15 and s = 10.
4
(iv) the value of r and of s when θ = and A = 24.
3
(v) the value of θ and of s when r = 8 and A = 40.
(vi) the value of r and of θ when s = 40 and A = 500.

3. A wire of length 15 cm is bent to form a sector of radius 4 cm and angle at centre


θ. Find (i) θ (ii) area of sector.
b = 700 , the length of the chord AB is 12
4. In a sector AOB with centre O and AOB
cm. Find

(i) the radius OA,


(ii) perimeter of the segment.

5. In a circle centre O and radius 10 cm, AB is a chord of length 16 cm. Find


b correct to 3 decimal places,
(i) AOB
(ii) area of △ AOB,
(iii) area of sector AOB.

61 dkrbabajee@gmail.com
c
CHAPTER 4. CIRCULAR MEASURE

4.5 Problems involving length of arc and area of


sector.
Example 4.3 The diagram shows a sector OQP centre O with radius r
b = θ in radians.
in a right angled triangle OQR with QRO

(i) Show that the area of the


shaded
 region is given by
r2 2
+ 2θ − π
4 tan θ
(ii) In case θ = 0.77 and r = 10
cm, find the perimeter of the
shaded region.

Solution

(i)
1 π  π 1
area of sector OPQ = r 2 − θ = r2 − r2θ
2 2 4 2
r r
from △ OQR, tan θ = ⇒ QR =
QR tan θ
1 r2
area of △ OQR = × r × QR =
2 2 tan θ
area of shaded region = area of △ OQR − area of sector OPQ
 
r2 π 2 1 2 r2 2
area of shaded region = − r + r θ= + 2θ − π
2 tan θ 4 2 4 tan θ

(ii)
10 π 
QR = = 10.31 cm, arc PQ = 10 × − 0.77 = 8.01 cm
tan 0.77
√ 2
OR = 102 + 10.312 = 14.36 cm, P R = 14.36 − 10 = 4.36 cm
perimeter of shaded region= QR + arc PQ + PR = 10.31 + 8.01 + 4.36 = 22.7 cm

62 dkrbabajee@gmail.com
c
CHAPTER 4. CIRCULAR MEASURE

Example 4.4 The diagram shows two sectors OAB and OCD centre O
b = 2 radians.
and AOB

Given that OA = 5 cm and area of


shaded region is 39 cm2 , find the

(i) length of AC,

(ii) perimeter of shaded region.

Solution
(i)

let x = AC
1
area of sector OCD = (5 + x)2 (2) = x2 + 10x + 25
2
1
area of sector OAB = (5)2 (2) = 25
2
area of shaded region = area of sector OCD − area of sector OAB
39 = x2 + 10x + 25 − 25 ⇒ x2 + 10x − 39 = 0
(x − 3)(x + 13) = 0 ⇒ x = 3 or − 13 ⇒ x = 3 (x > 0).

(ii)

perimeter of shaded region= arc AB + arc CD + 2AC = (5)(2) + (5 + 3)(2) + 2(3) = 32 cm

Example 4.5 The diagram shows a semicircle PQR centre O with radius
b = θ radians.
r with P OQ

(i) In case θ = 1.5, calculate


perimeter of sector QOR
.
perimeter of sector QOP
(ii) Given that the area of sector
QOR is twice the area of sector
π
QOP, show that θ = . For
3
this value of θ, show that the
perimeter
√ of △ QPR is
(3 + 3)r.

63 dkrbabajee@gmail.com
c
CHAPTER 4. CIRCULAR MEASURE

Solution
(i)

perimeter of sector QOP = 2r + 1.5r = 3.5r


perimeter of sector QOR = 2r + (π − 1.5)r = (π + 0.5)r
perimeter of sector QOR (π + 0.5)r π + 0.5
= = = 1.04
perimeter of sector QOP 3.5r 3.5

(ii)
1
area of sector QOR = r 2 (π − θ)
2
1 2
area of sector QOP = r θ
2
area of sector QOR = 2 × area of sector QOP
1 2 1
r (π − θ) = 2 × r 2 θ
2 2
π
π − θ = 2θ ⇒ 3θ = π ⇒ θ =
3
r   π  r  
2 2
π  √
P R = 2r, QP = 2r 1 − cos = r, QR = 2r 1 − cos π − = 3r
3 3
√ √
perimeter of △ PQR = P R + QP + QR = 2r + r + 3r = (3 + 3)r

Exercise 4.3. [Problems involving length of arc and area of sector]


1. The diagram shows a chord of length 15 cm which divides a circle of radius
10 cm.

Find the area of each segment.

2. The diagram shows a sector OQP centre O with radius 15 cm in a right angled
b = 0.8 radians.
triangle OQR with QOR

Find

(i) the area of the shaded region,

(ii) the perimeter of the shaded region.

64 dkrbabajee@gmail.com
c
CHAPTER 4. CIRCULAR MEASURE

b = 1 radian.
3. The diagram shows two sectors OAB and OCD centre O and AOB
Given that AC = 2 cm and area of shaded
region is 18 cm2 , find the

(i) length of OA,

(ii) perimeter of shaded region.

4. The diagram shows a semicircle PQR centre O with radius 5 cm with


b = 2 radians.
P OQ

Calculate
area of shaded region PQXP
(i) ,
area of shaded region QYRQ
(ii) the perimeter of △ QPR.

5. The diagram shows a rectangle ABCD with BC = 12 cm and CD = 16 cm. The


midpoint of AB is O and DXC is an arc of a circle centre O.
b = 1.176 correct to 3
(i) Show that D OC
decimal places.

(ii) Find the radius OC correct to 3


decimal places.

(iii) Find the perimeter of the shaded


segment.

(iv) Find the area of the shaded segment.

65 dkrbabajee@gmail.com
c
CHAPTER 4. CIRCULAR MEASURE

4.6 Miscellaneous Examples


Example 4.6 The diagram shows a circle C1 touching a circle C2 at a
point E. Circle C1 has centre A and radius 10 cm and C2 has centre B
and radius 15 cm. Points C and D lie on C1 and C2 , respectively and CD
b = π radians.
is parallel to AB. C AE
4

Find

(i) the length of CD,

(ii) area of shaded region,

(iii) perimeter of shaded region.

Solution
(i)
π    
b = sin −1 DG −1 7.071
DG = CF = 10 sin = 7.071 cm, D BG = sin = 0.491 rad
4 DB 15
√ √
GB = DB 2 − DG2 = 152 − 7.0712 = 12.87 cm, AB = AE + EB = 10 + 15 = 25 cm
CD = F G = AB − (AF + GB) = 25 − (7.071 + 12.87) = 5.06 cm

(ii)
1  
2 π
area of sector CAE = (10) = 39.30
2 4
1
area of sector BDE = (15)2 (0.491) = 55.24
2
1 1
area of trapezium ACDB = (AB + CD) × CF = (25 + 5.06)(7.071) = 106.28
2 2
area of shaded region = area of trapezium − ( area of sector CAE + area of sector BDE)
area of shaded region = 106.28 − (55.24 + 39.30) = 11.7 cm2 .

66 dkrbabajee@gmail.com
c
CHAPTER 4. CIRCULAR MEASURE

(iii)

perimeter of shaded region = CD + arc CE + arc DE


 π
perimeter of shaded region = 5.06 + 10 × + (15 × 0.491) = 20.3 cm
4

Miscellaneous Exercise 4.
1. The diagram shows a major segment AXB of a circle centre O, radius 10 cm and
b = 2400 .
reflex AOB

Find

(i) area of major segment AXB,

(iii) perimeter of major segment AXB.

b = 300 . Triangle
2. The diagram shows a sector OAB centre O, radius 6 cm and AOB
b = 900 .
OXA is right-angled with O XA

Find

(i) area of shaded region,

(iii) perimeter of shaded region.

3. The diagram shows a sector AOB of a circle, centre 0 and radius 5 cm.
b = 1000 and AX and BX are tangents to the circle.
AOB

Find

(i) area of shaded region,

(iii) perimeter of shaded region.

4. The diagram shows a circle centre O and radius r cm. The region R1 is a minor
b = 1.4 rad. Region R2 is the major sector.
sector with angle at centre, P OQ
Find
Perimeter of R2
(i) ,
Perimeter of R1
Area of R2
(iii) .
Area of R1

67 dkrbabajee@gmail.com
c
CHAPTER 4. CIRCULAR MEASURE

5. The diagram shows a cross-section which consists of a rectangle ABCD of length


20 cm and width 10 cm and an arc BXC of a circle with centre O.

Find the area of the cross-section.

b = 600 .
6. A sector POQ has centre O, radius r cm and P OQ
length of arc PQ π
(i) Show that = .
length of chord PQ 3
area of sector POQ
(ii) Find the exact value of .
area of △ POQ
7.* The diagram shows a circle C1 touching a circle C2 at a point E. Circle C1 has
centre A and radius 8 cm and C2 has centre B and radius 12 cm. Points C and
b = 0.6 radians.
D lie on C1 and C2 , respectively and CD is parallel to AB. D BE
Find

(i) the length of CD,

(ii) area of shaded region,

(iii) perimeter of shaded region.

b = θ radians
8. The diagram shows two sectors OAB and OCD, centre O and AOB
and OA = 4 cm.
Given that the length of arc CD is 1.5 times
the length of arc AB, find

(i) length of AC,


area of shaded region
(ii) .
area of sector COD

9.* The diagram shows two circles C1 and C2 , touching at E. C1 has centre A and
radius 4.5 cm and C2 has centre B and radius 12.5 cm. A tangent touches the
circles C1 and C2 at the points C and D, respectively.

68 dkrbabajee@gmail.com
c
CHAPTER 4. CIRCULAR MEASURE

Find

(i) the length of CD,

(ii) area of shaded region,

(iii) perimeter of shaded region.

10. The diagram shows an isosceles triangle with OC = OD =10 cm and


angle COD =300 . OAB is a sector centre O and radius 8 cm.

Find

(i) area of shaded region,

(ii) perimeter of shaded region.

69 dkrbabajee@gmail.com
c
C HAPTER 5

Trigonometry 1

5.1 Objective
At the end of this chapter, students should be able to:

1. sketch and use graphs of the sine, cosine and tangent functions (for angles of
any size, and using either degrees or radians);

2. find the period, amplitude, minimum and maximum values of general trigonometric
functions;

3. use the exact values of the sine, cosine and tangent of 300 , 450 , 600 , and related
angles;

4. locate in which quadrant an angle lies and find the sign of trigonometric ratio
of the angle in this quadrant;

5. use the notations sin−1 x, cos−1 x, tan−1 x to denote the principal values of the
inverse trigonometric relations;
sin θ
6. use the identities = tan θ and sin2 θ + cos2 θ = 1;
cos θ
7. find all the solutions of simple trigonometrical equations lying in a specified
interval.

70 dkrbabajee@gmail.com
c
CHAPTER 5. TRIGONOMETRY 1

5.2 Graphs of Trigonometric functions


5.2.1 Simple form Trigonometric functions
The diagram shows the graph of y = sin x
in the interval −3600 ≤ x ≤ 3600. The
curve has a maximum of value 1 at
x = −2700 , 900 and a minimum of value
−1 at x = −900 , 2700. The curve cuts the
x− axis at
x = −3600 , −1800 , 00 , 1800 , 3600 . The
amplitude is 1 and the period (one
complete cycle) is 3600 . The range is
−1 ≤ y ≤ 1.
In case the angles are given in radians,
we replace 1800 by π, etc.
The diagram shows the graph of y = cos x
in the interval −3600 ≤ x ≤ 3600 . The
curve has a maximum of value 1 at
x = −3600 , 00 , 3600 and a minimum of
value −1 at x = −1800 , 1800 . The curve
cuts the x− axis at
x = −2700 , −900 , 900 , 2700 . The
amplitude is 1 and the period (one
complete cycle) is 3600 . The range is
−1 ≤ y ≤ 1

The diagram shows the graph of y = tan x


in the interval −3600 ≤ x ≤ 3600. The
curve has no maximum and no minimum
values and no amplitude. The curve cuts
the x− axis at
x = −3600 , −1800 , 00 , 1800 , 3600 . The
asymptotic lines are
x = −2700 , −900 , 900 , 2700 .
The period is 1800.

71 dkrbabajee@gmail.com
c
CHAPTER 5. TRIGONOMETRY 1

5.2.2 General form of Trigonometric functions


The diagram shows the graph of
y = a sin (bx) + c, a > 0 . The curve has a
maximum of value a + c at
2700 900
x=− , and a minimum of value
b b
900 2700
−a + c at x = − , .
b b
The curve cuts the line y = c (mid-value)
3600 1800 0 1800 3600
at x = − , − , 0, , . The
b b b b
amplitude is a and the period (one
3600
complete cycle) is . The range is
b
−a + c ≤ y ≤ a + c.

The diagram shows the graph of


y = a cos (bx) + c, a > 0.
The curve has a maximum of value a + c
0 0
at x = − 360b , 00 , 360
b
and a minimum of
1800 1800
value −a + c at x = − , . The
b b
curve cuts the line y = c (mid-value) at
2700 900 900 2700
x=− , − , , . The
b b b b
amplitude is a and the period (one
3600
complete cycle) is . The range is
b
−a + c ≤ y ≤ a + c.

The diagram shows the graph of


y = a tan (bx), a > 0.
The curve has no maximum and no
minimum values and no amplitude. The
curve cuts the x− axis at
3600 1800 0 1800 3600
x=− , − , 0, , . The
b b b b
asymptotic lines are
2700 900 900 2700
x=− , − , , .
b b b b
1800
The period is .
b

72 dkrbabajee@gmail.com
c
CHAPTER 5. TRIGONOMETRY 1

Example 5.1 Sketch the graphs of the following functions and find their
range, period and number of cycles:

(i) y = f (x) = sin (2x) + 1 for 00 ≤ x ≤ 3600 ,

(ii) y = f (x) = 3 cos (3x) − 1 for −π ≤ x ≤ π,

(iii) y = f (x) = 2 tan (2x) for 0 ≤ x ≤ 2π.

Solution
The diagram shows the graph of
f (x) = sin (2x) + 1. The maximum value of
f is 1 + 1 = 2 and its minimum value is
−1 + 1 = 0. The mid-value is 1. Range:
(i) 3600
0 ≤ f (x) ≤ 2. Period = = 1800.
2
360 − 0
There are = 2 cycles.
180

The diagram shows the graph of


f (x) = 3 cos (3x) − 1. The maximum value
of f is 3 − 1 = 2 and its minimum value is
−3 − 1 = −4. The mid-value is −1. Range:
(ii) 2π
−4 ≤ f (x) ≤ 2. Period = .
3
π − −π
There are 2π = 3 cycles.
3

The diagram shows the graph of


f (x) = 2 tan (2x). The curve cuts the
π 3π
x− axis at x = 0, , π, , 2π.
2 2
The asymptotic lines are
(iii) π 3π 5π 7π
x= , , , . Range: f (x) ∈ R.
4 4 4 4
π 2π − 0
Period = . There are π = 4 cycles.
2 2

Example 5.2 (i) Sketch and label on the same diagram the graphs of
y = sin (3x) and y = 3 cos x for 0 ≤ x ≤ π.

(ii) Hence find the number of solutions of the equation sin (3x) = 3 cos x
in the interval 0 ≤ x ≤ π.

73 dkrbabajee@gmail.com
c
CHAPTER 5. TRIGONOMETRY 1

Solution

(i)

(ii) From the diagram, the two curves intersect at only one point A in the interval
0 ≤ x ≤ π and there is one solution to the equation sin (3x) = 3 cos x in that
interval.

Exercise 5.1. [Graphs of Trigonometric functions]

1. Sketch the graphs of the following functions and find their range, period T and
number of cycles N:

(i) y = f (x) = 3 sin (2x) − 2 for 00 ≤ x ≤ 3600,


(ii) y = f (x) = 2 sin (2x) + 3 for 0 ≤ x ≤ π,
(iii) y = f (x) = 5 cos (2x) − 1 for 00 ≤ x ≤ 3600,
(iv) y = f (x) = 4 cos x + 2 for 0 ≤ x ≤ 2π,
(v) y = f (x) = tan (3x) for 00 ≤ x ≤ 1800 ,
(vi) y = f (x) = 5 tan (2x) for −π ≤ x ≤ π.

2. The function defined by f (x) = a sin (bx) + c, 0 ≤ x ≤ 3600 has a maximum value
of 7, a minimum value of −1 and a period of 1800. Find the values of a, b and c
and sketch the graph of y = f (x).

3. The function defined by f (x) = a cos (bx) + c, −π ≤ x ≤ π has a maximum value


of −1, a minimum value of −7 and a period of 2π. Find the values of a, b and c
and sketch the graph of y = f (x).
π
4. The function defined by f (x) = a tan (bx), 0 ≤ x ≤ π has a period of and
π 3
f = 4. Find the values of a and b and sketch the graph of y = f (x).
12

74 dkrbabajee@gmail.com
c
CHAPTER 5. TRIGONOMETRY 1

5. (i) Sketch and label on the same diagram the graphs of y = 3 sin x and
y = cos (3x) for 0 ≤ x ≤ π.
(ii) Hence find the number of solutions of the equation cos (3x) = 3 sin x in the
interval 0 ≤ x ≤ π.

5.3 Trigonometric ratios


5.3.1 Trigonometric ratios of special angles
θ sin θ cos θ tan θ

degrees radians

00 0 0 1 0

π 1 3 1
300 √
6 2 2 3
π 1 1
450 √ √ 1
4 2 2

π 3 1 √
600 3
3 2 2
π
900 1 0 ∞
2
1800 π 0 −1 0


2700 −1 0 −∞
2
3600 2π 0 1 0

75 dkrbabajee@gmail.com
c
CHAPTER 5. TRIGONOMETRY 1

5.3.2 Signs of Trigonometric Ratios and Quadrants

The diagram shows the graph of y = sin x and its signs in the different quadrants.
We find that sin x is +ve in the 1st quadrant (00 ≤ x ≤ 900 and −3600 ≤ x ≤ −2700 ) and
in the 2nd quadrant (900 ≤ x ≤ 1800 and −2700 ≤ x ≤ −1800 ). Also sin (−x) = − sin x

The diagram shows the graph of y = cos x and its signs in the different quadrants.
We find that cos x is +ve in the 1st quadrant (00 ≤ x ≤ 900 and −3600 ≤ x ≤ −2700 )
and in the 4th quadrant (2700 ≤ x ≤ 3600 and −900 ≤ x ≤ 00 ). Also cos (−x) = cos x

76 dkrbabajee@gmail.com
c
CHAPTER 5. TRIGONOMETRY 1

The diagram shows the graph of y = tan x and its signs in the different quadrants.
We find that tan x is +ve in the 1st quadrant (00 ≤ x ≤ 900 and −3600 ≤ x ≤ −2700 ) and
in the 3rd quadrant (1800 ≤ x ≤ 2700 and −1800 ≤ x ≤ −900 ). Also tan (−x) = − tan x
We denote the angle in the first quadrant as the basic angle α. We can represent the
angles in the other quadrants in terms of α.

Quadrant θ Sign

degrees radians sin θ cos θ tan θ

1st α, −(3600 − α) α, −(2π − α) + + +

2nd 1800 − α, −(1800 + α) π − α, −(π + α) + − −

3rd 1800 + α, −(1800 − α) π + α, −(π − α) − − +

4th 3600 − α, −α 2π − α, −α − + −

5.3.3 Principal values of Inverse of Trigonometric Ratios


y≥0 y<0
−1 π π
0 ≤ sin y ≤ − ≤ sin−1 y < 0
2 2
π π
0 ≤ cos−1 y ≤ < cos−1 y ≤ π
2 2
π π
0 ≤ tan−1 y < − < tan−1 y < 0
2 2

5.4 Solving Trigonometrical Equations


Example 5.3 Solve the following trigonometrical equations for
00 ≤ x ≤ 3600:
(i) sin x = 0.5 (ii) cos x = −0.5 (iii) tan x = 1
1 1
(iv) sin (3x + 150 ) = √ (v) cos (2x − 450 ) = − √
2 2

(i)

α = sin−1 (0.5) = 300


sin is +ve in 1st and 2nd quadrants
x = α, 1800 − α = 300 , 1500

77 dkrbabajee@gmail.com
c
CHAPTER 5. TRIGONOMETRY 1

(ii)

α = cos−1 (0.5) = 600


cos is -ve in 2nd and 3rd quadrants
x = 1800 − α, 1800 + α = 1200, 2400

(iii)

α = tan−1 (1) = 450


tan is +ve in 1st and 3rd quadrants
x = α, 1800 + α = 450 , 2250

(iv)

00 ≤ x ≤ 3600 ⇒ 150 ≤ 3x + 150 ≤ 10950


 
1
α = sin −1
√ = 450
2
sin is +ve in 1st and 2nd quadrants
3x + 150 = α, 1800 − α, .....
3x + 150 = 450 , 1350, 4050, 4950 , 7950 , 8550
3x = 300 , 1200 , 3900, 4800 , 7500 , 8400
x = 100 , 400 , 1300 , 1600 , 2500 , 2800

(v)

00 ≤ x ≤ 3600 ⇒ −450 ≤ 2x − 450 ≤ 6750


 
1
α = cos −1
√ = 450
2
cos is -ve in 2nd and 3rd quadrants
x = −α, 1800 − α, 1800 + α, .....
2x − 450 = −450 , 1350, 2250 , 4950 , 5850
2x = 00 , 1800, 2700 , 5400 , 6300
x = 00 , 900 , 1350 , 2700 , 3150

78 dkrbabajee@gmail.com
c
CHAPTER 5. TRIGONOMETRY 1

Exercise 5.2. [Trigonometric functions and equations]

1. Find the exact values of the following:


(i) sin 300 (ii) cos (π/4) (iii) tan 600

(iv) sin 1200 (v) cos (2π/3) (vi) tan 1350

(vii) sin (3π/4) (viii) cos 1500 (ix) tan (5π/6)

(x) sin 2250 (xi) cos (5π/4) (xii) tan 2400

(xiii) sin (4π/3) (xiv) cos 3000 (xv) tan (π/3)

(xvi) sin 3300 (xvii) cos (11π/6) (xviii) tan 3300

b = 450 , QPbR = 600 and PR=10 cm. Find the exact length
2. In a triangle PQR, P QR
of QR.
b = 450 , QPbR = 300 and QR=8 cm. Find the exact length
3. In a triangle PQR, P QR
of PR.
b = 300 , P RQ
4. In a triangle PQR, P QR b = 150 and QR=20 cm. Find the exact
length of PR.

5. In a triangle PQR, QPbR = θ0 , PQ=10 cm and√ PR=20 cm.


Given that the area of triangle PQR is 50 3, find the possible values of θ and
the corresponding exact values of the length of QR.

6. Solve the following trigonometrical equations for 00 ≤ x ≤ 3600 :


(i) 2 sin x = −1 (ii) cos x = 0.3 (iii) 3 tan x + 7 = 0

7. Solve the following trigonometrical equations for 0 ≤ x ≤ 2π, giving your


answers in terms of π where necessary: √
(i) 4 sin x = 3 (ii) 2 cos x + 1 = 0 (iii) 3 tan x = 1

8. Solve
√ the following0 trigonometrical equations for√00 ≤ x ≤ 3600 :
(i) 2 sin (2x + 15 ) = −1 (ii) 2 cos (3x − 600 ) = 3 (iii) tan (2x + 750 ) = −1

9. Solve the following trigonometrical equations for 0 ≤ x ≤ π, giving your answers


in terms of π: √ √
(i) 2 sin (2x + π/4) = 3 (ii) cos (2x − π/4) = −1 (iii) 3 tan (2x + π/3) = 3

79 dkrbabajee@gmail.com
c
CHAPTER 5. TRIGONOMETRY 1

5.5 Trigonometric Identities


Consider the triangle:

y x y
sin θ = , cos θ = , tan θ =
r r x
y
sin θ y
= xr = = tan θ
cos θ r
x
sin θ
tan θ =
cos θ
y 2 x2 y 2 + x2 r2
sin2 θ + cos2 θ = + = = = 1, since r 2 = y 2 + x2 (Pythagoras theorem).
r2 r2 r2 r2
sin2 θ + cos2 θ = 1
Some useful trigonometric identities are:

sin2 θ = 1 − cos2 θ
cos2 θ = 1 − sin2 θ

80 dkrbabajee@gmail.com
c
CHAPTER 5. TRIGONOMETRY 1

5.5.1 Prove Identities


Example 5.4 Prove the following identities:

(i) 2 cos2 x + 3 sin x − 3 ≡ −2 sin2 x + 3 sin x − 1,

(ii) 2 sin2 x − 5 cos x − 4 ≡ −2 cos2 x − 5 cos x − 2,


tan2 x + 2
(iii) ≡ 1 + cos2 x,
tan2 x + 1
tan x
(iv) ≡ sin x cos x,
tan2 x + 1
1 + cos x sin x 2
(v) + ≡ .
sin x 1 + cos x sin x

Solution

(i)

2 cos2 x + 3 sin x − 3 ≡ 2(1 − sin2 x) + 3 sin x − 3


≡ 2 − 2 sin2 x + 3 sin x − 3 ≡ −2 sin2 x + 3 sin x − 1

(ii)

2 sin2 x − 5 cos x − 4 ≡ 2(1 − cos2 x) − 5 cos x − 4


≡ 2 − 2 cos2 x − 5 cos x − 4 ≡ −2 cos2 x − 5 cos x − 2

(iii)
sin2 x
tan2 x + 2 cos2 x
+2

tan2 x + 1 sin2 x
+1
cos2 x
sin2 x+2 cos2 x
cos2 x
≡ sin2 x+cos2 x
cos2 x
2
sin x + 2 cos2 x cos2 x
≡ 2
× , since sin2 x + cos2 x = 1
cos x 1
≡ sin2 x + 2 cos2 x ≡ sin2 x + cos2 x + cos2 x ≡ 1 + cos2 x

81 dkrbabajee@gmail.com
c
CHAPTER 5. TRIGONOMETRY 1

(iv)
sin x
tan x cos x

tan2 x + 1 sin2 x
+1
cos2 x
sin x
cos x
≡ sin2 x+cos2 x
cos2 x
sin x cos2 x
≡ × ≡ sin x cos x
cos x 1

(v)

1 + cos x sin x (1 + cos x)2 + sin2 x


+ ≡
sin x 1 + cos x sin x(1 + cos x)
1 + 2 cos x + cos2 x + sin2 x 2 + 2 cos x
≡ ≡
sin x(1 + cos x) sin x(1 + cos x)
2(1 + cos x) 2
≡ ≡
sin x(1 + cos x) sin x

5.5.2 Use of Identities in Solving Trigonometrical Equations


Example 5.5 Solve the following trigonometrical equations for
00 ≤ x ≤ 3600:

(i) 2 cos2 x + 3 sin x − 3 = 0,

(ii) 2 sin2 x = 5 cos x + 4,

(iii) 2(sin x − cos x) = 3(sin x + cos x),

(iv) sin2 x = sin x cos x + 2 cos2 x.

Solution
(i)

2 cos2 x + 3 sin x − 3 = 0 ⇒ 2(1 − sin2 x) + 3 sin x − 3 = 0


2 sin2 x − 3 sin x + 1 = 0 ⇒ (2 sin x − 1)(sin x − 1) = 0
1
sin x = sin x = 1
2 
1
α = sin−1 = 300 x = 900
2
sin is +ve in 1st and 2nd quadrants
x = α, 1800 − α = 300 , 1500

82 dkrbabajee@gmail.com
c
CHAPTER 5. TRIGONOMETRY 1

(ii)

2 sin2 x = 5 cos x + 4 ⇒ 2(1 − cos2 x) = 5 cos x + 4


2 cos2 x + 5 cos x + 2 = 0 ⇒ (2 cos x + 1)(cos x + 2) = 0
1
cos x = − cos x = −2
2 

1
α = cos−1 = 600 no solution since − 1 ≤ cos x ≤ 1
2
cos is -ve in 2nd and 3rd quadrants
x = 1800 − α, 1800 + α = 1200 , 2400

(iii)

2 sin x − 2 cos x = 3 sin x + 3 cos x ⇒ sin x = −5 cos x


sin x
= −5 ⇒ tan x = −5
cos x
α = tan−1 (5) = 78.70
tan is -ve in 2nd and 4th quadrants
x = 1800 − α, 3600 − α = 101.30 , 281.30

(iv)

sin2 x − sin x cos x − 2 cos2 x = 0


(sin x − 2 cos x)(sin x + cos x) = 0
sin x = 2 cos x sin x = − cos x
tan x = 2 tan x = −1
α = tan−1 (2) = 63.40 α = tan (1) = 450
−1

tan is +ve in 1st and 3rd quadrants tan is -ve in 2nd and 4th quadrants
x = α, 1800 + α = 63.40 , 243.40 x = 1800 − α, 3600 − α = 1350 , 3150

Example 5.6 The function f : x 7→ 5 sin2 x + 2 cos2 x is defined for the


domain 0 ≤ x ≤ π.

(i) Express f (x) in the form a + b sin2 x, stating the values of a and b.

(ii) Hence find the values of x for which f (x) = 5 sin x.

(iii) Sketch the graph of f , stating its range.

83 dkrbabajee@gmail.com
c
CHAPTER 5. TRIGONOMETRY 1

Solution

(i)

f (x) = 5 sin2 x + 2 cos2 x = 5 sin2 x + 2(1 − sin2 x) = 2 + 3 sin2 x (a = 2, b = 3)

(ii)

f (x) = 5 sin x ⇒ 2 + 3 sin2 x = 5 sin x


3 sin2 x − 5 sin x + 2 = 0 ⇒ (3 sin x − 2)(sin x − 1) = 0
2
sin x = sin x = 1
3 
2 π
α = sin−1 = 0.730 x=
3 2
st nd
sin is +ve in 1 and 2 quadrants
x = α, π − α = 0.730, 2.41

(iii)
0 ≤ sin2 x ≤ 1 ⇒ 0 ≤ 3 sin2 x ≤ 3 ⇒ 2 ≤ 3 sin2 x + 2 ≤ 5 ⇒ 2 ≤ f (x) ≤ 5

Exercise 5.3. [Identities and their use in solving trigonometric equations]

1. Prove the following identities:

(i) 2 cos2 x − 5 sin x − 4 ≡ −2 sin2 x − 5 sin x − 2,


(ii) 9 sin2 x + 3 cos x − 7 ≡ −9 cos2 x + 3 cos x + 2,
2 tan2 x + 1
(iii) ≡ 2 − cos2 x,
tan2 x + 1
3 tan x 3
(iv) 2 ≡ sin x cos x,
2 tan x + 2 2

84 dkrbabajee@gmail.com
c
CHAPTER 5. TRIGONOMETRY 1

1 − sin x cos x 2
(v) + ≡ .
cos x 1 − sin x cos x
2. Solve the following trigonometrical equations for 00 ≤ x ≤ 3600 :

(i) 2 cos2 x = 5 sin x + 4,


(ii) 9 sin2 x + 3 cos x − 7 = 0,
(iii) 3(sin x − cos x) = 2(sin x + cos x),
(iii) 5(sin x + cos x) = 2(sin x − cos x),
(v) 2 sin2 x = cos x(sin x + cos x),
(vi) 6 tan x sin x = 5 sin x − cos x.

3. The function f : x 7→ 5 sin2 x + 2 cos2 x is defined for the domain 0 ≤ x ≤ π.

(i) Express f (x) in the form a + b cos2 x, stating the values of a and b.
(ii) Hence find the values of x for which f (x) = 2 cos x.
(iii) Sketch the graph of f , stating its range.

4. The function f : x 7→ 5 sin2 x − cos2 x is defined for the domain 0 ≤ x ≤ π.

(i) Express f (x) in the form a + b sin2 x, stating the values of a and b.
(ii) Hence find the values of x for which f (x) = 0.
(iii) Sketch the graph of f , stating its range.
1 1 2
5.* (i) Show that + = .
1 + sin θ 1 − sin θ cos2 θ
1 1 2
(ii) Show that + = .
1 + cos θ 1 − cos θ sin2 θ

(iii) Hence solve the equation


1 1 1 1
+ = + , for −π ≤ θ ≤ π.
1 + sin θ 1 − sin θ 1 + cos θ 1 − cos θ

5.6 Miscellaneous Exercises


tan x(1 − sin x) sin x
Example 5.7 (i) Show that = .
cos x 1 + sin x

tan x(1 − sin x) 1


(ii) Hence solve the equation = for 0 ≤ x ≤ 2π.
cos x 4

85 dkrbabajee@gmail.com
c
CHAPTER 5. TRIGONOMETRY 1

Solution

(i)
sin x
tan x(1 − sin x) cos x
(1
− sin x) sin x(1 − sin x)
= =
cos x cos x cos2 x
sin x(1 − sin x) sin x(1 − sin x) sin x
= 2 = = .
1 − sin x (1 − sin x)(1 + sin x) 1 + sin x

(ii)
sin x 1 1
= ⇒ 4 sin x = 1 + sin x ⇒ 3 sin x = 1 ⇒ sin x =
1 + sin x  4 3
1
α = sin−1 = 0.34
3
sin is +ve in 1st and 2nd quadrants
x = α, π − α = 0.34, 2.81

Miscellaneous Exercise 5.

1. The function f : x 7→ πa sin 2x + b is defined for the domain 0 ≤ x ≤ π. It is given


that f (0) = 1 and f = 4.
4
(i) Find the values of a and b.
(ii) Solve the equation f (x) = 0.
(iii) Sketch the graph of y = f (x).

2. The functions f : x 7→ 2 sin x − 1 and g : x 7→ cos x − 1 are defined for the domain
0 ≤ x ≤ π.

(i) On the same diagram, sketch the graphs of y = f (x) and y = g(x).
(ii) Show that the the equation f (x) = g(x) has only one solution and find it.
(iii) Hence, find the range of values of x for which f (x) < g(x).
b = 900 , ACB
3. In a triangle ABC, ABC b = θ0 and AC=10 cm.
The perimeter and area of triangle ABC are denoted by P cm and A cm2 .

(i) Show that A = 50 sin θ cos θ


(ii) Express P in terms of θ.
(iii) Hence, show that (P − 10)2 = 100 + 4A.

86 dkrbabajee@gmail.com
c
CHAPTER 5. TRIGONOMETRY 1

4. Prove the following identities:


(i) cos2 x(1 + tan2 x) = 1,
(ii) (2 sin x + cos x)2 + (sin x − 2 cos x)2 = 5,
 2
1 + sin x 1 + sin x
(iii) ≡ ,
cos x 1 − sin x
sin2 x
(iv) 1 − ≡ cos x,
1 + cos x
1 − 2 cos2 x
(v) ≡ sin x − cos x.
sin x + cos x
5. Solve the following trigonometrical equations for 00 ≤ x ≤ 3600 :
(i) 6 cos2 x + 7 sin x − 8 = 0,
(ii) 6 sin2 x + 5 cos x − 7 = 0,
(iii) tan2 x − 5 tan x + 6 = 0,
(iv) 4(3 cos x + sin x) = 3(cos x − 4 sin x),
(v) 2 sin x tan x = 3,
(vi) 3 tan2 x cos x = 8.
6. (i) Show that the equation tan x = cos x can be written as sin2 x + sin x − 1 = 0.
(ii) Hence solve the equation tan x = cos x, for 0 ≤ x ≤ 2π.
5 + 3 tan2 x
7. (i) Show that ≡ 3 + 2 cos2 x.
1 + tan2 x
5 + 3 tan2 x
(ii) Hence solve the equation = 7 cos x, for 00 ≤ x ≤ 3600.
1 + tan2 x
cos x 1
8. (i) Prove the identity ≡ − 1.
tan x(1 + sin x) sin x
cos x
(ii) Hence solve the equation = 2, for 00 ≤ x ≤ 3600 .
tan x(1 + sin x)
1 1 2 sin θ
9.* (i) Show that − = .
1 − sin θ 1 + sin θ cos2 θ
1 1 2 cos θ
(ii) Show that − = .
1 − cos θ 1 + cos θ sin2 θ

(iii) Hence solve the equation


8 8 1 1
− = − , for 00 ≤ θ ≤ 3600 .
1 − sin θ 1 + sin θ 1 − cos θ 1 + cos θ

87 dkrbabajee@gmail.com
c
CHAPTER 5. TRIGONOMETRY 1

10. The function f is such that f (x) = 3 sink x + 2 cosk x, for 0 ≤ x ≤ π, where k is a
constant.

(i) In the case where k = 1, solve the equation f (x) = 0


(ii) In the case where k = 2,
(a) Express f (x) in the form p + q sin2 x, stating the values of p and q.
(b) Sketch the graph of f , stating its range.

88 dkrbabajee@gmail.com
c
C HAPTER 6

Vectors 1

6.1 Objective
At the end of this chapter, students should be able to:
 
  x
x  −→
1. use standard notations for vectors, i.e. , xi + yj, y , xi + yj + zk, AB,
y
z
a;

2. carry out addition and subtraction of vectors and multiplication of a vector by


a scalar, and interpret these operations in geometrical terms;

3. calculate the magnitude of a vector and the scalar product of two vectors;

4. use the scalar product to determine the angle between two directions and to
solve problems concerning perpendicularity of vectors;

5. use unit vectors, displacement vectors and position vectors.

6.2 Standard Notation of vectors


 
x
A vector a in 2-D, where a = can also be written as xi + yj.
y
 
x
In 3-D a vector b =  y  can also be written as xi + yj + zk.
z
The vectors i, j and k are unit vectors along the x-axis, y-axis and z-axis respectively.

89 dkrbabajee@gmail.com
c
CHAPTER 6. VECTORS 1

6.3 Vector Operations


6.3.1 Addition and Subtraction of vectors
     
3 2 3
Example 6.1 Given that a =  4 , b =  −1  and c =  −5 ,
5 7 −2
evaluate

(i) a + b (ii) c − b (iii) b + a − c

Solution
(i)        
3 2 3+2 5
a + b =  4  +  −1  =  4 + −1  =  3  = 5i + 3j + 12k
5 7 5+7 12

(ii)        
3 2 3−2 1
c − b =  −5  −  −1  =  −5 − −1  =  −4  = i − 4j − 9k
−2 7 −2 − 7 −9
         
2 3 3 2+3−3 2
(iii) b + a − c =  −1  +  4  −  −5  =  −1 + 4 − −5  =  8  = 2i + 8j + 14k
7 5 −2 7 + 5 − −2 14

6.3.2 Scalar Multiplication


 
2
Given that x =  −1  and a constant k, the scalar multiplication
  3 
2 2k
kx = k  −1  =  −k .
3 3k

Example 6.2 Using the same vectors as in Example 6.1, evaluate


(i) 2a + 3b (ii) 5b + 3c − 4a

Solution
(i)
       
3 2 2(3) + 3(2) 12
2a + 3b = 2  4  + 3  −1  =  2(4) + 3(−1)  =  5  = 12i + 5j − 11k
5 7 2(5) + 3(−7) −11

90 dkrbabajee@gmail.com
c
CHAPTER 6. VECTORS 1

(ii)

5b + 3c − 4a
       
2 3 3 5(2) + 3(3) − 4(3)
= 5  −1  + 3  −5  − 4  4  =  5(−1) + 3(−5) − 4(4) 
7 −2 5 5(7) + 3(−2) − 4(5)
 
7
=  −36  = 7i − 36j + 9k
9

6.4 Parallel Vectors


Two vectors a and b are said to be parallel if a can be expressed as a multiple of b,
i.e. a = kb, where k is a scalar.
   
1 2
Example 6.3 Given that a =  2  and b =  4 , show that a and b
3 6
are parallel.

Solution    
2 1
Since b =  4  
= 2 2  = 2a, a and b are parallel.
6 3
   
2 −6
Example 6.4 Given that a =  p  and b =  9  are parallel,
−5 q
find the values of the constants p and q.

Solution
Method 1
a = kb where k is a constant
       
2 −6 2 −6k
 p  = k  9  ⇒  p  =  9k 
−5 q −5 qk
−6k = 2 p = 9k qk = −5
1 1 1
k=− p = 9 × − = −3 q = −5 ÷ − = 15
3 3 3

91 dkrbabajee@gmail.com
c
CHAPTER 6. VECTORS 1

Method 2: Using same ratio

2 : p : −5 = −6 : 9 : q
h i
2 : p : −5 × −3 = −6 : 9 : q
−6 : 3p : 15 = −6 : 9 : q
3p = 9 ⇒ p = 3, q = 15

Exercise 6.1. [Vector operations and parallel vectors]

1. Given that p = i − 2j + 3k, q = 4i − j + 5k and r = 2i − 3j + 4k, evaluate

(i) p + q (ii) p − r (iii) p − q − r (iv) q − p + r


(v) 3p + 2q (vi) 3p − 4r (vii) 3p + 5q − 3r (viii) 6q − 2p + 7r

2. Given that a = −i + j + 4k, b = i − j − 4k, c = 2i + 2j + 8k and d = 2i − 2j − 8k.

(i) Determine with a reason which of these vectors are parallel.


(ii) If c and e = pi − 3j + qk are parallel, find the values of p and q.

6.4.1 Geometrical Interpretation


Addition Law of Vectors

c = a + b is
called the
Triangle Law of
vectors.

92 dkrbabajee@gmail.com
c
CHAPTER 6. VECTORS 1

d = a + b + c.

Scalar Multiplication and Negative Vectors

a and b are parallel (scalar


multiplication).
c = −d is the vector d with equal
length but in opposite direction.

6.5 Dot product of vectors


   
x1 x2
The dot product of of two vectors a =  y1  and b =  y2  is given by
z1 z2
   
x1 x2
a · b =  y1  ·  y2  = x1 x2 + y1 y2 + z1 z2
z1 z2
   
1 2
Example 6.5 Given that a =  −2  and b =  −1 , evaluate the dot
3 4
product a · b.

93 dkrbabajee@gmail.com
c
CHAPTER 6. VECTORS 1

Solution   
1 2
a · b =  −2  ·  −1  = (1 × 2) + (−2 × −1) + (3 × 4) = 2 + 2 + 12 = 16
3 4

6.6 Perpendicular Vectors


Two vectors a and b are said to be perpendicular if a · b = 0.
   
2 1
Example 6.6 Given that a =  1  and b =  −6 , show that a and
4 1
b are perpendicular.

Solution    
2 1
a · b =  1  ·  −6  = (2 × 1) + (1 × −6) + (4 × 1) = 2 − 6 + 4 = 0
4 1

Since a · b = 0, a and b are perpendicular.


Exercise 6.2. [Dot-product and perpendicular vectors]

1. Evaluate
       
2 1 2 1
(i)  1  ·  2  (ii)  −1  ·  4 
−1 0 5 2
       
−1 −2 2 1
(iii)  −1  ·  1  (iv)  −1  ·  2 
5 3 0 4

2. Evaluate
(i) (2i − j + 3k) · (−i + j − k) (ii) (3i + 2j − 2k) · (i − 2j + 3k)

(iii) (−i + 4j − k) · (2i + 3j − 2k) (iv) (5i + 6j − 4k) · (−3i + 2j + k)

3. If a = 2i − j + 2k and b = pi + 2j − 3k, find the value of p if


(i) a · b = 5 (ii) a · b = −2 (iii) a · b = 0

4. Show that a = 3i − 2j + k and b = 5i + 6j − 3k are perpendicular.

5. The vector 3i + kk is perpendicular to 6i + 5j − 3k, find the value of k.

6.* Given that a = i + 3j + k, b = pi + qj + 7k and c = 3i + 2j − 2k, find the value of


p and of q if a and b are perpendicular and b and c are perpendicular.

94 dkrbabajee@gmail.com
c
CHAPTER 6. VECTORS 1

6.7 Magnitude of a vector


 
x p
The magnitude or length of a vector a =  y  = xi + yj + zk is given by |a| = x2 + y 2 + z 2 .
z
 
2
Example 6.7 Given that a =  −3 , find the length of a.
6
p √
Solution |a| = 22 + (−3)2 + 62 = 49 = 7.

6.8 Unit vector


a
A unit vector b
a in the direction of the vector a is given by b
a= .
|a|
a |a|
The unit vector has magnitude |b a| = = = 1.
|a| |a|
Also, we have b = |b|ba where b is a vector of length |b| in the direction of a.

Example 6.8 Given that a = 2i + j + 2k, find a unit vector and a vector
of length 9 in the direction of a.

Solution √ √
|a| = 22 + 12 + 22 = 9 = 3
a 1 2 1 2
b
a= = (2i + j + 2k) = i + j + k
|a| 3 3 3 3
9
b = |b|b a = (2i + j + 2k) = 6i + 3j + 6k
3

6.9 Angle between two vectors


a·b
The angle between 2 vectors a and b is given by cos θ = .
|a| |b|

Case 1: If a · b > 0, θ is acute.

Case 2: If a · b < 0, θ is obtuse.

Case 3: If a · b = 0, θ = 900 (a and b are perpendicular).


   
1 3
Example 6.9 Find the angle between a =  2  and b =  −2 .
1 4

95 dkrbabajee@gmail.com
c
CHAPTER 6. VECTORS 1

Solution
a · b = (1 × 3) + (2 × −2) + (1 × 4) = 3,
√ √ p √
|a| = 12 + 22 + 12 = 6, |b| = 32 + (−2)2 + 42 = 29,
   
a·b 3
θ = cos −1
= cos−1
√ √ = 76.90 .
|a| |b| 6 29
Exercise 6.3. [Magnitude of a vector, angle between two vectors]

1. Find a unit vector in the direction of the following vectors, leaving your answers
in exact form where necessary:

(i) 3i + 4j (ii) 2i + 3k (iii) 2i − j − 2k
√ √
(iv) 2i − 6j + 9k (v) 3i − 2j + k (vi) − 3i + j − 5k

2. Given that a = 3i + 5j + k, b = −i + 6j − 2k and c = 3i − 2j + 3k, find a unit vector


parallel to:
(i) a + 4b (ii) 3a − 2c (iii) − a − 2b + 4c

3. Given that a = 2i + 3j − 6k, find a unit vector and a vector of length 3.5 in the
direction of a.

4. A vector a = 4i + 4j + pk has length 9, find the two unit vectors in the direction
of a.

5. Find the angle between the following pairs of vectors and state its type:
(i) 3i − 2j, 2i − 3j (ii) 3i − 4j, 2i + 3j

(iii) 3i − j + 2k, 4i − j + 8k (iv) 4i + 4j − 7k, i + 2j + 2k


       
−7 1 2 3
(v)  4 ,  1  (vi)  −4 ,  −2 
3 1 −1 −5

6. Given that a = pi + 4j + 8k and b = 2i − 2j + k, find the value of p if a · b = 2 and


hence find the angle between a and b.

7.* Find all angles in the triangle ABC with A(1, 1, −3), B(−2, 3, 1) and C(4, −3, 2).
     
2 2 p
8. Given that a =  −2  , b =  6  and c =  p , find
1 3 p+1

(i) the angle between a and b,


(ii) the value of p for which b and c are perpendicular.

96 dkrbabajee@gmail.com
c
CHAPTER 6. VECTORS 1

6.10 Position Vectors


−→
Position vector of a point A relative to the origin is given by OA.
−→ −→ −−→
The Triangle Law of vectors for △OAB gives AB = AO + OB.
−→ −→ −→ −−→ −→
Since AO = −OA, we have AB = OB − OA .
−→ −→ −→ −→
In parallelogram PQRS, P Q = SR, P S = QR .

6.10.1 Position vector of the Midpoint of two points


Suppose M is the midpoint of A and B. Then
−−→ −−→
AM = MB
−−→ −→ −−→ −−→
OM − OA = OB − OM
−−→ −→ −−→
2OM = OA + OB
−→ −−→
−−→ OA + OB
OM =
2

97 dkrbabajee@gmail.com
c
CHAPTER 6. VECTORS 1

6.10.2 Position vector of a point between two points


Suppose P lies between A and B such that AP : P B = r : s. Then A, P and B are
−→ −−→ r
collinear points such that AP = k P B, k = . Also,
s
−→ r −−→
AP = P B
s
−→ −−→
sAP = r P B
−→ −→ −−→ −→
sOP − sOA = r OB − r OP
−→ −→ −−→
(s + r)OP = sOA + r OB
−→ −−→
−→ sOA + r OB
OP =
s+r

Example 6.10 The position vectors of A, B and C are given by


−→ −−→ −→
OA = 2i + 3j − 4k, OB = 5i − j + 2k and OC = pi − 9j + 14k. Find

(i) the position vector of D such that ABCD is a parallelogram,

(ii) the value of p if A, B and C are collinear,

(iii) the position vector of Q if AQ : QB = 3 : 4.

Solution

(i)
    
5 2 3
−→ −−→ −→ 
AB = OB − OA = −1  −  3  =  −4 
2 −4 6
−−→ −→
DC = AB
−→ −−→ −→
OC − OD = AB
     
p 3 p−3
−−→ −→ −→ 
OD = OC − AB = −9  −  −4  =  −5  = (p − 3)i − 5j + 8k
14 6 8

98 dkrbabajee@gmail.com
c
CHAPTER 6. VECTORS 1

(ii)
     
p 5 p−5
−−→ −→ −−→ 
BC = OC − OB = −9  −  −1  =  −8 
14 2 12
−→ −−→
AB = k BC
   
3 (p − 5)k
 −4  =  −8k 
6 12k
1
− 8k = 4 ⇒ k =
2
p−5
(p − 5)k = 3 ⇒ = 3 ⇒ p − 5 = 6 ⇒ p = 11
2

(iii)
−→ 3 −−→
AQ = QB
4
−→ −−→
4AQ = 3QB
−→ −→ −−→ −→
4OQ − 4OA = 3OB − 3OQ
     
2 5 23
−→ −→ −−→
7OQ = 4OA + 3OB = 4  3  + 3  −1  =  9 
−4 2 −10
−→ 23 9 10
OQ = i+ j− k
7 7 7

99 dkrbabajee@gmail.com
c
CHAPTER 6. VECTORS 1

Example 6.11 Consider the cuboid OABCDEFG. The unit vectors i, j


−→ −→ −→
and k are along OA, OC and OG respectively. Also OC = 10 cm,
OA = 15 cm and OG = 5 cm and X and Y are the midpoint of DE and
GD respectively.

Find
−−→
(i) OX in terms of i, j and k,
−−→
(ii) OY in terms of i, j and k,
b .
(iii) X OY

Solution

(i)
−→ −−→ −→ −−→ 1 −−→ 1 −→ 1
OA = 15i, AD = OG = 5k, DX = DE = OC = (10i) = 5j
2 2 2
−−→ −→ −−→ −−→
OX = OA + AD + DX
−−→
OX = 15i + 5j + 5k

(ii)
−→ −−→ 1 −→ 15
OG = 5k, GY = OA = i
2 2
−−→ −→ −− →
OY = OG + GY
−−→ 15
OY = i + 5k
2

100 dkrbabajee@gmail.com
c
CHAPTER 6. VECTORS 1

(iii) b is the angle between −


X OY
−→ −−→
OX and OY .
   15 
15  
−−→ −−→    2  15
OX · OY = 5 ·  0  = 15 × + (5 × 0) + (5 × 5) = 137.5,
2
5 5
s 
√ √ 2 √
−−→ 2 2 2
−−
→ 15
|OX| = 15 + 5 + 5 = 275, |OY | = + 02 + 52 = 81.25,
2
−−→ −−→ !  
OX · OY 137.5
θ = cos −1
−−→ −−→ = cos
−1
√ √ = 23.10 .
|OX| |OY | 275 81.25

     
2 4 2
−→ −→ −→
Example 6.12 Given that OP =  2 , OQ =  −2 , OR =  3 
−1 2 x
 
−2
−→
and OS =  0 , find
y
−→
(i) a unit vector in the direction of P Q,
b = 900 ,
(ii) the value of x such that P OR
−→
(iii) the values of y for which |P S| = 6 units.

Solution
(i)
     
4 2 2
−→ −→ −→ 
P Q = OQ − OP = −2  −  2  =  −4 
2 −1 3
−→ p √
|P Q| = 22 + (−4)2 + 32 = 29
 
−→ 2
−→ PQ 1
Unit vector in the direction P Q = −→ = √  −4 
|P Q| 29 3

(ii) b is the angle between −


P OR
→ −→ b = 900 , −
OP and OR. Since P OR
→ −→
OP · OR = 0.
   
2 2
 2 · 3 =0
−1 x
4 + 6 − x = 0 ⇒ x = 10

101 dkrbabajee@gmail.com
c
CHAPTER 6. VECTORS 1

(iii)
     
−2 2 −4
−→ −→ −→ 
P S = OS − OP = 0  −  2  =  −2 
y −1 y+1
−→ −→ 2
|P S| = 6 ⇒ |P S| = 62 = 36
−→
|P S|2 = (−4)2 + (−2)2 + (y + 1)2 = 20 + (y + 1)2
20 + (y + 1)2 = 36 ⇒ (y + 1)2 = 16

y + 1 = ± 16 = ±4
y + 1 = −4 or y + 1 = 4
y = −5 or y = 3

−→ −→
Example 6.13 Given that OP = i + 2j + 4k and OQ = 4i − 2j + 4k,

(i) Calculate QPbO.


−→
(ii) Find a vector of length 25 in the direction P Q.
−→
(iii) Given also that OR = 3i + xk, where x is a constant and P R = 2P Q,
find the possible values of x.

Solution
−→ −→
(i) QPbO is the angle between P Q and P O.
       
4 1 3 −1
−→ −→ −→  −→
P Q = OQ − OP = −2  −  2  =  −4  , P O =  −2 
4 4 0 −4
   
3 −1
−→ −→   
PQ · PO = −4 · −2  = (3 × −1) + (−4 × −2) + (0 × −4) = 5,
0 −4
−→ p √ −→ p √
|P Q| = 32 + (−4)2 + 02 = 25 = 5, |P O| = (−1)2 + (−2)2 + (−4)2 = 21,
−→ −→ !  
PQ · PO 5
θ = cos−1
−→ −→ = cos
−1
√ = 77.40.
|P Q| |P O| 5 21

102 dkrbabajee@gmail.com
c
CHAPTER 6. VECTORS 1

(ii)
−→ −→
P Q = 3i − 4j, |P Q| = 5
−→
−→ PQ 1
Unit vector in the direction P Q, b
a = −→ = (3i − 4j)
|P Q| 5
25
b = |b|b
a = (3i − 4j) = 15i − 20j
5

(iii)
     
3 1 2
−→ −→ −→     
P R = OR − OP = 0 − 2 = −2 
x 4 x−4
−→ −→ −→ 2 −→
|P R| = 2|P Q| ⇒ |P R| = 22 |P Q|2 = 4 × 52 = 100
−→
|P R|2 = 22 + (−2)2 + (x − 4)2 = 8 + (x − 4)2
8 + (x − 4)2 = 100 ⇒ (x − 4)2 = 92
√ √
x − 4 = ± 92 ⇒ x = 4 ± 92

Example 6.14 Relative to an  origin


 O, the position
 vectors of the
 points

2 3 10
−→ −→ −→
P, Q and R are given by OP =  4 , OQ =  −4  and OR =  q ,
1 3 8
−→ −→ −→
where q is a constant. Given that OR = mOP + nOQ, where m and n are
constants, find the values of m, n and q.

Solution
     
10 2m 3n
 q  =  4m  +  −4n 
8 m 3n
   
10 2m + 3n
 q  =  4m − 4n 
8 m + 3n
2m + 3n = 10 q = 4m − 4n m + 3n = 8
Replace m = 8 − 3n in 2m + 3n = 10, we have
2(8 − 3n) + 3n = 10 ⇒ 16 − 6n + 3n = 10 ⇒ 3n = 6 ⇒ n = 2
m = 8 − 3(2) = 2, q = 4m − 4n = 4(2) − 4(2) = 0

103 dkrbabajee@gmail.com
c
CHAPTER 6. VECTORS 1

Exercise 6.4. [Position vectors]


−→
1. The position vectors of A, B and C are given by OA = 3i + 2j + 4k,
−−→ −→
OB = 5i + 2j − k and OC = 14i + 9j + 11k. Find

(i) the position vector of D such that ABCD is a parallelogram,


(ii) the position vector of R if AR : RB = 2 : 3.
−→ −→
2. Consider the cuboid OABCDEFG. The unit vectors i, j and k are along OA, OC
−→
and OG respectively. Also OC = 8 cm, OA = 12 cm and OG = 4 cm and X and Y
are the midpoint of DE and GD respectively.

Find
−−→
(i) OX in terms of i, j, and k.
−−→
(ii) OY in terms of i, j, and k.
b .
(iii) X OY

       
−2 2 2 y
−→ −→ −→ −→
3. Given that OP =  3 , OQ =  −1 , OR =  x  and OS =  −1 ,
10 4 1 6
find
−→
(i) a unit vector in the direction of P Q,
(ii) the value of x such that P ORb = 900 ,
−→
(iii) the values of y for which |P S| = 6 units.

4. Relative to an origin O, the position vectors of the points P, Q and R are given by
     
1 −2 −1
−→  −→ −→
OP = −3 , OQ =  1  and OR =  q , where q is a constant. Given
4 1 14
−→ −→ −→
that OR = mOP + nOQ, where m and n are constants, find the values of m, n
and q.

5. Relative to the origin O, the position vectors of A and B are i + 2j + 3k and


3i + 4j + 5k, respectively. The point C is such that OC = 3 OA. Find

(i) the length of OC,


b .
(ii) the angle ACB

104 dkrbabajee@gmail.com
c
CHAPTER 6. VECTORS 1

6.* Relative to the origin O, the position vectors of P and Q are j + 2k and qi + 4j,
respectively.
−→
(i) Find the unit vector in the direction P Q in the case q = 6.
b = cos−1 (0.4).
(ii) Find the value of q for which angle P OQ

7. Relative to the origin O, the position vectors of A and B are 3i − 4j + k and


5i + λj + 7k, respectively.
−→
Find the values of λ for which |AB| = 7.

8.* Relative to the origin O, the position vectors of A and B are i − 2j + 2k and
8i + j + 4k, respectively.
−→ −−→ −→
Given that OC is parallel to OB and has same magnitude as OA, find the
possible position vectors of C.

9.* Relative to the origin O, the position vectors of A and B are pi + qj + 4k and
2i + 4j + 5k, respectively.
−→ b = 900 .
Find the values of p and q for which |OA| = 9 and AOB

10.* Relative to the origin O, the position vectors of P and Q are 2i − j + 2k and
ai + 3j + qk, respectively.

−→ −→
(i) Find the values of a and b for which OP is parallel to OQ.
(ii) Express a in terms of b in case angle POQ is 900 .
3 −→
(iii) In the case where b = a, find the values of a for which |P Q| = 9.
2

105 dkrbabajee@gmail.com
c
CHAPTER 6. VECTORS 1

6.11 Miscellaneous Exercises


Example 6.15 Consider the triangular prism having a rectangular base
PSUR, with SU = 4 units and RU = 8 units. △ PQR and △ TSU are
isosceles triangles with PQ = QR = 3 units. X and Y are midpoints of QT
and SU respectively. O is the midpoint of PR and unit vectors i, j and k
−→ −−→ −→
are along OR, OY and OQ respectively.

Find
−→
(i) |OQ|,
−−→ −−→
(ii) XR, XY in terms of i, j and k,
b .
(iii) RXY
Solution
(i) OP Q is a right-angled triangle with PQ = 3 units and OP
√ = 2 units.√
Using Pythagoras theorem, OQ2 + OP 2 = P Q2 ⇒ OQ = 32 − 22 = 5.
−→ √ −→ √
∴, OQ = 5k and |OQ| = 5.
(ii)
−−→ 1 −→ 1 −−→ 1
QX = QT = OY = (8j) = 4j,
2 2 2
−→ 1 −→ 1 −→ 1
OR = P R = SU = (4i) = 2i
2 2 2
−−→ −−→ −→ −→
XR = XQ + QO + OR
−−→ √ √
XR = −4j − 5k + 2i = 2i − 4j − 5k
−−→ −−→ −→ −→ √
XT = QX = 4j, Y T = OQ = 5k
−−→ −−→ −→
XY = XT + T Y
−−→ √
XY = 4j − 5k.

106 dkrbabajee@gmail.com
c
CHAPTER 6. VECTORS 1

(iii) b is the angle between −


RXY
−→ −−→
XR and XY .
   
2 0 √ √
−−→ −−→   ·  4  = (2 × 0) + (−4 × 4) + (− 5 × − 5) = −11,
XR · XY = −4
√ √
− 5 − 5
q √ √ q √ √
−−→ −−→
|XR| = 2 + (−4) + (− 5) = 25 = 5, |XY | = 02 + 42 + (− 5)2 = 21,
2 2 2

−−→ −−→ !  
XR · XY −11
θ = cos −1
−−→ −−→ = cos
−1
√ = 118.70.
|XR| |XY | 5 21

Miscellaneous Exercise 6.

1. Find all angles in the triangle ABC with A(2, 1, 3), B(−1, 2, 3) and C(0, −3, 4).

2. Relative to the origin O, the position vectors of A and B are i + 2j + k and −2i + k,
respectively. The point P is such that OP = OA + kAB. Find
b
(i) the angle AOB,
(ii) the value of k such that OP is perpendicular to AB.

3. Consider the triangular prism having a rectangular base PSUR, with


SU = 8 units and RU = 10 units. △ PQR and △ TSU are isosceles triangles
with PQ = QR = 5 units. X and Y are midpoints of QT and SU respectively.
−→ −−→ −→
O is the midpoint of PR and unit vectors i, j and k are along OR, OY and OQ
respectively.

Find
−→
(i) |OQ|,
−−→ −−→
(ii) XR, XY in terms of i, j and k,
b .
(iii) RXY

−→ −→
4. Given that OP = 4i + 3j − 4k and OQ = 4i − 2j + 8k,

(i) calculate QPbO.


−→
(ii) Find a vector of length 39 in the direction P Q.
−→
(iii) Given also that OR = −4i + 9j + xk, where x is a constant and P R = 2P Q,
find the possible values of x.

107 dkrbabajee@gmail.com
c
CHAPTER 6. VECTORS 1

4
5. If the cosine of the angle between the vectors 6i + λj − 2k and −2i + 4j − 4k is ,
21
find the value of λ.

6. In triangle POQ, O is the origin, P and Q have the position vectors 4i + 3j − 4k


and i + 2j + 6k, respectively.

(i) Show that triangle POQ is isosceles.


b
(ii) Find P OQ.
−→
7. The vector OP has a magnitude of 9 units and is parallel to the vector i + 2j + 2k
−→
and The vector OQ has a magnitude of 18 units and is parallel to the vector
4i − 7j − 4k.
b
(i) Find P OQ.
−→
(ii) Find a unit vector in the direction P Q.

8.* Relative to the origin O, the position vectors of A and B are pi + qj + 2k and
4i − 3j + k, respectively.
−→ b = 900 .
Find the values of p and q for which |OA| = 3 and AOB

9. Relative to the origin O, the position vectors of P and Q are i−j+2k and −i+j+pk,
respectively.
−→ −→
(i) Find the value of p for which OP is parallel to OQ.
−→ −→
(ii) Find the value of p for which OP is perpendicular to OQ.
−→
(iii) Find the values of p for which |P Q| = 3.

10.* Relative to the origin O, the position vectors of A, B and C are 5i−2j+ k, 2i+ j+ k
and 3i − 4j − 2k, respectively.
b
(i) Find ABC.
(ii) Find the position vector of D given that ABCD is parallelogram.
−−→ −→
(iii) The point E lies on BA produced such that BE = BA.
Find the position vector of E.
(iv) The line CE cuts the line AD at X.
Find the position vector of X.

108 dkrbabajee@gmail.com
c
C HAPTER 7

Series

7.1 Objective
At the end of this chapter, students should be able to:

1. use the expansion of (a + b)n , where n is a positive integer;

2. find a specific term in the binomial expansion;

3. recognise arithmetic progressions and use the formulae for the nth term and
for the sum of the first n terms to solve problems;

4. recognise geometric progressions and use the formulae for the nth term and for
the sum of the first n terms to solve problems;

5. use the condition for the convergence of a geometric progression, and the formula
for the sum to infinity of a convergent geometric progression.

7.2 Binomial expansion


The binomial expansion of (a + b)n , where n is a positive integer is given by

(a + b)n = n
C0 an b0 + n C1 an−1 b1 + n C2 an−2 b2 + . . . + n Cr an−r br + . . . + n Cn an−n bn ,

n!
where n Cr = and r! = r × (r − 1) × (r − 2) × . . . × 2 × 1.
r!(n − r)!
Therefore, we have

n n! n n! n n! n(n − 1)
C0 = = 1, 0! = 1 C1 = =n C2 = =
0!(n − 0)! 1!(n − 1)! 2!(n − 2)! 2!
n n! n(n − 1)(n − 2) n n!
C3 = = ... Cn = =1
3!(n − 3)! 3! n!(n − n)!

and the binomial expansion simplifies to

(a + b)n = an + n C1 an−1 b1 + n C2 an−2 b2 + . . . + n Cr an−r br + . . . + bn , (7.1)

109 dkrbabajee@gmail.com
c
CHAPTER 7. SERIES

where n Cr can be calculated using a calculator.


Expansion (7.1) is in ascending order of a (we start from smallest power of a) and is
in descending order of b (we start from highest power of b).

Example 7.1 (a) Expand (2 + 3x)5 in ascending powers of x.

(b) Expand (2 + 3x)5 in descending powers of x.


Solution

(a) We expand from the smallest power of x (independent term) up to


the highest power of x (x5 )

(2 + 3x)5 = 25 + 5 C1 (2)4 (3x)1 + 5 C2 (2)3 (3x)2 + 5 C3 (2)2 (3x)3 + 5 C4 (2)1 (3x)4 + (3x)5
= 32 + 240x + 720x2 + 1080x3 + 810x4 + 243x5

(b) We expand from the the highest power of x (x5 ) up to the smallest
power of x (independent term).

(2 + 3x)5 = (3x + 2)5


= (3x)5 + 5 C4 (2)1 (3x)4 + 5 C3 (2)2 (3x)3 + 5 C2 (2)3 (3x)2 + 5 C1 (2)4 (3x)1 + 25
= 243x5 + 810x4 + 1080x3 + 720x2 + 240x + 32

7.2.1 Finding a specific term in the binomial expansion


n
The (r + 1)th term in the expansion (a + b)n is given by Tr+1 = Cr an−r br .

Example 7.2 Find the coefficient of x3 in the expansion of


(i) (1 + 2x)6 (ii) (1 − 3x)(1 + 2x)6

Solution

(i)

a = 1, b = 2x, n = 6
n
Cr an−r br =6 Cr 16−r (2x)r = 6 Cr 2r xr
For x3 , we require r = 3
Coefficient of x3 =6 C3 × 23 = 160

(ii) We note that (1 − 3x) consists of an independent term and a term in x. So we


require the coefficient of x2 and x3 in the expansion of (1 + 2x)6 . For x2 , we
require r = 2 and coefficient of x2 = 6 C2 22 = 60.
So the expansion of (1+2x)6 is 60x2 +160x3 . We just need to find the coefficient of

110 dkrbabajee@gmail.com
c
CHAPTER 7. SERIES

x3 in the expansion of (1−3x)(60x2 +160x3 ) using the conventional multiplication


of numbers (we are required to put the expressions in descending power of x).

−3x 1
160x3 60x2 ×
−180x3 60x2
−480x4 160x3 0 +
−480x4 −20x3 60x2

∴ the coefficient of x3 in the expansion of (1 − 3x)(1 + 2x)6 is -20.


 5
3
Example 7.3 Find the coefficient of x in the expansion of 2x − .
x

Solution
Method 1
3
a = 2x, b = − , n = 5
x 
r
n n−r r 5 5−r 3
Cr a b = Cr (2x) − = 5 Cr 25−r x5−r (−3)r x−r = 5 Cr 25−r (−3)r x5−2r
x
1
For x , we require 5 − 2r = 1 ⇒ r = 2
Coefficient of x = 5 C2 × 25−2 (−3)2 = 720

Method 2
 r
3
r (2x)5−r − Term
 x 0
3
0 (2x)5 × − x5
 x 1
3
1 (2x)4 × − x3
 x 2
3
2 (2x)3 × − x1
x

The term in x corresponds to r = 2 and the solution follows as usual.


 9
3
Example 7.4 The independent term in the expansion of px + 2 is
x
28
. Calculate the value of p.
9

111 dkrbabajee@gmail.com
c
CHAPTER 7. SERIES

Solution
3
a = px, b = ,n=9
x2 r
n n−r r 9 3
9−r
Cr a b = Cr (px) = 9 Cr p9−r x9−r (3)r x−2r = 9 Cr p9−r (3)r x9−3r
x2
For x0 , we require 9 − 3r = 0 ⇒ r = 3
Independent term = 9 C3 × p9−3 (3)3 = 2268 × p6
28 1
2268 × p6 = ⇒ p6 =
9 729
 6
1 1
p6 = ⇒p= .
3 3

Example 7.5 The first three terms in the expansion of (2 + ax)n in


ascending powers of x are 32 + 240x + bx2. Find the values of the constants
n, a and b.

Solution

(2 + ax)n = 2n + n C1 2n−1(ax)1 + n C2 2n−2 (ax)2


n(n − 1) n−2 2 2
= 2n + n × 2n−1 ax + 2 ax
2
= 32 + 240x + bx2

Comparing coefficients, we have

n(n − 1) n−2 2
2n = 32 n × 2n−1 a = 240 b = 2 a
2
5(5 − 1) 5−2 2
2n = 25 5 × 24 a = 240 b= 2 3
2
n=5 a=3 b = 720

Example 7.6 (i) Find the first 3 terms in the expansion of (1 + ax)6 in
ascending powers of x.

(ii) Given that there is no term in x in the expansion of (1 −2x)2 (1 + ax)6 ,


find the value of a.

(iii) For this value of a, find the coefficient of x2 in the expansion of


(1 − 2x)2 (1 + ax)6 .

112 dkrbabajee@gmail.com
c
CHAPTER 7. SERIES

Solution
(i)

(1 + ax)6 = 1 + 6 C1 (ax)1 + + 6 C2 (ax)2


= 1 + 6ax + 15a2 x2 .

(ii)

(1 − 2x)2 (1 + ax)6 = (1 − 4x + 4x2 )(1 + 6ax + 15a2 x2 )


4x2 −4x 1
15a2 x2 6ax 1 ×
2
4x −4x 1
2
−24ax 6ax 0 +
15a2 x2 0 0
2 2
(4 − 24a + 15a )x (6a − 4)x 1
2
No term in x, 6a − 4 = 0 ⇒ a =
3
   2
2 2 2 2 16
(iii) The coefficient of x is 4 − 24a + 15a = 4 − 24 + 15 =− .
3 3 3
Exercise 7.1. [Binomial expansion]
1. Expand in ascending powers of x 
5 2 4 x 3
(i) (2 + 5x) (ii) (1 − 5x ) (iii) 3 +
2
2. Find the coefficient of x3 in the expansion of (i) (1 + 2x)4 , (ii) (2 − 3x)(1 + 2x)4 .
 12
5 9 1
3. Find the coefficient of x in the expansion of (i) (1 + 2x) , (ii) 1 − x .
2
 6
3
4. Find the independent term in the expansion of 2x − .
x
5. (i) Expand (x + 1)5 − (x − 1)5 .
√ √
(ii) Hence, find the exact value of ( 3 + 1)5 − ( 3 − 1)5
6. Find the first 4 terms of the expansion of (1 + 2x)7 in ascending powers of x.
 7
26
Hence, find the value of correct to 3 decimal places.
25
 8
2 2
7. The coefficient of x in the expansion of px + 2 is 1792. Calculate the value
x
of p.

113 dkrbabajee@gmail.com
c
CHAPTER 7. SERIES

 5
1 1
8. Find the value of the coefficient of in the expansion of 2x − .
x x
9. The first three terms in the expansion of (3 + ax)n in ascending powers of x are
81 − 216 x + bx2 . Find the values of the constants n, a and b.

10. (i) Find the first 3 terms in the expansion of (1 + ax)5 in ascending powers of x.
(ii) Given that there is no term in x in the expansion of (1 + 5x)2 (1 + ax)5 , find
the value of a.
(iii) For this value of a, find the coefficient of x2 in the expansion of
(1 + 5x)2 (1 + ax)5 .

7.3 Arithmetic Progression (A. P.)


7.3.1 nt h term
Consider the sequence 2, 5, 8, 11, . . ..
We observe that there is a difference of 3 between the consecutive terms i.e. 5−2 = 3,
8−5 = 3, 11−8 = 3. We therefore say that this sequence is an arithmetic progression.
An arithmetic progression is a sequence in which there is a common difference
between consecutive terms. We denote the first term, a and the common difference
d.
Consider the sequence a, a + d, a + 2d, . . . and Tn denote the nth term

T1 = a = a + (1 − 1)d
T2 = a + d = a + (2 − 1)d
T2 = a + 2d = a + (3 − 1)d
..
.
Tn = a + (n − 1)d

So the nth term of A.P. is given by Tn = a + (n − 1)d .

7.3.2 Sum of n terms in an A.P.


Let Sn denotes the sum of the n terms in an A.P.

Sn = T1 + T2 + T3 + . . . + Tn−2 + Tn−1 + Tn
Sn = a + (a + d) + (a + 2d) + . . . + (Tn − 2d) + (Tn − d) + Tn (7.2)
Sn = Tn + (Tn − d) + (Tn − 2d) + . . . + (a + 2d) + (a + d) + a (7.3)

114 dkrbabajee@gmail.com
c
CHAPTER 7. SERIES

(7.2) + (7.3) gives


2Sn = a + Tn + a + Tn + a + Tn + . . . + a + Tn + a + Tn + a + Tn
| {z }
n times
2Sn = n(a + Tn )
n
Sn = (a + Tn )
2
n 
Sn = 2a + (n − 1)d
2

Example 7.7 Consider the sequence 2, 5, 8, 11, . . ..

(i) Find the 20th term.

(ii) Find the sum of the first 25 terms.

(iii) Which term has value 86?

(iv) Find the number of terms whose sum is 3775.

Solution

(i)

a = 2, d = 5 − 2 = 3
Tn = a + (n − 1)d
T20 = 2 + (20 − 1)3 = 59.

(ii)
n
Sn = (2a + (n − 1)d)
2
25  
S25 = 2(2) + (25 − 1)(3) = 950.
2

(iii)

Tn = a + (n − 1)d
Tn = 2 + 3(n − 1)
Tn = 86 ⇒ 2 + 3(n − 1) = 86
3(n − 1) = 84 ⇒ n − 1 = 28 ⇒ n = 29

115 dkrbabajee@gmail.com
c
CHAPTER 7. SERIES

(iv)
n 
Sn = 2a + (n − 1)d
2
n   25
Sn = 2(2) + 3(n − 1) = (3n + 1)
2 2
n
Sn = 3375 ⇒ (3n + 1) = 3375
2
2
3n + n − 7550 = 0
p
−1 ± (−1)2 − 4(3)(−7550) −1 ± 301 151
n= = = 50 or −
2(3) 6 3
since n > 0, n = 50.

Example 7.8 Find the sum of all the multiples of 6 between 200 and 300
inclusive.

Solution Consider the sequence 204, 210, 216, . . . , 300. This is A.P. with first term
a = 204 and d = 210 − 204 = 6. We find the number of terms in the sequence using
Tn = 300 and then we calculate its sum.

Tn = a + (n − 1)d
Tn = 204 + 6(n − 1)
Tn = 300 ⇒ 204 + 6(n − 1) = 300
6(n − 1) = 96 ⇒ n − 1 = 16 ⇒ n = 17
n
Sn = (a + Tn )
2
17  
S17 = 204 + 300 = 4284.
2

Example 7.9 An arithmetic progression contains 20 terms. Given that


the ninth term is 29 and the sum of all terms in the progression is 930.
Calculate

(i) the first term and the common difference,

(ii) the sum of all positive terms in the progression.

116 dkrbabajee@gmail.com
c
CHAPTER 7. SERIES

Solution
(i) We have T8 = 29 and S20 = 930. We obtain two simultaneous equations in a and
d.

Tn = a + (n − 1)d
T8 = a + (8 − 1)d = a + 7d
T8 = 29
a + 7d = 29 (7.4)
n 
Sn = 2a + (n − 1)d
2
20  
S20 = 2a + (20 − 1)d = 10(2a + 19d)
2
S20 = 930 ⇒ 10(2a + 19d) = 930
2a + 19d = 93 (7.5)
(7.5) − 2×(7.4) gives
5d = 35 ⇒ d = 7
a = 29 − 7d = 29 − 7(7) = −20

(ii) We determine which term is the first positive term (e.g Tn > 0).

Tn = a + (n − 1)d
Tn = −20 + 7(n − 1)
Tn > 0 ⇒ −20 + 7(n − 1) > 0
20
7(n − 1) > 20 ⇒ n > + 1 ⇒ n > 3.85 ⇒ n = 4
7
There are 3 negative terms and 17 positive terms. The sum of the positive
terms is given by S20 − S3 .
n 
Sn = 2a + (n − 1)d
2
3 
S3 = 2(−20) + 7(3 − 1) = −39
2
Sum of all positive terms = S20 − S3 = 930 − −39 = 930 + 39 = 969.

Example 7.10 A circle is divided into 6 sectors in such a way that the
angles of the sectors are in arithmetic progression. The angle of the
largest sector is 5 times the angle of the smallest sector. Given that the
radius of the circle is 9 cm, find the area of the smallest sector.
Solution
The circle is divided into 6 sectors. This implies that we have 6 angles in an arithmetic

117 dkrbabajee@gmail.com
c
CHAPTER 7. SERIES

progression. Let a be the smallest angle. The largest angle is T6 = 5a and the sum of
all angles is S6 = 2π.

T6 = 5a
n
Sn = (a + Tn )
2
6 
S6 = a + 5a = 18a
2
π
S6 = 2π ⇒ 18a = 2π ⇒ a =
9
1 2π
Area of smallest sector = (9) = 4.5π
2 9
Exercise 7.2. [Arithmetic Progression]

1. Consider the sequence 3, 7, 10, 13, . . ..

(i) Find the 15th term.


(ii) Find the sum of the first 20 terms.
(iii) Which term has value 151?
(iv) Find the number of terms whose sum is 4095.

2. Find the sum of

(i) all the even numbers between 100 and 200 inclusive.
(ii) all the multiples of 3 between 200 and 300 inclusive.
(iii) all the multiples of 5 between 100 and 500 inclusive.
(iv) all the multiples of 8 between 300 and 400 inclusive.

3. Consider the following arithmetic progression 3, 9, 15, ...., 525. Find

(i) the number of terms in this progression,


(ii) the 16th term,
(iii) the sum of all the terms in the progression.

4. An arithmetic progression has fourth term 15 and ninth term 35. Find the first
term and the common difference.

5. An arithmetic progression contains 20 terms. Given that the tenth term is 45


and the sum of all terms in the progression is 1030. Calculate

(i) the first term and the common difference,


(ii) the sum of all positive terms in the progression.

118 dkrbabajee@gmail.com
c
CHAPTER 7. SERIES

6. An arithmetic progression has first term a = 55 and common difference d = −12.

(i) Show that the nth term is given by Tn = 67 − 12n.


(ii) Show that the sum of the first n terms is given by Sn = 61n − 6n2 .
(iii) Show that Tn = Sn − Sn−1 .
(iv) Find the first negative term.
(v) Find the number of terms for which Sn = −435.

7. A circle is divided into 12 sectors in such a way that the angles of the sectors are
in arithmetic progression. The angle of the largest sector is 3 times the angle of
the smallest sector. Given that the radius of the circle is 18 cm, find the area of
the smallest and largest sector.

8. If the sum of the first 20 terms of an arithmetic progression is 1220 and that the
sum of the next 20 terms is 3640, find

(i) the first term and the common difference,


(ii) the sum of the first 12 terms of the progression.

9. If 1st term of an arithmetic progression is 3 and the 21st term is 8.

(i) Find the common difference.


(ii) Find the least value of n such that the sum of the first n terms exceeds 20.

10. The sizes of the angles of a quadrilateral follow an arithmetic progression such
that the largest angle exceeds the smallest angle by 900 . Find all angles of the
quadrilateral.

7.4 Geometric Progression (G. P.)


7.4.1 nth term
Consider the sequence 1, 3, 9, 27, . . .. We observe that
T2 3 T3 9 T4 27
= = 3, = = 3, = = 3.
T1 1 T2 3 T3 9
In this case we have a common ration of 3. We therefore refer this sequence as a
geometric progression.
A geometric progression is a sequence in which there is a common ratio between

119 dkrbabajee@gmail.com
c
CHAPTER 7. SERIES

every 2 consecutive terms. In this case, we denote the first term, a and common
ratio, r of a G.P. Consider the sequence a, ar, ar 2 , . . . and Tn denote the nth term

T1 = a = ar 1−1
T2 = ar = ar 2−1
T2 = ar 2 = ar 3−1
..
.
Tn = ar n−1

So the nth term of G.P. is given by Tn = ar n−1 .

7.4.2 Sum of n terms in an G.P.


Let Sn denotes the sum of the n terms in an G.P.

Sn = T1 + T2 + T3 + . . . + Tn−2 + Tn−1 + Tn
Sn = a + ar + ar 2 + . . . + ar n−3 + ar n−2 + ar n−1 (7.6)
rSn = ar + ar 2 + ar 3 . . . + ar n−2 + ar n−1 + ar n (7.7)

(7.7) - (7.6) gives


rSn − Sn = ar n − a
(r − 1)Sn = a(r n − 1)
a(r n − 1)
Sn = ,r>1
r−1
a(1 − r n )
Sn = ,r<1
1−r

Example 7.11 Consider the sequence 1, 3, 9, 27, . . ..

(i) Find the 15th term.

(ii) Find the sum of the first 18 terms.

(iii) Which term has value 243?

(iv) Find the number of terms whose sum is 1093.

120 dkrbabajee@gmail.com
c
CHAPTER 7. SERIES

Solution

(i)
3
a = 1, r = =3
1
Tn = ar n−1
T15 = 1(3)15−1 = 4782969.

(ii)

r=3>1
a(r n − 1)
Sn =
r−1
1(318 − 1)
S18 = = 193710244.
3−1

(iii)

Tn = ar n−1
Tn = 1(3)n−1 = 3n−1
Tn = 243 ⇒ 3n−1 = 243
3n−1 = 35 ⇒ n − 1 = 5 ⇒ n = 6

(iv)
a(r n − 1)
Sn =
r−1
1(3n − 1) 3n − 1
Sn = =
3−1 2
3n − 1
Sn = 1093 ⇒ = 1093 ⇒ 3n − 1 = 2186
2
3n = 2187 ⇒ 3n = 37 ⇒ n = 7.

Example 7.12 A company agrees to sponsor a football team in which


grants will be received each year. This grant will be Rs 2000 in 2013 and
will increase by 10% each year. Calculate

(i) the value of this grant in 2023,

(ii) the total amount the team will receive in the years 2013 to 2023
inclusive.

121 dkrbabajee@gmail.com
c
CHAPTER 7. SERIES

Solution

(i) Grant in 2013 = Rs 2000. Grant in 2014 will be 10% more than in 2013 i.e.
100%+ 10% = 110% × grant in 2014 and so on. We have a geometric progression
with a = 2000 and r = 1.1 > 1. The grant in 2023 will be the 11th (2023-2013+1)
term in the G.P.
Tn = ar n−1
T11 = (2000)(1.1)11−1 = 5187.48.

(ii)
a(r n − 1)
Sn =
r−1
(2000)(1.111 − 1)
S11 = = 37062.33.
1.1 − 1

7.4.3 Sum to infinity


1 1 1
Consider the sequence 1, , , , . . ..
2 22 23
1
It is a geometric progression with a = 1 and common ratio r = < 1.
 n−1 2
1
Tn = −→ 0 as n −→ 0.
2
Sn will also tend to a finite value which is known as the sum to infinity S∞ .
a
S∞ = , |r| < 1
1−r

1
Example 7.13 A geometric progression has common ratio − and the
3
sum of the first 3 terms is 105. Find

(i) the first term of the progression,

(ii) the sum to infinity.

122 dkrbabajee@gmail.com
c
CHAPTER 7. SERIES

Solution

(i)
1
r = − , S3 = 105
3
a(1 − r 3 )
S3 =
1 − r  
1 3
a 1 − −3 7
S3 = 1 = a
1 − −3 9
7 105 × 9
S3 = 105 ⇒ a = 105 ⇒ a = = 135
9 7

(ii)
a
S∞ =
1−r
135 135 × 3
= 1 = = 101.25
1 − −3 4

Example 7.14 The third, fourth and fifth consecutive terms of a


geometric progression are 2k + 3, k + 6 and k. Given that all terms of
the geometric progression are positive, calculate

(i) the value of the constant k,

(ii) the sum to infinity.

Solution

(i)
k+6 k
r= =
2k + 3 k+6
(k + 6) = k(2k + 3) ⇒ k 2 + 12k + 36 = 2k 2 + 6k
2

k 2 − 9k − 36 = 0 ⇒ (k + 3)(k − 12) = 0
k = −3 or12
since k > 0, k = 12

123 dkrbabajee@gmail.com
c
CHAPTER 7. SERIES

(ii)
12 2
r= =
12 + 6 3
4
T5 = 12 ⇒ ar = 12
12
a = 4 = 60.75
2
3
a 60.75
S∞ = = = 182.25
1−r 1 − 23

Exercise 7.3. [Geometric Progression]


1. Consider the sequence 6, 12, 24, 48, . . ..

(i) Find the 10th term.


(ii) Find the sum of the first 15 terms.
(iii) Which term has value 384?
(iv) Find the number of terms whose sum is 186.

2. Consider the following geometric progression 2, 4, 8, ...., 2048. Find

(i) the number of terms in this progression,


(ii) the 7th term in this progression,
(iii) the sum of all terms in the progression.

3. A company agrees to sponsor a basketball team in which grants will be received


each year. This grant will be Rs 20000 in 2014 and will increase by 10% each
year. Calculate

(i) the value of this grant in 2025,


(ii) the total amount the team will receive in the years 2014 to 2025 inclusive.

4. In a geometric progression, the 2nd term is 4 and the 5th term is 108. Find the
first term and common ratio.

5. The nth term of a geometric progression of positive terms is 16807 and the (n+4)th
term is 40353607. Find the common ratio.
1
6. A geometric progression has common ratio − and the sum of the first 3 terms
4
is 325. Find

(i) the first term of the progression,


(ii) the sum to infinity.

124 dkrbabajee@gmail.com
c
CHAPTER 7. SERIES

7. The third, fourth and fifth consecutive terms of a geometric progression are
12k − 1, 3k + 1 and k + 1. Given that all terms of the geometric progression are
positive, calculate

(i) the value of the constant k,


(ii) the sum to infinity.

8. Find the common ratio of a geometric series which has first term 5 and sum to
infinity 6.

9.* Given that a geometric series has first term 10 and common ratio r, where r > 0.
The sum of the first 7 terms is 3 times the sum of the 8th and 9th terms.
Prove that r 7 (4 − 3r 2 ) = 1.

10.* In the geometric progression 1 + e−5 + e−10 + e−15 + . . ., work out expressions for

(i) the sum of the first n terms, Sn ,


(ii) sum to infinity, S∞ .
e−5n
(iii) Hence, show that S∞ − Sn = .
1 − e−5

7.5 Miscellaneous Exercises


Example 7.15 The 1st term of an arithmetic progression is a and the
common difference is d, where d 6= 0.

(i) Write down expressions, in terms of a and d, for the 3rd term and
the 6th term.
The 1st term, the 3rd term and the 6th term of the arithmetic
progression are the three consecutive terms of a geometric
progression.

(ii) Show that a = 4d.

(iii) Find the common ratio of the geometric progression.

Solution

(i)

Tn = a + (n − 1)d ⇒ T3 = a + 2d, T6 = a + 5d

125 dkrbabajee@gmail.com
c
CHAPTER 7. SERIES

(ii)

a, a + 2d, a + 5d are in G.P.


a + 2d a + 5d
r= =
a a + 2d
(a + 2d) = a(a + 5d) ⇒ a2 + 4ad + 4d2 = a2 + 5ad
2

4d2 − ad = 0 ⇒ d(4d − a) = 0 ⇒ a = 4d (d 6= 0)

(iii)
a + 2d 6d 3
r= = = .
a 4d 2

Miscellaneous Exercise 7.
 10
4
1. Find the term independent of x in the expansion of x+ .
x
2. (i) Expand (x + 1)7 − (x − 1)7 .
√ √
(ii) Hence, find the exact value of ( 5 + 1)7 − ( 5 − 1)7
3. Find the coefficient of x2 in the expansion of (2 − 3x)2 (2 + x)7 .
4. (i) Find the first 3 terms in the expansion of (1 − 2x)6 in ascending powers of x.
(ii) Given that there is the coefficient of x2 in the expansion of
(1 + 2x + ax2 )(1 − 2x)6 is 48, find the value of a.
5. John is planning to give a sum of money as charity each year for 10 years. He
decides to give $100 in the first year and increase his contribution $30 each year.
Find
(i) how much he gives during the last year,
(ii) the total amount of money given as charity for the 10 years.
6. In an arithmetic progression, the 1st term is -10, the 15th term is 11 and the last
term is 41. Find the sum of all terms in the progression.
7. A parent deposits $1000 into a saving account for her daughter on each of her
birthday from her 10th to her 18th. The account pays an interest at 6% for each
complete year that the money is invested. How much money is in the account on
the day after 18th birthday.
8.* A geometric series has non-zero first term a and common ratio r, where 0 < r < 1.
Given that the sum of the first 8 terms of the series is half the sum to infinity,
find the value of r correct to 3 decimal places. Given also that the 17th term of
the series is 10, find a.

126 dkrbabajee@gmail.com
c
CHAPTER 7. SERIES

9.* The 1st term of an arithmetic progression is a and the common difference is d,
where d 6= 0.

(i) Write down expressions, in terms of a and d, for the 4th term and the 9th
term.
The 1st term, the 4th term and the 9th term of the arithmetic progression
are the three consecutive terms of a geometric progression.
(ii) Show that 9d = 2a.
(iii) Find the common ratio of the geometric progression.

10.* The sum of the first 70 terms of an arithmetic progression is 6387.5. The first,
third and seventh terms of the arithmetic progression are the 3 consecutive terms
of a geometric progression. Find

(i) the first term and common difference of the arithmetic progression,
(ii) the common ratio of the geometric progression.

127 dkrbabajee@gmail.com
c
C HAPTER 8

Differentiation 1

8.1 Objective
At the end of this chapter, students should be able to:

1. understand the idea of the gradient of a curve, and use the notations f ′ (x),
dy d2 y
f ′′ (x), and 2 ;
dx dx
2. use the derivative of xn (for any rational n), together with constant multiples,
sums, differences of functions;

3. use chain rule to find the derivative of composite functions;

4. apply differentiation to gradients, tangents and normals;

5. apply differentiation to increasing and decreasing functions;

6. locate stationary points (using second derivative test), and use information
about stationary points in sketching graphs;

7. apply differentiation to rates of change (including connected rates of change).

128 dkrbabajee@gmail.com
c
CHAPTER 8. DIFFERENTIATION 1

8.2 Gradient of a curve at a point

The gradient of a curve y = f (x) at a point


P (x, y) from first principles is given by

dy f (x + δx) − f (x)
= lim .
dx δx−→0 δx

Consider the curve y = f (x) = x2 .

dy f (x + δx) − f (x)
= lim
dx δx−→0 δx
(x + δx)2 − x2 x2 + 2x(δx) + (δx)2 − x2
= lim = lim
δx−→0 δx δx−→0 δx
= lim 2x + δx = 2x
δx−→0

8.3 Differentiation of xn together with constant multiples,


sums and differences
f (x) f ′ (x) f (x) f ′ (x)

c 0 ax + b a

ax2 + bx + c 2ax + b axn naxn−1


a na
c(ax + b)n nac(ax + b)n−1 −
xn xn+1
c nac √ 1
− x √
(ax + b)n (ax + b)n+1 2 x
√ a
ax + b √ ag(x) + bh(x) ag ′(x) + bh′ (x)
2 ax + b

We show that
1 a 1  na
= ax−n = −nax−n−1 = − n+1
dx xn dx x
and
1 √  1  1  1 1 −1 1 − 1 1 1 1
x = x2 = x2 = x 2 = 1 = √ .
dx dx 2 2 2 x2 2 x

129 dkrbabajee@gmail.com
c
CHAPTER 8. DIFFERENTIATION 1

Example 8.1 Differentiate w.r.t. x:


(i) x3 (ii) 3x4 (iii) 2x5 + 4x3 + 6x2
5 2
(iv) (v) 3x3 − 4x5 − (vi) (3x + 2)4
x6 x4
2 300 − 48x2 4
(vii) 3
(viii) (ix) √
(5 − x) 10x x
√ √ √
(x) 4 − 5x (xi) 2 + 4x x + 3x2 x

Solution
d 
(i) x3 = 3x3−1 = 3x2
dx
d 
(ii) 3x4 = 3(4) x4−1 = 12x3
dx
d 
(iii) 2x5 + 4x3 + 6x2 = 2(5)x5−1 + 4(3)x3−1 + 6(2)x2−1 = 10x4 + 12x2 + 12x
dx
 
d 5 −5(6) 30
(iv) 6
= 6+1 = − 7
dx x x x
 
d 2 −2(4) 8
(v) 3x − 4x − 4 = 3(3)x3−1 − 4(5)x4 − 4+1 = 9x2 − 20x4 + 5
3 5
dx x x x
!
d
(vi) (3x + 2)4 = 4(3)(3x + 2)4−1 = 12(3x + 2)3
dx
!
d 2 −2(3)(−1) 6
(vii) 3
= 3+1
=
dx (5 − x) (5 − x) (5 − x)4
   
d 300 − 48x2 1 30 24 −30 24 30 24
(viii) = − x = 1+1 − =− 2 −
dx 10x dx x 5 x 5 x 5
   
d 4 1  −1  1 1 3 2 2 2
(ix) √ = 4x 2 =4 − x− 2 −1 = −2x− 2 = − 3 = − 1 = − √
dx x dx 2 x2 x × x2 x x
!
d √ −5
(x) 4 − 5x = √
dx 2 4 − 5x
! !    
d √ 2
√ 1 3 5 3 3
−1 5 5
(xi) 4x x + 3x x = 4x 2 + 3x 2 = 4 x2 + 3 x 2 −1
dx dx 2 2
1 15 3 √ 15 √
= 6x 2 + x 2 = 6 x + x x
2 2

130 dkrbabajee@gmail.com
c
CHAPTER 8. DIFFERENTIATION 1

8.4 Chain Rule


dy df du
Let y = f (u(x)). The chain rule is given by = × = f ′ (u(x))u′ (x).
dx du dx
f (x) f ′ (x)

(u(x))n nu′ (x) u(x)n−1

1 −nu′ (x)
u(x)n u(x)n+1
p u′ (x)
u(x) p
2 u(x)

Example 8.2 Differentiate w.r.t. x:


√ 1
(i) (3x2 + 2)4 (ii) 4x3 + 5x2 + 2x + 1 (iii) √
x+x
Solution
d 
(i) (3x2 + 2)4 = 4(3x2 + 2)3 × 6x = 24x(3x2 + 2)3
dx
d √ 3 2
 12x2 + 10x + 2 6x2 + 5x + 1
(ii) 4x + 5x + 2x + 1 = √ =√
dx 2 4x3 + 5x2 + 2x + 1 4x3 + 5x2 + 2x + 1
    √
d 1 −1 1 −(1 + 2 x)
(iii) √ = √ × √ +1 = √ √
dx x+x ( x + x)2 2 x 2 x( x + x)2
Exercise 8.1. [Differentiation of simple functions and chain rule]
1. Differentiate w.r.t. x:
4
(i) x7 (ii) 3x8 (iii)
x3
18 9
(iv) √ (v) 4x7 − 2x5 + 8x3 − 4
(vi) (2x − 9)3
x x
√ √ 4
(vii) 12 + 4x (viii) 10 − 7x (ix)
(2x + 3)5
200x2 − 17x5 √ √ 12
(x) (xi) 3x + 2x x + 4x3 x (xii) √
4x8 x3
2. Differentiate w.r.t. x:
1
(i) (3x2 + 4)3 (ii) (6 − 2x3 )5 (iii)
(3x4
+ 2)3
√ 1 3
(iv) 3x3 + 2x (v) √ (vi) √ .
2x − 7x2 2 x+x

131 dkrbabajee@gmail.com
c
CHAPTER 8. DIFFERENTIATION 1

8.5 Tangents and Normal


A tangent to the curve is a line which touches the curve at only one point.
A line perpendicular to the tangent is called the normal.
dy
The gradient of the tangent at the point x = k is given by mT = .
dx x=k
1
Since mT × mN = −1, the gradient of the normal is given by mN = − .
mT
dy
If the tangent is the horizontal line y = c, then mT = = 0.
dx
dx
If the tangent is the vertical line x = c, then mN = = 0.
dy

Example 8.3 Find the equations of the tangent and normal

(i) to the curve y = x2 at the point where x = 2,


4
(ii) to the curve y = at the point where x = 3,
x2 − 2x
6
(iii) to the curve y = √ at the point where y = 3.
x

Solution

(i)

When x = 2, y = 22 = 4

dy dy 1
= 2x, mT = = 2(2) = 4, mN = −
dx dx x=2 4
Equation of tangent at (2, 4)
y−4
= 4 ⇒ y − 4 = 4x − 8 ⇒ y = 4x − 4
x−2
Equation of normal at (2, 4)
y−4 −1
= ⇒ 4y − 16 = −x + 2 ⇒ 4y + x = 18
x−2 4

This is illustrated in the diagram:

132 dkrbabajee@gmail.com
c
CHAPTER 8. DIFFERENTIATION 1

(ii)
4
When x = 3, y =
3
dy −4 −8(x − 1)
= 2 2
× 2x − 2 = 2
dx (x − 2x) (x − 2x)2

dy −8(3 − 1) 16 9
mT = = 2 2
= − , mN =
dx x=3 (3 − 2(3)) 9 16
 
4
Equation of tangent at 3,
3
4
y−3 −16
= ⇒ 9y − 12 = −16x + 48 ⇒ 9y + 16x = 60
x−3 9  
4
Equation of normal at 3,
3
4
y−3 9 64 17
= ⇒ 16y − = 9x − 27 ⇒ 16y = 9x − ⇒ 48y = 27x − 17
x−3 16 3 3

133 dkrbabajee@gmail.com
c
CHAPTER 8. DIFFERENTIATION 1

(iii)
6 √
When y = 3, √ = 3 ⇒ x = 2 ⇒ x = 22 = 4
x
1
dy −6 × 2 3
= 3 =− √
dx x 2 x x

dy −3 3 8
mT = = = − , mN =
dx x=4 4(2) 8 3
Equation of tangent at (4, 3)
y−3 −3
= ⇒ 8y − 24 = −3x + 12 ⇒ 8y + 3x = 36
x−4 8
Equation of normal at (4, 3)
y−3 8
= ⇒ 3y − 9 = 8x − 32 ⇒ 3y = 8x − 23
x−4 3

1
Example 8.4 The equation of a curve is y = 1 + .
x
(i) Show that the equation of the normal to the curve at the point A(1, 2)
is y = x + 1.
This normal meets the curve again at the point B.

(ii) Find the coordinates of B.

Solution
(i)

dy 1 dy
= − , mT = = −1, mN = 1
dx x dx x=1
Equation of normal at A (1, 2)
y−2
= 1⇒ y−2 =x−1⇒y = x+1
x−1

(ii)
1
y = x + 1, y =1+
x
1 1
x + 1 = 1 + ⇒ x = ⇒ x2 = 1 ⇒ x = ±1
x x
x-coordinate of B = −1, y = −1 + 1 = 0 ∴ B(−1, 0)

This is illustrated in the diagram:

134 dkrbabajee@gmail.com
c
CHAPTER 8. DIFFERENTIATION 1

Example 8.5 The curve y = x2 + 2x − 8 cuts the x-axis at P and Q.

(i) Find the coordinates of P and Q.

(ii) Find the equations of the tangents at P and Q.

(iii) The two tangents meet at R. Find the coordinates of R.

(iv) Find the acute angle between the two tangents.

Solution
(i)

y = x2 + 2x − 8 = 0 ⇒ (x + 4)(x − 2) = 0 ⇒ x = −4, or, 2


P=(-4,0) and Q=(2,0)

(ii)
dy
= 2x + 2
dx
dy
At P, = 2(−4) + 2 = −6 ⇒ m1 = −6
dx x=−4
equation of tangent at P(-4,0),
y−0
= −6 ⇒ y = −6x − 24
x − −4
dy
At Q, = 2(2) + 2 = 6 ⇒ m2 = 6
dx x=2
equation of tangent at Q(2,0),
y−0
= 6 ⇒ y = 6x − 12
x−2

135 dkrbabajee@gmail.com
c
CHAPTER 8. DIFFERENTIATION 1

(iii)

y = 6x − 12 .....(1), y = −6x − 24 .......(2)


6x − 12 = −6x − 24 ⇒ 12x = −12 ⇒ x = −1 ⇒ y = 6(−1) − 12 = −18
R=(-1,-18)

(iv)

m1 = −6, m2 = 6


−1 m1 − m2

θ = tan
1 + m1 m2
 
−6 − 6
−1 12
θ = tan = tan−1
= 18.90
1 + (−6 × 6) 35

This is illustrated in the diagram:

1
Example 8.6 A curve has equation y = and P (1, 1) is a point on the
x
curve.

(i) Find the equation of the tangent to the curve at P.

(ii) Find the acute angle that this tangent makes with the x-axis.

136 dkrbabajee@gmail.com
c
CHAPTER 8. DIFFERENTIATION 1

Solution
(i)
dy 1
=− 2
dx x
dy
At P, = −1 ⇒ mT = −1
dx x=1
equation of tangent at P(1,1),
y−1
= −1 ⇒ y = −x + 2
x−1

(ii)

tan θ = |mT | = 1 ⇒ θ = tan−1 (1) = 450

Exercise 8.2. [Tangent and normal]


1. Find the equation of the tangent and the normal to the following curves:

(i) y = x2 − 8x + 7 at the point where x = 3,


3 2
(ii) y = + 2 at the point where x = −2,
x x
(iii) y = 2x2 − 6x + 3 at the point where the tangent is parallel to the x-axis.
x2
2. The equation of a curve is y = 3 + 4x − .
2
(i) Show that the equation of the normal to the curve at the point A(2, 9) is
x + 2y = 20. This normal meets the curve again at the point B.
(ii) Find the coordinates of B.

137 dkrbabajee@gmail.com
c
CHAPTER 8. DIFFERENTIATION 1

3. The curve y = x2 − x − 6 cuts the x-axis at P and Q.


(i) Find the coordinates of P and Q.
(ii) Find the equations of the tangents at P and Q.
(iii) The two tangents meet at R. Find the coordinates of R.
(iv) Find the acute angle between the two tangents.
3
4. A curve has equation y = .
4x + 3
(i) Find the equation of the tangent to the curve at the point where y = −3.
(ii) Find the acute angle that this tangent makes with the x-axis.

8.6 Increasing and decreasing functions


dy dy
For an increasing function, > 0 whereas < 0 for a decreasing function.
dx dx
Example 8.7 Find the set of values of x for which y = x2 − 2x + 4 is an
increasing function.
Solution
dy
for an increasing function, >0
dx
dy
= 2x − 2 > 0 ⇒ 2x > 2 ⇒ x > 1
dx
Example 8.8 Find the set of values of x for which y = x3 − 6x2 + 9x − 2
is a decreasing function.
Solution
dy
For a decreasing function, <0
dx
dy
= 3x2 − 12x + 9 < 0 ÷ 3 ⇒ x2 − 4x + 3 < 0 ⇒ (x − 1)(x − 3) < 0
dx
Critical values of x are 1, 3
(x − 1) (x − 3) (x − 1)(x − 3)
x<1 −ve −ve +ve (> 0) ×
1 < x < 3 +ve −ve −ve (< 0) X
x>3 +ve +ve +ve (> 0) ×
∴ 1<x<3

138 dkrbabajee@gmail.com
c
CHAPTER 8. DIFFERENTIATION 1

This is illustrated in the diagram:

x3 x2
Example 8.9 Show that y = + + x + 1 is an increasing function
3 2
for all real values of x.

Solution
dy
we show that> 0 for all real values of x
dx
 2
dy 2 1 3
=x +x+1= x+ + >0
dx 2 4
Exercise 8.3. [Increasing and decreasing functions]

1. Find the set of values of x for which y = 3x2 − 6x + 7 is an increasing function.

2. Find the set of values of x for which y = x2 + 3x − 1 is a decreasing function.

3. Find the set of values of x for which y = 2x3 − 9x2 + 12x + 1 is an increasing
function.

4. Find the set of values of x for which y = 2x3 − 3x2 − 12x + 1 is a decreasing
function.

5. Show that y = x3 − 3x2 + 6x + 3 is an increasing function for all real values of x.

6. Show that y = 5 − 4x − x3 is a decreasing function for all real values of x.

139 dkrbabajee@gmail.com
c
CHAPTER 8. DIFFERENTIATION 1

8.7 Stationary Points


A stationary point can be a maximum or minimum point and at stationary points,
dy
the gradient of the tangent at that points is 0, i.e. = 0.
dx

8.7.1 Determining the nature of stationary points


The nature of the stationary point (a, b) of a curve can be determined by the second
derivative test:

d2 y
1 If 2 > 0, (a, b) is a minimum point.
dx x=a

d2 y
2 If 2 < 0, (a, b) is a maximum point.
dx x=a

d2 y
3 If 2 = 0, (a, b) is a point of inflexion.
dx x=a

Example 8.10 Find the stationary point/s of the following curve and
determine its or their nature. Also sketch the curve.
(i) y = x2 − 4x + 5 (ii) y = 12x − 2x2
3 2
(iii) y = 2x + 9x + 12x + 5

Solution

(i)
dy
y = x2 − 4x + 5, = 2x − 4
dx
dy
for stationary point = 0 ⇒ 2x − 4 = 0 ⇒ x = 2
dx
y = 22 − 4(2) + 5 = 1 Stationary point (2, 1)
d2 y
= 2 > 0 ⇒ (2, 1) is minimum.
dx2

To sketch the curve, we need the point where it cuts the y−axis (e.g. x = 0).
y = 5. So we have (0, 5). We draw a decreasing curve from (5, 0) to (2, 1). The
curve is decreasing for x < 2, reaches its minimum at x = 2 and is increasing
for x > 2.

140 dkrbabajee@gmail.com
c
CHAPTER 8. DIFFERENTIATION 1

(ii)
dy
y = 12x − 2x2 , = 12 − 4x
dx
dy
for stationary point = 0 ⇒ 12 − 4x = 0 ⇒ x = 3
dx
y = 12(3) − 2(3)2 = 18 Stationary point (3, 18)
d2 y
= −4 < 0 ⇒ (3, 18) is maximum.
dx2

To sketch the curve, we need the point where it cuts the y−axis (e.g. x = 0).
y = 0. So we have (0, 0). We draw a increasing curve from (0, 0) to (3, 18). The
curve is increasing for x < 3, reaches its maximum at x = 3 and is decreasing
for x > 3.

141 dkrbabajee@gmail.com
c
CHAPTER 8. DIFFERENTIATION 1

(iii)
dy
y = 2x3 + 9x2 + 12x + 5, = 6x2 + 18x + 12
dx
dy  
for stationary points = 0 ⇒ 6x2 + 18x + 12 = 0 ÷ 6
dx
x2 + 3x + 2 = 0 ⇒ (x + 1)(x + 2) = 0 ⇒ x = −1 or x = −2
y = 2(−1)3 + 9(−1)2 + 12(−1) + 5 = 0 or y = 2(−2)3 + 9(−2)2 + 12(−2) + 5 = 1
Stationary points (−1, 0) and (−2, 1)
d2 y
= 12x + 18
dx2
d2 y
= 12(−1) + 18 = 6 > 0 ⇒ (−1, 0) is minimum
dx2 x=−1

d2 y
= 12(−2) + 18 = −6 < 0 ⇒ (−2, 1) is maximum
dx2 x=−2

To sketch the curve, we need the point where it cuts the y−axis (e.g. x = 0).
y = 5. So we have (0, 5). We draw a decreasing curve from (0, 5) to (−1, 0).
The curve is decreasing for x > −1, reaches its minimum at x = −1 and is
increasing for x < −1 until it reaches its maximum at x = −2. Then the curve
is decreasing for x < −2.

142 dkrbabajee@gmail.com
c
CHAPTER 8. DIFFERENTIATION 1

Example 8.11 A solid cuboid has a base of length and width in the ratio
3 : 2. Given that the width is x cm, the height is y cm and the volume is
144 cm3 .

(i) Express y in terms of x.


480
(ii) Show that the total surface area of the cuboid, A = 3x2 + .
x
If x can vary,

(iii) Find the stationary value of A and determine whether it is


maximum or minimum.

Solution

(i)

Volume = Length × Width × Height


3 144 × 2 96
144 = x × x × y ⇒ y = 2
= 2.
2 3x x

(ii)
   
3 3
A= 2× x × x + 2 × x × y + (2 × x × y)
2 2
     
3 3 96 96
= 2× x×x + 2× x× 2 + 2×x× 2
2 2 x x
288 192 480
= 3x2 + + = 3x2 +
x x x

(iii)
dA 480
= 6x − 2
dx x
dA
for stationary points =0
dx
480 480 √
6x − 2 = 0 ⇒ 6x = 2 ⇒ 6x3 = 480 ⇒ x3 = 80 ⇒ x = 80 = 4.31
3

x x
480
A = 3(4.31)2 + = 167
4.31
d2 A 960 d2 A 960
= 6 + ⇒ = 6 + = 18 > 0 ⇒ A is minimum
dx2 x3 dx2 x=4.31 80

143 dkrbabajee@gmail.com
c
CHAPTER 8. DIFFERENTIATION 1

Example 8.12 A cylinder open at one end has total surface area of
96π cm 2 . The radius is r cm and the height is h cm.

(i) Express h in terms of r.


πr 3
(ii) Show that the volume of cylinder, V = 48πr − .
2
If r can vary,

(iii) Find the stationary value of V and determine whether it is


maximum or minimum.
Solution
(i)

Total Surface Area = πr 2 + 2πrh


" #
96 − r 2 48 r
πr 2 + 2πrh = 96π ÷ π ⇒ r 2 + 2rh = 96 ⇒ h = = −
2r r 2

(ii)

V = πr 2 h
 
2 48 r πr 3
= πr − = 48πr −
r 2 2

(iii)
dV 3πr 2
= 48π −
dr 2
dV
for stationary points =0
dr
3πr 2 3πr 2 √
48π − = 0 ⇒ 48π = ⇒ r 2 = 32 ⇒ r = 32 = 5.66
2 2
π(5.66)3
V = 48π(5.66) − = 569
2

d2 V d2 V
= −3πr ⇒ = −3π(5.66) = −53.3 < 0 ⇒ V is maximum
dr 2 dr 2
r=5.66

Exercise 8.4. [Stationary points, Minimum and maximum values]


1. Find the coordinates of the stationary points of each of the following curves and
determine their nature:
(i) y = 2x3 − 3x2 (ii) y = (x − 2) 3
√ − 12x
1 2 x
(iii) y = x + (iv) y = +
x x 2
144 dkrbabajee@gmail.com
c
CHAPTER 8. DIFFERENTIATION 1

2. A curve has equation y = x3 + ax + b, where a and b are constants.


(i) Given that the curve has a maximum point at (−1, 4), find the value of a
and of b.
(ii) Find the other stationary point of the curve.
3. A solid cuboid has a base of length and width in the ratio 2 : 1. Given that the
width is x cm, the height is y cm and the volume is 576 cm3 .
(i) Express y in terms of x.
1728
(ii) Show that the total surface area of the cuboid, A = 4x2 + .
x
If x can vary,
(iii) Find the stationary value of A and determine whether it is maximum or
minimum.
4. A solid cuboid has a base of length and width in the ratio 2 : 1. Given that the
width is x cm, the height is y cm and the total surface area is 48 cm2 .
(i) Express y in terms of x.
4
(ii) Show that the volume of the cuboid, V = 16x − x3 .
3
If x can vary,
(iii) Find the stationary value of V and determine whether it is maximum or
minimum.
5. A cylinder open at one end has total surface area 4.32π cm 2 . The radius is r cm
and the height is h cm.
(i) Express h in terms of r.
(ii) Show that the volume of cylinder, V = 2.16πr − 0.5πr 3.
If r can vary,
(iii) Find the stationary value of V and determine whether it is maximum or
minimum.
6. A cylinder open at one end has volume 125π cm 3 . The radius is r cm and the
height is h cm.
(i) Express h in terms of r.
250π
(ii) Show that the total surface area of cylinder, A = πr 2 + .
r
If r can vary,
(iii) Find the stationary value of A and determine whether it is maximum or
minimum.

145 dkrbabajee@gmail.com
c
CHAPTER 8. DIFFERENTIATION 1

8.8 Rate of Change


Rate is always associated with time. If we say that the rate of a change of a variable
dx
x, then we are referring to , i.e. how the variable x is changing with time. If
dt
dx dx
> 0, x increases with time. If < 0, x decreases with time.
dt dt
Example 8.13 If x = 4(1 − 2t)5 . Find the rate of change in x at t = 1.

Solution
dx
= 20(1 − 2t)4 (−2) = −40(1 − 2t)4 .
dt
dx
= −40(1 − 2(1))4 = −40.
dt t=1
The rate of decrease of x is 40 units per second.

8.9 Connected Variables


dy dy dx
If y is a function of x, then by chain rule = × .
dt dx dt
Example 8.14 The volume of water in a pool is 4x3 + 6x2 + 3x cm3 when
the depth of water is x cm. Water enters the pool through a tap at a
constant rate of 1800 cm3 /s. Find the rate at which the level of water is
rising when the depth of water in the pool is 30 cm.

Solution

V = 4x3 + 6x2 + 3x

dV 2 dV
= 12x + 12x + 3, = 12(30)2 + 12(30) + 3 = 11163
dx dx x=30
dV dV dV dx dx dx 1800
= 1800, = × ⇒ 1800 = 11163 × ⇒ = = 0.161 cm/s
dt dt dx dt dt dt 11163
Example 8.15 Liquid is poured into a container at a rate of 24 cm3 /s.
1
The volume of liquid in the container is V cm3 and V = (h2 + 4h) where
2
h is the height of liquid in the container. Find the rate at which h is
increasing when V = 16.

146 dkrbabajee@gmail.com
c
CHAPTER 8. DIFFERENTIATION 1

Solution
1
V = (h2 + 4h) = 16 ⇒ h2 + 4h − 32 = 0 ⇒ (h + 8)(h − 4) = 0 ⇒ h = 4 (h > 0)
2
dV 1 dV
= (2h + 4) = h + 2, =4+2=6
dh 2 dh h=4
dV dV dV dx dh dh 24
= 24, = × ⇒ 24 = 6 × ⇒ = = 4 cm/s
dt dt dh dt dt dt 6
Exercise 8.5. [Rate of change, Connected variables]

1. The radius of a sphere increases at a constant rate of 0.1 cm/s. At what rate is
the volume increasing when

(i) the radius is 6 cm,


(ii) its volume is 36π cm3 .

2. An ice cube has a volume which decreases at a constant rate of 3 cm3 /s. Find
the rate of change of the length of its side when

(i) its length is 8 cm,


(ii) volume of the cube is 1.728 cm3 .

3. When the depth of water in a vessel is x cm, its volume is V = 4x3 + 6x2 + 3x cm3 .
Water leaks from a hole in the vessel at a constant rate of 1800 cm3 /s. Find the
rate at which the water is rising when the depth of water in the vessel is 15 cm.

8.10 Miscellaneous Exercises


Example 8.16 The equation of a curve is given by y = x3 − 3x2 − 9x + 2.

(i) Find the coordinates of the stationary points on the curve and
determine their nature.

(ii) Show that the normal to  at the point where x = 1 intersects


 the curve
109
the y-axis at the point 0, − .
12
(iii) Find the rate of change of y at the point where x = 1 if x increases
at a rate of 0.1 unit per second.

147 dkrbabajee@gmail.com
c
CHAPTER 8. DIFFERENTIATION 1

Solution
(i)
dy
y = x3 − 3x2 − 9x + 2, = 3x2 − 6x − 9
dx
dy  
for stationary points = 0 ⇒ 3x2 − 6x − 9 = 0 ÷ 3
dx
2
x − 2x − 3 = 0 ⇒ (x + 1)(x − 3) = 0 ⇒ x = −1 or x = 3
y = (−1)3 − 3(−1)2 − 9(−1) + 2 = 7 or y = (3)3 − 3(3)2 − 9(3) + 2 = −25
Stationary points (−1, 7) and (3, −25)
d2 y
= 6x − 6
dx2
d2 y
= 6(−1) − 6 = −12 < 0 ⇒ (−1, 7) is maximum
dx2 x=−1

d2 y
= 6(3) − 6 = 12 > 0 ⇒ (3, −25) is minimum
dx2
x=3

(ii)

When x = 1, y = (1)3 − 3(1)2 − 9(1) + 2 = −9



dy 2 dy 1
= 3x − 6x − 9, mT = = −12, mN =
dx dx x=1 12
Equation of normal at (1, −9)
y+9 1
= ⇒ 12y + 108 = x − 1 ⇒ 12y = x − 109
x−1 12
normal cuts the y-axis, x = 0
 
109 109
12y = −109 ⇒ y = − ⇒ 0, −
12 12

(iii)

dy dx dy dy dx
= −12, = 0.1, = × = −12 × 0.1 = −1.2
dx x=1 dt dt dx dt
rate of decrease of y is 1.2 unit/s

Miscellaneous Exercise 8.
1. The equation of a curve is given by y = x3 − 3x2 + 1.

(i) Find the coordinates of the stationary points on the curve and determine
their nature.

148 dkrbabajee@gmail.com
c
CHAPTER 8. DIFFERENTIATION 1

(ii) Find the equation of the tangent and of the normal to the curve at the point
P where x = 1.
The tangent at P intersects the y−axis at Q and the normal at P intersects
the y-axis at R.
(iii) Find the area of the triangle PQR.

2. A rectangular block of ice has dimensions x cm by x cm by 10x.

(i) Find expressions for its total surface area, A cm2 and its volume, V cm3 in
terms of x.
(ii) Given that the ice is melting at a constant rate of 30 cm3 /s. Find the rate
of change of the total surface area when the volume is 10000 cm3 .

3. The equation of a curve is given by y = (3x + p)3 − 9x, where p is a constant.

(i) The curve has a maximum point at x = −1, find the value of p.
(ii) Find the coordinates of the other stationary point on the curve.

4. A solid cuboid has a square base of length x. Given that the height is y cm and
the volume is 27 cm3 .

(i) Express y in terms of x.


108
(ii) Show that the total surface area of the cuboid, A = 2x2 + .
x
If x can vary,
(iii) Find the stationary value of A and determine whether it is maximum or
minimum.

5. (i) Find the range of values of p for which f (x) = x3 + 3x2 + px + 3 is an


increasing function.
(ii) Find the range of values of q for which f (x) = x3 +qx2 +3x−1 is a decreasing
function.
b
6. A curve has equation y = ax + , where a and b are constants. The equation of
x
the normal to the curve at the point (1, 0) is x + 2y = 1.

(i) Find the values of a and b.


(ii) Using these values of a and b, find the coordinates of the other point where
the normal meets the curve again.

7. In the diagram, S is the point (0, 4) and T is the point (5, 0). The point Q with
coordinates (x, y) lies on ST. The points P and R lie on the x−axis and y-axis

149 dkrbabajee@gmail.com
c
CHAPTER 8. DIFFERENTIATION 1

respectively and OPQR is a rectangle.


(i) Find the equation of the line ST.

(ii) Show that the area of OPQR is


4
A = 4x − x2 .
5
(iii) Given that x can vary, find the
perimeter of OPQR for which A has
a stationary value.
8.* The equation of a curve C is given by y = x2 − x + 2. A line L has equation
y = x + p, where p is a constant.

(i) Find the value of p for which L is a tangent to C and find the point of
intersection of L and C.
(ii) Find the value of p for which L is a normal to C and find the points of
intersection of L and C.

9.* A door in the shape of a rectangle surmounted by a semi-circle whose diameter


is equal to the width of the rectangle. If the perimeter of the door is 5 m, the
radius of the semi-circle is r m and the height of the rectangle is h m,

(i) Express h in terms of r.


(ii) Show that the area of the door, A cm2 is given by
π 2
A = 5r − 2r 2 − r
2
5
(iii) Show that A has a maximum value when r = .
4+π
b = 900 , AC = x cm. Given that x can vary but the
10.* In a triangle ABC with B AC
total length of AC and AB is fixed and is 20 cm.

(i) Show that the area of the triangle, A cm2 is given by

x2
A = 10x −
2

(ii) Find the stationary value of A and determine whether it is minimum or


maximum.

150 dkrbabajee@gmail.com
c
C HAPTER 9

Integration 1

9.1 Objective
At the end of this chapter, students should be able to:
1. understand integration as the reverse process of differentiation, and integrate
(ax + b)n (for any rational n except −1), together with constant multiples, sums
and differences;
2. find the constant of integration and equation of a curve;
3. evaluate definite integrals (including simple cases of ’improper’ integrals) ;
4. use definite integration to find the area enclosed by a curve, the x-axis and the
lines parallel to the y-axis (vertical elements);
5. use definite integration to find the area enclosed by a curve, the y-axis and the
lines parallel to the x-axis (horizontal elements);
6. use definite integration to find the area enclosed by two curves;
7. use definite integration to find a volume of revolution formed when the region
enclosed by the curve, the x-axis and the lines parallel to the y-axis is rotated
about the x-axis;
8. use definite integration to find a volume of revolution formed when the region
enclosed by the curve, the y-axis and the lines parallel to the x-axis is rotated
about the y-axis;
9. use definite integration to find the volume formed when the region enclosed by
two curves is rotated about one of the axes.

9.2 Integration as a reverse process of differentiation


dy
If y is a function of x, finding is known as differentiation. To retrieve y, we must
Z dx
dy dy
integrate , that is y = dx.
dx dx

151 dkrbabajee@gmail.com
c
CHAPTER 9. INTEGRATION 1

dy dy
If y = x + 1, then = 1. If y = x + 2, then = 1.
dx dx
Z
dy dy
So, if = 1, y = dx = x + c, where c is known as the constant of integration.
dx dx
xn+1 dy n + 1 n+1−1
If y = , = x = xn
n + 1 dx n+1
Z
n xn+1
x dx = +c
n+1

(ax + b)n+1 dy a(n + 1)


If y = , = (ax + b)n+1−1 = (ax + b)n
a(n + 1) dx a(n + 1)
Z
(ax + b)n+1
(ax + b)n dx = +c
a(n + 1)
Z Z
1 (ax + b)−n+1 (ax + b)−(n−1)
dx = (ax + b)−n dx = +c= +c
(ax + b)n a(−n + 1) −a(n − 1)
Z
1 1
n
dx = − +c
(ax + b) a(n − 1)(ax + b)n−1

The table below shows the integral of some functions:


Z
f (x) f (x) dx

a ax + c

axn+1
axn +c
n+1
p(ax + b)n+1
p(ax + b)n +c
a(n + 1)
a a
− +c
xn (n − 1)xn−1
p p
− +c
(ax + b)n (n − 1)(ax + b)n−1

152 dkrbabajee@gmail.com
c
CHAPTER 9. INTEGRATION 1

Example 9.1 Integrate w.r.t. x:


(i) x3 (ii) 3x4 (iii) 3x2 − 2x + 4

x3 2x4
(iv) 2x3 + 6x2 − 2 (v) − 4x5 − (vi) 3x4 + 4x3 + 5x2 + 6x + 2
2 7
(vii) (3x + 2)(x − 4) (viii) (2x − 7)(4x2 + 8) (ix) (3x + 4)5
3

4 3 4x5 + x4 + 5
(x) (xi) (xii) .
x5 (3 − 2x)4 5x3

Solution
Z
x3+1 x4
(i) x3 dx = +c= +c
3+1 4
Z
4 3x5
(ii) 3x dx = +c
5
Z
3x3 2x2
(iii) 3x2 − 2x + 4 dx = − + 4x + c = x3 − x2 + 4x + c
3 2
Z
2x4 6x3 x4
(iv) 2x3 + 6x2 − 2 dx = + − 2x + c = + 2x3 − 2x + c
4 3 2
Z
x3 2x4 x4 4x6 2x5 x4 2x6 2x5
(v) − 4x5 − dx = − − +c= − − +c
2 7 2×4 6 5×7 8 3 35
Z
3x5 4x4 5x3 6x2
(vi) 3x4 + 4x3 + 5x2 + 6x + 2 dx = + + + + 2x + c
5 4 3 2
3x5 5x3
= + x4 + + 3x2 + 2x + c
5 3
Z Z
3x3 10x2
(vii) (3x + 2)(x − 4) dx = 3x2 − 10x − 8 dx = − − 8x + c = x3 − 5x2 − 8x + c
3 2
Z Z
3 2 4x6 28x3
(viii) (2x − 7)(4x + 8) dx = 8x5 + 16x3 − 28x2 − 56 dx = + 4x4 − − 56x + c
3 3
Z
(3x + 4)5+1 (3x + 4)6
(ix) (3x + 4)5 dx = +c= +c
3 × (5 + 1) 18
Z
4 4 1
(x) 5
dx = − 5−1
+c=− 4 +c
x (5 − 1)x x
Z
3 3 1
(xi) dx = − + c = +c
(3 − 2x)4 (4 − 1)(−2)(3 − 2x)4−1 2(3 − 2x)3

153 dkrbabajee@gmail.com
c
CHAPTER 9. INTEGRATION 1

Z Z
4x5 + x4 + 5 4x2 x 1 4x3 x2 1
(xii) 3
dx = + + 3
dx = + − +c
5x 5 5 x 5 × 3 5 × 2 (3 − 1)x3−1
4x3 x2 1
= + − 2 + c.
15 10 2x

9.3 Finding constant of integration and equation of


a curve
Example 9.2 Find the equation of the curve for the following:
dy
(i) given that = 2x2 − x + 3 and passing through the point (1, −1),
dx
dy 3
(ii) given that = − 5x + 1 and passing through the point
dx (2x + 1)2
(1, 0),
d2 y dy
(iii) given that = 4x + 3 and = 2 and y = 1 when x = 1.
dx2 dx

Solution

(i)
dy
= 2x2 − x + 3
dx Z
y= 2x2 − x + 3 dx
2x3 x2
y= − + 3x + c
3 2
2(1)3 (1)2 25
x = 1, y = −1 ⇒ −1 = − + 3(1) + c ⇒ c = −
3 2 6
2x3 x2 25
y= − + 3x −
3 2 6

154 dkrbabajee@gmail.com
c
CHAPTER 9. INTEGRATION 1

(ii)
dy 3
= − 5x + 1
dx (2x + 1)2
Z
3
y= − 5x + 1 dx
(2x + 1)2
3 5x2 3 5x2
y=− − + x + c ⇒ y = − − +x+c
(2 − 1)(2)(2x + 1)2−1 2 2(2x + 1) 2
3 5(1)2
x = 1, y = 0 ⇒ 0 = − − +1+c⇒c=2
2(2(1) + 1) 2
3 5x2
y=− − +x+2
2(2x + 1) 2

(iii)
d2 y
= 4x + 3
dx2 Z
dy
= 4x + 3 dx
dx
dy 4x2 dy
= + 3x + c ⇒ = 2x2 + 3x + c
dx 2 dx
dy
x = 1, = 2 ⇒ 2 = 2(1)2 + 3(1) + c ⇒ c = −3
dx
dy
= 2x2 + 3x − 3
dx Z
y= 2x2 + 3x − 3 dx
2x3 3x2 2x3 3x2
y= + − 3x + c ⇒ y = + − 3x + c
3 2 3 2
2(1)3 3(1)2 11
x = 1, y = 1 ⇒ 1 = + − 3(1) + c ⇒ c =
3 2 6
3 2
2x 3x 11
y= + − 3x +
3 2 6

155 dkrbabajee@gmail.com
c
CHAPTER 9. INTEGRATION 1

Exercise 9.1. [Integration as a reverse process of differentiation]

1. Integrate w.r.t. x:
(i) x4 (ii) 4x3 (iii) x2 + 2x − 5

x3 5x6
(iv) 6x3 + 3x2 − 2 − 5x4 −
(v) (vi) 2x5 + 5x3 + 3x2 + 4x + 3
3 7
(vii) (3x − 4)(x + 2) (viii) (7x + 2)(8x2 − 4) (ix) (4x + 3)5
3

5 2 2x6 + x5 + 4
(x) (xi) (xii) .
x4 (2 − 3x)3 4x4
2. Find the equation of the curve for the following:
dy
(i) given that = x + 4 and passing through the point (2, −2),
dx
dy
(ii) given that = 3x2 + 2x − 3 and passing through the point (1, 1),
dx
dy 4
(iii) given that = + 2x − 1 and passing through the point (1, 0),
dx (x + 1)2
dy √
(iv) given that = x + 1 and passing through the point (0, −3),
dx
dy1
(v) given that =√and passing through the point (0, 1),
dx
2x + 1
d2 y dy
(vi) given that 2 = 3x + 4 and = 1 and y = 2 when x = −1.
dx dx
dy
3. A curve is such that = 2x(2 − 3x). Given that the curve passes through the
dx
points (1, 5) and (−1, k), find the value of k.
dy
4. A curve is such that = ax + 1, where a is a constant. Given that the curve
dx
passes through the points (0, −2) and (−2, 0), find the equation of the curve.
dy k
5. A curve is such that = 2 , where k is a constant. Given that the curve passes
dx x
through thepoints (1, 3) and (2, 2), find the equation of the curve.
dy dy
6. Given that is directly proportional to (4x − 1)2 and that = 6 and y = 0
dx dx
1
when x = 0, find the value of y when x = .
2

156 dkrbabajee@gmail.com
c
CHAPTER 9. INTEGRATION 1

dy dy
7. Given that is inversely proportional to x3 and that = −6 and y = 0 when
dx dx
x = −1, express y in terms of x.
dy
8.* A curve is such that = 2x − 2. The line 4x + y = 2 is a tangent to the curve at
dx
the point P.

(i) Find the coordinates of P.


(ii) Find the equation of the curve.
dy
9.* A curve is such that = 3x2 − 6. The line 3y = x − 16 is a normal to the curve
dx
at the point P. Given the x−coordinate of P is positive, find

(i) coordinates of P,
(ii) the equation of the curve.
d2 y
10.* A curve is such that = 12x − 14. The curve has a minimum point at P (2, −4)
dx2
and a maximum point at Q.

(i) the equation of the curve,


(ii) coordinates of Q.

9.4 Definite Integrals


Definite
Z b integrals have a value and they are evaluated by
h ib
f (x) dx = F (x) + c = (F (b) + c) − (F (a) + c) = F (b) − F (a).
a a
In definite integrals, the constant of integration is eliminated.

Example
Z 2 9.3 EvaluateZthe following integrals: Z −1
3
4
(i) x dx (ii) 3x dx (iii) 3x2 + 2x dx
1 2 −2
Z 3 3 Z 2 Z 2
x −4 4

(iv) dx (v) (2x + 3) dx (vi) 5x − 1 dx
2 3x2 1 1
Z −1
1
(vii) 3
dx
−∞ x

157 dkrbabajee@gmail.com
c
CHAPTER 9. INTEGRATION 1

Solution

(i)
Z 2  2
x2 22 12 3
x dx = = − =
1 2 1 2 2 2

(ii)
Z 3  3
4 3x5 3(3)5 3(2)5 633
3x dx = = − =
2 5 2 5 5 5

(iii)
Z −1 h i−1
3x2 + 2x dx = x3 + x2 = ((−1)3 + (−1)2 ) − ((−2)3 + (−2)2 ) = 4
−2 −2

(iv)
Z 3 Z 3  2 3  2   2 
x3 − 4 x 4 x 4 3 4 2 4 11
dx = − 2 dx = + = + − + =
2 3x2 2 3 3x 6 3x 2 6 3(3) 6 3(2) 18

(v)
Z 2  2
4 (2x + 3)5 (2(2) + 3)5 (2(1) + 3)5 6841
(2x + 3) dx = = − =
1 10 1 10 10 5

(vi)
Z 2  2
√ 2 3 2 3 2 3 38
5x − 1 dx = (5x − 1) 2 = (5(2) − 1) 2 − (5(1) − 1) 2 =
1 15 1 15 15 15

(vii) This is an example of an improper integral:


Z −1  −1
1 1 1 1 1
3
dx = − 2 =− + 2
=−
−∞ x 2x −∞ 2 2(∞) 2

1
Note that −→ 0 as x −→ −∞.
2x2
Some properties of integral are:
Z a
1. f (x) dx = 0
a

158 dkrbabajee@gmail.com
c
CHAPTER 9. INTEGRATION 1

Z a Z b
2. f (x) dx = − f (x) dx
b a
Z b Z b Z b
3. mf (x) + ng(x) dx = m f (x) dx + n g(x) dx
a a a
Z c Z b Z c
4. f (x) dx = f (x) dx + f (x) dx
a a b

Exercise 9.2. [Definite integrals]

1. Evaluate
Z 3 the following integrals:
Z 2 Z 2
(i) x dx (ii) 5x6 dx (iii) 4x3 − 3x2 − 2x dx
1 1 1
Z 5 2 Z 2 Z 1
x −4
(iv) dx (v) (2x + 1)(3x + 5) dx (vi) (3x − 2)4 dx
2 3x2 1 0
Z 7 Z 1 Z 0
1 1 √
(vii) (4x − 1) dx (viii)
3 dx (ix) 1 − 3x dx
1
0 (1 + 2x)2 −1
Z 2 2
Z ∞ Z 9
2 1 1
(x) √ dx (xi) dx (xii) √ dx
−1 x+2 2 x4 0 x
Z 2
2. Find the value of k for which 6x + k dx = 12.
1
Z b
3. Find the value of b for which x2 dx = 9.
0
Z 3 √ 14
4. Find the value of a for which x + 3 dx = .
a 3
Z 4 Z 2
5. Given that f (x) dx = 8 and f (x) dx = 3, evaluate
Z 4 2 Z 4 1

(i) 4f (x) − 3x dx (ii) f (x) dx


2 1
Z 5 Z 5
6. Given that f (x) dx = 6 and g(x) dx = 4, evaluate
Z 5 1 Z 5 1

(i) f (x) dx (ii) 3f (x) + 2g(x) dx


1 1

159 dkrbabajee@gmail.com
c
CHAPTER 9. INTEGRATION 1

9.5 Area under a curve


9.5.1 Area enclosed by a curve, the x-axis and the lines parallel
to the y-axis (vertical elements)

Area enclosed by the curve y = f (x), the x-axis and the lines x = a and x = b is given
by
Z b Z b
A= y dx = f (x) dx.
a a

Example 9.4 Find the area enclosed by the curve y = x2 + 3, the x-axis
and between the lines x = 2 and x = 4.

Solution
Z 4 Z 4  4  3   3 
2 x3 4 2 74
A= y dx = x + 3 dx = + 3x = + 3(4) − + 3(2) =
2 2 3 2 3 3 3

Note: When the value of the integral is negative, it means that the area is below the
x-axis and we take the magnitude.

160 dkrbabajee@gmail.com
c
CHAPTER 9. INTEGRATION 1

9.5.2 Area enclosed by a curve, the y-axis and the lines parallel
to the x-axis (horizontal elements)

Area enclosed by the curve y = f (x), the y-axis and the lines y = a and y = b is given
by
Z b Z b
A= x dy = f −1 (y) dy.
a a

Example 9.5 Find the area enclosed by the curve y = x2 + 3, the x-axis
and between the lines y = 3 and y = 4.

Solution
p
y = f (x) = x2 + 3 ⇒ x = f −1 (y) = y − 3
Z 4 Z 4 p  4    
2 3 2 3 2 3 2
A= x dy = y − 3 dy = (y − 3) 2 = (4 − 3) 2 − (3 − 3) 2 =
3 3 3 3 3 3 3

161 dkrbabajee@gmail.com
c
CHAPTER 9. INTEGRATION 1

9.5.3 Area enclosed by two curves

Area enclosed by the curves y1 = f1 (x) and y2 = f2 (x) and the x-axis is given by
Z b Z b
A= y1 − y2 dx = f1 (x) − f2 (x) dx,
a a

where a and b are the x-coordinates of the points of intersection of the two curves.
Note: We subtract the curve y2 which is below from the curve y1 which is above.

Example 9.6 Find the area enclosed by the curves y = x2 + x − 2 and


y = 2x2 − x − 5.

Solution
y1 = x2 + x − 2, y2 = 2x2 − x − 5 ⇒ 2x2 − x − 5 = x2 + x − 2
x2 − 2x − 3 = 0 ⇒ (x + 1)(x − 3) = 0 ⇒ x = −1 or 3
Z 3 Z 3 Z 3
2 2
A= y1 − y2 dx = (x + x − 2) − (2x − x − 5) dx = −x2 + 2x + 3 dx
−1 −1 −1
 3
3  3   
x 2 3 2 (−1)3 2 34
= − + x + 3x = − + 3 + 3(3) − + (−1) + 3(−1) =
3 −1 3 3 3

162 dkrbabajee@gmail.com
c
CHAPTER 9. INTEGRATION 1

9.5.4 Area divided into two parts


Such cases occur when the area consists of one region above the x-axis and the other
one below the x-axis.
Example 9.7 Find the area enclosed by the curve y = x2 − 4x + 3, x-axis
and the lines x = 2 and x = 5.

Solution
A = A1 + A2 ,
where the curve meets the x-axis, x2 − 4x + 3 = 0 ⇒ (x − 1)(x − 3) = 0 ⇒ x = 1 or 3
Z 3  3 3
2 x 2
A1 = x − 4x + 3 dx = − 2x + 3x
2 3 2
 3   3 
3 2 2 2 2
= − 2(3) + 3(3) − + 2(2) + 3(2) = − , negative since the area is below x-axis
3 3 3
Z 5  3 5
x
A2 = x2 − 4x + 3 dx = − 2x2 + 3x
3
 33   3 3

5 2 3 2 20
= − 2(5) + 3(5) − + 2(3) + 3(3) =
3 3 3
2 20 22
A = |A1 | + A2 = + =
3 3 3
Exercise 9.3. [Application of integration: area]
1. Using vertical components, find the area with the given boundaries:
(i) the curve y = x2 + 1, the x−axis and the lines x = 1 and x = 2,

(ii) the curve y = x, the x−axis and the lines x = 1 and x = 9,
(iii) the curve y = x2 + 2, the x−axis and the lines x = −1 and x = 1,
(iv) the curve y = x2 + 2x, the x−axis and the lines x = 0 and x = 3,

163 dkrbabajee@gmail.com
c
CHAPTER 9. INTEGRATION 1

1
(v) the curve y = 3x 3 , the x−axis and the lines x = 1 and x = 8.
2. Evaluate:
Z 3 Z Z
15 4
2 1 2
(i) 2y + 3y dy (ii) √ dy (iii) (5 − y)− 3 dy
2 7 2y − 5 −3

3. Using horizontal components, find the area with the given boundaries:
(i) the curve y = x2 + 1, the y−axis and the lines y = 2 and y = 5,

(ii) the curve y = x, the y−axis and the lines y = 1 and y = 2,
(iii) the curve y 2 = 4 − x and the y−axis.
4. Find the area bounded by:
(i) the curve y = x2 and the line y = 3x,
(ii) the curves y = 2x2 + x − 1 and y = x2 − x + 2,
(iii) the curves y = x2 and y = 2 − x2 ,
(iv) the curves y = x2 − 3x + 2 and y = 5x − 4 − x2 .
5. Find the area enclosed by the curve y = x2 − 6x + 8, x-axis and the lines x = 3
and x = 5.
6. A curve has equation y = x(x2 − 1).
(i) Find the coordinates of the points where the curve cuts the coordinates axes.
Sketch the curve.
(ii) Find the total area enclosed by the curve and the x−axis.
7. The diagram shows the curve y = 3 + 2x − x2 and the tangent to the curve at the
point P(2, 3). The tangent meets the x−axis at Q.
(i) Find the equation of the tangent.

(ii) Find the coordinates of Q.

(iii) Calculate the area of the shaded


region.
8.* The diagram shows the curve y = (x − 1)(x − 2) and the normal to the curve at
the point P(3, 2). The normal meets the curve again at Q.
(i) Find the equation of the normal.

(ii) Find the coordinates of Q.

(iii) Calculate the area of the shaded


region.

164 dkrbabajee@gmail.com
c
CHAPTER 9. INTEGRATION 1

9.6 Volume of solids of revolution


9.6.1 Rotation about the x-axis of the region enclosed by the
curve, the x-axis and the lines parallel to the y-axis
Volume of the region enclosed by the curve y = f (x), the x-axis and the lines x = a
and x = b, rotated through 3600 about the x-axis is given by
Z b Z b
2
V =π y dx = π f (x)2 dx
a a

Example 9.8 Find the volume of the solid of revolution obtained by


rotating the region bounded by the curve y = x2 + 1, between the lines
x = 0, x = 3 and the x-axis through 3600 about the x-axis.

Solution
Z 3 Z 3
2
Volume = π y dx = π (x2 + 1)2 dx
0 0
Z 3  5 3
4 2 x 2x3
=π x + 2x + 1 dx = π + +x
0 5 3 0
 5  
3 2(3)3 348
=π + +3 −0 = π
5 3 5

9.6.2 Rotation about the y-axis of the region enclosed by the


curve, the y-axis and the lines parallel to the x-axis
Volume of the region enclosed by the curve y = f (x), the y-axis and the lines y = a
and y = b, rotated through 3600 about the y-axis is given by
Z b Z b
2
V =π x dy = π (f −1 (y))2 dy
a a

Example 9.9 Find the volume of the solid of revolution obtained by


rotating the region bounded by the curve y = 9x2 + 1, between the lines
y = 1, y = 10 and the y-axis through 3600 about the y-axis.

165 dkrbabajee@gmail.com
c
CHAPTER 9. INTEGRATION 1

Solution
y−1
y = 9x2 + 1 ⇒ x2 =
9
Z 10 Z 10
2 y−1
Volume = π x dy = π dy
1 1 9
Z 10  10   
(y − 1)2 (y − 1)2 (10 − 1)2 9
=π dy = π =π −0 = π
1 18 18 1 18 2

9.6.3 Volume formed when the region enclosed by two curves


is rotated about one of the axes
Volume enclosed by the curves y1 = f1 (x) and y2 = f2 (x) and rotated through 3600
about the x-axis is given by
Z b Z b
2 2
V = y1 − y2 dx = (f1 (x))2 − (f2 (x))2 dx,
a a

where a and b are the x-coordinates of the points of intersection of the two curves.
Volume enclosed by the curves y1 = f1 (x) and y2 = f2 (x) and rotated through 3600
about the y-axis is given by
Z d Z b
2 2
V = x1 − x2 dx = (f1 −1 (y))2 − (f2 −1 (y))2 dy,
c a

where c and d are the y-coordinates of the points of intersection of the two curves.

166 dkrbabajee@gmail.com
c
CHAPTER 9. INTEGRATION 1

20
Example 9.10 The diagram shows part of the curve y = .
x

The points A and B with coordinates (2, 10) and (5, 4) lie on the curve.
Find

(i) the equation of line AB,

(ii) the volume obtained when the shaded region is rotated through 4
right-angles about the x-axis,

(iii) the volume obtained when the shaded region is rotated through 4
right-angles about the y-axis.

Solution

(i)
4 − 10
mAB = = −2
5−2
Equation of AB at B(5, 4),
y−4
= −2 ⇒ y − 4 = −2x + 10 ⇒ y = −2x + 14
x−5

(ii)
Z 5  2 Z 5
2 20 400
Volume = π (−2x + 14) − dx = π 4x2 − 56x + 196 − dx
2 x 2 x2
 3
5
4x 400
=π − 28x2 + 196x +
3 x 2
 3
  
4(5) 2 400 4(2)3 2 400
=π − 28(5) + 196(5) + − − 28(2) + 196(2) +
3 5 3 2
 
1580 1472
=π − = 36π
3 3

167 dkrbabajee@gmail.com
c
CHAPTER 9. INTEGRATION 1

(iii)
14 − y 20 20
y = −2x + 14 ⇒ x = , y= ⇒x=
2 x y
Z 10  2  2 Z 10
14 − y 20 (14 − y)2 400
Volume = π − dy = π − 2 dx
4 2 y 4 4 y
 3
10
(14 − y) 400
=π − +
12 y 4
     
(14 − 10)3 400 (14 − 4)3 400 104 50
=π − + − − + =π − = 18π
12 10 12 4 3 3

Exercise 9.4. [Application of integration: volume]

1. Find the volume of the solid of revolution obtained by rotating the region bounded
by the curve y = x2 + 2, between the lines x = 0, x = 1 and the x-axis through
3600 about the x-axis.

2. Find the volume of the solid of revolution obtained by rotating the region bounded
1
by the curve y = , between the lines x = 1, x = 2 and the x-axis through 3600
x
about the x-axis.

3. Find the volume of the solid of revolution obtained by rotating the region bounded
by the curve y = x2 , between the lines x = −2, x = 2 and the x-axis through 3600
about the x-axis.

4. Find the volume of the solid of revolution obtained by rotating the region bounded
by the curve y = x(1 − x) and the x-axis through 3600 about the x-axis.

5. Find the volume of the solid of revolution obtained by rotating the region bounded
by the curve y = 2x2 + 1, between the lines y = 1, y = 3 and the y-axis through
3600 about the y-axis.

6. Find the volume of the solid of revolution obtained by rotating the region enclosed
by the curve y 2 = x and the line y = x about the x−axis.
4
7. The diagram shows part of the curve y = . The points A and B with coordinates
x
(1, 4) and (2, 2) lie on the curve.

168 dkrbabajee@gmail.com
c
CHAPTER 9. INTEGRATION 1

(i) the equation of line AB,

(ii) the volume obtained when the


shaded region is rotated through 4
right-angles about the x-axis,

(iii) the volume obtained when the


shaded region is rotated through 4
right-angles about the y-axis.

9.7 Miscellaneous Exercises


Example 9.11 The diagram shows part of the curve which is such that
dy
= 2x − 3.
dx

The line y = x − 5 is a normal to the curve at the point P and the curve
meets the normal again at Q. Find

(i) the coordinates of P,

(ii) the equation of the curve,

(iii) the coordinates of Q,

(iv) the area of the shaded region.

Solution
dy
y = x − 5 ⇒ mN = 1 ⇒ = −1 ⇒ 2x − 3 = −1
(i) dx
⇒ 2x = 2 ⇒ x = 1 ⇒ y = −4 ⇒ P (1, −4)
Z
(ii) y= 2x − 3 dx = x2 − 3x + c ⇒ −4 = 12 − 3(1) + c ⇒ c = −2 ⇒ y = x2 − 3x − 2

y = x − 5, y = x2 − 3x − 2 ⇒ x2 − 3x − 2 = x − 5 ⇒ x2 − 4x + 3 = 0
(iii)
(x − 1)(x − 3) = 0 ⇒ x = 3 ⇒ y = −2 ⇒ Q(3, −2)

169 dkrbabajee@gmail.com
c
CHAPTER 9. INTEGRATION 1

Z 3 Z 3
2
Area = (x − 5) − (x − 3x − 2) dx = −x2 + 4x − 3 dx
1 1
(iv)  3
3  3   3 
x 2 3 2 1 2 4
= − + 2x − 3x = − + 2(3) − 3(3) − − + 2(1) − 3(1) =
3 1 3 3 3

Miscellaneous Exercise 9.

1. Integrate w.r.t. x:
1 √ √ 3x − 4
(i) x3 − 3 (ii) x( x − 1) (iii) √
x x
√ 2 4
(iv) (3x + x) (v) (2x + 5)5 (vi)
(7x + 2)2
 3
2 1 √ √
(vii) (viii) √ (ix) x x + x2 x
1 − 4x 2x + 3
2. Evaluate
Z 8 the following integrals:
Z 4 Z 2
1 √ 2 1
(i) 1 dx (ii) x + √ dx (iii) 4x3 + 2 + 2 dx
1 4x 3 1 x 1 2x
Z 2 Z −2 Z 1

(iv) x(x2 − 1) dx (v) (x − 2)(x − 3) dx (vi) x2 (2 + 3 x) dx
0 −3 0
Z 4 Z 5 Z 4
2x + 1 √
(vii) √ dx (viii) (2x − 1)3 dx (ix) 3x + 4 dx
1 x 0 −1

dy
3. A curve is such that = 3x2 + 12x + 9. The curve has a minimum point at
dx
(−1, −3).

(i) Find the equation of the curve.


(ii) Find the other stationary point of the curve and determine its nature.
(iii) Sketch the curve.

4. The diagram shows part of the curve y = x2 + 2 and the line x = k, where k is a
positive constant.

Given that the area of region A and of


region B are in ratio 5 : 4, find the value
of k.


5. A curve has equation y = a x, where a is a constant.

(i) Find the value of a if the area of the region enclosed by the curve, x−axis
and the lines x = 1 and x = 4 is 14.

170 dkrbabajee@gmail.com
c
CHAPTER 9. INTEGRATION 1

(ii) Find the value of a if the volume generated is 30π when the region enclosed
by the curve, x−axis and the lines x = 1 and x = 4 is rotated 3600 around
the x−axis.

6.* The diagram shows part of the curve y = x and the line x = h, where h is a
positive constant.
(i) Find the value of h if the area of the
4
shaded region is .
3
(ii) Find the value of h if the volume
π
generated is when the shaded
6
region is rotated 3600 around the
x−axis.

7.* The equation of a curve is y = 6x − x2 − 8. Find

(i) the coordinates of the stationary point on the curve,


(ii) the volume obtained when the region bounded by the curve and the x-axis
is rotated through 3600 about the x-axis.

8.* The diagram shows part of the curve y = x2 and the line 2x + y = 3 intersecting
at A.
Find

(i) the coordinates of A,

(ii) the area of the shaded region,

(iii) the volume obtained when the


shaded region is rotated through
3600 about the x-axis.

9.* The diagram shows part of the curve y = x − x. The curve has a maximum
point at M and meets the x-axis at O and A.
(i) Find the coordinates of A and M.

(ii) Find the area of the shaded region.

(ii) Find the volume obtained when the


shaded region is rotated through 3600
about the x-axis.

171 dkrbabajee@gmail.com
c
CHAPTER 9. INTEGRATION 1

dy
10.* The diagram shows part of the curve which is such that = 4 − 2x. The line
dx
x+2y = 13 is a normal to the curve at the point P and the curve meets the normal
again at Q.
Find

(i) the coordinates of P,

(ii) the equation of the curve,

(iii) the coordinates of Q,

(iv) the area of the shaded region.

172 dkrbabajee@gmail.com
c
C HAPTER 10

Algebra

10.1 Objective
At the end of this chapter, students should be able to:
1. understand the meaning of |x|, and use relations such as |a| = |b| ⇔ a2 = b2 and
|x − a| < b ⇔ a − b < x < a + b or the graphical method in solving equations and
inequalities involving modulus;
2. divide a polynomial, of degree not exceeding 4, by a linear or quadratic polynomial,
and identify the quotient and remainder (which may be zero);
3. use the factor theorem and the remainder theorem, e.g. to find factors, solve
polynomial equations or evaluate unknown coefficients;
4. recall an appropriate form for expressing rational functions in partial fractions,
and carry out the decomposition, in cases where the denominator is no more
complicated than

(ax + b)(cx + d)(ex + f ),


(ax + b)(cx + d)2 ,
(ax + b)(x2 + c2 ),

and where the degree of the numerator does not exceed that of the denominator;
5. use the expansion of (1 + x)n , where n is a rational or negative and |x| < 1;
6. use partial fractions in binomial expansion.

10.2 Modulus Function


10.2.1 Graph of Modulus Function
The modulus, denoted by |x| is given by
(x if x≥0
|x| =
−x if x<0

173 dkrbabajee@gmail.com
c
CHAPTER 10. ALGEBRA

The graph of y = |x| is a V-shaped with


the vertex (0, 0). The vertex is obtained
by finding the value of x for which the
expression inside the modulus is zero e.g.
x = 0 and so y = |0| = 0. We note that the
graph of y = |x| can be obtained by
reflecting the negative part of the graph
of y = x in the x-axis.

10.2.2 Solving equations involving modulus


To solve equations involving modulus we use

|a| = |b| ⇔ a2 = b2 .

Example 10.1 Solve |x + 3| = 4.

Solution
Method 1
Squaring on both sides

(x + 3)2 = 42
x2 + 6x + 9 = 16
x2 + 6x − 7 = 0
(x + 7)(x − 1) = 0
x = −7 or 1.

Method 2

By graphical method, the solutions are


the points of the graph y = |x + 3| and
y = 4. The graph of y = |x + 3| is V-shaped
with vertex V (−3, 0). The intersection of
the line y = −(x + 3) and y = 4 gives the
first solution −(x + 3) = 4 ⇒ x = −7. The
intersection of the line y = x + 3 and y = 4
gives the second solution
x + 3 = 4 ⇒ x = 1.

Example 10.2 Solve |x| = |2x − 1|.

174 dkrbabajee@gmail.com
c
CHAPTER 10. ALGEBRA

Solution
Method 1
Squaring on both sides

x2 = (2x − 1)2
x2 = 4x2 − 4x + 1
3x2 − 4x + 1 = 0
(3x − 1)(x − 1) = 0
1
x = or 1.
3
Method 2

By graphical method, the solutions are


the points of the graph y = |x| and
y = |2x − 1|. The graph of y = |x| is
V-shaped with vertex V (0, 0). The graph
y = |2x
of   − 1| is V-shaped with vertex
1
V , 0 . The intersection of the line
2
y = −(2x − 1) and y = x gives the first
1
solution −(2x − 1) = x ⇒ x = . The
3
intersection of the line y = x and
y = 2x − 1 gives the second solution
x = 2x − 1 ⇒ x = 1.

10.2.3 Inequalities involving modulus


1 1
If 3 > 2, then −3 < −2 and < . So one must reverse the inequality sign when
3 2
multiplying by −1 or using the reciprocals.

Example 10.3 Solve |x| < a, a > 0.

Solution
Squaring on both sides,

x2 < a2
x2 − a2 < 0
(x − a)(x + a) < 0
−a < x < a.

Remark 10.1 1. If |x| > a, then x < −a or x > a.

175 dkrbabajee@gmail.com
c
CHAPTER 10. ALGEBRA

2. If |x − b| < a, then −a + b < x < a + b.

Example 10.4 Solve 2|x + 1| > |x − 1|.

Solution
Squaring on both sides,

4(x + 1)2 > (x − 1)2


4x2 + 8x + 4 > x2 − 2x + 1
3x2 + 10x + 3 > 0
(3x + 1)(x + 1) > 0
1
x < −3 or x > − .
3

Example 10.5 Solve |x| − 1 < |x − 2|.

Solution
Double squaring will remove the modulus
sign and it leads to a complicated
algebra. We use the graphical method.
The graph of y = |x − 2| is V-shaped with
vertex V (2, 0) and the graph of y = |x| − 1
is V-shaped with vertex occurring when
x = 0 (inside the modulus) and so the
vertex is V (0, −1). The two graph meet at
the point satisfying y = −(x − 2) and
3
y = x − 1 or −(x − 2) = x − 1 ⇒ x = .
2
From the diagram, it can be observed
that the graph y = |x| − 1 is below that of
3
y = |x − 2| when x < .
2
Exercise 10.1. [Modulus]

1. Solve using the algebraic method:


(i) |x + 1| = 1 (ii) |x + 3| = |x + 4| (iii) 2|x − 1| = |x + 1|

(iv) 3|2x + 1| = |x − 2| (v) |3x + 1| = 2|x − 3|

2. Sketch the following curve and find the vertex point:


(i) y = |x + 2| (ii) y = |3x − 5| (iii) y = |x − 2| + 1

(iv) y = |5x − 2| − 2 (v) y = 5 − 2|x + 1| (vi) y = 5|2x − 1| − 2x

176 dkrbabajee@gmail.com
c
CHAPTER 10. ALGEBRA

3. Solve using the graphical method:


(i) |x − 2| = 1 (ii) |x + 1| = 2x − 3 (iii) |x − 1| = |x + 2|

(iv) |2x + 1| = |x − 2| (v) 1 − |x| = |x| (vi) |4x − 3| = 2 − |x + 1|

4. Solve using the algebraic method:


(i) |x + 1| < |x − 2| (ii) 2|x| < |1 − x| (iii) |x + 2| > 3|x + 1|

(iv) |x| < |3x + 4| (v) |x| < |2x − 1|

5. Solve using the graphical method:


(i) |x + 1| < |x + 2| (ii) |x + 2| > 2x (iii) |x| + 1 < |x + 2|

(iv) |3x + 1| > 2 − |x − 1| (v) |2x + 3| > |3x + 2| (vi) |4x − 3| > |3x − 4|

10.3 Polynomials
10.3.1 Representation of Polynomials
A polynomial of degree n is given by

f (x) = an xn + an−1 xn−1 + ... + a1 x + a0 ,

where aj are coefficients of xj , j = 0..n.

1. If n = 0, the polynomial is a constant or independent of x.

2. If n = 1, the polynomial is linear.

3. If n = 2, the polynomial is quadratic.

4. If n = 3, the polynomial is cubic.

5. If n = 4, the polynomial is quartic.

10.3.2 Multiplication of Polynomials


Example 10.6 Expand (x + 1)(x + 2).

177 dkrbabajee@gmail.com
c
CHAPTER 10. ALGEBRA

Solution
Method 1
Using binomial expansion,

(x + 1)(x + 2) = (x × x) + (x × 2) + (1 × x) + (1 × 2)
= x2 + 2x + x + 2
= x2 + 3x + 2.

Method 2
Using the conventional method of multiplication,

x 1
x 2 ×
2x 2
x2 x 0 +
x2 +3x +2

10.3.3 Division of Polynomials


Dividend = Quotient × Divisor + Remainder

Example 10.7 Find the quotient and remainder when x2 − 2x + 2 is


divided by x + 1.

Solution
Method 1
Using long division,
  5
x2 − 2x + 2 : x + 1 = x − 3 +
x+1
− x2 − x
− 3x + 2
3x + 3
5

Method 2

x2 − 2x + 2 ≡ (x + 1)(px + q) + r (10.1)
Comparing coefficient of x2 in (10.1), we have 1 = p.
Comparing constant terms in (10.1), we have 2 = q + r.
Comparing coefficient of x in (10.1), we have −2 = p + q ⇒ q = −2 − p = −2 − 1 = −3
and r = 2 − q = 2 + 3 = 5. In both cases, the quotient is x − 3 and the remainder is 5.

178 dkrbabajee@gmail.com
c
CHAPTER 10. ALGEBRA

10.3.4 Remainder Theorem


When a polynomial f (x) is divided by x − a, then the remainder = f (a).
In the example 10.7, f (x) = x2 − 2x + 2 and f (−1) = (−1)2 − 2(−1) + 2 = 5.

10.3.5 Factor Theorem


When a polynomial f (x) is exactly divisible by x − a (or vanished at x = a), then the
remainder = f (a) = 0.

Example 10.8 Let f (x) = 2x3 + ax2 + bx − 20. Given that f (x) is divided
by (x − 1), the remainder is −42 and f (x) is exactly divisible by x + 1.

(i) find the value of a and of b.

(ii) with these values of a and b, solve the equation f (x) = 0.

Solution

(i) Procedure: By reminder theorem, f (1) = −42 and by factor theorem, f (−1) = 0.
Obtain two equations involving a and b and solve them simultaneously.

f (1) = 2 + a + b − 20 = −42,
a + b = −24 (10.2)

and

f (−1) = −2 + a − b − 20 = 0,
a − b = 22 (10.3)

(10.2) + (10.3) yields

2a = −2,
a = −1.
From (10.2),
b = −24 − a = −24 + 1 = −23.

(ii) Procedure: We know that x + 1 is a factor of f (x), divide f (x) by x + 1 to obtain


a quadratic factor and factorise the latter.

179 dkrbabajee@gmail.com
c
CHAPTER 10. ALGEBRA

By long division,
 
2x3 − x2 − 23x − 20 : x + 1 = 2x2 − 3x − 20
− 2x3 − 2x2
− 3x2 − 23x
3x2 + 3x
− 20x − 20
20x + 20
0

Or

2x3 − x2 − 23x − 20 ≡ (x + 1)(px2 + qx + r) (10.4)

Comparing coefficient of x3 in (10.4), we have 2 = p.


Comparing constant terms in (10.4), we have −20 = r.
Comparing coefficient of x2 in (10.4), we have −1 = p + q ⇒ q = −1 − p = −3.
So

2x3 + ax2 + bx − 20 = (x + 1)(2x2 − 3x − 20)


= (x + 1)(2x + 5)(x − 4)

5
So, x = −1, − or 4.
2

Example 10.9 Suppose x2 +x+1 is a factor of p(x) = x4 −x3 −9x2 −10x+a,

(i) find the value of a and the other quadratic factor.

(ii) solve the equation p(x) < 0, showing all your workings clearly.

Solution

(i) Procedure: Obtain the quadratic factor by long division



4 3 2
  8 + 1a
x − x − 9x − 10x + a : x2 + x + 1 = x2 − 2x − 8 +
x2 + x + 1
− x4 − x3 − x2
− 2x3 − 10x2 − 10x
2x3 + 2x2 + 2x
− 8x2 − 8x +a
8x2 + 8x +8

8 + 1a

180 dkrbabajee@gmail.com
c
CHAPTER 10. ALGEBRA

or by comparing coefficients:

x4 − x3 − 9x2 − 10x + a ≡ (x2 + x + 1)(px2 + qx + r) (10.5)

Comparing coefficient of x4 in (10.5), we have 1 = p.


Comparing constant terms in (10.5), we have a = r.
Comparing coefficient of x3 in (10.5), we have −1 = q + 1 ⇒ q = −2.
Comparing coefficient of x2 in (10.5), we have
−9 = r + q + 1 ⇒ r = −9 − q − 1 = −9 + 2 − 1 = −8.
So a = r = −8 and the quadratic factor is x2 − 2x − 8.

(ii) By completing to the square, show that x2 + x + 1 is always positive.


 2  2
2 1 1
x +x+1= x+ +1−
2 2
 2
1 3
= x+ + .
2 4
| {z } |{z}
+ve +ve

x4 − x3 − 9x2 − 10x − 8 < 0,


(x2 + x + 1)(x2 − 2x − 8) < 0, ÷ (x2 + x + 1)
x2 − 2x − 8 < 0,
(x − 4)(x + 2) < 0,
−2 < x < 4.

Exercise 10.2. [Polynomials]

1. Expand
(i) (x − 2)(x − 3) (ii) (2x + 5)(3x − 2) (iii) x(x + 1)(x − 2)

(iv) (x − 1)(x − 3)(2x + 1) (v) (x + 1)2 (x + 3) (vi) (2x − 5)2 (3x + 1)

2. Find the quotient Q and remainder r for the following dividends D and divisor
d:
(i) D = x2 − 3x + 2, d = x + 2
(ii) D = 2x2 − 5x + 1, d = x − 1
(iii) D = x3 − 6x2 − 5x − 1, d = x + 3
(iv) D = x3 − 2x + 4, d = 2x − 1
(v) D = x4 − 3x3 + 5x − 2, d = x + 1
(vi) D = x4 − x3 − 2x2 + 2x + 4, d = x2 − x − 6

181 dkrbabajee@gmail.com
c
CHAPTER 10. ALGEBRA

3. Use the factor theorem to solve the following equations:


(i) x3 + 3x2 + 2x = 0 (ii) x3 = 2x2 + 5x − 6
(iii) 2x3 − 7x2 + 4x + 4 = 0 (iv) 12x3 + 37x2 + 2x − 3 = 0
(v) x3 = 3x2 + 13x + 6 (vi) x3 + 4x2 + 4x + 3 = 0

4. Let f (x) = 2x3 + ax2 + bx+ 20. Given that f (x) is divided by (x+ 1), the remainder
is 42 and f (x) is exactly divisible by x − 1.

(i) find the value of a and of b.


(ii) with these values of a and b, solve the equation f (x) = 0.

5. Let f (x) = 2x3 −11x2 +ax+b. Given that f (x) is divided by (x−2), the remainder
is −6 and f (x) is exactly divisible by 2x − 1.

(i) find the value of a and of b.


(ii) with these values of a and b, solve the equation f (x) = 0.

6. Let f (x) = x3 + ax + b. Given that f (x) has (x + 1) and (x + 3) as two of its linear
factors,

(i) find the value of a and of b.


(ii) with these values of a and b, find the third factor.

7. Let f (x) = x3 + x2 + ax + b. Given that f (x) is exactly divisible by x2 − 1.

(i) find the value of a and of b.


(ii) with these values of a and b, express f (x) as a product of its linear factors.

8. Let f (x) = x3 + ax2 + bx + 6. f (x) leaves a remainder of 4 when divided by (x − 1)


and a remainder is −20 when divided by x + 2.

(i) find the value of a and of b.


(ii) with these values of a and b, solve the equation f (x) = 0.

9.* Suppose x2 + 2x + 2 is a factor of p(x) = x4 + 4x3 − 2x2 − 12x + a,

(i) find the value of a and the other quadratic factor.


(ii) solve the equation p(x) > 0, showing all your workings clearly.

10.* Given that p(x) = x4 + ax2 + 4 is exactly divisible by x2 − 4,

(i) find the value of a,


(ii) with this value of a , solve the equation p(x) = 0.

182 dkrbabajee@gmail.com
c
CHAPTER 10. ALGEBRA

10.4 Partial Fractions (P3 only)


1 1
Example 10.10 Simplify − .
x−1 x+1

Solution
1 1 (x + 1) − (x − 1)
− =
x−1 x+1 (x − 1)(x + 1)
2
=
(x − 1)(x + 1)

In this example, we have combined two fractions (partial) into a single fraction. Now,
we study how to decompose into partial fractions. First, we differentiate between
proper and improper fractions.
A proper fraction is such that the degree of the polynomial into the numerator is less
3
than that of the polynomial in the denominator, e.g. .
x+1
x
Otherwise the fraction is improper e.g. .
x+1

10.4.1 Type 1: Denominators with only distinct linear factors


(Proper Fractions)
A
For a linear term ax + b we get a contribution of .
ax + b
2
Example 10.11 Express into partial fractions.
(x − 1)(x + 1)

Solution
Method 1 (Comparing coefficients)
2 A B
= +
(x − 1)(x + 1) x−1 x+1
A(x + 1) + B(x − 1)
= .
(x − 1)(x + 1)

2 = A(x + 1) + B(x − 1) (10.6)


Set x = 1 in (10.6), we have 2 = 2A ⇒ A = 1.
Set x = −1 in (10.6), we have 2 = −2B ⇒ B = −1.

183 dkrbabajee@gmail.com
c
CHAPTER 10. ALGEBRA

Method 2: Cover-up Rule


2 A B
= + ,
(x − 1)(x + 1) x−1 x+1

2 2

A= = = 1,
(x − 1)(x + 1) 2
x=1
2 2

B= = = −1.
(x − 1)(x + 1) −2
x=−1

In both cases,
2 1 1
= − .
(x − 1)(x + 1) x−1 x+1

10.4.2 Type 2: Denominators with repeated linear factors (Proper


Fractions)
A B
For a repeated linear term, such as (ax+b)2 , we get a contribution of +
ax + b (ax + b)2

1
Example 10.12 Express into partial fractions.
(x + 1)(x − 1)2

Solution
1 A B C
= + +
(x + 1)(x − 1)2 x + 1 (x − 1) (x − 1)2
A(x − 1)2 + B(x − 1)(x + 1) + C(x + 1)
= .
(x + 1)(x − 1)2

1 = A(x − 1)2 + B(x − 1)(x + 1) + C(x + 1) (10.7)


1
Set x = 1 in (10.7), we have 1 = 2C ⇒ C = .
2
1
Set x = −1 in (10.7), we have 1 = 4A ⇒ A = .
4
2 1
Comparing coefficient of x in (10.7), we have 0 = A + B ⇒ B = −A = − .
4
To find B, we can also use differentiation. Differentiating (10.7) yields

0 = 2A(x − 1) + B(x − 1) + B(x + 1) + C (10.8)

184 dkrbabajee@gmail.com
c
CHAPTER 10. ALGEBRA

C 1
Put x = 1 in (10.8) yields 0 = 2B + C ⇒ B = − =− .
2 4
Finally,
1 1 1 1
2
= − + .
(x + 1)(x − 1) 4(x + 1) 4(x − 1) 2(x − 1)2

10.4.3 Type 3: Denominators with a quadratic factor (Proper


Fractions)
Ax + B
For a quadratic term ax2 + bx + c , we get a contribution of
ax2 + bx + c
5x + 1
Example 10.13 Express into partial fractions.
(x − 1)(x2 + 1)

Solution
5x + 1 A Bx + C
= +
(x − 1)(x2 + 1) x − 1 (x2 + 1)
A(x2 + 1) + (Bx + C)(x − 1)
= .
(x − 1)(x2 + 1)

5x + 1 = A(x2 + 1) + (Bx + C)(x − 1) (10.9)


Set x = 1 in (10.9), we have 6 = 2A ⇒ A = 3.
Comparing coefficient of x2 in (10.9), we have 0 = A + B ⇒ B = −A = −3.
Comparing constants in (10.9), we have 1 = A − C ⇒ C = A − 1 = 3 − 1 = 2.
To find B and C , we can also use differentiation. Differentiating (10.9) yields

5 = 2Ax + B(x − 1) + Bx + C (10.10)

Differentiating (10.10) yields

0 = 2A + B + B (10.11)

From (10.11), we get B = −A = −3 and put x = 0 in (10.10) yields


5 = −B + C ⇒ C = 5 + B = 5 − 3 = 2.
Finally,
5x + 1 3 −3x + 2
2
= + 2 .
(x − 1)(x + 1) x − 1 (x + 1)

185 dkrbabajee@gmail.com
c
CHAPTER 10. ALGEBRA

10.4.4 Improper Fractions


Example 10.14 Express the following into partial fractions:
x2 + 3 x3 + 2 x3
(i) (ii) 2 (iii)
(x + 1)(x − 3) x (3x − 1) (x − 1)(2x2 + 1)

Solution
(i) This is an improper fraction of Type 1:
Method 1
We make the improper fraction become proper using long division.
  2x + 6
x2 + 3 : x2 − 2x − 3 = 1 + 2
x − 2x − 3
− x2 + 2x + 3
2x + 6

2x + 6
is proper and is of Type 1.
(x + 1)(x − 3)

2x + 6 A B
= + ,
(x + 1)(x − 3) x+1 x−3

2x + 6 2(−1) + 6

A= = = −1,
(x + 1)(x − 3) −1 − 3
x=−1

2x + 6 2(3) + 6

B= = = 3.
(x + 1)(x − 3) 3+1
x=3

Method 2
x2 + 3 Q R
=P + + .
(x + 1)(x − 3) x+1 x−3
P (x + 1)(x − 3) + Q(x − 3) + R(x + 1)
= .
(x + 1)(x − 3)

x2 + 3 = P (x + 1)(x − 3) + Q(x − 3) + R(x + 1) (10.12)


Set x = 3 in (10.12), we have 12 = 4R ⇒ R = 3.
Set x = −1 in (10.12), we have 4 = −4Q ⇒ Q = −1.
Comparing coefficient of x2 in (10.12), we have P = 1.
Finally, in both cases,

x2 + 3 1 3
=1− + .
(x + 1)(x − 3) x+1 x−3

186 dkrbabajee@gmail.com
c
CHAPTER 10. ALGEBRA

(ii) This is an improper fraction of Type 2:

x3 + 2 Q R S
2
=P+ + + 2.
x (3x − 1) 3x − 1 x x
2 2
P x (3x − 1) + Qx + Rx(3x − 1) + S(3x − 1)
= .
x2 (3x − 1)

x3 + 2 = P x2 (3x − 1) + Qx2 + Rx(3x − 1) + S(3x − 1) (10.13)


1
Comparing coefficient of x3 in (10.13), we have 3P = 1 ⇒ P = .
3
1 55 Q 55
Set x = in (10.13), we have = ⇒Q= .
3 27 9 3
Set x = 0 in (10.13), we have S = −2.
Comparing coefficient of x2 in (10.13), we have
1 55
0 = −P + Q + 3R ⇒ 0 = − + + 3R ⇒ 0 = 18 + 3R ⇒ R = −6.
3 3
Finally,

x3 + 2 1 55 6 2
= + − − .
x2 (3x − 1) 3 3(3x − 1) x x2

(iii) This is an improper fraction of Type 3:

x3 Q Rx + S
2
=P + + 2 .
(x − 1)(2x + 1) x − 1 2x + 1
P (x − 1)(2x2 + 1) + Q(2x2 + 1) + (x − 1)(Rx + S)
= .
(x − 1)(2x2 + 1)

x3 = P (x − 1)(2x2 + 1) + Q(2x2 + 1) + (x − 1)(Rx + S) (10.14)


1
Comparing coefficient of x3 in (10.14), we have 2P = 1 ⇒ P = .
2
1
Set x = 1 in (10.14), we have 3Q = 1 ⇒ Q = .
3
1 1 1
Set x = 0 in (10.14), we have 0 = −P + Q − S ⇒ 0 = − + − S ⇒ S = − .
2 3 6
Comparing coefficient of x2 in (10.14),
2 1
we have 0 = −2P + 2Q + R ⇒ 0 = −1 + + R ⇒ R = .
3 3
Finally,

x3 1 1 2x − 1
2
= + + .
(x − 1)(2x + 1) 2 3(x − 1) 6(2x2 + 1)

187 dkrbabajee@gmail.com
c
CHAPTER 10. ALGEBRA

10.5 Binomial Expansion of (a + b)n, n is a negative


integer or a rational (P3 only)
The binomial expansion of (1 + ax)n is given by

n(n − 1) n(n − 1)(n − 2)


(1 + ax)n = 1 + nax + (ax)2 + (ax)3 + ...
2! 3!
1
The expansion is valid if |ax| < 1 ⇒ |x| < .
|a|
If n = −1, we have a simplified expansion

(1 + ax)−1 = 1 − ax + a2 x2 − a3 x3

Example 10.15 Express the following expansions in ascending powers


of x up to and including the term in x3 and state the values of x for which
they are valid
1 2+x
(i) (1 − 3x)−1 (ii) (1 − 2x) 2 (iii)
(3 − 2x)2

Solution

(i)
(−1)(−2) (−1)(−2)(−3)
(1 + 3x)−1 = 1 + (−1)(3x)1 + (3x)2 + (3x)3
2! 3!
= 1 − 3x + 9x2 − 27x3

1
The expansion is valid if |3x| < 1 ⇒ |x| < .
3
(ii)
  1
    
1 1 − 12 1
− 12 − 23
(1 − 2x) = 1 +
2 (−2x)1 + 2
(−2x)2 + 2
(−2x)3
2 2! 3!
1 1
= 1 − x − x2 − x3
2 2

1
The expansion is valid if |2x| < 1 ⇒ |x| < .
2

188 dkrbabajee@gmail.com
c
CHAPTER 10. ALGEBRA

(iii)
 −2
2+x −2 −2 2
= (2 + x)(3 − 2x) = (2 + x)3 1− x
(3 − 2x)2 3
 1  2  3 !
1 2 (−2)(−3) 2 (−2)(−3)(−4) 2
= (2 + x) 1 + (−2) − x + − x + − x
9 3 2! 3 3! 3
 
1 4 4 32
= (2 + x) 1 + x + x2 + x3
9 3 3 27
     
2 8 1 8 4 2 64 4
= + + x+ + x + + x3
9 27 9 27 27 243 27
2 11 4 100 3
= + x + x2 + x
9 27 9 243

2 3
The expansion is valid if x < 1 ⇒ |x| < .
3 2

10.5.1 Use of Partial Fractions in Binomial expansion


Partial Fractions is useful in Binomial expansion because it avoids multiplication of
terms in the expansions.

x2 + 7x + 7
Example 10.16 Let f (x) = . When x is sufficiently
(x − 1)(x − 2)(x + 2)
small for x4 and higher powers of x can be neglected, show that
7 7 67 37
f (x) ≈ + x + x2 + x3
4 2 16 8
Solution
f (x) is a proper fraction of Type 1 which can be decomposed using cover-up rule.

x2 + 7x + 7 A B C
= + + ,
(x − 1)(x − 2)(x + 2) x−1 x−2 x+2

x2 + 7x + 7 1+7+7

A= = = −5,
(x − 1)(x − 2)(x + 2) −1 × 3
x=1
2
x + 7x + 7 4 + 14 + 7 25

B= = = ,
(x − 1)(x − 2)(x + 2) 1×4 4
x=2
x2 + 7x + 7 4 − 14 + 7 1

C= = =− .
(x − 1)(x − 2)(x + 2) −3 × −4 4
x=−2

189 dkrbabajee@gmail.com
c
CHAPTER 10. ALGEBRA

We have
−5 25 1
f (x) = + − ,
x − 1 4(x − 2) 4(x + 2)
25 1
= 5(1 − x)−1 − (2 − x)−1 − (2 + x)−1 ,
4 4
 −1  −1
−1 25 1 1 1
= 5(1 − x) − 1− x − 1+ x ,
8 2 8 2
   
2 3 25 1 1 2 1 3 1 1 1 2 1 3
≈ 5(1 + x + x + x ) − 1+ x+ x + x − 1− x+ x − x
8 2 4 8 8 2 4 8
       
25 1 25 1 25 1 2 25 1
≈ 5− − + 5− + x+ 5− − x + 5− + x3
8 8 16 16 32 32 64 64
7 7 67 37
≈ + x + x2 + x3 .
4 2 16 8
Exercise 10.3. [Partial fractions and Binomial expansion (P3 only)]

1. Express the following as a single fraction:


1 1 1 1 2 3 2x + 1
(i) + (ii) − + 2
(iii) + 2
x+1 x−1 x + 2 x + 1 (x + 1) x+2 x +2
2. Express the following proper fractions of Type 1 into partial fractions:
4 x
(i) (ii)
(x − 2)(x + 2) (x − 2)(x + 2)
1 6x + 1
(iii) (iv)
(x − 3)(x + 1) (x − 1)(x − 2)(x − 3)
3. Express the following proper fractions of Type 2 into partial fractions:
1 x
(i) 2
(ii)
(x − 1)(x + 1) (x − 1)(x + 2)2
x2 + 1 6
(iii) 2 (iv)
x (2x + 1) x(2x − 3)2
4. Express the following proper fractions of Type 3 into partial fractions:
2 x
(i) 2
(ii)
(x + 1)(x + 1) (x + 1)(x2 + 1)
4x − x2 2x2 − x + 1
(iii) (iv)
(x + 2)(x2 + 2) (x − 3)(2x2 + 1)
5. Express the following improper fractions of Type 1 into partial fractions:
x2 x2 + 3x + 5
(i) (ii)
(x + 1)(x − 1) (x + 3)(x − 1)
2
3x x3 + 1
(iii) (iv)
(2x − 1)(3x + 2) x(x − 2)(x + 2)

190 dkrbabajee@gmail.com
c
CHAPTER 10. ALGEBRA

6. Express the following improper fractions of Type 2 into partial fractions:


x2 x3 + 1
(i) (ii)
(x − 1)2 x2 (2x + 1)
x3 + 5x2 + 2x − 1 x3 − x − 1
(iii) (iv)
(x + 1)(x − 3)2 (x + 2)(3x − 1)2
7. Express the following improper fractions of Type 3 into partial fractions:
5x2 + 1 x3 + 2
(i) 2 (ii) 2
x +5 (x + 1)(x + 2)
3 2
3x + 5x + 2 6x3 − 5x2 + 4x − 7
(iii) (iv)
x(2x2 + 5) (2x + 1)(2x2 + 3)
8. Express the following expansions in ascending powers of x up to and including
the term in x3 and state −2 of x for which they are valid
 the values
5x 1
(i) (1 + 2x)−1 (ii) 2 − (iii) (1 + 3x) 2
2
  32
1 4x 2+x
(iv) (8 − 5x)− 3 (v) 1 + (vi)
3 1−x
3+x √ 2 + 5x
(vii) 2
(viii) (5 − x) 1 + 4x (ix) √
(2 − 3x) 1+x
1
9. When (1 + ax)(1 + 3x) 3 , where a is a constant, is expanded in ascending powers
of x, the coefficient of the term in x is zero.

(i) Find the value of a.


1
(ii) Using this value of a, find the coefficient of x3 in the expansion of (1 + ax)(1 + 3x) 3 .

10. Express f (x) into partial fractions and hence obtain the first 4 terms of the
expansion of f (x) in ascending powers of x:
3 4
(i) f (x) = (ii) f (x) =
(x + 1)(x − 2) (2x + 3)(x + 1)2
1 x2
(iii) f (x) = (iv) f (x) =
(x − 1)(2x2 + 1) (x − 3)(x − 4)
3
x x3 + 3
(v) f (x) = (vi) f (x) =
(x − 5)(x − 2)2 (x + 1)(3x2 + 4)

191 dkrbabajee@gmail.com
c
CHAPTER 10. ALGEBRA

10.6 Miscellaneous Exercises


x3 + 1
Example 10.17 Let f (x) = .
(x2 + 2)(x − 1)
2 x−5
(i) Show that f (x) = 1 + +
3(x − 1) 3(x2 + 2)
(ii) When x is sufficiently small for x4 and higher powers of x can be
1 1 1 3
neglected, show that f (x) ≈ − − x − x2 − x3
2 2 4 4

Solution

(i) f (x) is an improper fraction of Type 3 which can be decomposed as follows:

x3 + 1 B Cx + D
2
=A+ + 2
(x + 2)(x − 1) x − 1 (x + 2)
A(x − 1)(x2 + 2) + B(x2 + 2) + (Cx + D)(x − 1)
= .
(x − 1)(x2 + 2)

x3 + 1 = A(x − 1)(x2 + 2) + B(x2 + 2) + (Cx + D)(x − 1) (10.15)


2
Set x = 1 in (10.15), we have 2 = 3B ⇒ B = .
3
Comparing coefficient of x3 in (10.15), we have 1 = A.
Comparing constants in (10.15), we have
4 5
1 = −2A + 2B − D ⇒ D = −2A + 2B − 1 = −2 + − 1 = − .
3 3

Comparing coefficient of x in (10.15), we have


5 1
0 = 2A − C + D ⇒ C = 2A + D = 2 − = .
3 3

192 dkrbabajee@gmail.com
c
CHAPTER 10. ALGEBRA

(ii)
2 x−5
f (x) = 1 + + ,
3(x − 1) 3(x2 + 2)
2 1
= 1 − (1 − x)−1 − (5 − x)(2 + x2 )−1 ,
3 3
 −1
2 −1 1 1 2
= 1 − (1 − x) − (5 − x) 1 + x ,
3 6 2
 
2 2 3
 1 5 2 1 3
≈1− 1+x+x +x − 5−x− x + x
3 6 2 2
2 2 2 2 5 1 5 1
≈ 1 − − x − x2 − x3 − + x + x2 − x3
 3 3  3 3 6 6 12  12  
2 5 2 1 2 5 2 2 1
≈ 1− − + − + x+ − + x + − − x3 ,
3 6 3 6 3 12 3 12
1 1 1 3
≈ − − x − x2 − x3 .
2 2 4 4

Miscellaneous Exercise 10.

1. Solve:
(i) 2|x + 1| = |3x + 2| (ii) 3|x + 1| > 2|x − 4| (iii) |4x − 1| < |3x − 2|

2. Solve for any real values of a:


(i) |2x + a| = |x + 2a| (ii) |2x − a| = |x + 2a|

3. Let f (x) = 2 x3 + x2 + ax + b. Given that f (x) is exactly divisible by 2x2 + 3x + 3,

(i) find the value of a and of b,


(ii) with this values of a and b, solve the equation f (x) < 0.

4. Let f (x) = x3 + ax2 + bx + c. f (x) leaves a remainder of −24 when divided by


(x − 1) and a remainder of −36 when divided by (x − 2). Also f (x) is exactly
divisible by (x − 5).

(i) Find the values of a, b and c.


(ii) with these values of a, b and c, solve the equation f (x) = 0.

5. Given that p(x) = x4 + x2 + a is exactly divisible by x2 + x + 1,

(i) find the value of a,


with this value of a, find the other quadratic factor.
(ii)

6. When (1 − x) 1 + ax, where a is a constant, is expanded in ascending powers of
x, the coefficient of the term in x is zero.

193 dkrbabajee@gmail.com
c
CHAPTER 10. ALGEBRA

(i) Find the value of a.



(ii) When a has this value, find the term in x2 in the expansion of (1 − x) 1 + ax ,
simplifying the coefficient.
2
7.* The first four terms in the expansion of (1 + ax)(1 + bx) 3 , where a and b are
1 22 2
constants, are 1 + x − x + kx3 . Find the possible values of a, b and k.
6 9
2x2 + x − 6
8. Let f (x) = .
(x − 2)(3x + 1)(x + 3)
4 9 55
(i) Show that f (x) = + + .
35(x − 2) 40(x + 3) 56(3 x + 1)
(ii) When x is sufficiently small for x3 and higher powers of x can be neglected,
53 2
show that f (x) ≈ 1 − 3 x + x.
6
x3 + 1
9. Let f (x) = .
(x2 + 1)(x + 2)
1−3x 7
(i) Show that f (x) = 1 + − .
5(x2 + 1) 5(x + 2)
(ii) When x is sufficiently small for x4 and higher powers of x can be neglected,
1 1 3 11 3
show that f (x) ≈ − x − x2 + x.
2 4 8 16
1 + 3x
10.* Let f (x) = .
(1 + ax)(1 + x)
(a − 3) 2
(i) Show that f (x) = + .
(a − 1)(1 + ax) (a − 1)(1 + x)
(ii) The coefficient of x in the expansion of f (x) is 0, find the value of a.

194 dkrbabajee@gmail.com
c
C HAPTER 11

Logarithms and exponential


functions

11.1 Objective
At the end of this chapter, students should be able to:

1. understand the relationship between logarithms and indices, and use the laws
of logarithms;

2. understand the definition and properties of ex and ln x, including their relationship


as inverse functions and their graphs;

3. use logarithms to solve equations of the form ax = b, and similar inequalities;

4. use logarithms to transform a given relationship to linear form, and hence


determine unknown constants by considering the gradient and/ or intercept.

11.2 Indices and Logarithms


The logarithmic form is obtained by making the power x in the index form subject of
formula:
x
a = }b ⇔ loga b = x ,
| {z | {z }
Index Form Logarithm Form

where a is the base.

195 dkrbabajee@gmail.com
c
CHAPTER 11. LOGARITHMS AND EXPONENTIAL FUNCTIONS

11.3 Laws of Logarithms


We recall the laws of indices.

a0 = 1, (11.1)
a1 = a, (11.2)
ax × ay = ax+y , (11.3)
ax ÷ ay = ax−y , (11.4)
1
a−x = x , (11.5)
a
(ax )y = axy = (ay )x , (11.6)
(a × b)x = ax × bx , (11.7)
 a x ax
= x, (11.8)
b b
x = y ⇔ ax = ay (11.9)
a = b ⇔ ax = bx . (11.10)

The laws of logarithms are given by

loga 1 = 0, (11.11)
loga a = 1, (11.12)
loga (x × y) = loga x + loga y, (11.13)
 
x
loga = loga x − loga y, (11.14)
y
loga xp = p loga x, (11.15)
logc b
loga b = , (11.16)
logc a
1
loga b = (11.17)
logb a
x = y ⇔ loga x = logb y. (11.18)

Laws (11.16) and (11.17) involves a change of base.


We note that

loga (x + y) 6= loga x + loga y


loga (x − y) 6= loga x − loga y.

196 dkrbabajee@gmail.com
c
CHAPTER 11. LOGARITHMS AND EXPONENTIAL FUNCTIONS

 − 12
49
Example 11.1 (i) Evaluate .
36
1 3
2− 2 × 4 2
(ii) Evaluate 1 .
86
(iii) Convert 31 = 3 in logarithmic form.

(iv) Convert log10 1000 = 3 in index form.


 
1
(v) Evaluate log2 .
8
1
(vi) Express 3 log5 p − log5 q + 2 as a single logarithm.
2
Solution
 − 21   12 √
49 36 36 6
(i) = =√ = .
36 49 49 7
1 3 3
2− 2 × 4 2 (22 ) 2 23 1 1
(ii) 1 = 1 1 = 1 1 = 23− 2 − 2 = 22 = 4.
8 6 2 × (23 )
2 6 2 ×22 2

(iii) log3 3 = 1.

(iv) 103 = 1000.


 
1
(v) log2 = log2 1 − log2 8 = log2 1 − log2 23 = log2 1 − 3 log2 2 = 0 − 3 = −3.
8
1 1 √
(vi) 3 log5 p − log5 q + 2 = log5 p3 − log5 q 2 + 2 log5 5 = log5 p3 − log5 q + log5 52
2  
3 √ 25p3
= log5 p − log5 q + log5 25 = log5 √ .
q

Example 11.2 If loga x =  q, express the following in terms of q:


1 √
(i) loga x3 (ii) loga 4
(iii) loga (a x)
x

Solution

(i)

loga x3 = 3 loga x, using law (11.15),


= 3q.

197 dkrbabajee@gmail.com
c
CHAPTER 11. LOGARITHMS AND EXPONENTIAL FUNCTIONS

(ii)
 
1
loga = loga x−4 , using law (11.5),
x4
= −4 loga x, using law (11.15),
= −4q

(iii)
√ 1
loga (a x) = loga (a × x 2 ),
1
= loga a + loga x 2 , using law (11.13),
1
= 1 + loga x, using laws (11.12) and (11.15),
2
q
=1+ .
2

Two special logarithms are lg (base 10: log10 ) and ln (Natural Logarithms: loge ). They
are available in a calculator.
loga b is defined for a > 0, a 6= 1 and b > 0.

11.4 The graphs of y = ex and y = ln x and their


relation

The graph of y = f (x) = ex meets the


y−axis at (0, 1) and the asymptote is the
x−axis meaning that such that the
distance between them approaches zero
as they tend to infinity. We note that
ex > 0 for all x. As x → ∞, ex → ∞. As
x → −∞, ex → 0.
The graph of y = f −1 (x) = ln x meets the
x−axis at (1, 0) and the asymptote is the
y−axis We note that ln x ≥ 0 for x ≥ 1 and
ln x < 0 for 0 < x < 1. As x → ∞,
ln x → ∞. As x → 0, ln x → −∞.
The graph of y = ln x is a reflection of the
graph of y = ex in the line y = x as shown
in the diagram.

198 dkrbabajee@gmail.com
c
CHAPTER 11. LOGARITHMS AND EXPONENTIAL FUNCTIONS

Example 11.3 Sketch the curve y = f (x) = e−x , x ∈ R and find its
range. Find f −1 (x) and sketch the graph of y = f −1 (x) clearly making
the relation between them.

Solution
The graph of y = e−x is the reflection of
the graph of y = ex in the y−axis. Since
e−x > 0, the range of f is f (x) > 0. To find
the inverse, let y = f (x).

e−x = y,
x = − ln y,
f −1 (x) = − ln x.

Since the domain of f −1 = Range of f ,


f −1 (x) = − ln x, x > 0. The graph of
y = f −1 (x) is obtained by reflecting the
graph of y = e−x in the line y = x.

Exercise 11.1. [Logarithmic and exponential functions]

1. Evaluate  − 23 1 1 1 2
2 9 8 2 × 32− 2 3− 6 × 3− 3
(i) 64 3 (ii) (iii) 1 (iv) 1 1
25 16 4 9 3 × 27− 2
2. Convert in logarithmic form:
(i) 23 = 8 (ii) 10−2 = 0.01 (iii) ex = 5

3. Convert in index form:


1
(i) log4 64 = 3 (ii) log36 6 = (iii) loge p = 2
2
4. Evaluate:  
1 √
3
(i) log3 9 (ii) log2 (iii) log121 11 (iv) loga a2
16
5. If loga x = q, express 
the following
 in terms of q:
1 √
(i) loga x5 (ii) loga (iii) log a ( ax)
x2
6. Express as a single logarithm:
1
(i) 5 log5 p + 4 log5 q (ii) 2 log2 p − 3 log2 q + 2 (iii) lg p + 2 lg q − lg r
2

199 dkrbabajee@gmail.com
c
CHAPTER 11. LOGARITHMS AND EXPONENTIAL FUNCTIONS

7. For the following functions f , find its inverse stating its domain and sketch
on a single diagram the graphs of y = f (x) and y = f −1 (x), making clear the
relationship between these two graphs:
x
(i) f (x) = e 2 (ii) f (x) = 1 + ex (iii) f (x) = 10(1 − e−x )

11.5 Solving equations and inequalities involving


logarithms
We make use of the laws of logarithms to solve equations.
Example 11.4 Solve the equation ln (2 + x) = 1 + ln x giving your answer
correct to 3 significant figures.
Solution
Procedure: We express as a single logarithm on both sides of the equation and apply
law (11.18):

ln (2 + x) = 1 + ln x,
ln (2 + x) = ln e + ln x, using law (11.12),
ln (2 + x) = ln (ex), using law (11.13),
2 + x = ex, using law (11.18),
2 = (e − 1)x,
2
x= = 1.16.
e−1

1
Example 11.5 Find the value of y for which 2 log4 y − log4 (y + 4) = .
2
Solution
Procedure: We express as a single logarithm on both sides of the equation and apply
law (11.18):
1
2 log4 y − log4 (y + 4) = ,
2
1
log4 y 2 − log4 (y + 4) = log4 4, using law (11.15) and (11.12),
 2  2
y 1
log4 = log4 4 2 , using law (11.14) and (11.15),
y+4
 2 
y
log4 = log4 2,
y+4

200 dkrbabajee@gmail.com
c
CHAPTER 11. LOGARITHMS AND EXPONENTIAL FUNCTIONS

y2
= 2, , using law (11.18),
y+4
y 2 = 2y + 8,
y 2 − 2y − 8 = 0,
(y − 4)(y + 2) = 0,
y = 4 or y = −2,
y = −2 ⇒ log4 (−2) is undefined,
∴ y = 4.

Example 11.6 Solve the equation ex + 3e−x = 4.


Solution
Procedure: Use the substitution y = ex to obtain a quadratic equation on y.

y = ex ,
1 1
e−x = x = , using law (11.5),
e y
" #
3
y + = 4 × 4,
y
y 2 + 3 = 4y,
y 2 − 4y + 3 = 0,
(y − 1)(y − 3) = 0,
y = 1 or y = 3,
ex = 1 or ex = 3,
x = ln 1 = 0 or x = ln 3.

Example 11.7 Solve the inequality e−2x − 2 > 0.


Solution

e−2x − 2 > 0,
e−2x > 2,
" #
− 2x > ln 2 × −1,

2x < − ln 2
1
x < − ln 2.
2

201 dkrbabajee@gmail.com
c
CHAPTER 11. LOGARITHMS AND EXPONENTIAL FUNCTIONS

Exercise 11.2. [Solving equations and inequalities involving logarithms]

1. Solve, giving your answer correct to 3 significant figures:


(i) ln (1 + x) = 3 + ln x (ii) ln (2 + x2 ) = 1 + 2 ln x

(iii) ln (1 + ex ) = 3 (iv) 2 ln (e−x − 2) + 1 = 0

2. Solve, giving your answer correct to 3 significant figures:


(i) 4x = 5 (ii) 2x+5 = 6x−3
3x + 1
(iii) = 3 (iv) 52x+1 − 11(5x ) + 2 = 0
3x − 1
3. Solve, giving your answer correct to 3 significant figures:
(i) e2x = 6 (ii) e1−3x = 4

(iii) e2x − 4ex − 5 = 0 (iv) ex + 12e−x = 7

4. Solve:
(i) log3 (x + 5) − log3 x = 2 (ii) log2 (x + 4) + log2 x = 5
1
(iii) 2 lg (2x + 5) = lg 4x + 1 (iv) 2 log9 x − log9 (x + 6) =
2
5. Solve the following inequalities:
(i) 2 − 3ex > 0 (ii) 3−x < 2

(iii) 2 < ex < 3 (iv) |2x − 3| < 2

11.6 Use of logarithms to transform a given relationship


to linear form
The linear form is given by Y = mX + c where m is the gradient and c is the
Y −intercept. We can make use of logarithms to reduce a non-linear equation to
linear form.

Example 11.8 The relationship between the variables x and y is y = a xk , where a


and k are constants. Five pairs of values of x and y are given in the following table:
By plotting a graph of lg y against lg x, estimate the values of a and k.

x 3 5 10 15 20
y 900 2000 5000 9000 16000

202 dkrbabajee@gmail.com
c
CHAPTER 11. LOGARITHMS AND EXPONENTIAL FUNCTIONS

4.50
Solution
4.00
Y
3.50

y = a xk , 3.00

2.50
lg y = lg (a xk ), using law (11.18), 2.00
k
lg y = lg a + lg x , using law (11.13), 1.50

1.00
lg y = k lg x + lg a, using law (11.15),
0.50

Y = mX + c, 0.00
1
- .00 0
- .50 0.00 0.50 1.00 1.50
Y = lg y, X = lg x, m = k, c = lg a. X

X = lg x 0.48 0.7 1 1.18 1.30


Y = lg y 2.95 3.30 3.70 3.95 4.20

From the diagram, c = 2.25 ⇒ a = 102.25 = 178. Two points on the line (0, 2.25) and
3.3 − 2.25
(0.7, 3.3) and so m = k = = 1.5.
0.7 − 0
Exercise 11.3. [Linear form]

1. By using logarithms, reduce the following non-linear equations in the linear


form Y = mX + c. (i) y = axk (ii) y = abx (iii) ax = by (iv) y = a ebx

2. The variables x and y are connected by the relation ay = 4x , where a is a constant.


The figure shows the straight line graph obtained by plotting values of the
variables indicated with the coordinates of two points on the line.

Estimate the value of a.

3. The figure shows the straight line graph obtained by plotting values of the
variables indicated with the coordinates of two points on the line.

Express y in terms of x.

4. The figure shows the straight line graph obtained by plotting values of the
variables indicated with the coordinates of two points on the line.

203 dkrbabajee@gmail.com
c
CHAPTER 11. LOGARITHMS AND EXPONENTIAL FUNCTIONS

Express y in terms of x.

5. The variables x and y are connected by the relation y = a bx , where a and b are
constants.
The figure shows the straight line graph obtained by plotting values of the
variables indicated with the coordinates of two points on the line.

Estimate the value of a and of b.

6. The relationship between the variables x and y is y = a xk , where a and k are


constants. Five pairs of values of x and y are given in the following table:
By plotting a graph of lg y against lg x, estimate the values of a and k.

x 1 1.5 2 2.5 3
y 320 380 600 870 1160

7. The relationship between the variables x and y is y = a bx , where a and b are


constants. Five pairs of values of x and y are given in the following table:
By plotting a graph of lg y against x, estimate the values of a and b.

x 1 5 10 15 20
y 20 80 600 4000 33000

11.7 Miscellaneous Exercises


Example 11.9 (i) Solve |ex − 3| = 1, giving your answers correct to 3
significant figures.

(ii) The relationship between the variables x and y is ay = b cx , where a,


b and c are constants.
Use logarithms to express y in terms of x.

204 dkrbabajee@gmail.com
c
CHAPTER 11. LOGARITHMS AND EXPONENTIAL FUNCTIONS

Solution

(i) Let y = ex , then |y − 3| = 1 ⇒ (y − 3)2 = 12 ⇒ y = 3 ± 1 = 4, or 2.

ex = 4 ⇒ x = ln 4 = 1.39 or ex = 2 ⇒ x = ln 2 = 0.693.

(ii)

ay = b cx ⇒ ln (ay ) = ln (b cx )
y ln a = ln b + ln cx
y ln a = (ln c)x + ln b
ln c ln b
y= x+
ln a ln a
y = (loga c)x + loga b

Miscellaneous Exercise 11.

1. Solve, giving your answer correct to 3 significant figures:


(i) (1.5)x = 15 (ii) 3x+1 = 2x (iii) 4x+1 × 5x−1 = 12

2. Solve, giving your answer correct to 3 significant figures:


(i) 3x = 5 + 14(3−x ) (ii) e3x − 9e2x + 14ex = 0 (iii) 4x = 2x+1 + 1

3. Solve:
(i) log2 (x + 3) + log2 (x − 3) = log2 (2x − 1) (ii) log3 (x + 8) + log3 x = 2

(iii) 2 lg 5 + lg (x − 1) = lg (3x − 4) + 1

4. Solve, giving your answer correct to 3 significant figures where necessary:


(i) |ex − 5| = 4 (ii) |5x − 2| = |5x + 1| (iii) |2(3x ) − 3| = |3x + 2|

5. For the following relations between the variables x and y, use logarithms to
express y in terms of x:
(i) 5y = 6x (ii) ey = 8(ex ) (iii) 4x × 3y = 12
3
6. Sketch the curve y = f (x) = ln (2x + 3), x > − and find its range.
2
Find f −1 (x) and sketch the graph of y = f −1 (x) clearly making the relation
between them.

7. The equation 22x − 5(2x ) + p = 0 has a solution at x = 1. Find

(i) the value of p,


(ii) the other solution, giving your answer correct to 3 significant figures.

205 dkrbabajee@gmail.com
c
CHAPTER 11. LOGARITHMS AND EXPONENTIAL FUNCTIONS

x

8. Given that y = 10 + 9 e 2 ,

(i) find the value of y when x = 0,


(ii) find the value of x when y = 15,
(iii) state the value of y when x becomes large.

9. The variables x and y are connected by the relation y = a (be)x , where a and b
are constants.
The figure shows the straight line graph obtained by plotting values of the
variables indicated with the coordinates of two points on the line.

Estimate the value of a and of b.

10.* The relationship between the variables x and y is xm y n = 100, where m and n
are constants. Four pairs of values of x and y are given in the following table:
By plotting a graph of lg y against lg x,

x 1 2 3 4
y 2.51 1.66 1.30 1.09

(i) estimate the values of m and n,


(ii) find the value of x when y = 2.

206 dkrbabajee@gmail.com
c
C HAPTER 12

Trigonometry 2

12.1 Objective
At the end of this chapter, students should be able to:

1. understand the relationship of the secant, cosecant and cotangent functions to


cosine, sine and tangent, and use properties and graphs of all six trigonometric
functions for angles of any magnitude;

2. use trigonometrical identities such as sec2 θ = 1 + tan2 θ and


cosec2 θ = 1 + cot2 θ for the simplification and exact evaluation of expressions
and in solving equations;

3. use the expansions of trigonometric ratios of compound angles (sin (A ± B),


cos (A ± B) and tan (A ± B)), the formulae for trigonometric ratios of double
angles (sin 2A, cos 2A and tan 2A) and the expressions of a sin θ + b cos θ in the
forms R sin (θ ± α) and R cos (θ ± α) to simplify expressions and solve equations.

12.2 Secant, cosecant and cotangent functions


The secant, cosecant and cotangent functions are defined by
1 1 1 cos x
sec x = , cosec x = and cot x = = . (12.1)
cos x sin x tan x sin x

207 dkrbabajee@gmail.com
c
CHAPTER 12. TRIGONOMETRY 2

The diagram shows the graphs of y = cos x


and y = sec x, for −2π ≤ x ≤ 2π (or we
could use −3600 ≤ x ≤ 3600 in case x is in
degrees.) When x = 0, cos x = sec x = 1.
When x = π, cos x = sec x = −1. When
π
x = , cos x = 0 and we cannot calculate
2
sec x. When cos x is small and positive,
sec x will be large and positive. When cos x
is small and negative, sec x will be large
and negative. Because the cosine graph is
periodic with period 2π, so too is the graph
of sec x.

The diagram shows the graphs of y = sin x


and y = cosec x, for −2π ≤ x ≤ 2π.
π
When x = , sin x = cosec x = 1. When
2

x= , cos x = cosec x = −1. When x = 0,
2
sin x = 0 and we cannot calculate cosec x.
When sin x is small and positive, cosec x
will be large and positive. When sin x is
small and negative, cosec x will be large
and negative. Because the sine graph is
periodic with period 2π, so too is the graph
of cosec x.

The diagram below shows the graph of


y = tan x, for −2π ≤ x ≤ 2π.

The diagram below shows the graph of


y = cot x, for −2π ≤ x ≤ 2π. When x is
small and positive (just above zero), so too
is tan x. So cot x will be large and positive.
π
When x is close to the value of tan x is
2
very large and positive, and so cot x will be
very small. Because the tangent graph is
periodic with period π, so too is the graph
of cot x.

208 dkrbabajee@gmail.com
c
CHAPTER 12. TRIGONOMETRY 2

12.3 Trigonometric Identities


We know that

sin2 x + cos2 x = 1. (12.2)

Dividing (12.2) by cos2 x, we

tan2 x + 1 = sec2 x. (12.3)

Dividing (12.2) by sin2 x, we

cot2 x + 1 = cosec2 x. (12.4)

Example 12.1 Prove the identity tan2 x + cot2 x + 2 ≡ cosec2 x sec2 x.

Solution

tan2 x + cot2 x + 2 ≡ tan2 x + 1 + cot2 x + 1,


≡ sec2 x + cosec2 x, using (12.3) and (12.4),
1 1
≡ + using (12.1) ,
cos x sin2 x
2

sin2 x + cos2 x
≡ ,
sin2 x cos2 x
1 1
≡ 2 , using (12.2),
sin x cos2 x
≡ cosec2 x sec2 x.

Example 12.2 Solve the following trigonometrical equations for


00 ≤ x ≤ 3600:
(i) sec2 x = tan x + 3 (ii) cot2 x − 5 cosec x + 7 = 0

Solution

(i)

tan2 x + 1 = tan x + 3, using (12.3)


tan2 x − tan x − 2 = 0
(tan x − 2)(tan x + 1) = 0
tan x = 2 tan x = −1
α = tan−1 (2) = 63.40 α = tan−1 (1) = 450
x = α, 1800 + α x = 1800 − α, 3600 − α
x = 63.40 , 243.40 x = 1350 , 3150

209 dkrbabajee@gmail.com
c
CHAPTER 12. TRIGONOMETRY 2

(ii)

cosec2 x − 1 − 5 cosec x + 7 = 0, using (12.4)


cosec2 x − 5 cosec x + 6 = 0
(cosec x − 2)(cosec x − 3) = 0
cosec x = 2 cosecx = 3
1 1
sin x = sin x =
2 3
   
1 1
α = sin −1
= 300 α = sin −1
= 19.50
2 3
x = α, 1800 − α x = α, 1800 − α
x = 300 , 1500 x = 19.50 , 160.50

Exercise 12.1. [Reciprocal of trigonometric functions and their identities]

1. (i) Sketch the graph of y = sec x for 0 ≤ x ≤ 2π and find the values of x in this
range for which sec x = 2.
(ii) Sketch the graph of y = cosec x for 00 ≤ x ≤ 3600 and find the values of x in

2 3
this range for which cosec x = − .
3
(iii) Sketch the graph of y = cot x for −π ≤ x ≤ π and find the values of x in this
range for which cot x = 0.

2. Find in surd form the values of:


(i) cot (5π/6) (ii) sec (π/4) (iii) cosec (5π/3)

3. Prove the following identities:


sec2 x − 1
(i) 2
≡ tan4 x
cot x
1 + tan2 x
(ii) 2 ≡ tan2 x
1 + cot x
(iii) sec x cosec x ≡ cot x + tan x
sin x
(iv) ≡ cosec x − cot x
1 + cos x
cos x
(v) ≡ sec x − tan x
1 + sin x
cos x 1 + sin x
(vi) + ≡ 2 sec x
1 + sin x cos x
tan x 1 + sec x
(vii) + ≡ 2 cosec x
1 + sec x tan x

210 dkrbabajee@gmail.com
c
CHAPTER 12. TRIGONOMETRY 2

4. Solve the following trigonometrical equations for 00 ≤ x ≤ 3600 :


(i) 4 cot x + 3 = 0 (ii) cosec x + 3 = 0

(iii) 2 sec x − 5 = 0 (iv) sec2 x − tan x − 3 = 0

(v) cot2 x − 2 cosec x + 1 = 0 (vi) tan2 x + sec x − 5 = 0

(vii) 10 cosec2 x = cot x + 12


1 + cos x
5.* (i) Prove that (cot x + cosec x)2 ≡ .
1 − cos x
(ii) Hence, solve the equation (cot x + cosec x)2 = 2 for 0 ≤ x ≤ 2π.

12.4 Compound angles


The addition and difference trigonometric formulae are given by

sin (A ± B) = sin A cos B ± cos A sin B, (12.5)


cos (A ± B) = cos A cos B ∓ sin A sin B, (12.6)
tan A ± tan B
tan (A ± B) = . (12.7)
1 ∓ tan A tan B

3 7
Example 12.3 Given that cos A = and sin B = , where A and B are
5 25
acute, find
(i) sin (A − B) (ii) cos (A + B) (iii) tan (A + B).

Solution
By simple Trigonometry,
3 4 4 24 7 7
cos A = , sin A = , tan A = , cos B = , sin B = , tan B = .
5 5 3 25 25 24
(i)

sin (A − B) = sin A cos B − cos A sin B,


   
4 24 3 7 3
= × − × = .
5 25 5 25 5

211 dkrbabajee@gmail.com
c
CHAPTER 12. TRIGONOMETRY 2

(ii)

cos (A + B) = cos A cos B − sin A sin B,


   
3 24 4 7 44
= × − × = .
5 25 5 25 125

(iii)
tan A + tan B
tan (A + B) = ,
1 − tan A tan B
4 7
3
+ 24 117
= 4 7
= .
1 − 3 × 24 44

Example 12.4 Solve tan (450 + x) = 3 tan (450 − x), for 00 < x < 1800 .

Solution
 
tan 450 + tan x tan 450 − tan x
=3 ,
1 − tan 450 tan x 1 + tan 450 tan x
1 + tan x 3(1 − tan x)
= ,
1 − tan x 1 + tan x
(1 + tan x)2 = 3(1 − tan x)2 ,
1 + 2 tan x + tan2 x = 3(1 − 2 tan x + tan2 x),
1 + 2 tan x + tan2 x = 3 − 6 tan x + 3 tan2 x,
" #
2 tan2 x − 8 tan x + 2 = 0 ÷ 2,

tan2 x − 4 tan x + 1 = 0,

tan x = 2 ± 22 − 1,
√ √
tan x = 2 + 3, tan x = 2 − 3,
√ √
x = tan−1 (2 + 3) = 750 , x = tan−1 (2 − 3) = 150 .

Exercise 12.2. [Compound angles]

1. Given that the angles A and B are acute, find the values of
(i) sin (A + B) (ii) cos (A + B) (iii) tan (A + B)
(iv) sin (A − B) (v) cos (A − B) (vi) tan (A − B)

for each the following trigonometric ratios:

212 dkrbabajee@gmail.com
c
CHAPTER 12. TRIGONOMETRY 2

3 12 8 24
(a) sin A = , sin B = (b) sin A = , cos B =
5 13 17 25
20 4 7 12
(c) cos A = , sin B = (d) cos A = , cos B =
29 5 25 37
2. By expressing 150 = 600 − 450 , find, in surd form, the exact values of
(i) sin 150 (ii) cos 150 (iii) tan 150

3. By expressing 750 = 300 + 450 , find the exact values of


(i) sin 750 (ii) cos 750 (iii) tan 750
24
4. Given that sin x = , where x is acute, find, in surd form, the exact values of
25
(i) sin (x + 600 ) (ii) cos (x + 600 ) (iii) tan (x + 600 )
(iv) sin (x − 600 ) (v) cos (x − 600 ) (vi) tan (x − 600 )

5. Prove the following identities:


cot x cot y + 1
(i) cot (x − y) =
cot x − cot y
(ii) sin (x + y) + cos (x − y) = (sin x + cos x)(sin y + cos y)
sin (x − y)
(iii) = tan x − tan y
cos x cos y
(iv) tan (x + 450 ) tan (450 − x) = 1
(v) sin (x + 600 ) − cos (x − 300 ) = 0
(vi) cos (x + 600 ) + sin (x − 300 ) = 0

6. Solve the following trigonometrical equations for 00 ≤ x ≤ 3600 :


(i) tan (x + 450 ) = 3 (ii) 2 tan (x − 450 ) = 1

(iii) 3 tan (x − θ) = 4, tan θ = 3 (iv) 3 tan (450 + x) = tan (450 − x)



7.* (i) Show that the equation sin (x + 450 ) = cos x can be written as tan x = 2−1.
(ii) Hence solve the equation sin (x + 450 ) = cos x for 00 ≤ x ≤ 3600.

8.* (i) Show that the equation cos (x − 600 ) = sin x can be written as tan x = 2+ 3.
(ii) Hence solve the equation cos (x − 600 ) = sin x for 00 ≤ x ≤ 3600 .

213 dkrbabajee@gmail.com
c
CHAPTER 12. TRIGONOMETRY 2

12.5 Double and Triple Angles

sin (2x) = sin (x + x)


= sin x cos x + cos x sin x
sin (2x) = 2 sin x cos x (12.8)

cos (2x) = cos (x + x)


= cos x cos x − sin x sin x
cos (2x) = cos2 x − sin2 x (12.9)
cos (2x) = 2 cos2 x − 1 (12.10)
cos (2x) = 1 − 2 sin2 x (12.11)

tan (2x) = tan (x + x)


tan x + tan x
=
1 − tan x tan x
2 tan x
tan (2x) = (12.12)
1 − tan2 x

sin (3x) = sin (2x + x)


= sin (2x) cos x + cos (2x) sin x
= 2 sin x cos2 x + (2 cos2 x − 1) sin x,
= 4 sin x cos2 x − sin x
= 4 sin x(1 − sin2 x) − sin x
sin (3x) = 3 sin x − 4 sin3 x (12.13)

cos (3x) = cos (2x + x)


= cos (2x) cos x − sin (2x) sin x
= (1 − 2 sin2 x) cos x − 2 sin2 x cos x
= cos x − 4 sin2 x cos x
= cos x − 4(1 − cos2 x) cos x
cos (3x) = 4 cos3 x − 3 cos x (12.14)

8
Example 12.5 If sin x = , where x is acute, evaluate:
17
(i) sin (2x) (ii) cos (2x) (iii) tan (2x)

214 dkrbabajee@gmail.com
c
CHAPTER 12. TRIGONOMETRY 2

15 8
Solution Using Pythagoras theorem, we have cos x = and tan x = .
17 15

8 15 240
(i) sin (2x) = 2 sin x cos x = 2 × × =
17 17 289
 2
8 161
(ii) cos (2x) = 1 − 2 sin2 x = 1 − 2 =
17 289
8

2 tan x 2 15 240
(iii) tan (2x) = 2 = 2 =
1 − tan x 1− 8 161
15

tan x + cot x
Example 12.6 (i) Prove the identity ≡ cosec (2x).
2
(ii) Find, in radians, all solutions of the equation
tan x + cot x = 8 cos (2x) in the interval 0 < x < π.

Solution

(i)
 
tan x + cot x 1 sin x cos x
≡ +
2 2 cos x sin x
1 sin2 x + cos2 x

2 sin x cos x
1

2 sin x cos x
1
≡ ≡ cosec (2x).
sin (2x)

215 dkrbabajee@gmail.com
c
CHAPTER 12. TRIGONOMETRY 2

(ii)
tan x + cot x
= 4 cos (2x)
2
cosec(2x) = 4 cos (2x)
1
= 4 cos (2x)
sin (2x)
2 × 2 sin (2x) cos (2x) = 1
1
sin (4x) =
2  
−1 1 π
Basic Angle = sin = ,
2 6
π 5π 13π 17π
4x = , , , since sin is +ve in 1st and 2nd quadrant and 0 < 4x < 4π,
6 6 6 6
π 5π 13π 17π
x= , , .
24 24 24 24

Example 12.7 Solve the equation sin x + 2 cos (2x) + 1 = 0 for


00 < x < 3600 .

sin x + 2 cos (2x) + 1 = 0


sin x + 2(1 − 2 sin2 x) + 1 = 0
sin x + 2 − 4 sin2 x + 1 = 0
4 sin2 x − sin x − 3 = 0
(4 sin x + 3)(sin x − 1) = 0
3
sin x = − , sin x = 1
4  
3
Basic Angle α = sin −1
= 48.60 , x = sin−1 (1) = 900
4
x = 1800 + 48.60 , 3600 − 48.60 , x = 900
x = 900 , 228.60 , 311.40 .

Exercise 12.3. [Double and triple angles]

1. Given that the angle x is acute, find the values of


(i) sin (2x) (ii) cos (2x) (iii) tan (2x)

for each the following trigonometric ratios:


3 12 24
(a) sin x = (b) cos x = (c) tan x =
5 13 7

216 dkrbabajee@gmail.com
c
CHAPTER 12. TRIGONOMETRY 2

2. Prove the following identities:

(i) cosec (2x) − cot (2x) ≡ tan x


(ii) sec (2x) + tan (2x) ≡ tan (450 + x)
(iii) cos (4x) ≡ 8 cos4 x − 8 cos2 x + 1

3. Solve the following trigonometrical equations for 00 ≤ x ≤ 3600 :


(i) 5 sin x cos x = 1 (ii) sin (2x) = tan x

(iii) cos (2x) + cos x = 0 (iv) cos (2x) + sin x = 0

(v) sin (2x) = cos x (vi) cos (2x) = 3 sin x + 2

(vii) 2 cos (2x) + cos x − 1 = 0 (viii) 6 cos (2x) − 5 cos x + 4 = 0

4. (i) Prove that tan x + cot x = 2 cosec (2x).


(ii) Hence solve the equation tan x + cot x = 8 sin (2x) for 0 < x < π.

5.* (i) Prove that cosec2 x − sec2 x = 4 cot (2x) cosec (2x).
(ii) Use your answer in part in (i) to find the exact value of
cosec2 (750 ) − sec2 (750 ).
(iii) Use your answer in part in (i) to solve the equation
cosec2 x − sec2 x = 3 cosec (2x) for 0 < x < π.
2 tan x
6.* (i) Show that sin (2x) = .
1 + tan2 x
1 − tan2 x
(ii) Show that cos (2x) = .
1 + tan2 x
(iii) Hence find the smallest value of x in degrees greater than zero so that
cos (2x) − sin (2x) = 1.

12.6 Express a cos θ±b sin θ as R cos (θ ± α) or R sin (θ ± α).


Example 12.8 Express 3 sin θ + 4 cos θ in the form:
(i) R sin (θ + α) (ii) R cos (θ − α) ,
where R > 0 and 00 < α < 900 , giving the value of α correct to 1 decimal
place.

217 dkrbabajee@gmail.com
c
CHAPTER 12. TRIGONOMETRY 2

Solution

(i)

R sin (θ + α) = R sin θ cos α + R cos θ sin α


= 3 sin θ + 4 cos θ

Comparing coefficient of sin θ, we have

R cos α = 3. (12.15)

Comparing coefficient of cos θ, we have

R sin α = 4. (12.16)

(12.15)2 + (12.16)2 gives

R2 (sin2 α + cos2 α) = 32 + 42 ,

R = 32 + 42 = 5.

(12.16) ÷ (12.15) gives


4
tan α = ,
3 
4
α = tan−1 = 53.10 .
3

∴ 3 sin θ + 4 cos θ = 5 sin (θ + 53.10 ).

(ii)

R cos (θ − α) = R cos θ cos α + R sin θ sin α


= 4 cos θ + 3 sin θ

Comparing coefficient of sin θ, we have

R sin α = 3. (12.17)

Comparing coefficient of cos θ, we have

R cos α = 4. (12.18)

218 dkrbabajee@gmail.com
c
CHAPTER 12. TRIGONOMETRY 2

(12.17)2 + (12.18)2 gives

R2 (sin2 α + cos2 α) = 32 + 42 ,

R = 32 + 42 = 5.

(12.17) ÷ (12.18) gives


3
tan α = ,
4 
3
α = tan−1 = 36.90 .
4

∴ 3 sin θ + 4 cos θ = 5 cos (θ − 36.90 ).


Example 12.9 (i) Express 5 cos θ − 12 sin θ in the form R cos (θ + α),
where R > 0 and 00 < α < 900 , giving the value of α correct to 1
decimal place.

(ii) Solve the equation 5 cos (2x) − 12 sin (2x) = 9, for −1800 < x < 1800 .
Solution
(i)

R cos (θ + α) = R cos θ cos α − R sin θ sin α


= 5 cos θ − 12 sin θ

Comparing coefficient of sin θ, we have

R sin α = 12. (12.19)

Comparing coefficient of cos θ, we have

R cos α = 5. (12.20)

(12.19)2 + (12.20)2 gives

R2 (sin2 α + cos2 α) = 52 + 122 ,



R = 52 + 122 = 13.

(12.19) ÷ (12.20) gives


12
tan α = ,
5 
12
α = tan −1
= 67.40 .
5

219 dkrbabajee@gmail.com
c
CHAPTER 12. TRIGONOMETRY 2

∴ 5 cos θ − 12 sin θ = 13 cos (θ + 67.40 ).

(ii)

5 cos (2x) − 12 sin (2x) = 9,


13 cos (2x + 67.40 ) = 9
9
cos (2x + 67.40 ) =
13 
9
Basic Angle = cos−1 = 46.20
13
2x + 67.40 = −46.20 , 46.20 , 313.80 , 406.20 , since −292.60 < 2x + 46.20 < 427.40 ,
2x = −113.60 , −21.20 , 246.40, 338.80,
x = −56.80 , −10.60 , 123.20 , 169.40 .

12.6.1 Finding the greatest and least values of R cos (θ ± α) or


R sin (θ ± α) in the interval 00 ≤ θ ≤ 3600
Case 1: R sin (θ + α)
The greatest value of R sin (θ + α) is R and it occurs at θ = 900 − α.
The least value of R sin (θ + α) is −R and it occurs at θ = 2700 − α.

Case 2: R sin (θ − α)
The greatest value of R sin(θ − α) is R and it occurs at θ = 900 + α.
The least value of R sin (θ − α) is −R and it occurs at θ = 2700 + α.

Case 3: R cos (θ + α)
The greatest value of R cos (θ + α) is R and it occurs at θ = 3600 − α.
The least value of R cos (θ + α) is −R and it occurs at θ = 1800 − α.

Case 4: R cos (θ − α)
The greatest value of R cos (θ − α) is R and it occurs at θ = α.
The least value of R cos (θ − α) is −R and it occurs at θ = 1800 + α.

Example 12.10 Find the least and greatest values of f (θ) and the values
0 0
of θ between√0 and 360 0at which they occur.
(i) f (θ) = 2 sin (θ + 45 ) (ii) f (θ) = 2 sin (θ − 600 )

(iii) f (θ) = 25 cos (θ + 16.30 ) (iv) f (θ) = 13 cos (θ − 22.60 )


√ √
(i) The greatest value√of 2 sin (θ + 450 ) is
√ 2 and it occurs at θ = 900 − 450 = 450 .
The least value of 2 sin (θ + 450) is − 2 and it occurs at θ = 2700 − 450 = 2250 .

220 dkrbabajee@gmail.com
c
CHAPTER 12. TRIGONOMETRY 2

(ii) The greatest value of 2 sin (θ − 600 ) is 2 and it occurs at θ = 900 + 600 = 1500 .
The least value of 2 sin (θ − 600 ) is −2 and it occurs at θ = 2700 + 600 = 3300 .

(iii) The greatest value of 25 cos (θ + 16.30 ) is 25 and it occurs at


θ = 3600 − 16.30 = 343.70. The least value of 25 cos (θ + 16.30 ) is −25 and it occurs
at θ = 1800 − 16.3 = 163.70.

(iv) The greatest value of 13 cos (θ − 22.60) is 13 and it occurs at θ = 22.60 .


The least value of 13 cos (θ − 22.60 ) is −13 and it occurs at
θ = 1800 + 22.60 = 202.60.

Exercise 12.4. [Application of compound angles]

1. Find the values of R and α, where 0 < α < 900 , for √ which:
(i) 12 cos θ + 5 sin θ ≡ R cos (θ − α) (ii) sin θ − 3 cos θ ≡ R sin (θ − α)

(iii) 24 sin θ + 7 cos θ ≡ R sin (θ + α) (iv) cos θ − 2 sin θ ≡ R cos (θ + α)

2. (i) Express 2 cos θ + 3 sin θ in the form R cos (θ − α), where R > 0 and
00 < α < 900 , giving the value of α correct to 1 decimal place.
(ii) Hence find the least and greatest values of 2 cos θ + 3 sin θ and the values of
θ between 00 and 3600 at which they occur.

3. (i) Express sin θ − cos θ in the form R sin (θ − α), where R > 0 and 0 < α < π/2.
(ii) Hence find the least and greatest values of sin θ − cos θ and the values of θ
between 0 and 2π at which they occur.

4. (i) Express 15 cos θ − 8 sin θ in the form R cos (θ + α), where R > 0 and
00 < α < 900 , giving the value of α correct to 1 decimal place.
(ii) Hence find the least and greatest values of 15 cos θ + 8 sin θ and the values of
θ between 00 and 3600 at which they occur.

5. (i) Express 20 sin θ + 21 cos θ in the form R sin (θ + α), where R > 0 and
0 < α < π/2.
(ii) Hence find the least and greatest values of 20 sin θ + 21 cos θ and the values
of θ between 0 and 2π at which they occur.

221 dkrbabajee@gmail.com
c
CHAPTER 12. TRIGONOMETRY 2

12.7 Miscellaneous Exercises



Example 12.11 (i) Express 3 cos θ + sin θ in the form R cos (θ − α),
giving the exact values of R and α (0 < α < π/2).

(ii) Find the least and greatest values of 3 cos θ + sin θ and the values
of θ between 0 and 2π at which they occur.

(iii) Solve the equation 3 cos (2x) + sin (2x) = 1, for 0 ≤ x ≤ π.

Solution

(i)

R cos (θ − α) = R cos θ cos α + R sin θ sin α



= 3 cos θ + sin θ

Comparing coefficient of sin θ, we have

R sin α = 1 (12.21)

Comparing coefficient of cos θ, we have



R cos α = 3. (12.22)

(12.21)2 + (12.22)2 gives



R2 (sin2 α + cos2 α) = 12 + ( 3)2 ,

R = 4 = 2.

(12.21) ÷ (12.22) gives


1
tan α = √ ,
3
 
−1 1 π
α = tan √ = .
3 6

∴ 3 cos θ + sin θ = 2 cos (θ − π/6).

(ii) The greatest value of 2 cos (θ − π/6) is 2 and it occurs at θ = π/6.


The least value of 2 cos (θ − π/6) is −2 and it occurs at θ = π + π/6 = 7π/6.

222 dkrbabajee@gmail.com
c
CHAPTER 12. TRIGONOMETRY 2

(iii)

3 cos (2x) + sin (2x) = 1,
2 cos (2x − π/6) = 1
1
cos (2x − π/6) =
2  
1
Basic Angle = cos−1 = π/3
2
2x − π/6 = π/3, 5π/3 since 0 ≤ 2x ≤ 2π,
2x = π/2, 11π/6
x = π/4, 11π/12.

Miscellaneous Exercise 12.


 
1 1
1. (i) Show that sin x − = 2 tan2 x.
1 − sin x 1 + sin x
1 1
(ii) Hence solve the equation − = cosec x for 0 < x < π.
1 − sin x 1 + sin x
2. (i) Prove the identity cosec (2x) − cot (2x) = tan x.
(ii) Hence solve the equation cosec (2x) − cot (2x) + 3 = 0 for 00 < x < 3600 .

3. If tan A = p, where A is acute and A + B = 450 , without using a calculator, find,


in terms of p,
(i) tan B (ii) tan (A − B)

4. (i) Prove the identity sin (x − 600 ) + cos (x − 300 ) = sin x.


(ii) Hence solve the equation sin (x − 600 ) + cos (x − 300 ) = 6 cos (2x) − 5
for 00 < x < 3600 .
0 0
√ 2 tan (30 + x) + 5 tan (x − 60 ) = 0 can be written in
5.* (i) Show that the equation
2
the form tan x + (14 3) tan x − 13 = 0
(ii) Hence, or otherwise, solve the equation 2 tan (300 + x) + 5 tan (x − 600 ) = 0,
for 00 ≤ θ ≤ 1800.

6.* (i) Prove that cosec2 x + sec2 x = 4 cosec2 (2x).


(ii) Use your answer in part in (i) to find the value of
cosec2 (150 ) + sec2 (150 ).
(iii) Use your answer in part in (i) to solve the equation cosec2 x + sec2 x = 9 for
0 < x < π.

223 dkrbabajee@gmail.com
c
CHAPTER 12. TRIGONOMETRY 2

7.* (i) Express sin x + cos x in the form R sin (x + α), where R > 0 and
0 < α < 900 .
(ii) Hence, solve the equation (1 + sin x)2 + (1 + cos x)2 = 2 for 00 ≤ x ≤ 3600 .

8.* (i) Solve the equation 4x3 −5x−1 = 0, giving your answers correct to 3 decimal
places, where necessary.
(ii) By expanding sin (3x) = sin (2x + x), show that sin (3x) = 3 sin x − 4 sin3 x.
(iii) Using your answers in parts (i) and (ii), solve the equation
sin (3x) + 2 sin x + 1 = 0 for 00 ≤ x ≤ 3600 .

9. (i) Express sin θ + 3 cos θ in the form R sin (θ + α), giving the exact values of R
and α (0 < α < π/2).

(ii) Find the least and greatest values of sin θ + 3 cos θ and the values of θ
between 0 and 2π at which they occur.

(iii) Solve the equation sin (2x) + 3 cos (2x) = 1, for 0 ≤ x ≤ π.

10.* In the diagram, AB= 8 cm, AD=17 cm, AC=10 cm, B AD b = θ, C AD


b = α and
b = 90 . Without using a calculator, find the values of
ABD 0

(i) sin (α + θ) (ii) cos (α + θ)


(iii) sin θ (iv) cos θ
(v) sin α (vi) cos α

224 dkrbabajee@gmail.com
c
C HAPTER 13

Differentiation 2

13.1 Objective
At the end of this chapter, students should be able to:

1. use the derivatives of exponential and trigonometric functions (ex , ln x, sin x,


cos x, tan x, together with constant multiples, sums, differences and composites);

2. use product and quotient rule in differentiation;

3. find and use the first derivative of a function which is defined implicitly;

4. find and use the first derivative of a function which is defined parametrically.

13.2 Derivatives of exponential and trigonometric


functions
The derivatives of exponential and trigonometric functions are given in the table
below:
f (x) f ′ (x)

eax+b aeax+b
a
ln (ax + b)
ax + b
sin (ax + b) a cos (ax + b)

cos (ax + b) −a sin (ax + b)

tan (ax + b) a sec2 (ax + b)

Example 13.1 Find the derivatives of: x


(i) cos (2x + 1) (ii) 3 sin (3x) (iii) −5 tan (iv) 3e1−3x
2
(v) ln (4x + 2) (vi) ln (tan x) (vii) sin2 x (viii) e2 cos (2x)

225 dkrbabajee@gmail.com
c
CHAPTER 13. DIFFERENTIATION 2

Solution
d
(i) (cos (2x + 1)) = −2 sin (2x + 1)
dx
d
(ii) (3 sin (3x)) = 3(3) cos (3x) = 9 cos (3x)
dx
 
d   x  1  
2 x 5  
2 x
(iii) −5 tan = −5 sec = − sec
dx 2 2 2 2 2
d 
(iv) 3e1−3x = 3(−3)e1−3x = −9e1−3x
dx
d 4
(v) (ln (4x + 2)) =
dx 4x + 2
d 1 d sec2 x 1
(vi) (ln (tan x)) = × (tan x) = = = sec x cosec x
dx tan x dx tan x cos x sin x
d d
(vii) (sin2 x) = 2 sin x × (sin x) = 2 sin x cos x = sin 2x
dx dx
d 2 cos (2x)  d
(viii) e = e2 cos (2x) × (2 cos (2x)) = −4 sin (2x) e2 cos (2x)
dx dx

13.3 Product and Quotient Rules


d dv du
The Product Rule is given by (uv) = u + v .
dx dx dx

du dv
d  u  v dx − u dx
The Quotient Rule is given by = .
dx v v2

Example 13.2 Find the derivative of:


x sin x
(i) x cos x (ii) x3 e−2x (iii) x2 ln x (iv) e3x sin (4x) (v) (vi)
x2 +1 x
Solution
d d d
(i) (x cos x) = x × (cos x) + cos x × (x) = x(− sin x) + cos x(1)
dx dx dx
= −x sin x + cos x
d  d −2x  d 
(ii) x3 e−2x = x3 × e + e−2x × x3 = x3 (−2e−2x ) + 3x2 e−2x
dx dx dx
= −2x3 e−2x + 3x2 e−2x = x2 e−2x (−2x + 3)

226 dkrbabajee@gmail.com
c
CHAPTER 13. DIFFERENTIATION 2

 
d 2 d d 1
(iii) (x ln x) = x2 × (ln x) + ln x × (2x) = x2 + ln x(x2 ) = x + 2x ln x
dx dx dx x
d 3x  d d 3x 
(iv) e sin (4x) = e3x × (sin (4x)) + sin (4x) × e
dx dx dx
= e3x (4 cos (4x)) + sin (4x)(3e3x ) = e3x (4 cos (4x) + 3 sin (4x)

  d d
d x (x + 1) × (x) − x × (x + 1) (x + 1)(1) − (x)(1) 1
(v) = dx dx = =
dx x+1 (x + 1) 2 (x + 1) 2 (x + 1)2

  d d
d sin x x× (sin x) − sin x × (x) x(cos x) − sin x(1) x cos x − sin x
(vi) = dx dx = =
dx x x2 x2 x2

Example 13.3 Find the stationary point of the curve y = x e−x and state
whether it is minimum or maximum.

Solution
dy
By Product Rule, = e−x − x e−x = e−x (1 − x)
dx
dy
for stationary point, = 0 ⇒ e−x (1 − x) = 0
dx
1
since e−x > 0, 1 − x = 0 ⇒ x = 1, y =
e
2
dy
2
= −e−x − (1 − x)e−x = (x − 2)e−x
dx
d2 y 1
2
=− <0
dx e
x=1

 
1
The stationary point is 1, and it is maximum.
e

e2x
Example 13.4 The curve y = has a stationary point for
sin x
π π
− < x < . Find the coordinates of the stationary point.
2 2

227 dkrbabajee@gmail.com
c
CHAPTER 13. DIFFERENTIATION 2

Solution
dy sin x(2e2x ) − cos x(e2x ) (2 sin x − cos x)e2x
By Quotient Rule, = = ,
dx sin2 x sin2 x
dy (2 sin x − cos x)e2x
for stationary point, =0⇒ = 0,
dx sin2 x
sin x
2 sin x − cos x = 0 ⇒ = 2 ⇒ tan x = 2
cos x
x = tan−1 (2) = 1.11, y = 10.2

The coordinates of the stationary point are (1.11, 10.2).


Exercise 13.1. [Derivatives, Product and Quotient Rules]
1. Find the derivatives of: x
(i) sin (2x + 1) (ii) 3 cos (3x + π/6) (iii) 2 tan (iv) 4e5x+4
5
(v) ln (2x + 3) (vi) ln (sin x) (vii) cos2 x (viii) sin3 (3x)

(ix) tan2 x (x) e2 sin (2x) (xi) e1−2 tan (3x) (xii) e2 sin x+5 cos x
2. Find the derivatives of:
(i) x sin x (ii) x2 e3x (iii) x3 ln x (iv) e2x tan x

x2 cos x
(v) (vi) (vii) sec (x) (viii) cosec(x)
5x − 2 x
sin2 x
(ix) cot x (x) e−x sin x (xi) (xii) x ln (cos x)
x
3. Find the smallest positive value of x for which there is a stationary value of the
following functions:
(i) 2x + 3 cos x (ii) x − 2 sin x (iii) 3 sin x + 5 cos x
4. Find the value of x for which the following
  functions have stationary values:
1
(i) 3x − e2x (ii) x2 + 3 ln x (iii) ln + 8x
x
5. For the following curves, find the gradient of the curve and the equations of
tangent and normal at the given point:
(i) y = sin x + cos x, x = π/2 (ii) y = x + ex , x = 0

(iii) y = 2 + x2 + ln x, x = 1 (iv) y = x sin x, x = π/2


6. Find the coordinates of a point of the curve y = 4x − 2 cos x when the tangent at
that point is parallel to the line y = 5x − 1.
7. Find the coordinates of a point of the curve y = 3 ln x + x when the normal at
that point is parallel to the line 2x + 3y = 4.

228 dkrbabajee@gmail.com
c
CHAPTER 13. DIFFERENTIATION 2

8. Find the stationary point of the curve y = (x + 3) e−x and state whether it is
minimum or maximum.
e2x π π
9. The curve y = has a stationary point for − < x < . Find the coordinates
cos x 2 2
of the stationary point.
ex
10. Find the stationary point of the curve y = and determine whether it is
x−1
minimum or maximum.

13.4 Implicit Differentiation


d dy d 2 d dy dy
If y is a function of x, then (y) = , (y ) = (y 2 ) × = 2y .
dx dx dx dy dx dx

Consider y = x2 + 2x + 2, then y is expressed as a function of x and we differentiate
dy 2x + 2 x+1 x+1
explicitly with respect to (w.r.t.) x to obtain = √ =√ = .
dx 2 x2 + 2x + 2 x2 + 2x + 2 y
dy
We can also write y 2 = x2 + 2x + 2. Differentiating implicitly w.r.t. x and making
dx
as a function of x and y, we have
dy dy 2x + 2 x+1
2y = 2x + 2 ⇒ = = .
dx dx 2y y
dy 1
Implicit differentiation can be used to derive = when y = ln x:
dx x
y = ln x ⇒ ey = x
dy
Differentiating implicitly w.r.t. x, ey =1
dx
dy 1 1
= y = .
dx e x
Implicit differentiation can also help to find the derivative of ax , where a is a constant:

y = ax ⇒ ln y = ln ax = x(ln a)
1 dy
Differentiating implicitly w.r.t. x, = ln a
y dx
dy
= (ln a)y = (ln a)(ax )
dx

229 dkrbabajee@gmail.com
c
CHAPTER 13. DIFFERENTIATION 2

Example 13.5 Find the gradient of the curve x3 − 5xy 2 + 2 = 0 at the


point (2, −1).

Solution:
Differentiating x3 − 5xy 2 + 2 = 0 implicitly w.r.t. x,
 
2 2 dy dy
3x − (1)5y − 5x 2y = 0 ⇒ 3x2 − 5y 2 − 10xy =0
dx dx
dy 3x2 − 5y 2
=
dx 10xy

dy 3(2)2 − 5(−1)2 7
= =−
dx 10(2)(−1) 20
x=2, y=−1

Example 13.6 Find the coordinates of the stationary points of the curve
x2 + xy + y 2 = 27.

Solution:

x2 + xy + y 2 = 27, (13.1)
dy dy
Differentiating implicitly w.r.t. x, 2x + y + x + 2y =0
dx dx
dy dy 2x + y
(x + 2y) = −(2x + y) ⇒ =−
dx dx x + 2y
dy
= 0 ⇒ 2x + y = 0 ⇒ y = −2x (13.2)
dx
Solve (13.1) and (13.2) x2 + x(−2x) + (−2x)2 = 27 ⇒ x2 − 2x2 + 4x2 = 27
3x2 = 27 ⇒ x2 = 9 ⇒ x = ±3, y = ∓6

The stationary points are (3, −6) and (−3, 6).

Example 13.7 The equation of a curve is given by 2x − 1 = x2 ln y.


dy −2y(x − 1)
(i) Show that = .
dx x3
(ii) Find the equation of the tangent to the curve at the point where
y = 1.

230 dkrbabajee@gmail.com
c
CHAPTER 13. DIFFERENTIATION 2

Solution:

(i)

2x − 1 = x2 ln y, (13.3)
x2 dy
Differentiating implicitly w.r.t. x,
2 = 2x ln y +
y dx
 
2x − 1 2x − 1 x2 dy
From (13.3), x ln y = ⇒2=2 +
x x y dx
2
 
2 x dy dy y 2 −2y(x − 1)
2=4− + ⇒ = 2 −2 =
x y dx dx x x x3

1
(ii) When y = 1, x = ,
2
 
dy −1 1
=  3 −1 = 4
dx 1 1 2
x= , y = 1
2 2
 
1
Equation of tangent at ,1
2
y−1
= 4 ⇒ y = 4x − 1
1
x−
2

Exercise 13.2. [Implicit Differentiation]


dy
1. Find for the following curve:
dx
(i) x2 + y 2 = 9 (ii) x2 − xy + y 2 = 0 (iii) y 2 = 3x + 2
1 1
(iv) + = ey (v) x ey = x + 2 (vi) 3 sin x + 2 sin y = 1
x y
dy
2. Find for the following curve:
dx
(i) y = 2x (ii) y = 3−x (iii) y = x(5x )

(iv) y = xx (v) xy = ex (vi) y x = ey

3. Find the gradient of the following curves at the given points:


(i) x3 + 2xy 2 = 3, (1, −1) (ii) x3 + xy + y 2 = 1, (−1, 1)

(iii) y ln x = y − 3, (1, 3) (iv) x2 + y 2 = 8, (−2, −2)

231 dkrbabajee@gmail.com
c
CHAPTER 13. DIFFERENTIATION 2

4. Find the coordinates of the stationary point/(s) of the following curve:


(i) x2 + y 2 = 1 (ii) x2 + xy + y 2 = 12 (iii) (1 + x)(1 + y) = x2

5. Find the equations of the tangents to the curve 2x2 + 3y 2 − 3x + 2y = 0 at the


points where x = 1.

6. Find the equation of the normal to the curve x2 + xy + y = 0 at the point where
x = 1.

7.* The equation of a curve is x2 y + xy 2 = 2. Find the coordinates of the point on the
curve at which the tangent is parallel to the x−axis.

8.* The equation of a curve is given by 3x − 2 = x2 ln y, y > 0.


dy y(4 − 3x)
(i) Show that = .
dx x3
(ii) Find the equation of the tangent to the curve at the point where y = 1.

13.5 Parametric Equations


By Chain Rule,
dy dy dt
= × ,
dx dt dx
dy
dy
= dt (13.4)
dx dx
dt

dy
Example 13.8 Find in terms of t for the following:
dx
(i) x = 3t2 and y = 2t3 ,

(ii) x = cos2 t and y = sin2 t,

(iii) x = t − ln t and y = t2 − ln t2 ,

(iv) x = 3(1 + cos2 t) and y = 2sin3 t.

232 dkrbabajee@gmail.com
c
CHAPTER 13. DIFFERENTIATION 2

Solution

(i)
dx dy
= 6t, = 6t2 ,
dt dt
dy
dy 6t2
= dt = = t.
dx dx 6t
dt

(ii)
dx dy
= −2 cos t sin t, = 2 sin t cos t,
dt dt
dy
dy 2 sin t cos t
= dt = = −1.
dx dx −2 cos t sin t
dt

(iii)
dx 1 dy 2t 2
=1− , = 2t − 2 = 2t − ,
dt t dt t t
dy 2 2(t2 − 1) 2(t + 1)(t − 1)
dy 2t −
= dt = t = t = t = 2(t + 1).
dx dx 1 t−1 t−1
1−
dt t t t

(iv)
dx dy
= −6 cos t sin t, = 6 sin2 t cos t,
dt dt
dy
dy 6 sin2 t cos t
= dt = = − sin t.
dx dx −6 cos t sin t
dt

Example 13.9 A curve P in xy plane is defined parametrically by


x = e2t , y = t3 , t ≥ 0.
dy
(i) Find in terms of t.
dx
(ii) Find the equation of the normal to the curve at the point where t = 1.

233 dkrbabajee@gmail.com
c
CHAPTER 13. DIFFERENTIATION 2

Solution
(i)
dx dy
= 2e2t , = 3t2 ,
dt dt
dy
dy 3t2
= dt = .
dx dx 2e2t
dt

(ii)

At t = 1, x = e2 and y = 1,

dy 3 2e2
= 2 ⇒ mN = −
dx 2e 3
t=1
Equation of normal at (e2 , 1) is
y−1 2e2
2
= − ⇒ 3y + 2e2 x = 2e4 + 3.
x−e 3

Exercise 13.3. [Parametric equations]


dy
1. Find in terms of t for the following:
dx
(i) x = 2t3 and y = 3t2 ,
(ii) x = sin2 t and y = cos2 t,
(iii) x = t + ln t and y = t2 − ln t2 ,
(iv) x = 2(1 + sin2 t) and y = 3 cos3 t,
(v) x = cos t and y = 2t − sin (2t),
t t2
(vi) x = and y = .
1−t 1−t
2. A curve P in xy plane is defined parametrically by x = e3t , y = t2 , t ≥ 0.
dy
(i) Find in terms of t.
dx
(ii) Find the equation of the normal to the curve at the point where t = 1.
3. Find the equations of the tangent and the normal to the curve defined parametrically
by x = t2 and y = 2t at the point where t = −2.
4. The parametric equations of a curve are given by x = cos3 t and y = sin2 t, where
0 < t < π/2.

234 dkrbabajee@gmail.com
c
CHAPTER 13. DIFFERENTIATION 2

dy
(i) Find in terms of t.
dx
3
(ii) Find the equation of the tangent to the curve at the point where y = .
4
5. Find the turning points of the curve whose parametric equations are given by
x = t and y = t3 − 3t.

6. The parametric equations of a curve are given by x = t2 + t and y = t2 + 2t. Find


dy
the value of at each of the points where y = 3.
dx
7. The parametric equations of a curve are x = t sin t, y = t cos t.
dy 4−π
Show that = at the point where t = π/4.
dx 4+π

13.6 Miscellaneous Exercises


Example 13.10 The parametric equations of a curve are
π
x = sin3 t, y = cos3 t, 0 < t < .
2
dy
(i) Express in terms of t.
dx
(ii) Show that the equation of the tangent to the curve at the point with
parameter t is y sin t + x cos t = sin t cos t.

(iii) Hence show that, if this tangent meets the x-axis at P and the y-axis
at Q, then the length of PQ is always equal to 1.

Solution
dx dy dy −3 cos2 t sin t cos t
(i) = 3 sin2 t cos t, = −3 cos2 t sin t ⇒ = 2 =−
dt dt dx 3 sin t cos t sin t

235 dkrbabajee@gmail.com
c
CHAPTER 13. DIFFERENTIATION 2

(ii)
cos t
mT = −
sin t
Equation of tangent at (sin3 t, cos3 t) is given by
y − cos3 t cos t
3 = −
x − sin t sin t
sin t(y − cos t) = − cos t(x − sin3 t)
3

y sin t − sin t cos3 t = −x cos t + sin3 t cos t


y sin t + x cos t = sin3 t cos t + sin t cos3 t
y sin t + x cos t = sin t cos t(sin2 t + cos2 t) ⇒ y sin t + x cos t = sin t cos t

(iii)

y = 0 ⇒ x cos t = sin t cos t ⇒ x = sin t ⇒ P (sin t, 0)


x = 0 ⇒ y sin t = sin t cos t ⇒ y = cos t ⇒ Q(0, cos t)
p p √
P Q = (0 − sin t)2 + (cos t − 0)2 = sin2 t + cos2 t = 1 = 1

Miscellaneous Exercise 13.

1. Find the x−coordinate of the stationary point of the curve y = ex cos x defined
for 0 ≤ x ≤ π and determine the nature of that point.
e2x
2. Find the x−coordinate of the stationary point of the curve y = defined for
sin x
0 < x < π/2.

3. The equation of a curve is given by x ey = 3x − 2.


dy 2
(i) Show that = .
dx x(3x − 2)
(ii) Find the equation of the normal to the curve at the point where y = 0.

4. The equation of a curve is 4xy(x + y) = 1. Show that there is only one point on
the curve at which the tangent is parallel to the x-axis, and find the coordinates
of this point.

5.* The equation of a closed curve is given by (x − y)2 + 2(x + y)2 = 24.
dy −(3x + y)
(i) Show that = .
dx x + 3y
(ii) Find the coordinates of the points where the tangent is parallel to the x−axis.
(iii) Find the coordinates of the points where the normal is parallel to the x−axis.

236 dkrbabajee@gmail.com
c
CHAPTER 13. DIFFERENTIATION 2

6. The parametric equations of a curve are x = 2t + sin (2t), y = 1 − cos (2t).


dy
Show that = tan t.
dx
π
7. The parametric equations of a curve are x = cos2 t, y = ln (tan t), where 0 < t < .
2
dy 2
(i) Show that =− 2 .
dx sin (2t)
(ii) Find the equation of the tangent to the curve at the point where y = 0.

8. The parametric equations of a curve are x = cos t, y = cos t + sin t, for


−π/2 < t < π/2.
dy
(i) Show that = 1 − cot t.
dx
(ii) Find the exact coordinates of the point where the tangent is parallel to the
line y = 2x + 1.
1 2
9.* The parametric equations of a curve are x = t − , y = t + .
t t
(i) Find the equation of the normal at the point where t = 1.
(ii) Find the coordinates of the point where this normal meets the curve again.

10.* The parametric equations of a curve are x = a cos t, y = a sin t, where a


π
is a positive constant and 0 < t < .
2
dy
(i) Express in terms of t.
dx
(ii) Show that the equation of the tangent to the curve at the point with parameter
t is y sin t + x cos t = a.
(iii) Hence show that, if this tangent meets the x-axis at P and the y-axis at Q,
then the length P Q = 2a cosec(2t).

237 dkrbabajee@gmail.com
c
C HAPTER 14

Integration 2

14.1 Objective
At the end of this chapter, students should be able to:
1. extend the idea of reverse differentiation to include the integration of eax+b ,
1
, sin (ax + b), cos (ax + b) and sec2 (ax + b);
ax + b
2. use trigonometrical relationships (such as double-angle formulae) to facilitate
the integration of functions such as cos2 x;
k f ′ (x)
3. recognise an integrand of the form using exact derivatives;
f (x)
4. integrate rational functions by means of decomposition into partial fractions;
5. recognise when an integrand can usefully be regarded as a product, and use
integration by parts to integrate;
6. use a given substitution to simplify and evaluate either a definite or an indefinite
integral;
7. use the trapezium rule to estimate the value of a definite integral, and use
sketch graphs in simple cases to determine whether the trapezium rule gives
an over-estimate or an under-estimate.

14.2 Integration of exponential and trigonometric


functions
The integration of exponential and trigonometric functions are given in the table
below:
Example 14.1 Integrate  x w.r.t x:
(i) 4 sin (4x) (ii) 3 cos (iii) sec2 (5x) (iv) tan2 (2x)
4
1
(v) e5x+2 (vi) (vii) sin2 x (viii) cos2 (3x)
4x + 1

238 dkrbabajee@gmail.com
c
CHAPTER 14. INTEGRATION 2

Z
f (x) f (x) dx

eax+b
eax+b +c
a
1 ln |ax + b|
+c
ax + b a
cos (ax + b)
sin (ax + b) − +c
a
sin (ax + b)
cos (ax + b) +c
a
tan (ax + b)
sec2 (ax + b) +c
a

Solution

(i) Z
4 cos (4x)
4 sin (4x) dx = − + c = − cos (4x) + c,
4
(ii) Z x x x
3
3 cos dx = sin + c = 12 sin + c,
4 1 4 4
4
(iii) Z
tan (5x)
sec2 (5x) dx = + c,
5
(iv) Z Z
2 tan (2x)
tan (2x) dx = sec2 (2x) − 1 dx = − x + c,
2
(v) Z
e5x+2
e5x+2 dx = + c,
5
(vi) Z
1 ln |4x + 1|
dx = + c,
4x + 1 4
(vii) Z Z
2 1 cos (2x) x sin (2x)
sin x dx = − dx = − + c,
2 2 2 4

239 dkrbabajee@gmail.com
c
CHAPTER 14. INTEGRATION 2

(viii) Z Z
2 1 cos (6x) x sin (6x)
cos (3x) dx = + dx = + + c.
2 2 2 12

Example 14.2 Evaluate


Z 1 Z 1 Z π/4 Z π/8
x 5
(i) e dx (ii) dx (iii) cos (3x)dx (iv) sin2 (2x)dx
0 0 3 − 2x 0 0

Solution
Z 1
(i) ex dx = [ex ]10 = e − 1
0
Z 1
5 5h i1 5 5
(ii) dx = − ln |3 − 2x| = − (ln 1 − ln 3) = ln 3
0 3 − 2x 2 0 2 2
Z π/4  π/4 √
sin (3x) 1 2
(iii) cos (3x) dx = = (sin (3π/4) − sin 0) =
0 3 0 3 6
Z π/8 Z π/8  π/8
2 1 cos (4x) x sin (4x)
(iv) sin (2x) dx = − dx = −
0 0 2 2 2 8 0
π sin (π/2) π 1
= − = − .
16 8 16 8
Exercise 14.1. [Integration of exponential and trigonometric functions]

1. Integrate w.r.t x:
3
(i) e3x (ii) 3 e−x (iii) e5x+3 (iv) 10e3−5x (v) −2e− 2 x (vi) 3e 2 −5
x

2. Evaluate,
Z 1 giving your
Z 1answer in exactZform: Z
4 3
2x x+1 3−x
e2− 3 dx
x
(i) e dx (ii) 3e dx (iii) e dx (iv)
0 −1 3 0

3. Integrate w.r.t x:
1 2 1 2 6 −5
(i) (ii) (iii) (iv) (v) (vi)
3x x 2x + 1 1 − 3x 3 + 2x 7 − 4x
4. Evaluate,
Z 3 giving your
Z 2 answer in exact
Z 4 form: Z −1
2 1 3 1
(i) dx (ii) dx (iii) dx (iv) dx
1 x 1 3x 3 x−2 −3 2−x
5. Integrate w.r.t x:
(i) 2 sin (3x) (ii) − 3 cos (7x) (iii) − 2 sin (5x − π/6) (iv) 3 cos (3x + π/3)
 
2 x
(v) 4 sec (vi) tan2 (x + π/4) (vii) cos2 x (viii) 3 sin2 (2x)
2

240 dkrbabajee@gmail.com
c
CHAPTER 14. INTEGRATION 2

6. Evaluate, giving your answer in exact form:


Z π/4 Z π/4 Z π/6
(i) sin (2x) dx (ii) cos (π/2 − 2x) dx (iii) sec2 (2x) dx
0 π/6 0
Z π/4 Z π/3 Z π/8
(iv) tan2 x dx (v) sin2 x dx (vi) cos2 (2x) dx
−π/4 0 0
Z k
1
7. (i) Find the exact value of k for which dx = 1.
2 x−1
Z k  
1 1 5
(ii) Find the value of k for which dx = ln .
0 2x + 3 2 3

8.* (i) Express 3 cos x − sin x in the form R cos (x + α), where R > 0 and
0 < α < π/2.
Z π/6
1
(ii) Hence, find the exact value of √ dx.
0 ( 3 cos x − sin x)2
9.* (i) By writing sin (3x) = sin(2x + x), show that sin (3x) = 3 sin x − 4 sin3 x.
Z π/3
(ii) Hence, evaluate sin3 x dx.
0

10.* (i) By writing cos (4x) = cos(2x + 2x), show that cos (4x) = 8 cos4 x − 8 cos2 x + 1.
Z π/4
(ii) Hence, evaluate cos4 x dx.
0

14.3 Integration using exact derivatives (P3 only)


Z
f ′ (x)
dx = ln |f (x)| + c.
f (x)

Example 14.3 Integrate w.r.t. x:


x2
(i) 3 (ii) cot (4x)
x +1

Solution

(i)

f (x) = x3 + 1, f ′ (x) = 3x2 ,


Z Z
x2 1 3x2 1
3
dx = 3
dx = ln |x3 + 1| + c,
x +1 3 x +1 3

241 dkrbabajee@gmail.com
c
CHAPTER 14. INTEGRATION 2

(ii)

f (x) = sin (4x), f ′ (x) = 4 cos (4x),


Z Z
1 4 cos (4x) 1

cot (4x) dx = dx = ln sin (4x) + c.
4 sin (4x) 4

Example 14.4 Evaluate:


Z 1 Z π/3
x
(i) dx (ii) tan (x) dx
0 x2 + 1 0

Solution

(i)

f (x) = x2 + 1, f ′ (x) = 2x,


Z 1 Z
x 1 2x 1h 2
i1 1
dx = dx = ln |x + 1| = ln 2,
0 x2 + 1 2 x2 + 1 2 0 2

(ii)

f (x) = cos x, f ′ (x) = − sin x,


Z Z h iπ/3
− sin x
tan x dx = − dx = − ln | cos x|
cos x 0
 
1
= − ln | cos (π/3)| + ln | cos 0| = − ln = ln 2.
2

14.4 Integration using Partial Fractions (P3 only)


It is easier to integrate expressions which can be decomposed into partial fractions.

Example 14.5 Evaluate the following integrals using partial fractions,


giving your answers in the form a ln b + c, where a, b and c are rational
numbers:
Z 1 Z 5
1 3x2 + 5
(i) dx (ii) 2
dx
0 (x + 3)(x + 4) 4 (x − 3)(x + 1)
Z 3 Z 1
3x3 − 1 2−x
(iii) 2
dx (iv) dx
1 x (3x − 1) 0 (x + 1)(x2 + 2)

242 dkrbabajee@gmail.com
c
CHAPTER 14. INTEGRATION 2

Solution

(i) The integrand is a proper partial fraction of Type 1:


1 A B
= + ,
(x + 3)(x + 4) x+3 x+4

1 1

A= = 1, B = = −1.
(x + 3)(x + 4) (x + 3)(x + 4)
x=−3 x=−4
Z 1 Z 1
1 1 1
dx = − dx
0 (x + 3)(x + 4) 0 x+3 x+4
h i1
= ln |x + 3| − ln |x + 4| = (ln 4 − ln 5) − (ln 3 − ln 4)
0
   
42 16
= 2 ln 4 − ln 5 − ln 3 = ln = ln .
5×3 15

(ii) The integrand is a proper partial fraction of Type 2:

3x2 + 5 A B C A(x + 1)2 + B(x + 1)(x − 3) + C(x − 3)


= + + = ,
(x − 3)(x + 1)2 x + 3 x + 1 (x + 1)2 (x − 3)(x + 1)2
3x2 + 5 = A(x + 1)2 + B(x + 1)(x − 3) + C(x − 3), (14.1)
put x = 3 in (14.1), 32 = 16A ⇒ A = 2
put x = −1 in (14.1), 8 = −4C ⇒ C = −2
comparing coefficient of x2 in (14.1), 3 = A + B ⇒ B = 3 − 2 = 1
Z 5 Z 5
3x2 + 5 2 1 2
2
dx = + − dx
4 (x − 3)(x + 1) 4 x+3 x + 1 (x + 1)2
h    
2 i5 1 2
= 2 ln |x − 3| + ln |x + 1| + = 2 ln 2 + ln 6 + − ln 5 +
x+1 4 3 5
   
1 4×6 1 24 1
= ln 4 + ln 6 − ln 5 − = ln − = ln − .
15 5 15 5 15

243 dkrbabajee@gmail.com
c
CHAPTER 14. INTEGRATION 2

(iii) The integrand is an improper partial fraction of Type 2:

3x3 − 1 B C D Ax2 (3x − 1) + Bx(3x − 1) + C(3x − 1) + Dx2


= A + + + = ,
x2 (3x − 1) x x2 (3x − 1) x2 (3x − 1)
3x3 − 1 = Ax2 (3x − 1) + Bx(3x − 1) + C(3x − 1) + Dx2 , (14.2)
3
comparing coefficient of x in (14.2), 3 = 3A ⇒ A = 1
put x = 0 in (14.2), −1 = −C ⇒ C = 1
1 8 D
put x = in (14.2), − = ⇒ D = −8
3 9 9
comparing coefficient of x2 in (14.2), 0 = −A + 3B + D ⇒ 3B = 9 ⇒ B = 3
Z 3 Z 3
3x3 − 1 3 1 8
2
dx = 1 + + − dx
1 x (3x − 1) 1 x x2 (3x − 1)
h 1 8 i3  1 8
 
8

= x + 3 ln |x| − − ln |3x − 1| = 3 + 3 ln 3 − − ln 8 − 1 − 1 − ln 2
x 3 1 3 3 3
 9
8 8 1 8 1 3 8
= − ln 4 + 3 ln 3 = (9 ln 3 − 16 ln 2) + = ln 16 +
3 3 3 3 3 2 3

(iv) The integrand is a proper partial fraction of Type 3:

2−x A Bx + C A(x2 + 2) + (Bx + C)(x + 1)


= + 2 = ,
(x + 1)(x2 + 2) x+1 x +2 (x + 1)(x2 + 2)
2 − x = A(x2 + 2) + (Bx + C)(x + 1), (14.3)
put x = −1 in (14.3), 3 = 3A ⇒ A = 1
comparing coefficient of x2 in (14.3), 0 = A + B ⇒ B = −A = −1
comparing constant terms in (14.3), 2 = 2A + C ⇒ 2 = 2 + C ⇒ C = 0
Z 1 Z 1
2−x 1 x
2
dx = − 2 dx
0 (x + 1)x + 2 0 x+1 x +2
Z 1 Z " #1 " #1
1 1 1 2x 1
= dx − 2 +2
dx = ln |x + 1| − ln |x2 + 2|
0 x + 1 2 0 x 2
0  0
1 1 1 1 8
= ln 2 − ln 3 + ln 2 = (3 ln 2 − ln 3) = ln .
2 2 2 2 3

Exercise 14.2. [Integration using exact derivatives and partial fractions (P3 only)]

1. Integrate w.r.t x using exact derivatives:


x3 ex sin x cos x
(i) 4 (ii) cot x (iii) tan 2x (iv) x
(v) (vi)
x +1 2e +1 2 − cos x 3 + 4 sin x

244 dkrbabajee@gmail.com
c
CHAPTER 14. INTEGRATION 2

2. Evaluate, giving your answer in exact form:


Z 1 Z π/2 Z 1 Z π/2
x2 x2 ex sin x
(i) dx (ii) dx (iii) dx (iv) dx
0 x3 + 1 π/6 x3 + 1 0 ex + 1 π/3 1 − cos x

3. Evaluate the following integrals using proper partial fraction of Type 1, giving
yourZanswers in the form a ln bZ+ c, where a, b and c are rational
Z 1 numbers:
1 4
1 5x + 2 1
(i) dx (ii) dx (iii) dx
0 (x + 1)(x + 2) 3 (x + 1)(x − 2) 0 (x + 1)(x + 2)(x + 3)

4. Evaluate the following integrals using proper partial fraction of Type 2, giving
yourZanswers in the form a lnZ b1+ c, where a, b and c are rational
Z 4 numbers:
1
3x2 + x + 1 3x2 + 4x + 2 1
(i) dx (ii) dx (iii) dx
0 x2 (x + 1) 0 (x + 1)(2x + 1)
2
3 (x − 2)(x − 1)
2

5. Evaluate the following integrals using proper partial fraction of Type 3, giving
yourZanswers in the form a ln b + Z c, where a, b and c are rational
Z 1 numbers:
1 1
4 + 4x − x2 1−x 2 − 18x − 9x2
(i) dx (ii) dx (iii) dx
0 (2 − x)(4 + x2 ) 0 (x + 1)(x2 + 1) 0 (2x + 3)(3x2 + 2)
6. Evaluate the following integrals using improper partial fraction of Type 1, giving
yourZanswers in the form a ln bZ+ c, where a, b and c are rational
Z 5 numbers:
2 2
x2 + 4x + 2 2x2 + 3x − 5 x2
(i) dx (ii) dx (iii) dx
0 (x + 1)(x + 3) 1 (x + 1)(2x − 1) 4 (x − 2)(x − 3)

7. Evaluate the following integrals using improper partial fraction of Type 2, giving
yourZanswers in the form a ln b Z+ c, where a, b and c are rational
Z 1 numbers:
3 3 1 3
x x +2 x3
(i) 2
dx (ii) 2
dx (iii) 2
dx
2 (x + 1)(x − 1) 0 (x + 1) (2x + 1) 0 (x + 2)(2x + 1)

8. Evaluate the following integrals using improper partial fraction of Type 3, giving
yourZanswers in the form a ln b + c, whereZ 1 a, b and c are rational numbers:
1
4 + 2x + 3x2 − x3 2 x3 − 19 x2 − 4 x − 6
(i) dx (ii) dx
0 (3 − x)(x2 + 1) 0 (2x + 1)(x2 + 2)
Z 1
2 x3 − 5 x2 + x − 3
(iii) dx
0 x(2x2 + 1)

14.5 Integration by parts (P3 only)


Consider the product rule
d dv du
(uv) = u + v .
dx dx dx

245 dkrbabajee@gmail.com
c
CHAPTER 14. INTEGRATION 2

Integrating on both sides, we have


Z Z Z
d dv du
(uv) dx = u dx + v dx,
dx dx dx
Z Z
dv du
uv = u dx + v dx,
dx dx
Z Z
dv du
u dx = uv − v dx,
dx dx
Z Z
u dv = uv − v du, (14.4)
Z b h ib Z b
u dv = uv − v du. (14.5)
a a a

Eq. (14.4) is called integration by parts.

Example 14.6 Integrate w.r.t. x:


(i) xex (ii) x2 sin x (iii) ln x

Solution

(i)

u = x, dv = ex
du = 1, v = ex ,
Z Z Z
xe dx = uv − v du = xe − ex dx = xex − x + c
x x

(ii)

u = x2 , dv = sin x
du = 2x, v = − cos x,
Z Z Z
2 2
x sin x dx = uv − v du = −x cos x + 2 x cos x dx

u = x, dv = cos x
du = 1, v = sin x,
Z Z Z
x cos x dx = uv − v du = x sin x − sin x dx = x sin x + cos x
Z
x2 sin x dx = −x2 cos x + 2x sin x + 2 cos x + c.

246 dkrbabajee@gmail.com
c
CHAPTER 14. INTEGRATION 2

(iii)

u = ln x, dv = 1
1
du = , v = x,
Z x Z Z   Z
1
ln x dx = uv − v du = x ln x − x× dx = x ln x − dx = x ln x − x + c.
x

Z 4
ln x
Example 14.7 Show that √ dx = 8 ln 2 − 4.
1 x

1 1
u = ln x, dv = √ = x− 2
x
1 1
du = , v = 2x 2 ,
x
Z 4 h i4 Z 4 h 1 i4 Z 4 1
ln x 1
√ dx = uv − v du = 2x ln x −
2 × 2x 2 dx
1 x 1 1 1 1 x
h 1 i4 Z 4 1
h 1
i4 h i
1 4
= 2x 2 ln x − 2x− 2 dx = 2x 2 ln x − 4x 2
1 1 1 1
2
= 4 ln 4 − (8 − 4) = 4 ln 2 − 4 = 8 ln 2 − 4.

Example 14.8
x
The diagram shows the curve y = (x − 2)e− 2 and its maximum point M.
The curve cuts x−axis at P and y−axis at Q.

(i) Find the x-coordinates of M.

(ii) Find the area of the region bounded by OP, OQ and the curve, giving
your answer in terms of e.

247 dkrbabajee@gmail.com
c
CHAPTER 14. INTEGRATION 2

Solution

(i)
dy 1  x −x
− x2 − x2
= e − (x − 2)e = 2 − e 2
dx 2 2
dy  x −x x
for maximum point =0⇒ 2− e 2 ⇒ 2 − = 0 ⇒ x = 4.
dx 2 2

(ii)
x
Where curve cuts x−axis, y = 0 ⇒ (x − 2)e− 2 = 0 ⇒ x = 2 ∴ P (2, 0)
Where curve cuts y−axis, x = 0 ⇒ y = (0 − 2)e0 = 0 ⇒ y = −2 ∴ Q(0, −2)
x
u = x − 2, dv = e− 2
x
du = 1, v = −2e− 2 ,
Z 2 h i2 Z 2 h i2 Z 2
− x2 − x2 x
area = (x − 2)e dx = uv − v du = − 2(x − 2)e − −2e− 2 dx
0 0 0 0 0
h i
x 2
h i
x 2 4
= − 2(x − 2)e− 2 − 4e− 2 = (0 − 4) − (4e−1 − 4) = −
0 0 e
4
area = , integral is negative since the region is below the x-axis.
e

Exercise 14.3. [Integration by parts (P3 only)]

1. Integrate w.r.t x by parts:


(i) 5x e3x (ii) x e−x (iii) x2 ex (iv) 3x sin x (v) x cos (2x)

(vi) x2 cos x (vii) ln (2x) (viii) x ln x (ix) x ln x

2. Evaluate, giving your answer in exact form:


Z π/2 Z π/4 Z 1 Z 1
2 x
(i) x cos x dx (ii) x sin x dx (iii) (x + 2)e dx (iv) x e−x dx
0 0 0 0
Z 2 Z 3 Z 2 Z 9
2 ln x
(v) ln x dx (vi) x ln x dx (vii) x ln x dx (viii) √ dx
1 2 1 1 x
3. A curve has equation y = x e−x .

(i) Find the coordinates of the stationary point of the curve and determine its
nature.
(ii) Find the area of the region bounded by the curve, x − axis and the lines
x = −1 and x = 0.

248 dkrbabajee@gmail.com
c
CHAPTER 14. INTEGRATION 2

4. A curve has equation y = x ln x, x > 0.

(i) Find the exact coordinates of the stationary point of the curve and determine
its nature.
(ii) Find the exact value of the area of the region bounded by the curve, x − axis
and the lines x = 1 and x = e.
x
5. The diagram shows the curve y = (x − 3)e− 3 and its maximum point M. The
curve cuts x−axis at P and y−axis at Q.

(i) Find the x-coordinates of M.

(ii) Find the area of the region bounded


by OP, OQ and the curve, giving your
answer in terms of e.

Example 14.9 Evaluate the following integrals using the respective


substitutions:
Z 1
1
(i) 2 2
dx, x = tan θ,
0 (1 + x )
Z 5
x
(ii) 3
dx, u = x − 3,
4 (x − 3)
Z 1 √
2
(iii) x2 1 − x2 dx, x = sin θ.
0

Solution

(i)

x = tan θ, dx = sec2 θ dθ, (1 + x2 )2 = (1 + tan2 θ)2 = sec4 θ,


π
x = 0 ⇒ θ = 0, x = 1 ⇒ θ = ,
4
Z 1 Z π 2 Z π Z π Z π
1 4 sec θ 4 1 4
2
4 1 cos 2θ
2 2
dx = 4
dθ = 2
dθ = cos θ dθ = + dθ
0 (1 + x ) 0 sec θ 0 sec θ 0 0 2 2
 π π  π 1
θ sin 2θ 4 π 1
= + = + sin = +
2 4 0 8 4 2 8 4

249 dkrbabajee@gmail.com
c
CHAPTER 14. INTEGRATION 2

(ii)

u = x − 3, du = dx, x = u + 3, x = 4 ⇒ u = 1, x = 5 ⇒ u = 2,
Z 5 Z 2 Z 2
x u+3 1 3
3
dx = 3
du = 2
+ 3 du
4 (x − 3) 1 u 1 u u
 2    
1 3 1 3 3 15
= − − 2 = − − − −1 − =
u 2u 1 2 8 2 8

(iii)
√ p
x = sin θ, dx = cos θ dθ, x2 1 − x2 = sin2 θ 1 − sin2 θ = sin2 θ cos θ,
1 π
x = 0 ⇒ θ = 0, x = ⇒ θ = ,
2 6
Z 1 √ Z π Z π
2 6 6
2 2
2
x 1 − x dx = sin θ cos θ cos θ dθ = (sin θ cos θ)2 dθ
0 0 0
Z π 2 Z π 2 Z π  π
6 sin 2θ 6 sin 2θ 6 1 cos 4θ θ sin 4θ 6
= dθ = dθ = − dθ = −
0 2 0 4 0 8 8 8 32 0
  √
π 1 2π π 3
= − sin = − .
48 32 3 48 64

Exercise 14.4. [ntegration using substitution (P3 only)]

1. Evaluate
Z 1 the following integrals using Z the respective substitutions:
1
x 2 ex
(i) 2 2
dx, u = x + 1 (ii) √ dx, u = 1 + ex
0 (x + 1) 1+e x
0
Z π/3 Z 4
sec2 x x
(iii) 3 dx, u = tan x (iv) 3
dx, u = x − 2
π/4 tan x 3 (x − 2)
Z 1/2 √ Z 4
2
1
(v) x 1 − x dx, x = cos θ (vi) 2
dx, u = ln x
0 2 x(ln x)
Z 1 Z 1
1 1
(vii) 2
dx, x = tan θ (viii) √ dx, x = sin θ
0 1+x 0 1 − x2
2. (i) Show that sin3 x = sin x − cos2 x sin x.
Z
(ii) Use the substitution u = cos x to find cos2 x sin x dx.
Z π/3
(iii) Hence, evaluate sin3 x dx.
0

250 dkrbabajee@gmail.com
c
CHAPTER 14. INTEGRATION 2

3. (i) Express x2 − 2x + 2 in the form (x − p)2 + q.


Z 2
1
(ii) Use the substitution x = 1 + tan θ to evaluate dx.
1 x2 − 2x + 2
4. (i) Express 3 − 2x − x2 in the form p − (x + q)2 .
Z 1
1
(ii) Use the substitution x + 1 = 2 sin θ to evaluate √ dx.
0 3 − 2x − x2
Z 1
r
2 1−x
5. Use the substitution x = sin θ to show that I = dx can be expressed
1/2 x
Z π/2
as I = 2 cos2 θ dθ and hence evaluate I.
π/4

14.6 Trapezium Rule


Z b  
f irst + last
f (x) dx ≈ h + all others , (14.6)
a 2
b−a
where h = is the width of the interval and n is the number of intervals (or
n
number of trapeziums). We note that we have (n + 1) coordinates:
     
x0 = a, y0 = f (a) , x1 , y1 = f (x1 ) , x2 , y2 = f (x2 ) , ...,
   
xn , yn = f (xn ) , xn+1 = b, yn+1 = f (b)

Z b
y0 + y1
n = 1, f (x) dx ≈ (b − a)
a 2
Z b  
b − a y0 + y2
n = 2, f (x) dx ≈ + y1
a 2 2
Z b  
b − a y0 + y3
n = 3, f (x) dx ≈ + y1 + y2
a 3 2

A function is said to be concave if every line segment joining two points on its graph
does not lie above the graph at any point.
A function is said to be convex if every line segment joining two points on its graph
does not lie below the graph at any point.
In general, the trapezium rule gives an underestimate for the integral of a concave

251 dkrbabajee@gmail.com
c
CHAPTER 14. INTEGRATION 2

function and an overestimate for the integral of a convex function.

Z 3
1
Example 14.10 Given that I = dx,
0 x+2
(i) find the true value of I,

(ii) use the trapezium rule to estimate I using


(a) 1 interval (b) 2 intervals (c) 3 intervals

(iii) compare your results with that in part (i).

Solution

(i)
" #3  
5
I= ln (x + 2) = ln 5 − ln 2 = ln = ln 2.5 = 0.9163.
2
0

3−0
h= =3
1  x 0 3
(ii) f irst + last
I ≈h + all others f (x) 0.5 0.2
2
 
0.5 + 0.2
I1 = 3 = 1.0500
2

3−0
h= = 1.5
2  x 0 1.5 3
f irst + last
I ≈h + all others f (x) 0.5 0.2857 0.2
2
 
0.5 + 0.2
I2 = 1.5 + 0.2857 = 0.9536
2

3−0
h= =1
3  x 0 1 2 3
f irst + last
I ≈h + all others f (x) 0.5 0.3333 0.25 0.2
2
 
0.5 + 0.2
I3 = 1 + 0.3333 + 0.25 = 0.9333
2

252 dkrbabajee@gmail.com
c
CHAPTER 14. INTEGRATION 2

(iii)

As shown in the figure, for 1 interval, we use one trapezium and the error
is 0.1337. For 2 intervals, we use 2 trapeziums and the error is 0.0393. For
3 intervals, we use 3 trapeziums and the error is 0.0170. As the number of
the intervals increases, the estimate becomes more accurate. In all 3 cases,
we observe that the it is an over-estimate since the area of the trapeziums is
greater than that of the curve (f (x) is convex).
Z π

2
Example 14.11 Given that I = x sin x dx, use the trapezium rule
0
with 3 ordinates to estimate I and determine whether it is over-estimate
or under-estimate.

Solution
π π
x 0
4 2
f (x) 0 0.7452 1.2533

π
−0
2 π
h= =
3 − 1 4 
f irst + last Exercise 14.5.
I ≈h + all others
2
 
π 0 + 1.2533
I≈ + 0.7452 = 1.0775
4 2

We observe that the it is an


under-estimate since the area of the
trapeziums is smaller than that of the
curve (f (x) is concave).

[Trapezium Rule]
Z 1
1
1. Given that I = dx,
0 x+3
(i) find the true value of I,
(ii) use the trapezium rule to estimate I using
(a) 1 interval (b) 2 intervals (c) 3 intervals

253 dkrbabajee@gmail.com
c
CHAPTER 14. INTEGRATION 2

(iii) compare your results with that in part (i).



2. The diagram shows the curve y = f (x) = x sin x.
Z π
2
Given that I = f (x) dx, use the
0
trapezium rule with 4 ordinates to
estimate I and determine whether it is
over-estimate or under-estimate.
3. Estimate the value of each definite integral using the trapezium rule:
(a) 3 ordinates (b) 3 intervals
Z 4 Z 3 Z π/3 Z 3
3 1 √
(i) x dx (ii) 2
dx (iii) cos x dx (iv) (ln x)2 dx
0 1 x 0 1
Z 4
4. (i) Use two trapezia of equal widths to estimate the value of ln x dx, giving
2
your answer correct to 3 significant figures.
(ii) Explain why with the aid of a sketch why this estimate is less than the true
value of the integral.
(iii) Use integration by parts to find the exact value of the integral, giving your
answer in the form ln (a) + b, where a and b are integers.

5. Use the trapezium rule with interval width 0.25 to find an approximate value
Z 2 x
e
of the integral dx, giving your answer correct to 2 decimal places.
1 x

254 dkrbabajee@gmail.com
c
CHAPTER 14. INTEGRATION 2

14.7 Miscellaneous Exercises



Example 14.12 The diagram shows the curve y = e−x x + 1 and its
maximum point M. The shaded region between the curve and the axes is
denoted by R.

(i) Find the x-coordinate of M.

(ii) Use the trapezium rule to estimate the area of the shaded region
using 4 intervals, giving your answer correct to 4 decimal places.

(iii) Find by integration the volume of the solid obtained when R is


rotated completely about the x-axis, giving your answer in terms
of π and e.
Solution
(i)
dy √ e−x
= −e−x x + 1 + √
dx 2 x+1
   
−x
√ 1 −x 2(x + 1) − 1
= −e x+1− √ = −e √
2 x+1 2 x+1
 
−x 2x + 1
= −e √
2 x+1
dy 1
= 0 ⇒ 2x + 1 = 0 ⇒ x = −
dx 2

(ii)

x −1 −0.75 −0.5 −0.25 0

f (x) 0 1.0585 1.1658 1.1120 1

255 dkrbabajee@gmail.com
c
CHAPTER 14. INTEGRATION 2

0 − −1
h= = 0.25
4 
f irst + last
I≈h + all others
2
 
0+1
I ≈ 0.25 + 1.0585 + 1.1658 + 1.1120 = 0.9591
2

(iii)
Z 0 Z 0
2
Volume = π y dx = π (x + 1)e−2x dx
−1 −1
−2x
u = x + 1,dv = e
e−2x
du = 1, v=− ,
2
Z 0 h i0 Z h Z
−2x
0
e−2x i0 0
e−2x
(x + 1)e dx = uv − v du = − (x + 1) − − dx
−1 −1 −1 2 −1 −1 2
h e−2x i0 h e−2x i0 1 1 e2 e2 3
= − (x + 1) − =− − + = −
 22 −1
 4 −1 2 4 4 4 4
e −3
Volume = π
4

Miscellaneous Exercise 14.

1. Integrate w.r.t x:  
2x 1
(i) 3 sin (2x) (ii) 4 cos (iii) sec2 (3x) (iv) tan2 x (v) e3x−2 (vi)
3 2x − 1
2. The diagram shows the region R bounded by the curve y = cos 2x and the lines
x = 0 and x = π/4
(i) Find the area of R.

(ii) Find the volume of solid formed


when R is rotated completely about
the x-axis, giving your answer in
terms of π.
1
3. The diagram shows the curves y = and y = 8x2 intersecting at the point P.
x
(i) Find the coordinates of P.

(ii) Find the exact value of the area of the


shaded region.

256 dkrbabajee@gmail.com
c
CHAPTER 14. INTEGRATION 2

4. The diagram shows the curve y = sin x + cos x on the interval 0 ≤ x ≤ π/2 and
its maximum point P.
(i) Find the exact coordinates of P.

(ii) Find the area of the shaded region.

(iii) Find the volume of solid formed


when the shaded region is rotated
completely about the x-axis, giving
your answer in terms of π.
d
5. (i) Show that (sec x) = sec x tan x.
dx
Z π/3 √
(ii) Hence, show that sec x tan x dx = 2 − 2
π/4

sec x(sec x + tan x)


(iii) By writing sec x = , show that
Z sec x + tan x
sec x dx = ln | sec x + tan x| + c.
Z 9
1
6. Let I = √ dx.
4 x(x − 1)
Z 3
√ 2
(i) Use the substitution u = x to show that I = du.
2 (u − 1)(u + 1)
(ii) Hence, find the exact value of I.

7. (i) Find the values of the constants P , Q, R and S such that

2x3 + 1 Q R S
2
=P + + 2 +
x (2x + 1) x x (2x + 1)
Z 2  
2x3 + 1 3 1 125
(ii) Hence, show that 2
dx = + ln .
1 x (2x + 1) 2 2 432
8.* The diagram shows the curve y = x2 e−2x and its maximum point M. The tangent
at the point P passes through the origin O.
(i) Find the x-coordinate of M.
1
(ii) Show that the x-coordinate of P is .
2
(iii) Find the exact value of the area of the
shaded region bounded by the curve,
the x-axis and the line x = 2.

257 dkrbabajee@gmail.com
c
CHAPTER 14. INTEGRATION 2

x
9.* The diagram shows the curve y = .
+4x2
(i) Find the coordinates of the two
turning points of the curve.

Use the substitution t = x2 + 4 to find


(ii) Z
x
2
dx.
x +4
(iii) Hence find the exact value of the
x-coordinate of the point P given that
the area of the shaded region is 2.

10.* The diagram shows the curve y = e−x x + 2 and its maximum point M. The
shaded region between the curve and the axes is denoted by R.
(i) Find the x-coordinate of M.

(ii) Use the trapezium rule to estimate


the area of the shaded region using 4
intervals, giving your answer correct
to 4 decimal places.

(iii) Find by integration the volume of


the solid obtained when R is rotated
completely about the x-axis, giving
your answer in terms of π and e.

258 dkrbabajee@gmail.com
c
C HAPTER 15

Numerical Solution of equations

15.1 Objective
At the end of this chapter, students should be able to:

1. locate approximately a root of an equation, by means of graphical considerations


and/or searching for a sign change;

2. understand the idea of, and use the notation for, a sequence of approximations
which converges to a root of an equation;

3. understand how a given simple iterative formula of the form xn+1 = g(xn )
relates to the equation being solved, and use a given iteration, or an iteration
based on a given rearrangement of an equation, to determine a root to a prescribed
degree of accuracy.

15.2 Root of an equation


The root of an equation f (x) = 0 is denoted by α such that f (α) = 0. The root of
an equation can be obtained in exact form (solutions are obtained by direct method).
Numerical methods are sought when it is not possible to obtain α in exact form.
Let us consider the quadratic equation f (x) = x2 − x − 1 = 0.
Using the Quadratic Formula, we obtain the two roots
√ √
1+ 5 1− 5
α1 = = 1.62, α2 = = −0.62.
2 2

15.3 Number of roots of an equation


The equation f (x) = 0 can be written as g(x) = h(x). The number of points of
intersection of the graphs of y = g(x) and y = h(x) give the number of roots of the
equation.

Example 15.1 Find the number of roots of the equation x2 − x − 1 = 0.

259 dkrbabajee@gmail.com
c
CHAPTER 15. NUMERICAL SOLUTION OF EQUATIONS

Solution
The equation x2 − x − 1 = 0 can be written
as x2 = x + 1. The diagram shows the
graphs of the curve y = x2 and the line
y = x + 1 which intersect at two points.
The number of roots of the equation
x2 − x − 1 = 0 is 2.

15.4 Location of the root


The Intermediate Value Theorem says
that if a continuous function f (x) in the
interval a ≤ x ≤ b has opposite signs
at the ends, that is f (a)f (b) < 0 , then
the root α of the equation f (x) = 0 lies
between a and b.

Example 15.2 Show that the positive root of the equation x2 − x − 1 = 0


lies between 1 and 2.
Solution
Let f (x) = x2 − x − 1. f (1) = −1 < 0 and f (2) = 1 > 0.
Since f (1) and f (2) have opposite signs or f (1)f (2) < 0, the positive root of f (x) = 0
lies between 1 and 2.

15.5 Determination of the root by Iterative Method

The diagram shows the graph of y = x2 − x − 1 which cuts the x-axis at the point
(α, 0). We know that the positive root α lies between 1 and 2. Suppose we want to

260 dkrbabajee@gmail.com
c
CHAPTER 15. NUMERICAL SOLUTION OF EQUATIONS

find an approximation to this root, one way is to consider the points (1, −1) and (2, 1).
The equation of the line joining these two points is given by y = 2x − 3 which cuts
the x−axis at (1.5, 0). So an approximation to the positive root is 1.5 which is nearer
to α = 1.62 than 1 or 2.
We need an iterative method to obtain successive approximations which converge to
α.

15.5.1 Converging Sequence


We introduce the concept of a converging sequence.
A sequence {xn }n=1,2,... is said to converge
to a fixed number α if lim xn = α.
n−→∞

For example, the sequence


1 1 1
, , , ... converge to 0 since
2 22 23
1
lim xn = lim =0
n−→∞ n−→∞ 2n

15.5.2 Fixed Point Method


The Fixed Point Method is an iterative method given by xn+1 = g(xn ), n = 1, 2, 3, ...
with x1 as the starting point. The method produces the successive approximations
x1 , x2 = g(x1 ), x3 = g(x2 ), x4 = g(x3 ), x5 = g(x4 ) , x6 = g(x5 ), x7 = g(x6 ) and so on.
To use the Fixed Point Method, we must

Step 1: rearrange the equation f (x) = 0 in the form x = g(x).

Step 2: use a starting point x1 which is either given or chosen as the midvalue of the
interval in which the root lies.

Step 3: use the iterative method for a suitable number of iterations so that we reach
the required degree of accuracy.

We can obtain different forms of x = g(x) from the equation f (x) = 0. Usually one
of the form will produce a sequence of approximations which converges to the root
while a different form can produce a sequence of approximations that diverges from
the root.
When the Fixed Point Method converges to the root α, then α = g(α).

Example 15.3 Use the Fixed Point Method to find the positive root of
x2 − x − 1 = 0 correct to 2 decimal places. Give the result of each iteration
to 4 decimal places.

261 dkrbabajee@gmail.com
c
CHAPTER 15. NUMERICAL SOLUTION OF EQUATIONS

Solution √
We rearrange x2 − x − 1 = 0 in the form x = x + 1 to obtain the iterative method
√ 1+2
xn+1 = xn + 1. Since the root lies between 1 and 2, we choose x1 = = 1.5. Then
2
√ √
x2 = x1 + 1 = 1.5 + 1 = 1.5811
√ √
x3 = x2 + 1 = 1.5811 + 1 = 1.6066
√ √
x4 = x3 + 1 = 1.6066 + 1 = 1.6145
√ √
x5 = x4 + 1 = 1.6145 + 1 = 1.6169
√ √
x6 = x5 + 1 = 1.6169 + 1 = 1.6177
√ √
x7 = x6 + 1 = 1.6177 + 1 = 1.6179

We observe that x5 = x6 = x7 = 1.62 correct to 2 decimal places. Thus α = 1.62. We


can also confirm that α = 1.62 correct to 2 decimal places since f (1.615)f (1.625) < 0.
The other form of x2 − x − 1 = 0 is given by x = x2 − 1 which results in the iterative
method xn+1 = xn 2 − 1. With x1 = 1.5, we have the sequence of approximations:

x2 = x1 2 − 1 = (1.5)2 − 1 = 1.25
x3 = x2 2 − 1 = (1.25)2 − 1 = 0.5625
x4 = x3 2 − 1 = (0.5625)2 − 1 = −0.6836
x5 = x4 2 − 1 = (−0.6836)2 − 1 = −0.5327
x6 = x5 2 − 1 = (−0.5327)2 − 1 = −0.7162
x7 = x6 2 − 1 = (−0.7162)2 − 1 = −0.4870

which diverge from the positive root.

Remark 15.1 We can use a calculator


p to obtain the approximations. We type
1.5 and press =. Then we type (Ans + 1) and press = to obtain x2 . Pressing =
successively gives x3 , x4 and so on.
Usually an iterative method xn+1 = g(xn ) will converge if |g ′ (x)| < 1.
Exercise 15.1. [Fixed point iterative method]

1. By sketching a suitable pair of graphs, find the number of roots of the following
equations:
(i) x2 = 5x + 3 (ii) x3 = x + 1
x
(iii) 3x + 1 = e (iv) 2x ex = 1
1
(v) x4 − 3x + 1 = 0 (vi) x2 − 1 =
x
(vii) 3x + 1 = 5(3x − 1) (viii) sin x = x − 1, x ∈ [0, 2π]
(ix) x2 = cos x, x ∈ [0, 2π] (x) ex = tan x, x ∈ [0, 2π]

262 dkrbabajee@gmail.com
c
CHAPTER 15. NUMERICAL SOLUTION OF EQUATIONS

2. Show one of the root of following equations lies in the following interval:
(i) x2 = 5x + 3, [5, 6] (ii) x3 = x + 1, [1, 1.5]
(iii) 3x + 1 = ex , [1.5, 2] (iv) 2x ex = 1, [0, 1]
1
(v) x4 − 3x + 1 = 0, [0, 1] (vi) x2 − 1 = , [1.2, 1.4]
x
(vii) 3x + 1 = 5(3x − 1), [0, 1] (viii) sin x = x − 1 [1.9, 2]
(ix) x2 = cos x, [0.5, 1] (x) ex = tan x, [1, 1.5]
3. Use the following Fixed Point Methods xn+1 = g(xn ) with a suitable starting
point to find the root of the equations in part 2, giving your answer correct to 2
decimal places.
√ Give the result of each iteration
p to 4 decimal places.
(i) g(xn ) = 5xn + 3 (ii) g(xn ) = (xn + 1)
3

e−xn
(iii) g(xn ) = ln (3xn + 1) (iv) g(xn ) =
2
r
x4n + 1 1
(v) g(xn ) = (vi) g(xn ) = 1 +
3 xn
 xn 
1 3 +6
(vii) g(xn ) = ln (viii) g(xn ) = 1 + sin xn
ln 3 5

(ix) g(xn ) = cos xn (x) g(xn ) = tan−1 (exn )

15.6 Application of Fixed iteration Method


Example 15.4 The diagram shows a sector OPQ of a circle with centre
π
O and radius 5 cm. The angle P OQ is α radians, where 0 < α < . The
2
△ ORQ is a right-angled triangle and its area is equal to the area of the
region QRP.
(i) Show that α satisfies the
equation sin (2x) = x.

(ii) By sketching a suitable pair


of graphs, show that this
equation has exactly one root in
π
the interval 0 < x < .
2
(iii) Use the iterative formula
xn+1 = sin (2xn ) with x1 = 1
to find α correct to 2 decimal
places. Give the result of each
iteration to 4 decimal places.

263 dkrbabajee@gmail.com
c
CHAPTER 15. NUMERICAL SOLUTION OF EQUATIONS

Solution

(i)
1
area of △ ORQ = × OR × QR
2
1 25 25
= (5 cos α)(5 sin α) = (2 sin α cos α) = sin (2α)
2 4 4
1 25
area of sector OPQ = (5)2 (α) = α
2 2
area of region QRP = area of sector OPQ − area of △ ORQ
25 25
= α− sin (2α)
2 4
area of △ ORQ = area of region QRP
25 25 25
sin (2α) = α − sin (2α)
4 2 4
sin (2α) = 2α − sin (2α)
2 sin (2α) = 2α ⇒ sin (2α) = α

The diagram shows the graph of y = x and


(ii) y = sin (2x) which intersects at only one
π
point in the interval 0 < x < .
2

(iii)

x2 = sin (2x1 ) = sin 2(1) = 0.9093


x3 = sin (2x2 ) = sin 2(0.9093) = 0.9695
x4 = sin (2x3 ) = sin 2(0.9695) = 0.9330
x5 = sin (2x4 ) = sin 2(0.9330) = 0.9567
x6 = sin (2x5 ) = sin 2(0.9567) = 0.9419
x7 = sin (2x6 ) = sin 2(0.9486) = 0.9514

We observe that x5 = x6 = x7 = 0.95 correct to 2 decimal places.


Thus α = 0.95. We can also confirm that α = 0.95 correct to 2 decimal places
since f (0.945)f (0.955) < 0 with f (x) = sin (2x) − x.

264 dkrbabajee@gmail.com
c
CHAPTER 15. NUMERICAL SOLUTION OF EQUATIONS

Example 15.5 The equation of a curve is y = x2 + x ln x.

(i) Show that x−coordinate of the stationary point of the curve satisfies
the equation 2x + ln x + 1 = 0.

(ii) By sketching a suitable pair of graphs, show that this equation has
exactly one root.

(iii) Show by calculation that the root lies between 0.1 and 0.3.

(iv) Use the iterative formula xn+1 = e−(2xn +1) to find α correct to 2
decimal places. Give the result of each iteration to 4 decimal places.
Solution
(i)
dy d 2 d d
= (x ) + ln x (x) + x (ln x)
dx dx dx   dx
dy 1
= 2x + ln x(1) + x = 2x + 1 + ln x
dx x
dy
For stationary point = 0 ⇒ 2x + 1 + ln x = 0
dx

The diagram shows the graph of y = ln x


(ii) and y = −2x − 1 which intersects at only
one point.

(iii) Let f (x) = 2x + 1 + ln x. f (0.1) = −1.10 < 0 and f (0.3) = 0.40 > 0.
Since f (0.1) and f (0.3) have opposite signs or f (0.1)f (0.3) < 0, the root of
f (x) = 0, α lies between 0.1 and 0.3.
0.1 + 0.3
(iv) Since the root lies between 0.1 and 0.3, we choose x1 = = 0.2.
2
x2 = e−(2x1 +1) = e−(2(0.2)+1) = 0.2466
x3 = e−(2x2 +1) = e−(2(0.2466)+1) = 0.2247
x4 = e−(2x3 +1) = e−(2(0.2247)+1) = 0.2347
x5 = e−(2x4 +1) = e−(2(0.2347)+1) = 0.2300
x6 = e−(2x5 +1) = e−(2(0.2300)+1) = 0.2322
x7 = e−(2x6 +1) = e−(2(0.2322)+1) = 0.2312

265 dkrbabajee@gmail.com
c
CHAPTER 15. NUMERICAL SOLUTION OF EQUATIONS

We observe that x5 = x6 = x7 = 0.23 correct to 2 decimal places.


Thus α = 0.23. We can also confirm that α = 0.23 correct to 2 decimal places
since f (0.225)f (0.235) < 0.
Z α
1
Example 15.6 Given that x e−x dx = ,
0 2
(i) Show that α satisfies the equation 2(x + 1) = ex .

(ii) By sketching a suitable pair of graphs, show that this equation has
exactly one root in the interval 0 < x < 2.

(iii) Show by calculation that the root lies between 1.6 and 1.8.
 
(iv) Use the iterative formula xn+1 = ln 2(xn + 1) to find α correct to 2
decimal places. Give the result of each iteration to 4 decimal places.

Solution

(i)

u = x, dv = e−x
du = 1, v = −e−x ,
Z α h iα Z α h iα Z α
x x
x e dx = uv − v du = − x e − −e−x (1) dx
0 0 0 0 0
h iα
x −x −α
= −x e −e = 1 − (α + 1)e
0
Z α
1
x ex dx =
0 2
1 1
1 − (α + 1)e−α = ⇒ (α + 1)e−α = ⇒ 2(α + 1) = eα
2 2

The diagram shows the graph of y = ex


(ii) and y = 2(x + 1) which intersects at only
one point in the interval 0 < x < 2.

(iii) Let f (x) = 2(x + 1) − ex . f (1.6) = 0.25 > 0 and f (1.8) = −0.45 < 0.
Since f (1.6) and f (1.8) have opposite signs or f (1.6)f (1.8) < 0, the root of
f (x) = 0, α lies between 1.6 and 1.8.

266 dkrbabajee@gmail.com
c
CHAPTER 15. NUMERICAL SOLUTION OF EQUATIONS

1.6 + 1.8
(iv) Since the root lies between 1.6 and 1.8, we choose x1 = = 1.7.
2
   
x2 = ln 2(x1 + 1) = ln 2(1.7 + 1) = 1.6864
   
x3 = ln 2(x2 + 1) = ln 2(1.6864 + 1) = 1.6813
   
x4 = ln 2(x3 + 1) = ln 2(1.6813 + 1) = 1.6795
   
x5 = ln 2(x4 + 1) = ln 2(1.6795 + 1) = 1.6788
   
x6 = ln 2(x5 + 1) = ln 2(1.6788 + 1) = 1.6785

We observe that x4 = x5 = x6 = 1.68 correct to 2 decimal places.


Thus α = 1.68. We can also confirm that α = 1.68 correct to 2 decimal places
since f (1.675)f (1.685) < 0.
Exercise 15.2. [Application of fixed point iteration method]
1
1. (i) Show by calculation that the root of the equation 2x = (x + 3) lies between
2
−3 and −2.
(ii) Use the iterative formula xn+1 = 2(1+xn ) − 3 to find the root correct to
2 decimal places. Give the result of each iteration to 4 decimal places.

2. (i) By sketching a suitable pair of graphs, show that the equation ex = x2 + 2


has exactly one root.
(ii) Use the iterative formula xn+1 = ln (x2n + 2) with x1 = 1.3 to find the root
correct to 2 decimal places. Give the result of each iteration to 4 decimal
places.
6
3. (i) By sketching a suitable pair of graphs, show that the equation ln (10x) =
x
has exactly one root.
6
(ii) Use the iterative formula xn+1 = with x1 = 2 to find α correct to
ln (10xn )
4 decimal places. Give the result of each iteration to 6 decimal places.

4. (i) By sketching suitable graphs, show that the equation x2 + 1 = cot x has only
π
one root in the interval 0 < x < .
2
 
−1 1
(ii) Use the iterative formula xn+1 = tan with x1 = 0.6 to determine
1 + x2n
the root correct to 2 decimal places. Give the result of each iteration to
4 decimal places.

267 dkrbabajee@gmail.com
c
CHAPTER 15. NUMERICAL SOLUTION OF EQUATIONS

b = 2θ, arc PQ has


5. In a sector POQ with radius r and centre O. Given that P OQ
length 10 cm and PQ has length 8 cm.

(i) Show that θ satisfies the equation 5 sin x = 4x.


(ii) By sketching a suitable pair of graphs, show that this equation has exactly
one root in the interval 0 < x < π/2.
(iii) Show by calculation that the root lies between 1 and 1.5.
5 sin (xn )
(iv) Use the iterative formula xn+1 = to find θ correct to 2 decimal
4
places. Give the result of each iteration to 4 decimal places.

6. The diagram shows a sector OPQ of a circle with centre O and radius 5 cm.
π
The angle P OQ is α radians, where 0 < α < . The △ ORQ is a right-angled
2
triangle and the area of the region QRP is 6.25 cm2 .
(i) Show that α satisfies the equation
2x − sin (2x) = 1.

(ii) By sketching a suitable pair of


graphs, show that this equation has
exactly one root in the interval
π
0<x< .
2
(iii) Use the iterative formula
sin (2xn ) + 1
xn+1 = with x1 = 1 to
2
find α correct to 2 decimal places.
Give the result of each iteration to
4 decimal places.
7. The equation of a curve is y = x cos x.

(i) Show that x−coordinate of the stationary point of the curve satisfies the
equation x tan x = 1.
(ii) By sketching a suitable pair of graphs, show that this equation has exactly
one root α in the interval 0 < x < π/2.
(iii) Show by calculation that the root lies between 0.5 and 1.
 
−1 1
(iv) Use the iterative formula xn+1 = tan to find α correct to 2 decimal
xn
places. Give the result of each iteration to 4 decimal places.

268 dkrbabajee@gmail.com
c
CHAPTER 15. NUMERICAL SOLUTION OF EQUATIONS

Z α
1
8. Given that (1 − sin x) dx = ,
0 2
(i) Show that α satisfies the equation 2x = 3 − 2 cos x.
(ii) By sketching a suitable pair of graphs, show that this equation has exactly
one root in the interval 0 < x < π/2.
3 − 2 cos xn
(iii) Use the iterative formula xn+1 = with x1 = 0.8 to find α correct
2
to 2 decimal places. Give the result of each iteration to 4 decimal places.

9.* (i) Express y = 1 − 5x − 2x2 in the form y = a − 2(x + b)2 and find its turning
point.
(ii) Hence, by sketching a suitable pair of graphs, show that the equation
x3 + 2x2 + 5x − 1 = 0 has exactly one root α.
(iii)Show by calculation that α lies between 0 and 0.5.
1
(iv) Use the iterative formula xn+1 = − (x3n + 2x2n − 1) to find α correct to
5
3 decimal places. Give the result of each iteration to 5 decimal places.
xn a
10. The sequence of values given by the iterative formula xn+1 = + with initial
√ 2 xn
value x1 = 1.5 converges to 2.

(i) Find the value of a.


(ii) Use the iterative formula together with the value of a obtained in part (i) to
find α correct to 2 decimal places. Give the result of each iteration to
4 decimal places.

15.7 Miscellaneous Exercises


Example15.7 The sequence of values given by the iterative formula
4 1
xn+1 = xn + 3 with initial value x1 = 1.5 converges to α.
5 xn
(i) Use this iterative formula to find α correct to 2 decimal places. Give
the result of each iteration to 4 decimal places.

(ii) State an equation satisfied by α, and hence find the exact value of
α.

269 dkrbabajee@gmail.com
c
CHAPTER 15. NUMERICAL SOLUTION OF EQUATIONS

Solution

(i)
   
4 1 4 1
x2 = x1 + = 1.5 + = 1.4370
5 x1 3 5 (1.5)3
   
4 1 4 1
x3 = x2 + = 1.4370 + = 1.4192
5 x2 3 5 (1.4370)3
   
4 1 4 1
x4 = x3 + = 1.4192 + = 1.4152
5 x3 3 5 (1.4192)3
   
4 1 4 1
x5 = x4 + = 1.4152 + = 1.4144
5 x4 3 5 (1.4152)3
   
4 1 4 1
x6 = x5 + = 1.4144 + = 1.4143
5 x5 3 5 (1.4144)3
   
4 1 4 1
x7 = x6 + = 1.4143 + = 1.4142
5 x6 3 5 (1.4143)3

We observe that x5 = x6 = x7 = 1.41 correct to 2 decimal places. Thus α = 1.41.

(ii)
 
4 1
x= x+ 3
5 x
4 4 1
x= x+
5 5 x3
1 4 1
x=
5 5 x3
4 1 1 √
x = 3 ⇒ x4 = 4 ⇒ x = 4 4 = 2 2 = 2
x √
∴ α= 2

Miscellaneous Exercise 15.


 
9 1
1. The sequence of values given by the iterative formula xn+1 = xn + 3 with
10 xn
initial value x1 = 1.75 converges to α.

(i) Use this iterative formula to find α correct to 2 decimal places. Give the
result of each iteration to 4 decimal places.
(ii) State an equation satisfied by α, and hence find the exact value of α.
b = θ, arc PQ has
2. In a sector POQ with radius r and centre O. Given that P OQ
2
length 5 cm and the area of triangle POQ is 26 cm .

270 dkrbabajee@gmail.com
c
CHAPTER 15. NUMERICAL SOLUTION OF EQUATIONS

(i) Show that θ satisfies the equation 25 sin x = 52x2 .


(ii) By sketching a suitable pair of graphs, show that this equation has exactly
one root in the interval 0 < x < π/2.
(iii) Show by calculation that the root lies between 0.4 and 0.5.
r
25
(iv) Use the iterative formula xn+1 = sin xn to find θ correct to 2 decimal
52
places. Give the result of each iteration to 4 decimal places.

3. The equation of a curve is y = x e−x .

(i) The tangent at the point where x = α is parallel to the line 2y = x + 1.


Show that α satisfies the equation 2(1 − x) = ex .
exn
(ii) Use the iterative formula xn+1 = 1 − with initial value x1 = 0.3 to find
2
α correct to 3 decimal places. Give the result of each iteration to 5 decimal
places.

4. The equation x3 − x − 1 = 0 has one real root.

(i) Show by calculation that this root lies between x = 1 and x = 2.


(ii) Prove that, if a sequence of values given by the iterative formula
2x3n + 1
xn+1 = 2 converges, then it converges to this root.
3xn − 1
(iii) Use this iterative formula to calculate the root correct to 2 decimal places.
Give the result of each iteration to 4 decimal places.

5. (i) Show that the equation x3 + 2x2 − 5 = 0 may be arranged in the form
r
a
x= and state the values of a and b.
x+b
r
a
(ii) Hence using the iterative formula xn+1 = with x1 = 1 together
xn + b
with the values of a and b to find the root of the equation correct to 2 decimal
places. Give the result of each iteration to 4 decimal places.

6. The equation of a curve is given by y = x3 + 2x2 + x + 1.

(i) Find the turning points of the curve and determine their nature.
(ii) Hence, sketch the curve and show that it has only one real root α.
(iii) Show by calculation that the root lies between −2 and −1.

271 dkrbabajee@gmail.com
c
CHAPTER 15. NUMERICAL SOLUTION OF EQUATIONS

(iv) Prove that, if a sequence of values given by the iterative formula


2x3n + 2x2n − 1
xn+1 = converges, then it converges to this root.
3x2n + 4xn + 1
(v) Use this iterative formula to find the root correct to 3 decimal places. Give
the result of each iteration to 5 decimal places.
2
7. The equation of a curve is given by y = ex cos x.
(i) Show that x−coordinate of the stationary point of the curve satisfies the
equation 2x − tan x = 0.
(ii) By sketching a suitable pair of graphs, show that this equation has exactly
one root α in the interval 0 < x < π/2.
(iii) Show by calculation that the root lies between 1 and 1.5.
(iv) Use the iterative formula xn+1 = tan−1 (2xn ) to find α correct to 2 decimal
places. Give the result of each iteration to 4 decimal places.
Z α
8. Given that ln x dx = 2,
1

(i) Show that α satisfies the equation x(ln x − 1) = 1.


(ii) By sketching a suitable pair of graphs, show that this equation has exactly
one root.
(iii) Show by calculation that the root lies between 3 and 4.
(iv) Use the iterative formula xn+1 = e(1+ xn ) to find α correct to 2 decimal places.
1

Give the result of each iteration to 4 decimal places.


9.* The diagram shows a sector OPQ of a circle with centre O and radius 5 cm.
π
The angle P OQ is α radians, where 0 < α < . The △ ORQ is a right-angled
2
triangle and the perimeter of the region QRP is 10 cm.
(i) Show that sin x − cos x can be
expressed as R sin (x − θ), where
R > 0 and 0 < θ < π/2.

Show that α satisfies the equation


(ii) √
2 sin (x − π/4) + x = 1.

(iii) Use the iterative formula


 
−1 1 − xn π
xn+1 = sin √ + with
2 4
x1 = 0.9 to find α correct to
2 decimal places. Give the result of
each iteration to 4 decimal places.

272 dkrbabajee@gmail.com
c
CHAPTER 15. NUMERICAL SOLUTION OF EQUATIONS

10.* (i) By sketching a suitable pair of graphs, show that the equation
π
cosec(x) = x2 − 1, where x is in radians, has a root in the interval 0 < x < .
2
(ii) Verify by calculation that this root lies between 1.2 and 1.6.
r
1 + sin x
(iii) Show that this root also satisfies the equation x = .
sin x
(iv) Use an iterative formula based on the equation in part (iii) to determine the
root correct to 2 decimal places. Give the result of each iteration to
4 decimal places.

273 dkrbabajee@gmail.com
c
C HAPTER 16

Vectors 2

16.1 Objective
At the end of this chapter, students should be able to:

1. understand the significance of all the symbols used when the equation of a
straight line is expressed in the form r = a + tb;

2. determine whether two lines are parallel, intersect or are skew;

3. find the angle between two lines, and the point of intersection of two lines when
it exists;

4. find the perpendicular distance of a line from a point;

5. find a vector v perpendicular to both vectors v1 and v2 ;

6. understand the significance of all the symbols used when the equation of a
plane is expressed in either of the forms ax + by + cz = d or (r − a) · n = 0 ;

7. determine whether a line lie in a plane, it is parallel to the plane or it intersects


with a plane at a point.

8. find the equation of a plane given

(a) a point on the plane and a normal vector to the plane,


(b) three points on the plane,
(c) two lines;

9. find the angle between two planes;

10. find the angle between a line and a plane;

11. find the perpendicular distance of a point from a plane;

12. find the line of intersection of two planes.

274 dkrbabajee@gmail.com
c
CHAPTER 16. VECTORS 2

16.2 Line
The equation of a straight line in 2 dimensions is given by y = |{z}
m x+ c .
|{z}
gradient y-intercept
In analogy and from the diagram below, r = a + λ b is the vector equation of a
line that is parallel to a vector b (also called the direction vector) and which passes
through a point A with position vector a.

−→
AR is parallel to b.
−→
AR = λ b,
r − a = λ b,
r = a + λ b.

Example 16.1 Find the equation vector form for the following

(a) line passing through the point with position vector i + j + k and
parallel to 2i − j − 2k.

(b) a line passing through a point A(1, 2, 3) and point B(−1, 3, 2).

Solution
(a)

a = i + j + k, b = 2i − j − 2k,
r = a + λ b ⇒ r = i + j + k + λ (2i − j − 2k).

(b)
−→
a = OA = i + 2j + 3k,
−→ −−→ −→
b = AB = OB − OA = −i + 3j + 2k − (i + 2j + 3k) = −2i + j − k,
r = a + λ b ⇒ r = i + 2j + 3k + λ (−2i + j − k)

275 dkrbabajee@gmail.com
c
CHAPTER 16. VECTORS 2

16.3 Point on a line


To show that a point lie on a line, we verify that we obtain the same value when
equating the position vector of the point with the line.

Example 16.2 Show that the point with the position vector i + 3j − 2k
lies on the line with vector equation r = 5i + j + 2k + λ (2i − j + 2k).

Solution

r = (5 + 2λ)i + (1 − λ)j + (2 + 2λ)k = i + 3j − 2k.

We have
5 + 2λ = 1, 1 − λ = 3, 2 + 2λ = −2,
λ = −2, λ = −2, λ = −2

this implies that λ has the same value, the point lies on the same line.

16.4 Pairs of lines


16.4.1 Parallel lines
Two lines L1 and L2 with their respective direction vectors b1 and b2 are said to be
parallel if b1 = kb2 , where k is a scalar.

Example 16.3 Show that the lines


       
1 3 2 6
L1 : r =  0  + t  −1  , L2 : r =  1  + s  −2 
2 4 3 8

are parallel.

Solution
   
3 6
b1 =  −1  , b2 =  −2  , b2 = 2b1 .
4 8

Since b1 and b2 are parallel, L1 and L2 are also parallel.

16.4.2 Intersection of two lines


Two lines intersect at a point otherwise they are skew.

276 dkrbabajee@gmail.com
c
CHAPTER 16. VECTORS 2

Example 16.4 Find the point of intersection of the lines

r = i + k + λ (i − 2j − 3k) and r = 3i − 5k + µ (−i + 3j + 3k).

Solution
   
1+λ 3−µ
r =  −2λ  , r =  3µ .
1 − 3λ −5 + 3µ
Equating, we have

1 + λ = 3 − µ, (16.1)
−2λ = 3µ, (16.2)
1 − 3λ = −5 + 3µ. (16.3)

We solve eqs. (16.1) and (16.2). From eq. (16.1), λ = 2 − µ.


Replacing λ = 2 − µ in (16.2), we get 2(2 − µ) = −3µ ⇒ µ = −4 and λ = 6.
Substituting µ = −4 and λ = 6 in eq. (16.3), we have 1 − 3λ = −17 = −5 + 3µ.
Since µ = −4 and λ = 6 satisfy eq. (16.3),
 the lines intersect.
1+6 7
Point of intersection =  −2(6)  =  −12 .
1 − 3(16) −17

16.4.3 Skew Lines


When two lines do not intersect, they are called skew lines.

Example 16.5 Determine whether the lines with their respective vector
equations

r = i + k + λ (−i + 3j + 3k) and r = 2i + 3j + µ (i − j + k)

intersect or not.

Solution
   
1−λ 2+µ
r =  3λ  , r =  3 − µ .
1 + 3λ µ
Equating, we have

1 − λ = 2 + µ, (16.4)
3λ = 3 − µ, (16.5)
1 + 3λ = µ. (16.6)

277 dkrbabajee@gmail.com
c
CHAPTER 16. VECTORS 2

We solve eqs. (16.4) and (16.5). From eq. (16.4), −µ = 1 + λ.


Replacing −µ = 1 + λ in (16.5), we get 3λ = 3 + 1 + λ ⇒ λ = 2 and µ = −3.
Substituting µ = −3 and λ = 2 in eq. (16.6), we have 1 + 3λ = 7 6= −3 = µ.
Since µ = −3 and λ = 2 do not satisfy eq. (16.6), the lines are skew.
Since they do not intersect, they are skew.

Example 16.6 The lines with their respective vector equations

r = ai + j + k + λ (3i − 2j + k) and r = −2i + 3j + µ (2i + 3j − k)

intersect , find the value of a and the position vector of the point of
intersection.

Solution
   
a + 3λ −2 + 2µ
r =  1 − 2λ  , r =  3 + 3µ .
1+λ −µ
Equating, we have

a + 3λ = −2 + 2µ, (16.7)
1 − 2λ = 3 + 3µ, (16.8)
1 + λ = −µ. (16.9)

We solve eqs. (16.8) and (16.9). From eq. (16.9), µ = −1 − λ.


Replacing µ = −1 − λ in (16.8), we get 1 − 2λ = 3 − 3 − 3λ ⇒ λ = −1 and µ = 0.
Substituting µ = 0 and λ = −1in eq. (16.7), we have a − 3 = −2 ⇒ a = 1.
−2
Point of intersection =  3  = −2i + 3j.
0
Exercise 16.1. [Vector equation of line]

1. Find the vector equation form for the following:

(i) line passing through the point with position vector i + 2j + 3k and parallel
to 2i − j.
(ii) a line passing through the point A(−1, 2, 6) and point B(0, −2, 2).

2. Show that the point with the position vector i−2j+ 3k lies on the line with vector
equation r = i − 2j + 3k + λ (2i + 2j − 5k).

3. Find the value of a and of b for which the point with the position vector ai + bj
lies on the line with vector equation r = 4i + j − 2k + λ (i + j + k).

4. Determine whether the following lines are parallel, skew or intersecting. If they
intersect, find the position vector of their point of intersection.

278 dkrbabajee@gmail.com
c
CHAPTER 16. VECTORS 2

(i) L1 : r = i − j − k + λ (−3i + 5j − 3k), L2 : r = −2i + j + k + µ (6i − 10j + 6k)


(ii) L1 : r = 5i + 3j + 4k + λ (3i − 3j − k), L2 : r = 3i + 2j + 2k + µ (9i − 9j − 3k)
(iii) L1 : r = −i − j − k + λ (−4i − 5j − 3k), L2 : r = 2i + 4j − 3k + µ (−10i − 15j + 5k)
(iv) L1 : r = i + 3j + k + λ (−2i + j − 2k), L2 : r = 2i + j + 3k + µ (i − 2j − 2k)
(v) L1 : r = i + 2j + 5k + λ (2i − 3j − 2k), L2 : r = 6i + 2k + µ (−i − 4j − k)
(vi) L1 : r = 5i + 2j + 5k + λ (−3i + 3j − k), L2 : r = −i + 12j + 3k + µ (3i − 2j + k)

5.* The lines L1 and L2 with their respective vector equations


r = ai + 2j + k + λ (−4i + j − k), r = 2i + 5j + µ (3i + 2k)
intersect , find the value of a and the position vector of the point of intersection.

16.5 Angle between two lines


Step 1: Obtain the direction vector b1 from the line L1 with vector equation
r1 = a1 + λb1 .
Obtain the direction vector b2 from the line L2 with vector equation
r2 = a2 + λb2 .

Step 2: The angle θ between two lines L1 and L2 is the angle between b1 and b2 and it
b1 · b2
is given by cos θ = .
|b1 | |b2 |

Example 16.7 Find the acute angle between the lines

r = 4i − j + k + λ (i + 2k) and r = i − j + 2k + µ (3i + 2j − k).

Solution
   
1 3
b1 =  0  , b2 =  2  ,
2 −1
√ √
b1 · b2 = (1 × 3) + (0 × 2) + (2 × −1) = 1, |b1 | = 5, |b2 | = 14,
   
|b1 · b2 | 1
θ = cos −1
= cos−1
√ √ = 83.10 .
|b1 | |b2 | 5 14

279 dkrbabajee@gmail.com
c
CHAPTER 16. VECTORS 2

Method 1

Step 1: Let Q be the foot of the


perpendicular from P. Since Q lies
−→ −→ −→
on the line, we find P Q = OQ − OP
in terms of λ.
−→
Step 2: P Q is perpendicular to the direction
−→
vector b. We use P Q · b = 0 to
obtain an equation in λ.

Step 3: By solving the equation we obtain a


−→ of λ and we find the distance
value
|P Q|.
Method 2: Pythagoras Theorem

Step 1: Let Q be the foot of the


perpendicular from P. Find
−→ −→
AP = OP − a and its magnitude
−→
|AP |.
−→ −→
Step 2: AQ is a projection of AP along the
−→ −→
line (e.g AQ = AP cos θ).
Using dot-product
−→ −→
AP · b = |AP | |b| cos θ
−→ −→
−→ | AP · b|
|AQ| = |AP cos θ| =
|b|

Step 3: Using Pythagoras


q Theorem,
−→ −→ −→
|P Q| = |AP |2 − |AQ|2 .

16.6 Perpendicular distance of a line from a point


Suppose we want to find the perpendicular distance of a line with vector equation
−−→
r = a + λb, from a point P with position vector OP.

Example 16.8 Find the perpendicular distance from the point with
position vector 3i − k to the line r = i − 2j + k + λ (3i + 4j + 5k)

280 dkrbabajee@gmail.com
c
CHAPTER 16. VECTORS 2

Solution
Method 1
   
3 1 + 3λ
−→  −→
OP = 0  and any point Q on the line has position vector OQ =  −2 + 4λ .
−1 1 + 5λ
   
−2 + 3λ 3
−→ −→ −→ 
P Q = OQ − OP = −2 + 4λ  , b =  4 
2 + 5λ 5
   
−2 + 3λ 3
−→   
PQ · b = 0 ⇒ −2 + 4λ · 4 =0
2 + 5λ 5
3(−2 + 3λ) + 4(−2 + 4λ) + 5(2 + 5λ) = 0 ⇒ 50λ = 4 ⇒ λ = 0.08
   
−1.76 −44
−→  −→ 1 
OQ = −1.68  P Q = −42 
25
2.4 60
−→ 1 hp i √292
|P Q| = (−44)2 + (−42)2 + (60)2 = .
25 5
Method 2:
     
3 1 3
−→
b =  4 , a =  −2  , OP =  0 
5 1 −1
 
2
−→ −→ −→
AP = OP − a =  2  , |AP |2 = 22 + 22 + (−2)2 = 12
−2
   
2 3 √ √
−→   
AP · b = 2 · 4  = (2 × 3) + (2 × 4) + (−2 × 5) = 4, |b| = 32 + 42 + 52 = 50
−2 5
s  2 r √
−→ q
−→ |AP · b| 4 −→ −→ 2 −→ 2 4 292 292
|AQ| = = √ ⇒ |P Q| = |AP | − |AQ| = 12 − √ = =
|b| 50 50 25 5

Exercise 16.2. [Angle between two lines and perpendicular distance of a line from
a point]

1. Find the angle between the two lines

(i) r = 3i − 2j + k + λ (i + j + k) and r = 2i + 3j + 5k + µ (2i + 3j + 4k),


(ii) r = i − j + k + λ (−2i + 6j − 9k) and r = i + k + µ (i + 4j + 8k),
(iii) r = 4j + 3k + λ (2i − j − 2k) and r = i + 3j + 3k + µ (−6i − 6j + 7k),

281 dkrbabajee@gmail.com
c
CHAPTER 16. VECTORS 2

(iv) r = i − 5j + λ (3i − 4j + 5k) and r = 5i + k + µ (6i + 3j + 2k),


(v) r = 2i − 3j + 6k + λ (i − 2j + 2k) and r = 6i − 2j − 3k + µ (4i + 5j + 3k).

2. Find the position vector of the foot of the perpendicular from the point P with
position vector p to the line with vector equation r. Hence, find the perpendicular
distance from the point to the line.

(i) p = 4i + 3j − k and r = i + 5j − 3k + λ (2i − j + 2k),


(ii) p = −2i + 2j − 5k and r = 2i + 3j + 4k + λ (i + 3j + 2k),
(iii) p = 6i + 3j − 2k and r = −2i + j − 2k + λ (i − 3j),
(iv) p = 2i + 3j + k and r = i + j + k + λ (2i − j + 4k).

3. Find the perpendicular distance from the point P with position vector p to the
line with vector equation r.

(i) p = 8i + 4j − k and r = 4j − 3k + λ (5i + 12k),


(ii) p = 2i + 3j + 4k and r = i − 3j + λ (−2i + j − 2k),
(iii) p = 5i + 3j + 2k and r = 2i − 5j − k + λ (8i − 4j − k),
(iv) p = 5i + 12k and r = 8i + 4j − k + λ (4j − 3k),

16.7 Finding a vector v perpendicular to both vectors


v1 and v2
16.7.1 Method 1: Dot products
Let v = ai + bj + ck, v1 = a1 i + b1 j + c1 k and v2 = a2 i + b2 j + c2 k.

Step 1: Carry out v · v1 = 0 and v · v2 = 0 to obtain two simultaneous equations in 3


unknowns:
a1 a + b1 b + c1 c = 0,
a2 a + b2 b + c2 c = 0.

Step 2: Solve the simultaneous equations to obtain expressions for b and c in terms of
a only.

Step 3: Reduce the ratio a : b : c in simplest form to obtain v.

Example 16.9 Find a perpendicular vector to both vectors 3i + j − k and


i + 4j − 2k.

282 dkrbabajee@gmail.com
c
CHAPTER 16. VECTORS 2

Solution
         
a a 3 a 1
Let v =  b . Now  b  ·  1  = 0 and  b  ·  4  = 0 yield
c c −1 c −2

3a + b − c = 0, (16.10)
a + 4b − 2c = 0 (16.11)
5
2×(16.10) − (16.11) gives 5a − 2b = 0 ⇒ b = a
2
11
c = 3a + b = a
2
h 5 11 i 2
a : b : c = a : a : a × = 2 : 5 : 11
2 2 a
∴ v = 2i + 5j + 11k

16.7.2 Method 2: Cross Products


Babajee ? has proved that the cross product method can be obtained from the dot
product method to find the perpendicular vector.

Step 1:

v = v1 × v2

i j k

= a1 a2 a3
b1 b2 b3

which is the determinant of a 3 × 3 matrix.

Step 2: To calculate the determinant,



i j k

a1 a2 a3 = i a2 a3 − j a1 a3 + k a1 a2
b2 b3 b1 b3 b1 b2
b1 b2 b3

where the determinants of 2 × 2 matrices are computed as usual.

283 dkrbabajee@gmail.com
c
CHAPTER 16. VECTORS 2

In example 16.9,

i j k

v = 3 1 −1
1 4 −2

1 −1
= i − j 3 −1 + k 3 1
4 −2 1 −2 1 4
= (−2 + 4)i − (−6 + 1)j + (12 − 1)k = 2i + 5j + 11k

The cross product method is more simpler to use.

16.8 Planes
The vector equation of a plane is given by ax + by + cz = d .

n = ai + bj + ck is called the normal vector of the plane.


 
x
If r =  y  and A is a point with position vector a on the plane, then the equation
z
of the plane is given by r · n = a · n .

16.8.1 Point on a plane


Suppose that the equation of a plane is given by r · n = d, a point A with position
vector a

1. lies on the plane if a · n = d,

2. does not lie on the plane if a · n 6= d.

Example 16.10 Verify whether the points A (1, 1, 0) and B (2, −1, 2) lie
on the plane x + y + z = 2.

284 dkrbabajee@gmail.com
c
CHAPTER 16. VECTORS 2

Solution
   
1 1
a =  1 , n =  1 , a · n = 1 + 1 + 0 = 2
0 1
   
2 1
b=  −1  , n =  1  , a · n = 2 − 1 + 2 = 3 6= 2.
2 1

Therefore point A lies on the plane while point B does not lie on the plane.

16.8.2 Line and plane


Consider a line with vector equation r = a + λb and a plane ax + by + cz = d. The
normal to the plane is n.

Case 1: If r · n = d, then the line lie in the plane.

Case 2: If r · n = c 6= d, where c is a constant independent of λ, then the line is parallel


to the plane.

Case 3: If r · n = d is an equation involving λ, the line intersect with the plane at the
point whose position vector can be obtained by solving for λ.

In cases 1 and 2, the normal to the plane is perpendicular to the direction vector of
the line.
Example 16.11 Determine whether the given lines are parallel to,
contained or intersect the plane 2x + y + z = 4. It they intersect, find
the position vector of their point of intersection.

(i) r = i + j + k + λ (−2i + 3j + k)

(ii) r = 2i − j + 3k + λ (i − j − k)

(iii) r = −i + 2j − 4k + λ (i + j + k)

Solution

(i)
   
1 − 2λ 2
r =  1 + 3λ  , n =  1  , r · n = 2(1 − 2λ) + (1 + 3λ) + (1 + λ) = 4
1+λ 1

The line lies on the plane.

285 dkrbabajee@gmail.com
c
CHAPTER 16. VECTORS 2

(ii)
   
2+λ 2
r =  −1 − λ  , n =  1  , r · n = 2(2 + λ) + (−1 − λ) + (3 − λ) = 6 6= 4
3−λ 1

The line is parallel to the plane.


(iii)
   
−1 + λ 2
r =  2 + λ , n =  1 
−4 + λ 1
r · n = 2(−1 + λ) + (2 + λ) + (−4 + λ) = 4 ⇒ 4λ = 8 ⇒ λ = 2
   
−1 + 2 1
Point of Intersection is  2 + 2  =  4 
−4 + 2 −2

The line intersects the plane at (1, 4, −2).

16.8.3 Finding the equation of a plane


Case 1: Given a point with position vector a and the normal vector n
 
x
The equation of the plane is given by  y  · n = a · n.
z

Example 16.12 Find the equation of the plane containing the point with
position vector 2i − 3j + k and having the vector i − 2j − k perpendicular
to the plane.
Solution
   
2 1
a =  −3  , n =  −2  ,
1 −1
 
x
 y ·n= a·n
z
       
x 1 2 1
 y  ·  −2  =  −3  ·  −2 
z −1 1 −1
x − 2y − z = 2 + 6 − 1 ⇒ x − 2y − z = 7

286 dkrbabajee@gmail.com
c
CHAPTER 16. VECTORS 2

Case 2: Given three points A , B and C


−→ −−→
Step 1: Find two direction vectors, e.g. AB and BC.
−→ −−→
Step 2: Obtain the normal vector (e.g use cross product to get n = AB × BC).
 
x
−→
Step 3: The equation of the plane is given by  y  · n = OA · n.
z

Example 16.13 Find the equation of the plane containing the points A
(5, 1, 1), B (3, −2, 0) and C (2, 3, −4).

Solution
     
5 3 2
−→   −−→  −→
OA = 1 , OB = −2  , OC =  3 
1 0 −4
   
−2 −1
−→ −−→ −→  −−→ −→ −−→
AB = OB − OA = −3  , BC = OC − OB =  5 
−1 −4

i j k
−→ −−→
n = AB × BC = 2 −3 −1
−1 5 −4

−3 −1 −2 −1 −2 −3
= i − j
−1 −4 + k −1 5

5 −4
= (12 + 5)i − (8 − 1)j + (−10 − 3)k = 17i − 7j − 13k
 
x
 y ·n=− →
OA · n
z
       
x 17 5 17
 y  ·  −7  =  1  ·  −7 
z −13 1 −13
17x − 7y − 13z = 65

Case 3: Given two lines with vector equation r1 = a1 + λb1 and r2 = a2 + µb2
Step 1: Obtain the two direction vectors b1 and b2 .
Step 2: Obtain the normal vector (e.g use cross product to get n = b1 × b2 ).
 
x
Step 3: The equation of the plane is given by  y  · n = a1 · n.
z

287 dkrbabajee@gmail.com
c
CHAPTER 16. VECTORS 2

Example 16.14 Find the equation of the plane containing the lines L1
and L2 with respective vector equations r1 = i + 2j − k + λ(i + j + k) and
r2 = 3i + j + k + µ(−i + j − k).

Solution
     
1 −1 1
b1 =  1  , b2 =  1  , a1 =  2 
1 −1 −1

i j k

n = b1 × b2 = 1 1 1
−1 1 −1

1 1 1 1 1 1
= i − j + k
1 −1 −1 −1 −1 1
= (−1 − 1)i − (−1 + 1)j + (1 + 1)k = −2i + 2k
 
x
 y  · n = a1 · n
z
       
x −2 1 −2
 y · 0 = 2 · 0 
z 2 −1 2
h i
− 2x + 2z = −4 ÷ 2 ⇒ −x + z = −2

Exercise 16.3. [Planes]


1. Find a vector perpendicular to the vectors v1 and v2 :
(i) v1 = i + j + k and v2 = 2i − 3j + 4k,
(ii) v1 = 3i − 5j + 7k and v2 = −3i + j − k,
(iii) v1 = −i + 3k and v2 = −5j + 2k,
(iv) v1 = −2i − 5j − 4k and v2 = −3i + 2j + 7k,
(v) v1 = 12i + 8j − 9k and v2 = i + j − k.
2. Verify whether the points A (1, 1, 1) and B (1, −1, −1) lie on the plane
2x − 3y + 4z = 1.
 
3
3. Show that the point (1, 2, −3) lies in both the planes with equations r ·  1  = 2
  1
2
and r ·  5  = 15.
−1

288 dkrbabajee@gmail.com
c
CHAPTER 16. VECTORS 2

4. Determine whether the given lines are parallel to, contained or intersect the
plane x + 2y + 5z = 8. It they intersect, find the position vector of their point
of intersection.
(i) r = i + j + k + λ (3i + j − k)
(ii) r = 2i − j + 3k + λ (4i + 3j − 2k)
(iii) r = −i + 2j − 4k + λ (2i + 2j + 3k)
5. Find the position vector of the point of intersection of the following line L and
plane Π:
(i) L : r = i + j + k + λ (3i + j − k), Π : 2x + 4y − 5z = 2,
(ii) L : r = 2i − j + 3k + λ (4i + 3j − 2k), Π : x + y + z = 7,
(iii) L : r = −i + 2j − 4k + λ (2i + 2j + 3k), Π : −3x + 3y + z = 1.
6. Find the equation of the following planes in the form ax + by + cz = d:
(i) passing through the point (1, 2, −1) and normal to the vector 2i − j − k,
(ii) passing through the point (1, 2, −1) and parallel to the plane 5x+y+7z = 20,
(iii) passing through the point (1, −1, −3) and parallel to both vectors −i + j and
j − k,
(iv) passing through the points A (1, 2, 3), B (2, −1, 2) and C (3, 1, −1),
(v) passing through the points A (4, −1, 2), B (0, 0, 3) and C (−1, 2, 0),
(vi) containing the lines L1 and L2 with respective vector equations
r1 = 3i + 4j − k + λ(2i + 3j + 4k) and r2 = 3i + j + 5k + µ(i + 2j + k),
(vii) containing the lines L1 and L2 with respective vector equations
r1 = 4i + 3j + k + λ(i + 2j + 2k) and r2 = j + 4k + µ(4i + 2j − 3k).

16.8.4 Angle between two planes


Step 1: Obtain the normal vector n1 from the plane Π1 .
Obtain the normal vector n2 from the plane Π2
Step 2: The angle θ between two planes Π1 and Π2 is the angle between n1 and n2 and
n1 · n2
it is given by cos θ = .
|n1 | |n2 |

In case we need to find the acute angle we use |n1 · n2 |.

Example 16.15 Find the acute angle between the planes


Π1 : x + y − 2z = 5 and Π2 : 2x + 3y − z = 1.

289 dkrbabajee@gmail.com
c
CHAPTER 16. VECTORS 2

Solution
   
1 2
n1 =  1  , n2 =  3  ,
−2 −1
√ √
n1 · n2 = (1 × 2) + (1 × 3) + (−2 × −1) = 7, |n1 | = 6, |n2 | = 14,
   
|n1 · n2 | 7
θ = cos −1
= cos−1
√ √ = 40.20 .
|n1 | |n2 | 6 14

16.8.5 Angle between a plane and a line


Step 1: Obtain the normal vector n from the equation of plane Π .
Obtain the direction vector b from the vector equation of the line r.

n·b
Step 2: Find the angle between n and b using cos θ = .
|n| |b|

Step 3: The angle between the plane and the line is

(i) 900 − θ if θ is acute,


(ii) θ − 900 if θ is obtuse.

Example 16.16 Find the angle between the plane Π : 2x + y + 3z = 5


and the line r = 6i + j + 5k + λ(2i − 3j − 2k).

Solution
   
2 2
n =  1  , b =  −3  ,
3 −2
√ √
n · b = (2 × 2) + (1 × −3) + (3 × −2) = −5, |n| = 14, |b| = 17,
   
n·b −5
θ = cos−1
= cos−1
√ √ = 108.90
|n| |b| 14 17
angle between the plane and the line is θ − 900 = 108.90 − 900 = 18.90 .

16.8.6 Finding the perpendicular distance of a point from a


plane
Method 1
Suppose we want to find the perpendicular distance of a plane with equation
ax + by + cz = d from a point P(x1 , y1, z1 ).

290 dkrbabajee@gmail.com
c
CHAPTER 16. VECTORS 2

Step 1: Let Q be the foot of the perpendicular from P.


−→ −→
Since P Q is parallel to the normal, n to the plane, we have P Q = k n,
where k is a constant.
−→ −→ −→
Step 2: We obtain OQ = OP + kn and OQ · n = d as an equation in k.

Step 3: By solving the equation we obtain a value of k and we find the distance
−→
|P Q| = |k| |n|.

General Formula
   
a x1
−→
n =  b  , OP =  y1 
c z1
 
x1 + ka
−→ −→
OQ = OP + kn =  y1 + kb 
z1 + kc
−→
OQ · n = d ⇒ a(x1 + ka) + b(y1 + kb) + c(x1 + kc) = d
−(ax1 + by1 + cz1 ) + d
(a2 + b2 + c2 )k + ax1 + by1 + cz1 = d ⇒ k =
a2 + b2 + c2
−→ |ax1 + by1 + cz1 − d| √ 2 |ax1 + by1 + cz1 − d|
|P Q| = |k| |n| = 2 2 2
a + b2 + c2 = √ .
a +b +c a2 + b2 + c2

Method 2

Step 1: Let Q be the foot of the


perpendicular from P. Choose a
point A on the plane by setting two
variables to zero in the equation of
−→ −→ −→
the plane. AP = OP − OA and its
−→
magnitude |AP |.
−→ −→
Step 2: QP is a projection of AP along the
−→ −→
line (e.g QP = AP cos θ).
Using dot-product
−→ −→
AP · n = |AP | |n| cos θ
−→ −→
QP −→ |AP · n|
| | = |AP cos θ| =
|n|

291 dkrbabajee@gmail.com
c
CHAPTER 16. VECTORS 2

Example 16.17 Find the foot of the perpendicular of the point i − 2j − 3k


from the plane 5x − 4y + z = 8. Hence obtain the perpendicular distance.

Solution
Method 1
   
5 1
−→
n =  −4  , OP =  −2 
1 −3
−→ −→
Since P Q is parallel to n, P Q = k n
 
1 + 5k
−→ −→
OQ = OP + kn =  −2 − 4k 
−3 + k
−→
OQ · n = 8 ⇒ 5(1 + 5k) − 4(−2 − 4k) + (−3 + k) = 8
1
5 + 25k + 8 + 16k − 3 + k = 8 ⇒ k = −
  21
8
−→ 2 
∴ OQ = −19 
21
−32
−→ 1p 2 1√
|P Q| = |k| |n| = 5 + (−4)2 + 12 = 42.
21 21
We can obtain the perpendicular distance directly using Method 2.

let, x = y = 0 in 5x − 4y + z = 8 ⇒ z = 8
   
0 1
−→   −→ −→ −→ 
OA = 0 , AP = OP − OA = −2 
8 −11
   
1 5 √
−→
AP · n =  −2  ·  −4  = (1 × 5) + (−2 × −4) + (−11 × 1) = 2, |n| = 42
−11 1
−→ √ √
−→ |AP · n| 2 2 42 42
|QP | = =√ = =
|n| 42 42 21

16.8.7 Intersection of 2 planes


Two planes Π1 : a1 x+ b1 y + c1 z = d1 and Π2 : a2 x+ b2 y + c2 z = d2 intersect in a line with
vector equation r = a + λb. There are two methods to obtain the vector equation of
the plane.

292 dkrbabajee@gmail.com
c
CHAPTER 16. VECTORS 2

Method 1

Step 1: Obtain the normal vector n1 from the plane Π1 .


Obtain the normal vector n2 from the plane Π2

Step 2: Find the direction vector b (e.g. b = n1 × n2 ).

Step 3: Obtain a, point on both planes by setting x = 0 in both equations of the planes
and obtain the values of y and z.

Method 2

Step 1: Obtain a, point on both planes by setting x = 0 in both equations of the planes
and obtaining the values of y and z.

Step 2: Obtain c another point on both planes by setting y = 0 in both equations of the
planes and obtaining the values of x and z.

Step 3: Find the direction vector b = c − a.

Example 16.18 Find the vector equation of the line of intersection of the
planes Π1 : x + y − 3z = 6 and Π2 : 2x − y + z = 4.

Solution
Method 1
   
1 2
n1 =  1  , n2 =  −1 
−3 1

i j k

b = n1 × n2 = 1 1 −3
2 −1 1

1 −3 1 −3
= i − j + k 1 1
−1 1 2 1 2 −1
= (1 − 3)i − (1 + 6)j + (−1 − 2)k = −2i − 7j − 3k.

To obtain a, set x = 0 in

x + y − 3z = 6, 2x − y + z = 4 (16.12)

to obtain

y − 3z = 6, −y + z = 4 (16.13)

293 dkrbabajee@gmail.com
c
CHAPTER 16. VECTORS 2

 
0
Solving (16.13) we get y = −9 and z = −5 so that a =  −9 .
−5
The vector equation of the line is

r = a + λb = −9j − 5k + µ(2i + 7j + 3k), µ = −λ.

Method 2
Obtain another point c by setting y = 0 in (16.12) to obtain

x − 3z = 6, 2x + z = 4 (16.14)
 18   
2
18 8 
7
 9 
Solving (16.14) we get x = and z = − so that c = 0 and b = c − a = 7 .
7 7 7
− 87 3
Exercise 16.4. [Angle between planes, perpendicular distance of a point from a
plane, intersection of two planes]

1. Find the acute angle between the following planes Π1 and Π2 :

(i) Π1 : 2x − y − 3z = 7 and Π2 : x + 2y + 2z = 0,
(ii) Π1 : x + y + z = 4 and Π2 : 2x − 3y = 6,
(iii) Π1 : x + y − 2z = 7 and Π2 : 2x + y + 3z = 5.

2. Find the angle between the following line L and plane Π:

(i) L : r = 3i + j + λ(i + 2k) and Π : 4x − y = 8,


(ii) L : r = i + j + λ(−2i + j − k) and Π : 3x − y + z = 7,
(iii) L : r = i + 6j − 3k + λ(i − 2j + 2k) and Π : (r − 3i) · (2i − 3j + 6k) = 0.

3. For each of the following, find the foot of the perpendicular of the point P from
the plane Π. Hence obtain the perpendicular distance.

(i) P : (2, −3, 6) and Π : 2x − 3y + 6z = 0,


(ii) P : (3, 1, −2) and Π : 2x + y − 2z = 8,
(iii) P : (1, 2, 3) and Π : r · (i + 2j − 7k) = 9.

4. For each of the following, find the perpendicular distance of the point P from the
plane Π.

(i) P : (4, −1, 5) and Π : 3x + 6y + 2z = 0,


(ii) P : (1, −2, 3) and Π : 2x + y − 2z = 6,
(iii) P : (3, 2, 1) and Π : r · (i − 2j + 3k) = 9.

294 dkrbabajee@gmail.com
c
CHAPTER 16. VECTORS 2

5. For the following planes Π1 and Π2 , find the vector equation of the line of their
intersection:
(i) Π1 : 3x − 2y + z = 4 and Π2 : 4x + y − z = 5,
(ii) Π1 : x + y − 3z = 6 and Π2 : 2x − y + z = 4,
   
1 2
(iii) Π1 : r ·  −2  = 3 and Π2 : r ·  −1  = 4,
1 5
   
1 2
(iv) Π1 : r ·  1  = 2 and Π2 : r ·  1  = 0.
−2 −1
−→
6.* The position vectors of A, B and C, relative to the origin O are given by OA = 2i,
−−→ −→
OB = i + 2j and OC = i + 2j + 2k, respectively.
(i) Find the cartesian equations of the planes ABC and OAB.
(ii) Find the angle between the two planes.
(iii) Find the vector equation of the line of their intersection.

16.9 Miscellaneous Exercises


Example 16.19 The planes Π1 and Π2 have equations x + 3y + 4z = 0
and −3x + 4y + z = 0.

(i) find the acute angle between the two planes.

(ii) find the vector equation of the line of intersection of the two planes.

(iii) find the perpendicular distance of the point P(1, 3, −5) to the plane
Π1 .

Solution
(i)
   
1 −3
n1 =  3  , n2 =  4 
4 1
√ √
n1 · n2 = (1 × −3) + (3 × 4) + (4 × 1) = 13, |n1 | = 26, |n2 | = 26,
     
|n1 · n2 | 13 1
θ = cos −1
= cos −1
√ √ = cos−1
= 600 .
|n1 | |n2 | 26 26 2

295 dkrbabajee@gmail.com
c
CHAPTER 16. VECTORS 2

 
0
(ii) The origin (0, 0, 0) lies on both planes. So ~a =  0 .
0

i j k

b = n1 × n2 = 1 3 4
−3 4 1

3 4 1 4 1 3

= i −j
+ k
4 1 −3 1 −3 4
= (3 − 16)i − (1 + 12)j + (4 + 9)k = −13i − 13j + 13k.

The vector equation of the line is

r = a + λb = µ(i + j − k), µ = −13λ.

(iii)
   
1 1
−→
n1 =  3  , OP =  3 
4 −5
−→ −→
Since P Q is parallel to n1 , P Q = k n1
 
1+k
−→ −→
OQ = OP + kn1 =  1 + 3k 
−5 + 4k
−→
OQ · n1 = 0 ⇒ 1(1 + k) + 3(3 + 3k) + 4(−5 + 4k) = 0
5
1 + k + 9 + 9k − 20 + 16k = 0 ⇒ 26k = −10 ⇒ k = −
13
−→ 5√
|P Q| = |k| |n1 | = 26.
13

Example 16.20 The point A has position vector j and the line l has
equation r = 8i − k + t(−6i + j + 4k). The point B lies on the line l such
that AB is perpendicular to l.

(i) find the position vector of B.


The plane Π has equation x − y + 3z = 20.

(ii) find the position vector of the point C of intersection of l and Π.

(iii) find the equation of the plane ABC.

296 dkrbabajee@gmail.com
c
CHAPTER 16. VECTORS 2

Solution

(i)
     
0 −6 8 − 6t
−→   −−→
OA = 1 ,b= 1  , OB =  t 
0 4 −1 + 4t
     
8 − 6t 0 8 − 6t
−→ −−→ −→  − 1 =  t−1 
AB = OB − OA = t
−1 + 4t 0 −1 + 4t
−→
AB · b = 0 ⇒ −6(8 − 6t) + t − 1 + 4(−1 + 4t) = 0 ⇒ 53t = 53 ⇒ t = 1
   
8 − 6(1) 2
−−→   =  1  = 2i + j + 3k
OB = 1
−1 + 4(1) 3

(ii)
   
1 8 − 6t
−→
n1 =  −1  , OC =  t 
3 −1 + 4t
−→
OC · n1 = 20 ⇒ 8 − 6t − t + 3(−1 + 4t) = 0 ⇒ 5t = 15 ⇒ t = 3
   
8 − 6(3) −10
−→   =  3  = −10i + 3j + 11k
OC = 3
−1 + 4(3) 11

297 dkrbabajee@gmail.com
c
CHAPTER 16. VECTORS 2

(iii)
   
2 −10
−→ −−→ −→   −→ −→ −→
AB = OB − OA = 0 , AC = OC − OA =  2 
3 11

i j k
−→ −→
n2 = AB × AC = 2 0 3
−10 2 11

0 3 2 3 2 0
= i −j
−10 11 + k −10 2
2 11
= (0 − 6)i − (22 + 30)j + (4 − 0)k = −6i − 52j + 4k
 
x
 y  · n1 = − →
OA · n1
z
       
x −6 0 −6
 y  ·  −52  =  1  ·  −52 
z 4 0 4
− 6x − 52y + 4z = −52

Miscellaneous Exercise 16.

1. The planes Π1 and Π2 have equations 2x + 3y − 3z = 2 and 4x − 3y + 2z = 1.

(i) find the acute angle between the two planes.


(ii) find the vector equation of the line of intersection of the two planes.
(iii) find the perpendicular distance of the point P(1, 0, 1) to the plane Π1 .

2. The point A has position vector j and the line l has equation
r = i+j+t(i+3j+2k). The point B lies on the line l such that AB is perpendicular
to l.

(i) find the position vector of B.


The plane Π has equation x − y − 2z = 5.
(ii) find the position vector of the point C of intersection of l and Π.
(iii) find the equation of the plane ABC.

3. Two lines have vector equations r = i + 3j + 2k + λ(4i − 2j + k) and


r = 3i + 8j + 7k + µ(2i − 3j − k). Show that the lines intersect and find the position
vector of the point of intersection.

298 dkrbabajee@gmail.com
c
CHAPTER 16. VECTORS 2


1
4. Find the equation of a line which is perpendicular to the plane r ·  −2  = 3
4
and which passes through the point (1, −3, 2).

5. Two lines l and m vector equations r = 2i − j + 4k + λ(i + j − k) and


r = −2i + 2j + k + µ(−2i + j + k), respectively.

(i) Show that l and m do not intersect.


(ii) The point P lies on l and the point Q has position vector 2i − k.
Given that the line PQ is perpendicular to l, find the position vector of P.
(iii) Verify that Q lies on m and that PQ is perpendicular to m.
   
2 1
6. Find the point of intersection of the line r =  0  + λ 1  and the plane

  −1 3
1
r·  1  = 5.
−1

7. The point A has position vector i + 2j + 3k, the line L has vector equation
r = i + 4j − k + λ(i − j + k) and the plane P has equation −x + 2y + 2z = 2.

(i) Find the point of intersection of L and P .


(ii) Find the length of the perpendicular from A to P .
(iii) Find the acute angle between L and P .

8.* Find the cartesian equation of the plane which passes the point (3, −4, 1) and
which is parallel to the plane containing the point (1, 2, −1) and the line
r = λ(i + j + k).
   
3 2
9. Two planes with vector equations r ·  1  = 2 and r ·  5  = 15 intersect in
1 −1
the line L.

(i) Find the direction vector of the line L.


(ii) Show that the point (1, 2, −3) lies in both planes and write down a vector
equation for the line L.
−→ −−→
10.* Consider the following points with position vectors OA = 4i+k, OB = 5i−2j−2k,
−→ −−→
OC = 4i + j and OD = −i − 4k.

(i) Calculate the acute angle between lines AB and CD.

299 dkrbabajee@gmail.com
c
CHAPTER 16. VECTORS 2

(ii) Prove that the lines AB and CD intersect and find the position vector of the
point of their intersection.
(iii) The point P has position vector i + 5j + 6k. √
Show that the perpendicular distance from P to the line AB is 3.

300 dkrbabajee@gmail.com
c
C HAPTER 17

Differential Equations

17.1 Objective
At the end of this chapter, students should be able to:

1. understand differential equations involving an independent variable x and dependent


variable y and how to obtain when y is a function of x;

2. formulate a simple statement involving a rate of change as a differential equation,


including the introduction of a constant of proportionality if necessary;

3. find by integration a general form of solution for a first order differential equation
by separation of variables;

4. use an initial condition to find a particular solution;

5. interpret the solution of a differential equation in the context of a problem


being modelled by the equation.

17.2 Understanding Differential equations


Let x be an independent variable and y be a dependent variable.
dy
Suppose y = ex , then = ex = y is called a differential equation.
dx
Some examples of differential equations are
dy
−y =0 (17.1)
dx
d2 y dy
+ − 2y = 0. (17.2)
dx2 dx
The order of a differential equation is the highest order of the derivative present in
it.
Differential equation (17.1) is first order since it contains the first order derivative
as the highest order derivative.
Differential equation (17.2) is second order since it contains the second order derivative

301 dkrbabajee@gmail.com
c
CHAPTER 17. DIFFERENTIAL EQUATIONS

as the highest order derivative.


We now show how to obtain differential equation when y is a function of x.

dy y
Example 17.1 (i) If y = Ax, show that = .
dx x
√ dy y
(ii) If y = A x, show that = .
dx 2x
Solution
dy y
(i) y = Ax ⇒ =A= .
dx x
√ dy A y 1 y
(ii) y=A x⇒ = √ =√ × √ = .
dx 2 x x 2 x 2x

17.3 Solving Differential Equations: Separation of


Variables
17.3.1 General Solutions
The general solution of differential equation is obtained by the method of separation
of variables. The method consists of

Step 1: separating all expressions involving y on the left and all expressions involving
x on the right,

Step 2: integrate w.r.t. y on the left and integrate w.r.t. x on the right (use only a
constant of integration)

Step 3: express y in terms of x

Example 17.2 Find the general solutions for the following differential
equations expressing y in terms of x:
dy dy dy y dy y2
(i) = x (ii) = y (iii) = (iv) =− 2
dx dx dx x dx cos x

Solution
Z Z
x2
(i) dy = x dx ⇒ dy = x dx ⇒ y = +c
2

302 dkrbabajee@gmail.com
c
CHAPTER 17. DIFFERENTIAL EQUATIONS

(ii)
Z Z
1 1
dy = dx ⇒ dy = dx ⇒ ln y = x + c
y y
let c = ln A ⇒ ln y = x + ln A ⇒ ln y − ln A = x
y y
ln = x ⇒ = ex ⇒ y = A ex .
A A

(iii)
Z Z
1 1 1 1
dy = dx ⇒ dy = dx ⇒ ln y = ln x + c
y x y x
let c = ln A ⇒ ln y = ln x + ln A ⇒ ln y = ln (Ax) ⇒ y = Ax.
Z Z
1 1 1 1
(iv) − 2 dy = sec2 x dx ⇒ − 2 dy = sec2 x dx ⇒ = tan x + c ⇒ y =
y y y tan x + c

17.3.2 Particular Solutions


Particular solutions are obtained by finding the constant of integration using the
initial or boundary conditions.

Example 17.3 Find the particular solutions for the following


differential equations expressing y in terms of x:
dy
(i) = 2y given that y = 1 when x = 0.
dx
dy x3
(ii) = 2 given that y = 3 when x = 2.
dx y
dy
(iii) (x2 − 1) = y given that y = 1 when x = 2.
dx
Solution

(i)
Z Z
1 1
dy = 2 dx ⇒ dy = 2 dx ⇒ ln y = 2x + c
y y
x = 0, y = 1 ⇒ ln 1 = 0 + c ⇒ c = 0
ln y = 2x ⇒ y = e2x .

303 dkrbabajee@gmail.com
c
CHAPTER 17. DIFFERENTIAL EQUATIONS

(ii)
Z Z
2 3 2 y3 x4
y dy = x dx ⇒ y dy = x3 dx ⇒ = +c
3 4
33 24
x = 2, y = 3 ⇒= +c⇒9=4+c⇒c=5
3 r4
y3 x4 4
3 3x + 60
= +5⇒y = .
3 4 4

(iii)
Z Z
1 1 1 1
dy = 2 dx ⇒ dy = dx
y x −1 y x2 − 1
1 1 A B
2
= = + ,
x −1 (x − 1)(x + 1) x−1 x+1

1 1 1 1 1

A= = , B= = =−
(x − 1)(x + 1) 2 (x − 1)(x + 1) −2 2
x=1 x=−1
1 1 1
2
= −
x −1 2(x − 1) 2(x + 1)
Z Z  
1 1 1
dy = − dx
y 2(x − 1) 2(x + 1)
1 1
ln y = ln (x − 1) − ln (x + 1) + c
2 2
1 1 1
x = 2, y = 1 ⇒ ln 1 = ln 1 − ln 3 + c ⇒ c = ln 3
2 2 2
1 1 1
ln y = ln (x − 1) − ln (x + 1) + ln 3
2 2 2 s s
  !
1 3(x − 1) 3(x − 1) 3(x − 1)
ln y = ln ⇒ ln y = ln ⇒y= .
2 (x + 1) (x + 1) (x + 1)

Exercise 17.1. [Solving differential equations]


dy y−B
1. (i) If y = Ax + B, show that = .
dx x
dy ny
(ii) If y = Axn , show that = .
dx x
dy
(iii) If y = Aenx , show that = ny.
dx
x−1 dy
(iv) If y = , show that (x2 − 1) = 2y.
x+1 dx

304 dkrbabajee@gmail.com
c
CHAPTER 17. DIFFERENTIAL EQUATIONS

2. Find the general solutions for the following differential equations expressing y
in terms of x:
dy dy dy 2 dy y2
(i) = x − 1 (ii) =y+1 (iii) = −y (iv) =
dx dx dx dx x
2 2
dy y dy cos y dy dy y+3
(v) = 2 (vi) =− (vii) = ex−y (viii) =
dx x dx x dx dx x−2
3. Find the particular solutions for the following differential equations expressing
y in terms of x:
dy
(i) = 3y given that y = 1 when x = 2.
dx
dy x2
(ii) = 3 given that y = 3 when x = 3.
dx y
dy
(iii) (x2 − 4) = 4y given that y = 1 when x = 1.
dx
dy √
(iv) ex = y given that y = 9 when x = 0.
dx
dy
(v) y = cos x given that y = 1 when x = π/2.
dx
Z
u
4. (i) Find du, where p is a constant.
u2 + p
y dy y2 − 3
(ii) Hence, by solving the differential equation = 2 given that y = 2
x dx x +1
when x = 0.

5. If y = 1 when x = 0, find the coordinates of the point whose curve represented by


dy
3y 2 = 4e−2x crosses the x−axis.
dx
1
6. (i) Convert into partial fractions.
x(x + 1)
dy
(ii) Solve the differential equation xy(x + 1) − (1 + y 2) = 0 given that y = 1
dx
when x = 1.

7. The gradient of a curve at the point (x, y) is proportional to ex+y . At the origin,
the gradient is 2.
dy
(i) Show that = 2(ex+y ).
dx
 
1
(ii) By solving the above differential equation, show that y = ln .
3 − 2 ex

305 dkrbabajee@gmail.com
c
CHAPTER 17. DIFFERENTIAL EQUATIONS

Z
cos u
8. (i) Find du.
sin u
dy
(ii) Hence, solve the differential equation sin x = (y + 1) cos x given that
dx
y = 0 when x = π/6
Z
sin u
9. (i) Find du.
cos u
dy
(ii) Hence, solve the differential equation x sin (2y) = cos (2y) given that
dx
y = π when x = 1.
1
10. (i) Convert into partial fractions.
y(y − 1)2
dy
(ii) Express x in terms of y by solving the differential equation = y(y − 1)2
dx
given that y = 2 when x = ln 2 − 1.

17.4 Formulation of Differential Equation


Differential equations can be formulated from rate of change using a constant of
proportionality if necessary.

306 dkrbabajee@gmail.com
c
CHAPTER 17. DIFFERENTIAL EQUATIONS

Example 17.4 Write the differential equation for the following


statements. Use k as the constant of proportionality.

(i) The radius, r is increasing at a constant rate.

(ii) The length of a metal, x increases at a rate proportional to its length.

(iii) When heated, the length, L of a rod increases at a rate proportional


to the square of its length.

(iv) The number N of bacteria increases at a rate which is inversely


proportional to its value.

(v) The temperature T of a hot object decreases at a rate proportional to


its value at any time.

(vi) The rate of cooling of an object is proportional to the difference


between the temperature of a body, θ and temperature of its
surrounding, θs .

(vii) At any time t days, the birth rate of fish is equal to one hundredth
of the number N of fish present. Fish are taken from the lake at the
rate of R per day.

Solution
dr
(i) = k (Rate is a constant)
dt
dx dx
(ii) ∝x⇒ = kx (Rate is positive)
dt dt
dL dL
(iii) ∝ L2 ⇒ = kL2 (Rate is positive)
dt dt
dN 1 dN k
(iv) ∝ ⇒ = (Rate is positive)
dt N dt N
dT dT
(v) ∝ −T ⇒ = −kT (Rate is negative)
dt dt
dθ dθ
(vi) ∝ (θs − θ) ⇒ = k(θ − θs ) (Rate is negative since θ − θs < 0)
dt dt
dN N N − 100R N
(vii) = −R= ( will increase rate while R will decrease rate)
dt 100 100 100

307 dkrbabajee@gmail.com
c
CHAPTER 17. DIFFERENTIAL EQUATIONS

Example 17.5 A disease is spreading through a population. Let P be


the proportion infected at time t years.
The rate of increase of P is proportional to the product of proportion of
people infected and the proportion not infected.

(i) Show that P satisfies the differential equation


dP
= kP (1 − P ), where k is a constant.
dt

(ii) Show that the solution of this differential equation can be written
in the form:

A ekt
P = , where A is another constant.
1 + A ekt
Initially the proportion infected is 2% and half the population is
infected after 2 years.

(iii) Find the value of A and of k.

(iv) What happens to the population when t becomes very large?

Solution
dP dP
(i) Proportion not infected is 1 − P . ∝ P (1 − P ) ⇒ = kP (1 − P ).
dt dt

308 dkrbabajee@gmail.com
c
CHAPTER 17. DIFFERENTIAL EQUATIONS

(ii)
Z Z
1 1
dP = k dt ⇒ dP = k dt
P (1 − P ) P (1 − P )

1 1

A= = 1, B= =1
(1 − P )P (1 − P )P
P =1 P =0
1 1 1
= +
P (1 − P ) (1 − P ) P
Z   Z
1 1
+ dP = k dt
(1 − P ) P
− ln (1 − P ) + ln P = kt + c
let c = ln A ⇒ − ln (1 − P ) + ln P = kt + ln A
 
P
− ln (1 − P ) + ln P − ln A = kt ⇒ ln = kt
A(1 − P )
P
= ekt ⇒ P = Aekt − P Aekt ⇒ P + P Aekt = Aekt
A(1 − P )
A ekt
P (1 + Aekt ) = Aekt ⇒ P =
1 + A ekt

(iii)
A e0
when t = 0, P = 0.02 ⇒ 0.02 =
1 + A e0
A 0.02 1
0.02 = ⇒ 0.02 + 0.02A = A ⇒ 0.02 = 0.98A ⇒ A = =
1+A 0.98 49
1 kt kt
e e
P = 49 1 kt =
1 + 49 e 49 + ekt
e2k
when t = 2, P = 0.5 ⇒ 0.5 = 2k
⇒ 24.5 + 0.5e2k = e2k
49 + e
1 √
24.5 = 0.5e2k ⇒ e2k = 49 ⇒ 2k = ln 49 ⇒ k = ln 49 = ln 49 = ln 7 = 1.95
2

(iv)
e1.95t 1
P = ⇒ P =
49 + e1.95t 49e−1.95t + 1
1
as t −→ ∞, e−1.95t −→ 0 ⇒ P −→ =1
0+1
The proportion infected is 1 meaning the whole population will be infected.

309 dkrbabajee@gmail.com
c
CHAPTER 17. DIFFERENTIAL EQUATIONS

Example 17.6 The rate of increase of a variable x is proportional to


dx
(100 − x)2 . When t = 0, x = 0 and = 10
dt
dx
(i) Show that x satisfies the differential equation 1000 = (100 − x)2 .
dt
(ii) Show that the solution of this differential equation can be written
in the form:
1000
x = 100 − .
t + 10
(iii) Find the value of x when t = 90.

(iv) Find the value of t when x = 50.

(v) Sketch the graph of x against t and explain what happens to x when
t becomes very large.

Solution

(i)
dx dx
∝ (100 − x)2 ⇒ = k(100 − x)2
dt dt
dx 10 1
when t = 0, = 10 ⇒ 10 = k(100 − 0)2 ⇒ k = =
dt 10000 1000
dx 1 dx
= (100 − x)2 ⇒ 1000 = (100 − x)2
dt 1000 dt

(ii)
Z Z
1000 1000
dx = dt ⇒ dx = dt
(100 − x)2 (100 − x)2
1000
=t+c
100 − x
1000
when t = 0, x = 0 ⇒ = 0 + c ⇒ c = 10
100 − 0
1000 1000 1000
= t + 10 ⇒ = 100 − x ⇒ x = 100 −
100 − x t + 10 t + 10

1000
(iii) when t = 90, x = 100 − = 90
90 + 10

310 dkrbabajee@gmail.com
c
CHAPTER 17. DIFFERENTIAL EQUATIONS

(iv)
1000
when x = 50, 50 = 100 −
t + 10
1000 1000
= 50 ⇒ t + 10 = = 20 ⇒ t = 20 − 10 = 10
t + 10 50

(v)
1000
x = 100 −
t + 10
1
as t −→ ∞, −→ 0 ⇒ x −→ 100 − 0 = 100
t + 10

Exercise 17.2. [Formulation of Differential Equation]


1. Write the differential equation for the following statements. Use k as the constant
of proportionality.

(i) The radius, r is decreasing at a constant rate.


(ii) The velocity v of a body falling from rest in a medium is decreasing at a
rate proportional to its velocity.
(iii) During the initial stages of the growth of yeast cells in a culture, the number
of cells present increases in proportion to the number already formed n.
(iv) A particle moves in a straight line such that its rate of velocity is inversely
proportional to its velocity v.
(v) The temperature T of a hot object decreases at a rate proportional to the
square of its value at any time.
(vi) The gradient at any point of a curve is proportional to the square root of the
x−coordinate.

311 dkrbabajee@gmail.com
c
CHAPTER 17. DIFFERENTIAL EQUATIONS

(vii) At any time t days, the birth rate of fish is equal to one eightieth of the
number N of fish present. Fish are taken from the lake at the rate of R per
day.

2. A particle moves in a straight line with an acceleration (rate of velocity) which


is inversely proportional to its velocity.
Given that the acceleration is 1 m/s2 when the velocity is 10 m/s at time t = 1.
dv 10
(i) Show that v satisfies the differential equation = .
dt v
(ii) Show that the solution of this differential equation can be written in the
form v 2 = 20(t + 4).

3. A circular patch of oil on the surface of water has radius r m at time t minutes.
The rate of increase of r is inversely proportional to r. Given that when t = 0,
r = 1 and when t = 50, r = 4, find the value of t when r = 2.

4. The rate at which the atoms in a mass of radioactive material are disintegrating
is proportional to N, the number of atoms present at any time t. Initially, the
number of atoms is N0 .

(i) Form a differential equation and solve it.


(ii) Sketch the graph of N against t.

5. Grain is pouring from a hopper on to a barn floor where it forms a conical pile
whose height h is increasing at a rate that is inversely proportional to h2 . The
initial height of the pile is h0 and the height doubles after time T .  
3 3 7t
Form a differential equation involving h and t and show that h = h0 1 + .
T
6. A disease is spreading through a population. Let P be the proportion infected at
time t years.
The rate of increase of P is proportional to the product of proportion of people
infected and the proportion not infected. Initially the proportion infected is 10%
and three-quarter of the population is infected after 3 years.
Form a differential equation involving P and t and show that P can be expressed
as:
e(ln 3)t
P = .
9 + e(ln 3)t

7. At any time t days, the birth rate of fish is equal to one fiftieth of the number N
of fish present. Fish are taken from the lake at the rate of R per day. Initially,
there are N0 fishes in the lake.

312 dkrbabajee@gmail.com
c
CHAPTER 17. DIFFERENTIAL EQUATIONS

dN N
(i) Show that N satisfies the differential equation = − R.
dt 50
(ii) Show that the solution of this differential equation can be written in the
form:
N = N0 e0.02t + 50R(1 − e0.02t ).

8. A colony of ants is growing at a rate proportional to the number of organisms,


N present at any time t hours. Initially, there are 1000 ants.

(i) Form a differential equation involving N and t and show that N = 1000ekt ,
where k is a constant.
(ii) Given that the colony increases by 50% in 2 hours, find the value of k.
(iii) Find the time by which the population of the ant colony will triple.

9. The rate of increase of a variable x is proportional to (10 − x)3 . When t = 0, x = 0


dx
and = 100
dt
dx
(i) Show that x satisfies the differential equation 10 = (10 − x)3 .
dt
(ii) Show that the solution of this differential equation can be written in the
form:  
1
x = 10 1 − √ .
1 + 20t
(iii) Find the value of x when t = 1.2.
(iv) Find the value of t when x = 9.
(v) Sketch the graph of x against t and explain what happens to x when t
becomes very large.

10.* The rate of cooling of an object is proportional to the difference between the
temperature of a body, θ and temperature of its surrounding, θs .
A pan of water at 750 C is standing in a kitchen whose temperature is steady at
250 C.

(i) Show that after cooling t minutes, the water temperature θ can be modeled
by the equation θ = 25 + 50 e−kt , where k is a constant.
(ii) Given that after 10 minutes, the temperature has fallen to 500 C, find the
value of k.
(iii) Find the temperature after 20 minutes.
(iv) Sketch the graph of θ against t.

313 dkrbabajee@gmail.com
c
CHAPTER 17. DIFFERENTIAL EQUATIONS

17.5 Miscellaneous Exercises


Example 17.7 A reservoir has a horizontal square base of length 10 m.
At the time t = √0, it is empty and water begins to flow into it at a rate
proportional to h, where h is the depth of water at time t s.
dh
When h = 1, = 0.1.
dt
dh √
(i) Show that h satisfies the differential equation = 0.1 h.
dt
(ii) By solving the differential equation, express t in terms of h.

(iii) Find the time at which the depth of water reaches 0.25 m.

Solution
(i)
dV √ dh √ dh √
∝ h⇒A ∝ h ⇒ 102 ∝ h
dt dt dt
dh √
100 =k h
dt
dh √
when h = 1, = 0.1 ⇒ 100 × 0.1 = k 1 ⇒ k = 10
dt
dh √ dh √
100 = 10 h ⇒ = 0.1 h.
dt dt

(ii)
Z Z
1 − 12
√ dh = 0.1 dt ⇒ h dh = 0.1 dt
h
1
h2 √
1 = 0.1t + c ⇒ 2 h = 0.1t + c
2

when t = 0, h = 0 ⇒ 2 0 = 0 + c ⇒ c = 0
√ √
2 h = 0.1t ⇒ t = 20 h

(iii) when h = 0.25, t = 20 0.25 = 10 s
Miscellaneous Exercise 17.
1. The gradient of a curve at the point (x, y) is proportional to x2 y. At the point
(-1,1), the gradient is 3.
dy
(i) Show that = 3x2 y.
dx
314 dkrbabajee@gmail.com
c
CHAPTER 17. DIFFERENTIAL EQUATIONS

3 +1
(ii) By solving the above differential equation, show that y = ex .
Z
2. (i) Find x ln x dx.

y 2 dy
(ii) Hence, solve the differential equation = ln x given that
x dx
y = 2 when x = 1.
Z
sin x
3. (i) Find dx.
cos3 x
dy
(ii) Hence, by solving the differential equation ey cos3 x = sin x given that
dx
 
cos (2x) + 4
y = 0 when x = π/6, show that y = ln
3 cos (2x) + 3
2y 2
4. (i) Convert into partial fractions.
1 − y2
dy 1
(ii) Express x in terms of y by solving the differential equation 2 = 2 −1
dx y
given that y = 2 when x = −4.
Z  2 
1 1 y
5. (i) Show that 2
dy = ln 2
+ c.
y(y + 1) 2 y +1
dy
(ii) Given that y = 1 when x = 0, solve the differential equation = y(y 2 + 1),
dx
expressing x in terms of y in the form k ln f (y), where k is a constant to be
determined.
dy
6. Express y in terms of x by solving the differential equation (2x + 1)(x + 1) =y
dx
9
given that y = when x = 1.
2
7. A virus has infected the population of rabbits on an isolated island and the
evidence suggests that the growth in the number of rabbits infected is proportional
to the number already infected. Initially, 25 rabbits were recorded infected.

(i) Form a differential equation that models the growth in the number of rabbits
infected, N.
(ii) Thirty days after initial evidence was collected, 50 rabbits were infected.
After how many further days does the model predict that 200 rabbits will
be infected.

315 dkrbabajee@gmail.com
c
CHAPTER 17. DIFFERENTIAL EQUATIONS

8. A reservoir has a horizontal rectangular base of length 10 m and width of 5 m. At


the time t = 0, it is empty and water begins to flow into it at a rate proportional
2
to h 3 , where h is the depth of water at time t s.
dh
When h = 1, = 0.1.
dt
dh 2
(i) Show that h satisfies the differential equation = 0.1 h 3 .
dt
(ii) By solving the differential equation, express t in terms of h.
(iii) Find the time at which the depth of water reaches 0.216 m.

9. The mass of a substance formed in a chemical reaction, t seconds after the start
of the reaction is x grams. At any time the rate of formation of the substance is
dx
proportional to (100 − x). When t = 0, x = 0 and = 1.
dt
dx
(i) Show that x and t satisfy the differential equation = 0.01(100 − x).
dt
(ii) By solving the differential equation, express x in terms of t.
(iii) Find x when t = 50, giving your answer correct to 1 decimal place.
(iv) State what happens to the value of x as t becomes very large.

10.* Two chemical, P and Q are involved in a reaction. The masses of P and Q at
any time t hours are p and q, respectively.
The rate at which p is increasing at time t is proportional to the product of the
two masses. It is given that the masses of P and Q have a constant mass of
10 kg.

(i) Form a differential equation involving p and t.


(ii) Initially the mass of P is 5 kg and it is 7 kg at t = 3, find the mass of Q at
t = 5.

316 dkrbabajee@gmail.com
c
C HAPTER 18

Complex Numbers

18.1 Objective
At the end of this chapter, students should be able to:

1. understand the idea of a complex number;

2. perform operations of complex numbers expressed in cartesian form x + iy:

(a) addition and subtraction of complex numbers,


(b) multiplication of complex numbers,
(c) conjugate and modulus of a complex number,
(d) division of complex numbers;

3. use the result that, for a polynomial equation with real coefficients, any non-real
roots occur in conjugate pairs;

4. solve simultaneous equations involving complex numbers;

5. find the two square roots of a complex number;

6. represent complex numbers geometrically by means of an Argand diagram;

7. find the argument of a complex number;

8. express a complex number in polar and exponential forms;

9. understand in simple terms the geometrical effects of conjugating a complex


number and of adding, subtracting, multiplying and dividing two complex numbers
(including equilateral triangles);

10. illustrate simple equations and inequalities involving complex numbers by means
of loci in an Argand diagram, e.g. |z − a| < k, |z − a| < |z − b|, arg(z − a) = α;

11. find the greatest and least values of |z| and arg(z) in a circle.

317 dkrbabajee@gmail.com
c
CHAPTER 18. COMPLEX NUMBERS

18.2 Idea of complex number


Consider the equation x2 + 4 = 0. Comparing with the standard quadratic equation,
we have a = 1, b = 0 and c = 4.
Since b2√ − 4ac = 02 − 4(1)(4) = −16 <
√0, the equation
√ √ has no real roots.
2
Let i = −1, then x = −4 ⇒ x = ± −4 = ± 4 −1 = ±2i.
i is called the imaginary number.
z is called a complex number if z = x + iy, where x ∈ R is the real part of z (also
denoted by Re(z)) and y ∈ R is the imaginary part of z (also denoted by Im(z)).

18.3 Operations involving complex numbers


Let z1 = x1 + iy1 and z2 = x2 + iy2 be two complex numbers where x1 , x2 , y1 , y2 ∈ R.

18.3.1 Powers of i
√ 1 i
If i = −1, then i2 = −1, i3 = i2 × i = −i, i4 = (i2 )2 = (−1)2 = 1 and = 2 = −i.
i i

18.3.2 Addition and Subtraction of two complex numbers


We first collect the real parts and the imaginary parts and perform the addition to
obtain z1 + z2 = x1 + iy1 + x2 + iy2 = (x1 + x2 ) + (y1 + y2 )i.
Similarly, with subtraction we have z1 − z2 = x1 + iy1 − (x2 + iy2 ) = (x1 − x2 ) + (y1 − y2 )i.

Example 18.1 Let z1 = 3 + 2i and z2 = 1 − i. Find


(i) z1 + z2 (ii) z1 − z2
Solution

(i) z1 + z2 = 3 + 2i + 1 − i = (3 + 1) + (2 − 1)i = 4 + i

(ii) z1 − z2 = 3 + 2i − (1 − i) = (3 − 1) + (2 + 1)i = 2 + 3i

18.3.3 Multiplication of two complex numbers


The multiplication of two complex number makes use of i2 = −1.

z1 × z2 = (x1 + iy1 )(x2 + iy2 )


= x1 x2 + ix1 y2 + i(y1 x2 ) + i2 y1 y2
= (x1 x2 − y1 y2 ) + i(x1 y2 + x2 y1 )

Example 18.2 Let z1 = 3 + 2i and z2 = 1 − i. Find z1 × z2 .

318 dkrbabajee@gmail.com
c
CHAPTER 18. COMPLEX NUMBERS

Solution
z1 × z2 = (3 + 2i)(1 − i)
= 3 − 3i + 2i − 2i2 = (3 + 2) − 3i + 2i = 5 − i

18.3.4 Conjugate of a complex number


The conjugate of a complex number z = x + iy is given by z = x − iy. Only the sign of
the imaginary part of z is reversed.

Example 18.3 Let z1 = 3 + 2i and z2 = 1 − i. Find z1 and z2 .


Solution
z1 = 3 − 2i and z2 = 1 + i.

18.3.5 Modulus of a complex number


p
The modulus of a complex number z = x + iy is given by |z| = x2 + y 2 .

Example 18.4 Let z1 = 3 + 2i and z2 = 1 − i. Find |z1 | and |z2 |.


Solution
√ √ p √
|z1 | = 32 + 22 = 13 and |z2 | = 12 + (−1)2 = 2.

18.3.6 Division of two complex numbers


The modulus of a complex number is related to its conjugate:
z × z = (x + iy)(x − iy) = x2 − i2 y 2 = x2 + y 2 = |z|2 .
This property z × z = |z|2 is used in the division of two complex numbers:

z1 z1 z2 z1 z2
= =
z2 z2 z 2 |z2 |2

so that the denominator becomes real.


z1
The division of two complex numbers, is carried out as follows:
z2
Step 1: Find the conjugate of z2 , denoted by z2 .

Step 2: Find the product z1 z2 to get the numerator.

Step 3: Find |z2 |2 to get the denominator.


z1
Example 18.5 Let z1 = 3 + 2i and z2 = 1 − i. Find .
z2

319 dkrbabajee@gmail.com
c
CHAPTER 18. COMPLEX NUMBERS

Solution
z2 = 1 + i, |z2 |2 = 12 + (−1)2 = 2
z1 z1 z2 (3 + 2i)(1 + i)
= 2
=
z2 |z2 | 2
z1 3 + 3i + 2i + 2i2 3 − 2 + 3i + 2i 1 + 5i 1 5
= = = = + i
z2 2 2 2 2 2
Exercise 18.1. [Operation on complex numbers]

1. For the following complex numbers z1 and z2 , express the following


z1 z2
(i) z1 + z2 (ii) z1 − z2 (iii) z1 × z2 (iv) |z1 |2 (v) |z2 |2 (vi) (vii)
z2 z1
in the form a + bi, where a and b are real numbers:

(a) z1 = 1 + 2i and z2 = 3 + 4i,


(b) z1 = 7 + 12i and z2 = 4 − 3i,
(c) z1 = 2 − 5i and z2 = −3 − 7i,
√ √
(d) z1 = 1 + 2i and z2 = −2 + 3 2i.

2. Express the following in the form a + bi:


i+1
(i) (3 + 2i)2 (ii) (1 + i)3 (iii) 2i(4i − 5) (iv) (v) (1 + 2i)5 − (1 − 2i)5
i

18.4 Polynomial with real coefficients


18.4.1 Quadratic Equations
Quadratic equations having real coefficients have roots which can occur in conjugate
forms:
az 2 + bz + c = 0

−b ± b2 − 4ac
z=
2a
2
If b − 4ac = −k < 0, k > 0
√ √ √
b2 − 4ac = −k = ki

−b ± ki
z=
2a
Example 18.6 Solve z 2 − z + 1 = 0 giving your answers in the form a + bi.

320 dkrbabajee@gmail.com
c
CHAPTER 18. COMPLEX NUMBERS

Solution
√ √
a = 1, b = −1, c = 1, b2 − 4ac = (−1)2 − 4(1)(1) = −3 ⇒ b2 − 4ac = 3i
√ √
−(−1) ± 3i 1 3
z= = ± i.
2 2 2
If z = a + bi is a solution of a quadratic equation x2 + px + q = 0, then z = a − bi is a
solution of the quadratic equation.
To find p and q, we have

(x − z)(x − z) = 0
x2 − (z + z)x + zz = 0
x2 − 2Re(z)x + |z|2 = 0
p = −2Re(z), q = |z|2 .

Example 18.7 If 2 + 3i is a root of a quadratic equation, find

(i) the other root,

(ii) the quadratic equation.

Solution

(i) Let z = 2 + 3i. z = 2 − 3i is the other root.

(ii) Re(z) = 2, p = −2Re(z) = −4, |z|2 = 22 + 32 = 13, q = |z|2 = 13.


The quadratic equation is given by x2 − 4x + 13 = 0.

18.4.2 Higher order Polynomials with real coefficients


Example 18.8 If 1 and 1 + i are roots of a cubic equation, find

(i) the other root,

(ii) the cubic equation.

Solution

(i) Let z = 1 + i. z = 1 − i is the other root.

(ii) Re(z) = 1, p = −2Re(z) = −2, |z|2 = 12 + 12 = 2, q = |z|2 = 2.


The quadratic equation is given by x2 − 2x + 2 = 0.
The cubic equation is given by (x − 1)(x2 − 2x + 2) = 0 ⇒ x3 − 3x2 + 4x − 2 = 0

321 dkrbabajee@gmail.com
c
CHAPTER 18. COMPLEX NUMBERS

using the conventional method of multiplication:

x2 −2x 2
x −1 ×
−x2 2x −2
x3 −2x2 2x 0 +
x3 −3x2 +4x −2

Example 18.9 (i) Verify that z = −1 + i is a root of the polynomial


x4 + 2x2 + 4x + 8 = 0 and write a second complex root of the equation.

(ii) Find the two other roots of the equation.


Solution

(i) Let z = −1 + i.

(−1 + i)4 + 2(−1 + i)2 + 4(−1 + i) + 8


= [(−1)4 + 4(−1)3 i + 6(−1)2 i2 + 4(−1)i3 + i4 ] + 2[(−1)2 + 2i(−1) + i2 ] + 4[−1 + i] + 8
= [1 − 4i − 6 + 4i + 1] + 2[1 − 2i − 1] + [−4 + 4i] + 8 = 0

z = −1 − i is the second complex root.

(ii) Re(z) = −1, p = −2Re(z) = 2, |z|2 = (−1)2 + 12 = 2, q = |z|2 = 2.


The quadratic equation is given by x2 + 2x + 2 = 0.
 
x4 + 2x2 + 4x + 8 : x2 + 2x + 2 = x2 − 2x + 4
− x4 − 2x3 − 2x2
− 2x3 + 4x
3 2
2x + 4x + 4x
4x2 + 8x + 8
− 4x2 − 8x − 8
0
√ √ √
x2 − 2x + 4 = 0 ⇒ x = 1 ± 12 − 4 = 1 ± −3 = 1 ± 3 i

18.5 Simultaneous equations involving complex number


Example 18.10 Solve the simultaneous equations
u + v = i ...... (1) u + iv = 3 ....... (2)
giving your answer in the form a + bi.

322 dkrbabajee@gmail.com
c
CHAPTER 18. COMPLEX NUMBERS

Solution
i−3
(1) − (2) ⇒ (1 − i)v = i − 3 ⇒ v =
1−i
2
(i − 3)(1 + i) i + i − 3 − 3i −2i − 4
v= 2 2
= = = −2 − i .....(3)
1 + (−1) 2 2
replace (3) in (1), u − 2 − i = i ⇒ u = 2 + 2i

18.6 Square roots of a complex number



Example 18.11 Find the two square roots of 3 + 4i, giving your answer in the form
a + bi.
Solution
Method
√ 1
Let 3 + 4i = a + bi, where a and b are real coefficients.

3 + 4i = (a + bi)2 ⇒ 3 + 4i = (a2 − b2 ) + 2abi


comparing real part, a2 − b2 = 3 (18.1)
2
comparing imaginary part, 2ab = 4 ⇒ ab = 2 ⇒ b = (18.2)
a
solving (18.1) and (18.2), we have
4
a2 − 2 = 3 ⇒ a4 − 3a2 − 4 = 0
a
let x = a2 > 0, x2 − 3x − 4 = 0 ⇒ (x − 4)(x + 1) = 0 ⇒ x = 4 or − 1
√ 2
a2 = 4 ⇒ a = ± 4 = ±2, b = = ±1
±2

3 + 4i = 2 + i, 2 − i

Babajee ? developed a simple method for finding the square root of a complex
number.
If a + bi is a square root of z, then −(a + bi) is also a root.

Re(z) + |z| Im(z)


a2 = , b= .
2 2a
Method 2

z = 3 + 4i, Re(z) = 3, Im(z) = 4, |z| = 32 + 42 = 5
3+5 √ Im(z) 4
a2 = = 4 ⇒ a = 4 = 2, b = = =1
2 2a 2(2)
a + bi = 2 + i, −(a + bi) = −2 − i

323 dkrbabajee@gmail.com
c
CHAPTER 18. COMPLEX NUMBERS

Exercise 18.2. [Polynomials involving complex numbers]

1. Solve the following equations giving your answers in the form a + bi, where
necessary:
(i) x2 + 2x +
√ 3=0 (ii) x2 − 4x + 5 = 0 (iii) 2x2 + x + 1 = 0
(iv) x2 + (2 3)ix − 4 = 0 (v) x3 − 3 x2 + 4 x − 2 = 0 (vi) 3 x3 + 10 x2 + 13 x + 10 = 0

2. For each of the following complex number z, find its conjugate and find quadratic
equation with z as one of the root: √
√ 3 7
(i) 1 + i (ii) 7 − 2i (iii) − 3 − 2i (iv) − 5 + 2i (v) + i
4 4
3. For the following values of a and b which are roots of a cubic equation, find the
other root and the cubic equation:
1 4
(i) a = 3, b = 2i (ii) a = −1, b = 4 − 5i (iii) a = 0, b = − i
3 3
4. Show that 1 + 3i is a root of the cubic equation x3 + 6 x + 20 = 0 and find the two
other roots.

5. (i) Verify that 1 + 2i is a root of the polynomial x4 + 4x2 + 4x + 15 = 0 and write


a second complex root of the equation.
(ii) Find the two other roots of the equation.

6. Solve the simultaneous equations


2u + iv = 5 ...... (1) −2u + 3v = 5i ....... (2)
giving your answer in the form a + bi.

7. Evaluate in the form a + bi: p √


√ √ √ √ √
(i) 5 + 12i (ii) 5 − 12i (iii) 7 − 24i (iv) 15 + 8i (v) 4 + 3i (vi) 1 − 3i

18.7 Modulus and Argument


18.7.1 Argand diagram
A complex number z = x + iy is represented on an Argand diagram with coordinates
P(x, y).

324 dkrbabajee@gmail.com
c
CHAPTER 18. COMPLEX NUMBERS

18.7.2 Argument of a complex number


The argument θ = arg(z) of a complex number z = x + iy can be calculated based on
the position of (x, y):

Coordinates Quadrant Sign of θ θ


y
x > 0, y > 0 1st +ve tan−1
x
y
x < 0, y > 0 2nd +ve tan−1 +π
x
y
x < 0, y < 0 3rd -ve tan−1 −π
x
y
x > 0, y < 0 4th -ve tan−1
x

Usually, arg(z) > 0 if y > 0 and arg(z) < 0 if y < 0.

Example 18.12 Find the arguments of the following complex numbers:


(i) 1 + 2i (ii) − 1 + 3i (iii) − 4 − i (iv) 5 − 2i
Solution
 
st 2
−1
(i) (1, 2) is in the 1 quadrant and θ is +ve. So θ = tan = 1.107
1
 
nd −1 −3
(ii) (−1, 3) is in the 2 quadrant and θ is +ve. So θ = tan + π = 1.893
1
 
rd −1 −1
(iii) (−4, −1) is in the 3 quadrant and θ is -ve. So θ = tan − π = −2.900
−4
 
th −1 −2
(iv) (5, −2) is in the 4 quadrant and θ is -ve. So θ = tan = −0.381
5

18.7.3 Modulus-Argument form (polar)


If r = |z| and θ = arg(z), then z = x + iy = r cos θ + i(r sin θ).

18.7.4 Exponential form


If r = |z| and θ = arg(z), then z = x+iy = r(cos θ+i sin θ) = r eiθ since eiθ = cos θ+i sin θ.

Example 18.13 Express the following complex numbers into polar and exponential
forms: √
(i) 1 − i (ii) − 1 − 3i (iii) 3 + 4i

325 dkrbabajee@gmail.com
c
CHAPTER 18. COMPLEX NUMBERS

Solution
p √
(i) r = |z| = 12 + (−1)2 = 2.  
th −1 −1 π
(1, −1) is in the 4 quadrant and θ is -ve. So θ = tan =− .
 π √  π 1 
 4 π 
√ √ π √
Polar form: 1 − i = 2 cos − + 2 sin − i = 2 cos − 2 sin i.
√ 4− π i 4 4 4
Exponential form: 1 − i = 2e 4 .
q √ √
(ii) r = |z| = (−1)2 + (− 3)2 = 4 = 2.
√ !
√ − 3 2π
(−1, − 3) is in the 3rd quadrant and θ is -ve. So θ = tan−1 −π =− .
−1 3
       
√ 2π 2π 2π 2π
Polar form: −1 − 3i = 2 cos − + 2 sin − i = 2 cos − 2 sin i.
√ 3 3 3 3

Exponential form: −1 − 3i = 2e− 3 i .
√ √
(iii) r = |z| = 32 + 42 = 25 = 5.  
st −1 4
(3, 4) is in the 1 quadrant and θ is +ve. So θ = tan = 0.927.
3
Polar form: 3 + 4i = 5 cos (0.927) + 5 sin (0.927)i.
Exponential form: 3 + 4i = 5e0.927i .

Example 18.14 The variable complex number z is given by z = 1 − cos 2θ + i sin 2θ,
π π
where θ takes all values in the interval − < θ < .
2 2
π
(i) Show that the modulus of z is 2 sin θ and the argument of z is − θ.
2
1
(ii) Prove that the real part of is independent of θ.
z
Solution

(i)
p p
|z| = (1 − cos 2θ)2 + (sin 2θ)2 = 1 − 2 cos 2θ + cos2 2θ + sin2 2θ
√ p p p
= 1 − 2 cos 2θ + 1 = 2(1 − cos 2θ) = 2 × 2 sin2 θ = 4 sin2 θ = 2 sin θ.
     
−1 sin 2θ −1 2 sin θ cos θ −1 cos θ
θ = tan = tan = tan
1 − cos 2θ 2 sin2 θ sin θ
 π  π
= tan−1 (cot θ) = tan−1 tan −θ = − θ,
2 2
π  sin π − θ cos θ
2 =
since tan −θ = π
= cot θ.
2 cos 2 − θ sin θ

326 dkrbabajee@gmail.com
c
CHAPTER 18. COMPLEX NUMBERS

(ii)

z = 1 − cos 2θ − i sin 2θ, |z|2 = (2 sin θ)2 = 4 sin2 θ


1 z 1 − cos 2θ − i sin 2θ
= 2 =
z |z| 4 sin2 θ
 
1 1 − cos 2θ 2 sin2 θ 1
Re = 2 = 2 = which is a constant.
z 4 sin θ 4 sin θ 2

18.8 Geometrical Interpretation


Let z1 = x1 + iy1 and z2 = x2 + iy2 be two complex numbers where x1 , x2 , y1 , y2 are
real. Let P(x1 , y1) represents z1 , Q(x2 , y2 ) represents z2 and O(0, 0) represents the
origin.

18.8.1 Addition

Let also R represents z1 + z2 , then OPQR is a parallelogram.

Example 18.15 If z1 = 1 + i and z2 = 2 + 3i, explain with a diagram the geometrical


interpretation of z1 + z2 .
Solution
z1 + z2 = 1 + i + 2 + 3i = 3 + 4i. If P(1, 1) represents z1 , Q(2, 3) represents z2 , O(0, 0)
represents the origin and R(3, 4) represents z1 + z2 , then OPQR is a parallelogram.

327 dkrbabajee@gmail.com
c
CHAPTER 18. COMPLEX NUMBERS

18.8.2 Multiplication
The complex numbers can be written in polar form:
z1 = r1 eiθ1 and z2 = r2 eiθ2 , where r1 and r2 are the magnitudes of z1 and z2 and θ1 and
θ2 are the arguments of z1 and z2 respectively.

z1 z2 = r1 eiθ1 r2 eiθ2 = r1 r2 ei(θ1 +θ2 )


= r1 r2 cos (θ1 + θ2 ) + ir1 r2 sin (θ1 + θ2 )

Therefore we have |z1 z2 | = |z1 | × |z2 | and arg(z1 z2 ) = arg(z1 ) + arg(z2 ) .

Furthermore |z 2 | = |z × z| = |z| × |z| = |z|2 and arg(z 2 ) = arg(z) + arg(z) = 2 arg(z).


In general, |z n | = |z|n and arg(z n ) = n arg(z).

18.8.3 Division
z1 r1 eiθ1 r1 i(θ1 −θ2 )
= = e
z2 r2 eiθ2 r2
r1 r1
= cos (θ1 − θ2 ) + i sin (θ1 − θ2 )
r2 r2
 
z1 |z1 | z1
Therefore we have = and arg = arg(z1 ) − arg(z2 ) .
z2 |z2 | z2
 
z1
b
The last property tells us that P OQ = arg = |arg(z1 ) − arg(z2)| .
z2
In case |arg(z1 ) − arg(z2 )| is reflex, obtuse P OQb = 2π − |arg(z1) − arg(z2 )|

328 dkrbabajee@gmail.com
c
CHAPTER 18. COMPLEX NUMBERS


Example
√ 18.16 If O, P and Q represent the origin, the complex numbers 1 + 3i and
b (between 0 and π).
3 + i, respectively. Find the angle P OQ
Solution
√ √
arg(1 + 3i) = π/3,
√ arg( 3+
√ i) = π/6,
b = |arg(1 + 3i) − arg( 3 + i)| = π/3 − π/6 = π/6
P OQ

Conjugate
Let z = x + iy = r cos θ + i(r sin θ), where r is the magnitude of z and θ is the argument
of z respectively. Then z = x − iy, |z| = r and arg(z) = −θ. If P represents z, Q
represents z and O is the origin, then Q is the reflection of P in the real axis and
OPQ is an isosceles triangle with P OQ b = 2θ.

Example 18.17 Let z = 1 + i.

(i) Find the modulus and arguments of z and z,

(ii) If P represents z, Q represents z and O is the origin, show that OP Q is an


isosceles right-angled triangle.

Solution

p √
(i) r = |z| = 12 + (1)2 = 2.  
1 π
(1, 1) is in the 1st quadrant and θ is ve. So arg(z) = tan−1
= .
p √ 1 4
r = |z| = 12 + (−1)2 = 2.  
th −1 −1 π
(1, −1) is in the 4 quadrant and θ is -ve. So arg(z) = tan =− .
1 4

329 dkrbabajee@gmail.com
c
CHAPTER 18. COMPLEX NUMBERS

(ii) b = arg(z) − arg(z) = π .


Since |z| = |z|, OP = OQ. P OQ
2

Equilateral Triangles

1 3
Let P represents any complex number z and u = + i.
2 2
z
Let Q represents the complex number q = zu and R the complex number r = .
u
Then △ OPQ and △ OPR are equilateral.

Example 18.18 Let P represents the complex number 3 + i, Q represents the complex
number a + bi and O represents the origin. If △ OPQ is equilateral, find the possible
values of a and of b where a and b are real and exact.
Solution

330 dkrbabajee@gmail.com
c
CHAPTER 18. COMPLEX NUMBERS


1 3 z
Let z = 3 + i and u = + i. Then the possible values of Q are zu and :
2 2 u
√ ! √ ! √ !
1 3 3− 3 1+3 3
zu = (3 + i) + i = + i
2 2 2 2
√ ! √ !
z 3+i 3+ 3 1−3 3
= √ = + i
u 1
+ 3i 2 2
2 2

Example 18.19 Given that z1 = 2 + 4i and z2 = 3 + i,

(i) find in the form x + iy, where x and y are real, the complex numbers z1 + z2 , z1 z2
z1
and .
z2
(ii) find the arguments of z1 , z2 and z1 z2 and verify that
arg(z1 z2 ) = arg(z1 ) + arg(z2 ).

(iii) In an Argand diagram with origin O, the points P, Q and R represent z1 , z2 and
z1 + z2 respectively.
State fully the geometrical relationship between PR and OQ.
z1 b = π.
(iv) State the argument of and prove that P OQ
z2 4
Solution

(i) z1 + z2 = 2 + 4i + 3 + i = 5 + 5i and z1 z2 = (2 + 4i)(3 + i) = 6 + 2i + 12i − 4 = 2 + 14i.


z1 z1 z2 (2 + 4i)(3 − i)
= 2
= =1+i
z2 |z2 | 32 + 12
(ii)
   
−1 4 −1 1
arg(z1) = tan = 1.107, arg(z2) = tan = 0.322
2 3
 
−1 14
arg(z1z2 ) = tan = 1.429 = 1.107 + 0.322 = arg(z1 ) + arg(z2 )
2

(iii) Since OPRQ is a parallelogram, PR is parallel to OQ.


   
b = arg(z1 ) − arg(z2 ) = arg z1 −1 1 π
(iv) P OQ = tan = .
z2 1 4

331 dkrbabajee@gmail.com
c
CHAPTER 18. COMPLEX NUMBERS

Exercise 18.3. [Polar and exponential form, Geometrical Interpretation]


1. Express the following complex numbers into polar and exponential forms giving
your answers in exact form where necessary:
(i) 1√+ i (ii) 1√− i (iii) − 1√+ i (iv) − 1 √
−i
(v) √3 + i √ (vi)√ 3 −√ i (vii) −√ 3 +√ i (viii) −√ 3 −√
i
(ix) 2 + 6i (x) 2 − 6i (xi) − 2 + 6i (xii) − √ 2 − 6i
(xiii) 12 − 5i (xiv) − 15 − 8i (xv) − 4 + 5i (xvi) 1 + 2i
2. The variable complex number z is given by z = sin 2θ + i(1 − cos 2θ), where θ takes
π π
all values in the interval − < θ < .
2 2
(i) Show that the modulus of z is 2 sin θ and the argument of z is θ.
1
(ii) Prove that the imaginary part of is independent of θ.
z
3. If z1 = 1−i and z2 = 3+2i, explain with a diagram the geometrical interpretation
z1
of (i) z1 + z2 (ii) z1 × z2 (iii) 2z1 (iv)
z2
4. If O, P and Q represent the origin, the complex numbers z1 and z2 , respectively.
Find the angle P OQ b (between 0 and π) in radians for the following:
√ √
(i) z1 = 1 + i, z2 = 1 + 3i√ (ii) z1 = −1 + i, z2 = −1 +√ 3i
(iii) z1 = 1 − i, z2 = −1 + 3i (iv) z1 = −1 − i, z2 = 1 − 3i

5. Let z = 1 − 3i.
(i) Find the modulus and arguments of z and z,
(ii) If P represents z, Q represents z and O is the origin, show that OP Q is an
isosceles triangle.
6.* If P represents z, Q represents z and O is the origin and OP Q is an isosceles
triangle with OP = 2 and P OQ b = π/3, find the modulus of z and the possible
values of arg(z). Hence, find the possible values of z.
7.* Let P represents the complex number 1 + 3i, Q represents the complex number
a + bi and O represents the origin. If △ OPQ is equilateral, find the possible
values of a and of b.
8.* Given that z1 = 4 − 2i and z2 = −1 + 3i,
(i) find in the form x + iy, where x and y are real, the complex numbers z1 + z2 ,
z1
z1 z2 and .
z2
(ii) find the arguments of z1 , z2 and z1 z2 and verify that
arg(z1 z2 ) = arg(z1) + arg(z2 ).

332 dkrbabajee@gmail.com
c
CHAPTER 18. COMPLEX NUMBERS

(iii)In an Argand diagram with origin O, the points P, Q and R represent z1 , z2


and z1 + z2 respectively.
State fully the geometrical relationship between PR and OQ.
z1 b = 3π .
(iv) State the argument of and prove that P OQ
z2 4

18.9 Loci
Let z = x + iy be a variable complex number and a = p + qi and b = s + ti be two fixed
complex numbers. In an Argand diagram, z is represented by the point P(x, y) and a
and b by the points A(p, q) and B(s, t) respectively.

18.9.1 Circle
The equation of a circle with centre (p, q) and radius r is given by (x−p)2 +(y−q)2 = r 2 .
Consider the equation |z − a| = r, then |z − a|2 = r 2 which gives
|(x − p) + i(y − q)|2 = r 2 ⇒ (x − p)2 + (y − q)2 = r 2 .
1. |z − a| = r is the equation of a circle centre a and radius r.

2. |z − a| < r is the region inside a circle centre a and radius r.

3. |z − a| > r is the region outside a circle centre a and radius r.

1 2 3

Example 18.20 Draw and describe the locus of points for the following:

(i) |z + 1| = 2,

(ii) |z| ≤ 3,

(iii) |z − (1 + i)| > 4.

333 dkrbabajee@gmail.com
c
CHAPTER 18. COMPLEX NUMBERS

Solution

(i) |z − (−1)| = 2 is a circle with centre A(−1, 0) and radius 2.

(ii) |z| ≤ 3 is the region inside the circle with centre A(0, 0) and radius 3.

(iii) |z − (1 + i)| > 4 is the region outside the circle with centre A(1, 1) and radius 4.
Note the dotted line because of strict inequality >.

334 dkrbabajee@gmail.com
c
CHAPTER 18. COMPLEX NUMBERS

18.9.2 Perpendicular Bisector


Consider the equation |z − a| = |z − b|.

|z − a|2 = |z − b|2
(x − p)2 + (y − q)2 = (x − s)2 + (y − t)2
x2 − 2px + p2 + y 2 − 2qy + q 2 = x2 − 2sx + s2 + y 2 − 2ty + t2
2(p − s)x + 2(q − t)y = p2 + q 2 − (s2 + t2 )

y − q+t
2  p−s
p+s = −
x− 2 q−t

which is the equation of the perpendicular bisector of A and B.

1. |z − a| = |z − b| is the perpendicular bisector of A and B. In this case AP = BP .

2. |z − a| < |z − b| is the region on the left of the perpendicular bisector of A and B


where AP < BP .

3. |z − a| > |z − b| is the region on the right of the perpendicular bisector of A and


B where AP > BP .

335 dkrbabajee@gmail.com
c
CHAPTER 18. COMPLEX NUMBERS

Example 18.21 Draw and describe the locus of points for the following:

(i) |z| = |z − 2|,

(ii) |z − 3| < |z + 1|,

(iii) |z − 1 + i| ≥ |z − 2 + 3i|.

Solution

(i) A(0, 0) and B(2, 0). |z| = |z − 2| represents the perpendicular bisector of A and
B with AP = P B.

(ii) |z − 3| < |z − (−1)|. A(3, 0) and B(−1, 0). |z − 3| < |z + 1| represents the region of
the left on the perpendicular bisector of A and B with AP < P B.

336 dkrbabajee@gmail.com
c
CHAPTER 18. COMPLEX NUMBERS

(iii) |z −(1−i)| ≥ |z −(2−3i)|. A(1, −1) and B(2, −3). |z −1+i| ≥ |z −2+3i| represents
the region of the left on the perpendicular bisector of A and B with AP ≥ P B.

18.9.3 Half lines


Consider the equation arg(z − a) = θ. Then
y−q
tan θ = ⇒ y = tan θx − p tan θ + q
x−p

is the equation of a straight line with gradient tan θ. Since −π < θ ≤ π,


arg(z − a) = θ is the half-line starting from A and making an angle θ with the
horizontal Real axis.

Example 18.22 Draw and describe the locus of points for the following:
π
(i) arg(z) = ,
4
π
(ii) 0 < arg(z + 2) < ,
3
π
(iii) arg(z − (1 + i)) > .
6

337 dkrbabajee@gmail.com
c
CHAPTER 18. COMPLEX NUMBERS

Solution
π π
(i) arg(z) = is the half-line starting from A (0, 0) and making an angle with
4 4
the horizontal Real axis.

π
(ii) 0 < arg(z + 2) < is the region inside the half-line starting from A (−2, 0) and
3
π
making an angle with the horizontal Real axis.
3

π
(iii) arg(z − (1 + i)) >is the region outside the half-line starting from A (1, 1)
6
π
and making an angle with the horizontal Real axis.
6

338 dkrbabajee@gmail.com
c
CHAPTER 18. COMPLEX NUMBERS

Greatest and least value of |z| and arg(z) in a circle


Consider the circle |z − a| = r with centre A representing a and radius r.

AP = AQ = r and OA = |a|.
The least value of |z| is given by OP = OA − AP = |a| − r and the greatest value is
given by OQ = OP + P Q = |a| + r.

We draw the tangents OB and OD.

The greatest value of arg(z) is given by the greatest angle between the x-axis and a
line joining the origin to a point on the circle.  
b = arg(a) + sin −1 r
The greatest value of arg(z) is given by arg(a) + AOD .
  |a|
b −1 r
The least value of arg(z) is given by arg(a) − AOB = arg(a) − sin .
|a|

Example 18.23 (i) Draw and describe the locus of points for |z − (3 + 4i)| = 2.

(ii) Determine the greatest and least value of |z| in this region.

(iii) Determine the greatest and least value of arg(z) in this region.
Solution

(i) |z − (3 + 4i)| = 2 is a circle with centre A(3, 4) and radius r = 2.

339 dkrbabajee@gmail.com
c
CHAPTER 18. COMPLEX NUMBERS

√ √
(ii) Let a = 3 + 4i be the centre. |a| = 32 + 42 = 25 = 5. Greatest value of |z| is
given by |a| + r = 5 + 2 = 7 and least value of |z| is given by |a| − r = 5 − 2 = 3.
 
st −1 4
(iii) (3, 4) is in 1 quadrant and arg(a) = tan = 0.927.
    3
r 2
Also sin−1 = sin−1 = 0.412.
|a| 5  
−1 r
Greatest value of arg(z) is given by arg(a) + sin = 0.927 + 0.412 = 1.34
 |a| 
r
and least value of arg(z) is given by arg(a) − sin−1 = 0.927 − 0.412 = 0.515.
|a|

Example 18.24 Calculate the greatest value of |z| in the region whose the points
represent the complex number z satisfying both inequalities |z − i| ≤ 1 and
0 ≤ arg(z + 1) ≤ π/4.
Solution

The greatest value of |z| in this region is the distance OP.


1
AP = 1, OB = 1 and AB = = 1.414 so that BP = AP + AB = 2.414.
cos π/4
Using cosine rule, OP 2 = 12 + 2.4142 − 2(1)(2.414) cos π/4 ⇒ OP = 1.85.

340 dkrbabajee@gmail.com
c
CHAPTER 18. COMPLEX NUMBERS

Exercise 18.4. [Loci]


1. On separate Argand diagrams, describe and sketch the locus of the point P
representing the complex number z such that
(i) |z − 2| = 1 (ii) |z − 3i| ≤ 1 (iii) |z + 3| ≥ 3 (iv) |z + 2i| < 2 √
(v) |z − 2 + 2i| ≥ 3 (vi) |z − 2 − i| ≤ 1 (vii) |z + 2 + 3i| ≤ 5 (viii) |z − 3 + i| < 5
2. On separate Argand diagrams, describe and sketch the locus of the point P
representing the complex number z such that
(i) |z − 2| = |z + 2| (ii) |z − 1| ≥ |z + i| (iii) |z| ≤ |z + 1|
(iv) |z + 2 + i| = |z − 2 + i| (v) |z + 4 − 3i| ≤ |z − 3 + 4i| (vi) |z − 3 − 2i| ≥ |z + 2 + 3i|
3. On separate Argand diagrams, describe and sketch the locus of the point P
representing the complex number z such that
(i) arg(z − 1) = π/2 (ii) arg(z + 2) = −π/4 (iii) arg(z + 2i) = −3π/4
(iv) arg(z − 1 − i) = π (v) arg(z + 3 + i) = π/2 (vi) arg(z + 2 − 3i) = −5π/6
4. Sketch the following on separate Argand diagrams
(i) 1 ≤ |z − 2| < 2 (ii) π/6 ≤ arg(z + i) ≤ π/3
(iii) 2 ≤ |z − 2 + i| ≤ 3 (iv) − π/4 ≤ arg(z − 1) ≤ π/4
5. (i) Describe the locus of points for |z + 4 − 3i| = 1.
(ii) Determine the greatest and least value of |z| in this region.
(iii) Determine the greatest and least value of arg(z) in this region.

6. Calculate the greatest value of |z| in the region whose the points represent the
complex number z satisfying both inequalities |z − 2i| ≤ 1 and
0 ≤ arg(z + 2) ≤ π/4.

7.* (i) Sketch, on an Argand diagram, the locus of the complex number z satisfying
arg(z) = π/6.
√ 4
(ii) If z represents the point (x, y) in part (i), show that |z+4 3i|2 = x2 + 8x + 48.
3

(iii) Hence, by completing to the square, show that |z + 4 3i| ≥ 6.

18.10 Miscellaneous Exercises



Example 18.25 The complex number 3 + i is denoted by w.
(i) Express w in the form r(cos θ + i sin θ) where r > 0 and −π < θ ≤ π.
Hence or otherwise find the modulus and arguments of w 2 and w 3 .

(ii) Show that w is a root of z 2 − 2 3z + 4 = 0 and find the other root.

341 dkrbabajee@gmail.com
c
CHAPTER 18. COMPLEX NUMBERS

(iii) Sketch an Argand diagram showing points representing complex numbers i and
w. Shade the region whose points represent every complex number z satisfying
both inequalities |z − i| ≤ 1 and arg(z) ≥ arg(w).

Solution
q√ √ √
(i) r = |w| = ( 3)2 + 12 = 4 = 2. ( 3, 1) is in 1st quadrant and
 
−1 1 π
θ = arg(a) = tan √ = .
3 6
 
w = 2 cos π6 + i sin π6 .
π
|w 2 | = 22 = 4, arg(w 2) = 2 arg(w) = .
3
π
|w 3 | = 23 = 8, arg(w 3) = 3 arg(w) = .
2
√ √ √ √ √
(ii) ( 3 + i)2 − 2 3( 3 + i)
√ + 4 = 3 + 2 3i − 1 − 6 − 2 3i + 4 = 0.
The other root is w = 3 − i.

(iii) |z − i| ≤ 1 is the region inside a circle with centre A(0, 1) and radius r = 1.
π
arg(z) ≥ is the region above the half-line starting from B(0, 0) and making an
6
π
angle with the horizontal Real axis. The required region is the intersection
6
of the two regions.

Miscellaneous Exercise 18.

1. Given that z1 = 3 − 2i and z2 = −4 + 3i, evaluate


z2
(i) 3z1 + iz2 (ii) z1 × z2 (iii) z2 2 (iv)
z1
2. (i) Find the two square roots of −16 − 30i and denote them by z1 and z2 .
z1
(ii) Find and its argument.
z2

342 dkrbabajee@gmail.com
c
CHAPTER 18. COMPLEX NUMBERS

(iii) Sketch an Argand diagram showing points A and B representing the complex
numbers z1 and z2 .
b = π, where O is the origin
Using your answer in part (ii), show that AOB
and show on the same diagram that the point B can be obtained by the 1800
rotation of point A around O.

3. If one root of the equation x2 + ax + b = 0 is 2 − i. Find the other root and the
value of a and of b.

4. A, B and C are represented by the complex numbers z1 = 2 − 3i, z2 = 3 + 2i and


z3 respectively.
z1 b = π/2 where O is the
(i) Find and its argument and deduce that AOB
z2
origin.
(ii) Find z3 if OACB is a parallelogram.

5. The complex number 1 + 3i is denoted by w.

(i) Express w in the form r(cos θ + i sin θ) where r > 0 and −π < θ ≤ π.
Hence or otherwise find the modulus and arguments of w 2 and w 3 .
(ii) Show that w is a root of z 2 − 2z + 4 = 0 and find the other root.
(iii) Sketch an Argand diagram showing points representing complex numbers
i and w. Shade the region whose points represent every complex number z
satisfying both inequalities |z − i| ≤ 2 and arg(z) ≥ arg(w).

6. The complex number u = 1 − i is a root of the equation x3 − 5x2 + 8x − k = 0,


where k is a real constant.

(i) Find the value of k.


(ii) Write down the other complex root of the equation and find the real root.
(iii) Sketch an Argand diagram showing point representing complex number u.
Shade the region whose points represent every complex number z satisfying
both inequalities |z| ≤ |z − 1| and 0 < arg(z − u) < 2π/3.

7. A complex number u is denoted by u = 1 + 2i and another complex number w has


modulus 4 and argument π/3. Find in the form a + bi, where a and b are real,
u
the complex numbers: (i) w (ii) u × w (iii)
w
8.* (i) Find the two square roots of 9 − 40i giving your answers in the form a + bi,
where a and b are real constants.
(ii) Hence, solve the equation z 2 − 5z + 10i + 4 = 0 giving your answers in the
form a + bi, where a and b are real.

343 dkrbabajee@gmail.com
c
CHAPTER 18. COMPLEX NUMBERS

(iii) Sketch on an Argand diagram the points A and B representing the two roots
of the above equation.
b where O is the origin.
Find the value of AOB
2+i
9.* The complex number w is given by .
3−i
(i) Express w in the form a + bi, where a and b are real constants.
(ii) Find the modulus and argument of w.
(iii) On an Argand diagram draw the locus of points representing every complex
1
number z satisfying |z − w| = . Hence, find the greatest value of
4
|z| and of arg(z) in this region.

1 3
10.* The complex number u and w are given by u = − + i and w = 1 + i.
2 2
(i) Find the modulus and arguments of u and w.
(ii) State the modulus and argument of uw and the modulus and argument of
w
.
u
(iii) In an Argand diagram, the points O, P, Q and R represent the origin, w, uw
w √
and , respectively. Show that P OQb = 2π/3 and P Q = 6. Hence, explain
u
why P, Q and R are vertices of an equilateral triangle.
(iv) In an Argand diagram, the vertices of an equilateral triangle lie on a circle
with centre at the origin. One of the vertices represents the complex number
w. Find the complex numbers represented by the other two vertices. Give
your answers in the form a + bi, where a and b are real and exact.

344 dkrbabajee@gmail.com
c
Answers

Exercise 1.1
 2  2
2 3 7 1 16
1. (i) (x − 4) − 1 (ii) x + + (iii) 3 x + −
2 4 3 3
 2
1 25 1 2 22 2
(iv) 6 x− − (v) (x + 4) − (vi) − (x − 1) + 5
12 24 2 3
 2  2  2
3 29 3 9 4 3 3
(vii) − x + + (viii) −7 x − + (ix) − x+ +
2 4 14 28 3 4 2
   
3 7 3 1 16 1
2. (i) (4, −1) , min, x = 4 (ii) − , , min, x = − (iii) − , − , min, x = −
2 4 2 3 3 3
   
1 25 1 22
(iv) ,− , min, x = (v) −4, − , min, x = −4 (vi) (1, 5) , max, x = 1
12 24 12 3
     
3 29 3 3 9 3 3 3 3
(vii) − , , max, x = − (viii) , , max, x = (ix) − , , max, x = −
2 4 2 14 28 14 4 2 4

3. b = −4, c = 11 4. p = 1, q = 1 5. a = 15, c = −36 6. a = 2, b = 12, c = 3

Exercise 1.2
1
1. (i) 2, 3 (ii) − 3, 1 (iii) − 1, 5 (iv) − 9, −8 (v) − 1,
3
1 5 3 2 5 3
(vi) ,3 (vii) − , 1 (viii) − , − (ix) − ,
3 2 2 3 3 4
√ √
√ √ √ −3 ± 37 9 ± 41
2. (i) 2 ± 2 (ii) − 2 ± 3 (iii) 1 ± 3 (iv) (v)
2 10
√ √ √ √
−7 ± 65 −1 ± 13 −3 ± 3 5 ± 2 10
(vi) (vii) (viii) (ix)
8 3 2 6

3. (i) S = 5, P = 6, x2 − 5x + 6 = 0 (ii) S = −4, P = 3, x2 + 4x + 3 = 0


(iii) S = 3, P = −10, x2 − 3x − 10 = 0 (iv) S = −1, P = −12, x2 + x − 12 = 0
3 1 11 3
(v) S = , P = , 2x2 − 3x + 1 = 0 (vi) S = , P = − , 10x2 − 11x − 6 = 0
2 2 10 5

4. x2 + 2x − 1 = 0 5. (i) Real and Distinct (ii) Not Real (iii) Real and equal (iv) Real and distinct

4 1 10
6. p ≤ 4 7. k = ± , x = ± 8. k = 2 or − 10. b2 − 4ac = (k + 1)2 ≥ 0
3 2 9

345 dkrbabajee@gmail.com
c
Exercise 1.3

3 1 5 1 1
1. (i) x = ±1, ±5 (ii) z = ±1 (iii) y = 2, − (iv) y = , (v) y = ,4 (vi) x = −1, −
2 4 12 2 2

2. (i) 0, 2 (ii) 0 (iii) 3, 4 (iv) − 2, 0 (v) − 2 (vi) ± 1


s √
1 33 1 1 1+ 21
3. (i) (ii) 21 (iii) 8 (iv) 4. (i) ± ± (ii) 1 (iii) 1, 4 5. ±
4 16 3 2 5

Exercise 1.4

1. (i) x = 2, y = 3; x = −12, y = 17 (ii) x = 2, y = 5; x = 4, y = 9


11
(iii) x = −1, y = −2; x = −2, y = −1 (iv) x = 1, y = −2; x = 6, y =
2
46 9 1 2 8 9
(v) x = −2, y = 3; x = , y=− (vi) x = , y=− ;x= , y=−
13 13 5 5 5 5
1 1 2
2. (i) − 2 < x < −1 (ii) x < or x > (iii) − 2 ≤ x ≤
3 2 5
1
(iv) x ≤ −4 or x ≥ 3 (v) − 3 < x < −1 (vi) x ≤ − or x ≥ 1
5
√ √
(vii) − 4 < x < 0 (viii) − 3 ≤ x ≤ 3 (ix) x ≤ −3 − 6 or x ≥ −3 + 6

7
3. (i) p ≤ −9 or p ≥ −1 (ii) p ≤ 0 or p ≥ 8 4. (i) − 4 < a < 12 (ii) − 1 < a < 5. 0 < k < 4
2

Miscellaneous Exercise 1

1 4
1. ± , ±1 2. x = −4, −3, 1, 2 3. 0 < k < 4. 3 < k < 4 5. (i) c < 1
2 9

5. (ii) c = 0 < 1, yes, (0, 0), (4, 4) 6. (i) b = −10, c = 21 (ii) d > 4 7. r = 3, (2, 1); r = −3, (−2, −1)
 2  
3 1 3 1 1
8. (i) x− − (ii) Minimum pt ,− (iii) 2, −
2 4 2 4 4

9. (i) − 4 < p < 4 (ii) p = 4, (−2, 2); p = −4, (2, −2) 10. p = 1, q = 3, r = 2

Exercise 2.1

1. (i) yes (ii) yes (iii) no (iv) no

2. (i) f (x) ≥ 1 (ii) f (x) ≤ 1 (iii) − 5 ≤ f (x) ≤ 1 (iv) − 4 ≤ f (x) ≤ 6


17
(v) 3 ≤ f (x) ≤ 21 (vi) − 15 ≤ f (x) ≤ 3 (vii) − 10 ≤ f (x) ≤ 49 (viii) 1 ≤ f (x) ≤
8
(ix) − 4 ≤ f (x) ≤ 0 (x) 0 < f (x) ≤ 1 (xi) − 27 ≤ f (x) ≤ 1 (xii) f (x) < 0

346
1 9
3. (i) a = 2, b = −2 (ii) p = − 4. (i) a = 4, b = −3 (ii) p = 1,
2 4

5. (i) 1 ≤ f (x) ≤ 17 (ii) 2 ≤ x ≤ 3 (iii) 0 ≤ x ≤ 8 (iv) p = 2

Exercise 2.2

1. (i) f 2 (x) = x + 6, g 2 (x) = 4x + 3, f g(x) = 2x + 4, gf (x) = 2x + 7


x − 24
(ii) f 2 (x) = 25x + 24, g 2 (x) = , f g(x) = x, gf (x) = x
25
(iii) f 2 (x) = 4x − 15, g 2 (x) = x4 − 2x2 , f g(x) = 2x2 − 7, gf (x) = 4 x2 − 20 x + 24
5−x x−1
(iv) f 2 (x) = 4 x − 9, g 2 (x) = x, f g(x) = , x 6= 1, gf (x) = , x 6= 2
x−1 x−2

2. (i) f h(x) (ii) gf (x) (iii) f g(x) (iv) hf (x) (v) hg(x) (vi) gh(x)

1
3. (i) f 2 (x) = 4x + 3, −5 ≤ f 2 (x) ≤ 11 (ii) − 1, (iii) − 1 4. p < −6 5. (i) a = 2, b = 1 (ii) 3
2
Exercise 2.3

1. (i) yes (ii) no (iii) yes (iv) no (v) yes (vi) no

5−x
2. (i) f −1 (x) = x − 1, −1 ≤ x ≤ 3 (ii) f −1 (x) =
,2≤x≤5
3
r
√ x+3
(iii) f (x) = 1 − x − 3, 3 ≤ x ≤ 4
−1
(iv) f (x) = −1 +
−1
, −3 ≤ x ≤ −1
2
r
√ 4−x
(v) f −1 (x) = 2 − 2 − x, −14 ≤ x ≤ 2 (vi) f −1 (x) = −1 + , −8 ≤ x ≤ 4
3

x + 4 −1 1 + 3x 3
3. (i) f −1 (x) = , g (x) = , x 6= 0, x = −1,
3 2x 2

2 2x − 1 5 ± 21
4. (i) f −1
(x) = , x 6= 1 (ii) − 4 (iii) 1 (iv) − 1, 2 5. (i) g −1
(x) = , x 6= 3 (ii)
x−1 x−3 2

Miscellaneous Exercise 2
3x + 3
1. a = 4, b = 5 2. (i) f −1 (x) = , x 6= 1 (ii) m = 1, c = −2
x−1
1 11 1 − 11x
3. (i) x = −1 (ii) gf (x) = , x 6= − , (f g)−1 (x) = , x 6= 0
6 x + 11 6 6x

x − 3 −1 1 − 2x
f −1 (x) = , g (x) = , x 6= 0 4. p ≤ 1 5. (i) k = −9, 15 (ii) x = −3, 9 6. (i) p = −7, 1
2 3x
1 √
6. (ii) x = ± 7. (a) A = 4 (b) (i) (x − 3)2 − 9 (ii) 3 (ii) f −1 (x) = 3 − x + 9, −9 ≤ x ≤ −8
2

8. (a) (i) a = −1 (ii) x = −1 (b) (i) h(x) ≥ −2 (ii) h−1 (x) = (x + 2)2 − 4, x ≥ −2.

347

9. (a) a = 3, b = 2 (b) x = −1, 2 (c) (x + 1)2 − 1, h−1 (x) = −1 − x + 1, x ≥ −1
r
2 11 − x
10. (i) 11 − 2(x − 2) (ii) f (x) ≤ 11 (iv) B = 2 (v) g −1
(x) = 2 + , x ≤ 11, g −1 (x) ≥ 2
2

Exercise 3.1
√ √ √ √ √
1. (i) 2 (ii) 2 (iii) 5 (iv) 193 (v) 2 5 (vi) 2 10 (vii) 5 (viii) 97 (ix) 10
   
5 9
2. − 2, 4 3. − 2 4. 15.3 5. (i) (2, 1) (ii) (−4, −3) (iii) 3, (iv) ,4 (v) (3, 3)
2 2
     
3 7 5
5. (vi) (0, 3) (vii) −2, (viii) 1, (ix) (−3, −4) 6. a = −3, b = 3 7. (2, 2) 8. (ii) 1,
2 6 2

Exercise 3.2
3 12 1 4 4 4
1. (i) 0 (ii) undefined (iii) (iv) (v) 2 (vi) (vii) − (viii) (ix)
4 7 3 3 9 3

2. (i) yes (ii) no (iii) yes (iv) yes 3. (i) 3 (ii) 5

4. (i) parallel (ii) perpendicular (iii) perpendicular (iv) parallel

1 5 4
5. (i) m = 1, c = 7 (ii) m = −3, c = 4 (iii) m = ,c=− (iv) m = − , c = 3
2 6 3

6. (i) y = 2x − 4 (ii) y = −3x + 10 (iii) 4y = 3x (iv) 7x + 3y = 1

7. (i) y = 1 (ii) x = −4 (iii) 4y = 3x + 1 (iv) 7y = 12x − 26 (v) y = 2x − 3


(vi) 3y = x + 9 (vii) 4x + 3y = −1 (viii) 9y = 4x − 34 (ix) 3y = 4x − 24

8. (i) perpendicular (ii) perpendicular (iii) parallel (iv) parallel

9. (i) y = 2x − 1 (ii) 4y = 3x − 24 (iii) 5x + 4y = 54 (iv) y = −3x + 3

10. (i) x + y = 7 (ii) 2x + y = 3 (iii) y = 2x + 13 (iv) 3y = 8x − 2

Exercise 3.3
3
1. (i) 8y = 6x − 15 (ii) 5x − 3y = 4 (iii) x + y + 7 = 0 (iv) 6y = 4x + 5 (v) y = −3 (vi) x =
2

2. (5, −2) 3. (3, 3) 5. (i) A(1, −1), B(0.5, −1.5) (ii) 2x + 2y = −1 (iii) − 0.5

6. (i) 18.40 (ii) 78.10 (iii) 37.90

Miscellaneous Exercise 3
   
4 1 7 1
1. (i) L1 : y = x − 1, L2 : 2x + y = 3 (ii) , 2. (i) X(−0.5, 2), Y = (−2, −1) (ii) − ,
3 3 5 5

348
 
31 15
3. (i) 4x + 2y = 7 (ii) y = 9x − 12 (iii) , , 4.60 4. (i) A(−3, 6), B(1, 2) (ii) y = x + 5
22 22

√ √ √
4. (iii) 5 2 5. (i) 5x + 3y = 6, C(0, 2) (ii) AC = BC = 17, AB = 34 6. (i) 3x + 4y = 23, C(5, 2)
   
161 40 16 98
6. (ii) 17.1 7. (i) BD : 5x + 2y = 25, AC : 5x + 2y = −4 (ii) D ,− ,C ,−
29 29 29 29

8. A(−8, 0), M (4, 6), B(2, 10), D(6, 2) 9. (i) QR : 2y = x, RS : 2x + y = 25 (ii) R(10, 5)

10. (i) M (−1, 5), BD : y = 3x + 8 (ii) AB : y = x + 10, CD : 2y = x + 6 (iii) B(1, 11), D(−2, 2)

Exercise 4.1
π π 7π 11π 3π 5π
1. (i) (ii) (iii) (iv) (v) (vi) 2. (i) 0.524 (ii) 1.047 (iii) 1.222
6 3 18 18 4 6

2. (iv) 1.920 (v) 2.356 (vi) 2.618 3. (i) 450 (ii) 1200 (iii) 900 (iv) 200 (v) 1050 (vi) 720

Exercise 4.2

1. (a) (i) 23.6 (ii) 43.6 (iii) 118 (iv) 82.8 (v) 42.1 (b) (i) 16 (ii) 32 (iii) 64 (iv) 34.9 (v) 29.5

2
2. (i) s = 8, A = 40 (ii) r = 5, A = 15 (iii) θ = , A = 75 (iv) r = 6, s = 8 (v) θ = 1.25, s = 10
3
7
2. (vi) r = 25, θ = 1.6 3. (i) (ii) 14 4. (i) 10.5 (ii) 24.8 5. (i) 1.855 (ii) 48.0 (iii) 92.8
4

Exercise 4.3

1. area of segment ABD= 35.4 cm2 , area of segment OACB = 279 cm2

2. (i) 25.8 (ii) 34.0 3. (i) 8 (ii) 22 4. (i) 4.70 (ii) 23.8 5. (ii) 14.422 (iii) 33.0 (iv) 26.3

Miscellaneous Exercise 4

1. (i) 166 (ii) 59.2 2. (i) 1.63 (ii) 6.95 3. (i) 7.98 (ii) 20.6 4. (i) 2.02 (ii) 3.49 5. 257

2π 5
6. (ii) √ 7. (i) 5.85 (ii) 12.0 (iii) 21.1 8. (i) 2 (ii) 9. (i) 15 (ii) 22.1 (iii) 37.8
3 3 9

10. (i) 8.24 (ii) 13.4

Exercise 5.1

1. (i) − 5 ≤ f (x) ≤ 1, T = 1800, N = 2 (ii) 1 ≤ f (x) ≤ 5, T = π, N = 1

(iii) − 6 ≤ f (x) ≤ 4, T = 1800 , N = 2 (iv) − 2 ≤ f (x) ≤ 6, T = 2π, N = 1


π
(v) f (x) ∈ R, T = 600 , N = 3 (vi) f (x) ∈ R, T = ,N =4
2

349
2. a = 4, b = 2, c = 3 3. a = 3, b = 1, c = −4 4. a = 4, b = 3 5. (ii) 1

Exercise 5.2

1 1 √ 3 1
1. (i) (ii) √ (iii) 3 (iv) (v) − (vi) − 1
2 2 2 2

1 3 1 1 1 √
(vii) √ (viii) − (ix) − √ (x) − √ (xi) − √ (xii) 3
2 2 3 2 2
√ √
3 1 √ 1 3 1
(xiii) − (xiv) (xv) 3 (xvi) − (xvii) (xviii) − √
2 2 2 2 3
√ √ √ √ √
2. 5 6 3. 8 2 4. 10 2 5. θ = 60, QR = 10 3; θ = 120, QR = 10 7

6. (i) 2100, 3300 (ii) 72.50 , 287.50 (iii) 113.20 , 293.20 7. (i) 0.848, 2.294 (ii) 2π/3, 4π/3

7. (iii) π/6, 7π/6 8. (i) 1050 , 1500 , 2850 , 3300 (ii) 100 , 300 , 1300 , 1500 , 2500 , 2700

8. (iii) 300 , 1200 , 2100 , 3000 9. (i) π/24, 5π/24 (ii) 5π/8 (iii) 0, π/2, π, 3π/2

Exercise 5.3

2. (i) 2100, 3300 (ii) 48.20 , 109.50 , 250.50, 311.80 (iii) 78.70 , 258.70 (iv) 113.20, 293.20

2. (v) 450 , 153.40 , 2250 , 333.4 (vi) 18.40, 26.60 , 206.60, 198.40 3. (i) a = 5, b = −3 (ii) 0

3. (iii) 2 ≤ f (x) ≤ 5 4. (i) a = −1, b = 6 (ii) 0.421, 2.72 (iii) − 1 ≤ f (x) ≤ 5 5. (iii) ± π/4, ± 3π/4

Miscellaneous Exercise 5

1. (i) a = 3, b = 1 (ii) 1.74, 2.97 2. (ii) 0.464 (iii) 0 < x < 0.464 3. (ii) 10 sin θ + 10 cos θ + 10

5. (i) 300 , 41.80 , 138.20 , 1500 (ii) 600 , 70.50 , 289.50, 3000 (iii) 63.40 , 71.60, 243.40, 251.60

5. (iv) 150.60, 330.60 (v) 600 , 3000 (vi) 70.50 , 289.50 6. (ii) 0.667, 2.48 7. (ii) 600 , 3000

8. (ii) 19.50 , 160.50 9. (iii) 26.60 , 206.60 10. (i) 2.55 (ii) (a) p = 2, q = 1 (b) 2 ≤ f (x) ≤ 3

Exercise 6.1

1. (i) 5i − 3j + 8k (ii) − i + j − k (iii) − 5i + 2j − 6k (iv) 5i − 2j + 6k


(v) 11i − 8j + 19k (vi) − 5i + 6j − 7k (vii) 17i − 2j + 22k (viii) 36i − 23j + 52k

2. (i) a, b and c parallel (b = −a, d = −2a = 2b) (ii) p = −3, q = −12

Exercise 6.2

1. (i) 4 (ii) 8 (iii) 16 (iv) 0 2. (i) − 5 (ii) − 7 (iii) 12 (iv) − 7

3. (i) 6.5 (ii) 3 (iii) 4 5. k = 6 6. p = 8, q = −5

350
Exercise 6.3

1 1 √ 1
1. (i) (3i + 4j) (ii) √ (2i + 3k) (iii) (2i − j − 2k)
5 7 3
1 1 1 √ √
(iv) (2i − 6j + 9k) (v) √ (3i − 2j + k) (vi) (− 3i + j − 5k)
11 14 3

1 1 1
2. (i) √ (−i + 29j − 7k) (ii) √ (3i + 19j − 3k) (iii) √ (11i − 25j + 15k)
9 11 379 971

1 1 1 1
3. (2i + 3j − 6k), (2i + 3j − 6k) 4. p = ±7, (4i + 4j + 7k), (4i + 4j − 7k)
7 2 9 9

5. (i) 22.60 , acute (ii) 109.40, obtuse (iii) 30.60 , acute (iv) 94.20, obtuse (v) 900 , right

5. (vi) 47.70 , acute 6. p = 1, 85.60 b = 55.60, ACB


7. ABC b = 38.90 , B AC
b = 85.50

3
8. (i) 103.80 (ii) p = −
11

Exercise 6.4

19
1. (i) 16i + 9j + 6k (ii) i + 2j + 2k 2. (i) 12i + 4j + 4k (ii) 6i + 4k (iii) 23.10
5

1 √
3. (i) √ (4i − 4j − 6k) (ii) − 2 (iii) 0, −4 4. m = 3, n = 2, q = −7 5. (i) 126 (ii) 170
68

1 1
6. (i) (6i + 3j − 2k) (ii) ± 2 7. λ = −7, −1 8. ± (8i + j + 4k)
7 3

3 − 2b 38
9. p = 4, q = −7; p = −8, q = −1 10. (i) a = −6, b = 6 (ii) a = (iii) − ,6
2 13

Miscellaneous Exercise 6

b = 60.90 , ACB
b = 37.10 , B AC
b = 82.00 7
1. ABC 2. (i) 100.50 (ii) k =
13

3. (i) 3 (ii) 4i − 5j − 3k, 5j − 3k (iii) 112.80 4. (i) 40.80 (ii) − 15j + 36k (iii) x = −28 or 20

−−→ −−→ √ −−→ √ 1


5. 3 6. (i) |OP | = |OQ| = 41, |P Q| = 110 (ii) 110.00 7. (i) 131.80 (ii) √ (5i − 20j − 14k)
621

41 38
8. p = 1, q = 2; p=− ,q=− 9. (i) − 2 (ii) 1 (iii) 1, 3
25 25

2 −−→ 17 16
10. (i) 44.20 (ii) 6i − 7j − 2k (iii) 11i − 8j + k (iv) λ = µ = , OX = i− j−k
3 3 3

351
Exercise 7.1

1. (i) 32 + 400 x + 2000 x2 + 5000 x3 + 6250 x4 + 3125 x5 (ii) 1 − 20 x2 + 150 x4 − 500 x6 + 625 x8

27 9 1 99
1. (iii) 27 + 2 x+ 4 x2 + 8 x3 2. (i) 32, (ii) − 8 3. (i) 4032, (ii) − 4. − 4320
4

5. 1 + 14 x + 84 x2 + 280 x3 , 1.316 6. (i) 2 + 20 x2 + 10 x4 (ii) 152 7. p = 2 8. − 40

9. n = 4, a = −2, b = 216 10. (i) 1 + 5 a x + 10 a2 x2 (ii) a = −2 (iii) − 35.

Exercise 7.2

1. (i) 59 (ii) 820 (iii) 38 (iv) 45 2. (i) n = 51, 7650 (ii) n = 34, 8517 (iii) n = 81, 24300

2. (iv) n = 13, 4576 3. (i) 88 (ii) 93 (iii) 23232 4. a = 3, d=4

π 27 81
5. (i) a = −72, d = 13 (ii) 1267 6. (iv) − 5 (v) 15 7. a = , π, π
12 2 2

8. (i) a = 32.5, d = 3 (ii) 588 9. (i) d = 0.25, (ii) n = 6 10. 450 , 750 , 1050 , 1350

Exercise 7.3

1. (i) 3072 (ii) 196602 (iii) 7 (iv) 5 2. (i) 11 (ii) 128 (iii) 4094 3. (i) 57062.33 (ii) 427685.68

4 1 81
4. a = , r = 3 5. r = 7 6. (i) 400 (ii) 320 7. (i) k = (ii)
3 3 4

1 1 − e−5n 1
8. r = 10. (i) (ii)
6 1 − e−5 1 − e−5

Miscellaneous Exercise 7

1. 258048 2. (i) 2 + 42 x2 + 70 x4 + 14 x6 (ii) 3712 3. − 1536 4. (i) 1 − 12 x + 60 x2 (ii) a = 12

5. (i) $370 (ii) $2350 6. n = 35, 543 7. 1593.85 8. r = 0.917, a = 40

5
9. (i) a + 3d, a + 8d (ii) r = 10. (i) a = 5, d = 2.5 (ii) r = 2
3

Exercise 8.1

12 9
1. (i) 7x6 (ii) 24 x7 (iii) − (iv) − √
x4 x3
36 2 1 7 1
(v) 28 x6 − 10 x4 + 24 x2 + (vi) 6 (2 x − 9) (vii) √ (viii) − √
x5 x+3 2 10 − 7x
−40 51 x3 − 1200 √ √ 9
(ix) 6 (x) (xi) 14 x5 + 3 x + 3 (xii) √
(2 x + 3) 4 x7 4
x

352
2 4 36 x3
2. (i) 18 x 3 x2 + 4 (ii) − 480 x2 3 − x3 (iii) − 4
(3 x4 + 2)

9 x2 + 2 7x − 1 3 ( x + 1)
(iv) √ (v) 3 (vi) − √ √ 2
2 3 x3 + 2x (2x − 7x2 ) 2 x (2 x + x)

Exercise 8.2

1. (i) Tangent: 2x + y = −2, Normal: 2y = x − 19 (ii) Tangent: 4y = −x − 6, Normal: y = 4x + 7

1 (iii) Tangent: y = −1.5, Normal: x = 1.5 2. (ii) B(7, 6.5)

3. (i) P(−2, 0), Q(3, 0) (ii) Tangent at P: y = −5x − 10, Tangent at Q: y = 5x − 15


 
1 25
3. (iii) R , − (iv)22.60 4. (i) y = −12x − 15 (ii) 85.20
2 2

Exercise 8.3
3
1. x > 1 2. x < − 3. x < 1 or x > 2 4. − 1 < x < 2
2
Exercise 8.4

1. (i) maximum pt (0, 0), minimum pt (1, −1) (ii) maximum pt (0, −8), minimum pt (2, −40)

(iii) maximum pt (−1, −2), minimum pt (1, 2) (iv) minimum pt 4, 32

288
2. (i) a = −3, b = 2 (ii) (1, 0) 3. (i) y = (iii) x = 6, A = 432, minimum
x2
8 2 64 2.16
4. (i) y = − x (iii) x = 2, V = , maximum 5. (i) h = − 0.5r
x 3 3 r
125
5. (iii) r = 1.2, V = 1.728π, maximum 6. (i) h = (iii) r = 5, A = 75π, minimum
r2
Exercise 8.5

1
1. (i) 14.4π cm3 /s (ii) 3.6π cm3 /s 2. (i) − cm/s (decrease) (ii) −0.694 cm/s (decrease) 3. 0.624 cm/s
64

Miscellaneous Exercise 8

1. (i) Maximum pt (0, 1), Minimum pt (2, −3) (ii) tangent: y = −3x + 2, normal: 3y = x − 4
 
4 5 dA
1. (iii) Q(0, 2), R 0, − , 2. (i) A = 42x2 , V = 10x3 (ii) = −8.4 cm2 /s (decrease)
3 3 dt
 
1 27
3. (i) p = 2 (p 6= 4) (ii) − , 4 4. (i) y = 2 (iii) x = 3, A = 54, minimum
3 x
 
2 5
5. (i) p < 3 (ii) − 3 < q < 3 6. (i) a = 1, b = −1 (ii) − ,
3 6

353
4
7. (i) y = − x + 4 (iii) x = 2.5, perimeter = 9 8. (i) 1, (1, 2) (ii) 2, (0, 2), (2, 4)
5

5 − πr − 2r
9. (i) h = 10. (ii) 50, maximum
2

Exercise 9.1

x5 1 3
1. (i) +c (ii) x4 + c (iii) x + x2 − 5 x + c
5 3
3x4 x4 5x7 x6 5x4
(iv) + x3 − 2x + c (v) − x5 − +c (vi) + + x3 + 2x2 + 3x + c
2 12 49 3 4
28x6 16x3 (4x + 3)6
(vii) x3 + x2 − 8 x + c (viii) − 7x4 + − 8x + c (ix) +c
3 3 24
5 1 x3 x2 1
(x) − +c (xi) +c (xii) + − 3 +c
3x3 3(2 − 3x)2 6 8 3x

x2 + 8 x − 24 2(x + 1) x + 1 − 11
2. (i) y = (ii) y = x3 + x2 − 3 x + 2 (iii) y =
2 3
4 √ x3 + 4 x2 + 7 x + 8
(iv) y = − + x2 − x + 2 (v) y = 2x+1 (vi) y =
x+1 2

2 3(1 − x2 )
3. 9 4. y = x2 + x − 2 5. y = 1 + 6. 1 7. y = 8. (i) (−1, 6)
x x2
 
2 3 3 2 1 17
8. (ii) y = x − 2x + 3 9. (i) (1, −5) (ii) y = x − 6x 10. (i) y = 2x − 7x + 4x (ii) ,
3 27

Exercise 9.2

635 3 77 11 1 14 1
1. (i) 4 (ii) (iii) 5 (iv) (v) (vi) (vii) 15 (viii) (ix) (x) 4 (xi)
7 5 2 5 3 9 24

1. (xii) 6 2. 3 3. 3 4. 0 5. (i) 14 (ii) 11 6. (i) − 6 (ii) 26

Exercise 9.3

10 52 14 135 121 14 7 32
1. (i) (ii) (iii) (iv) 18 (v) 2. (i) (ii) 2 (iii) 3 3. (i) (ii) (iii)
3 3 3 4 6 3 3 3

9 32 8 8 1
4. (i) (ii) (iii) (iv) 5. 2 6. (i) (0, 0), (1, 0), (−1, 0) (ii)
2 3 3 3 2
 
7 1 28 500
7. (i) 2x + y = 7 (ii) Q(3.5, 0) (iii) 8. (i) 3y + x = 9 (ii) − , (iii)
12 3 9 81

Exercise 9.4

83 1 64 1 4 2
1. π 2. π 3. π 4. π 5. π 6. π/6 7. (i) 2 x + y = 6 (ii) π (iii) π
15 2 5 30 3 3

354
Miscellaneous Exercise 9

x4 1 x2 2 √ √ √
1. (i) + 2 +c (ii) − x x+c (iii) 2x x − 8 x + c
4 2x 2 3
12 √ x2 (2x + 5)6 4
(iv) 3x3 + x2 x + +c (v) +c (vi) − +c
5 2 12 7(7x + 2)
1 √ 2 √ 2 √
(vii) +c (viii) 2x + 3 + c (ix) x2 x + x3 x + c
(1 − 4x)2 5 7

9 26 69 149 32 34
2. (i) (ii) (iii) (iv) 2 (v) (vi) (vii) (viii) 820 (ix) 14
8 3 4 6 21 3
1
3. (i) y = x3 + 6x2 + 9x + 1 (ii) Max. pt (−3, 1) 4. 2 5. (i) 3 (ii) 2 6. (i) (ii) 1
2
4 5 62 1 π
7. (i) (3, 1) (ii) π 8. (i) (1, 1) (ii) (iii) π 9. (i) A(1, 0), M (0.25, 0.25) (ii) (iii)
3 3 15 6 30
 
7 19 125
10. (i) (1, 6) (ii) y = 3 + 4x − x2 (iii) , (iv)
2 4 48

Exercise 10.1
7
1 1
1. (i) − 2, 0 (ii) − (iii)
, 3 (iv) − 1, − (v) − 7, 1
2
3 7
     
5 2 1
2. (i) (−2, 0) (ii) ,0 (iii) (2, 1) (iv) , −2 (v) (−1, 5) (vi) , −1
3 5 2

1 1 1 2 4
3. (i) 1, 3 (ii) 4 (iii) − (iv) − 3, (v) ± (vi) ,
2 3 2 3 5
1 1 5 1 1
4. (i) x < (ii) −1 < x < (iii) − <x<− (v) x < −2, x > −1 (iv) x < ,x>1
2 3 4 2 3
3 1 1
5. (i) x > − (ii) x < 2 (iii) x > − (iv) x < − , x > 0 (v) −1 < x < 1 (vi) x < −1, x > 1
2 2 2

Exercise 10.2

1. (i) x2 − 5 x + 6 (ii) 6 x2 + 11 x − 10 (iii) x3 − x2 − 2 x


(iv) 2 x3 − 7 x2 + 2 x + 3 (v) x3 + 5 x2 + 7 x + 3 (vi) 12 x3 − 56 x2 + 55 x + 25

2. (i) Q = x − 5, r = 12 (ii) Q = 2x − 3, r = −2 (iii) Q = x2 − 9x + 22, r = −67

x2 x 7 25
(iv) Q = + − ,r= (v) Q = x3 − 4 x2 + 4 x + 1, r = −3 (vi) Q = x2 + 4, r = 6x + 28
2 4 8 8
1 1 1
3. (i) 0, −1, −2 (ii) − 2, 1, 3 (iii) 2, 2, − (iv) −3, − , (v) − 2, −0.54, 5.54 (vi) − 3
2 3 4
5 1
4. (i) a = 1, b = −23 (ii) 1, , −4 5. (i) a = 13, b = −4 (ii) 1, , 4 6. (i) a = −13, b = −12
2 2

355
6 (ii) (x − 4) 7. (i) a = −1, b = −1 (ii) (x + 1)2 (x − 1) 8. (i) a = −4, b = 1 (ii) − 1, 2, 3

9. (i) a = −16, x2 + 2x − 8 (ii) x < −4 or x > 2 10. (i) a = −5 (ii) ± 1, ±2

Exercise 10.3

2x x+3 5x2 + 5x + 8
1. (i) (ii) (iii)
(x − 1)(x + 1) (x + 2)(x + 1)2 (x + 2)(x2 + 2)

1 1 1 1
2. (i) − + (ii) +
x+2 x−2 2(x + 2) 2(x − 2)
1 1 7 13 19
(iii) − + (iv) − +
4(x + 1) 4(x − 3) 2(x − 1) x − 2 2(x − 3)

1 1 1 1 1 2
3. (i) − − (ii) − +
4(x − 1) 4(x + 1) 2 (x + 1)2 9(x − 1) 9(x + 2) 3 (x + 2)2
5 2 1 2 4 4
(iii) − + (iv) − +
2 x + 1 x x2 3 x 3(2x − 3) (2 x − 3)2

−x + 1 1 x+1 1
4. (i) 2
+ (ii) 2

x +1 x+1 2(x + 1) 2(x + 1)
x+2 2 6x−1 16
(iii) 2 − (iv) +
x +2 x+2 19(2x2 + 1) 19(x − 3)

1 1 9 5
5. (i) 1 + − (ii) 1 + −
2(x − 1) 2(x + 1) 4(x − 1) 4(x + 3)
1 3 4 1 9 7
(iii) + − (iv) 1 − + −
2 14(2 x − 1) 7(3 x + 2) 4x 8(x − 2) 8(x + 2)

2 1 1 7 2 1
6. (i) 1 + + (ii) + − +
x − 1 (x − 1)2 2 2(2x + 1) x x2
1 159 77 1 1 1 5
(iii) 1 + + + (iv) − − −
16(x + 1) 16(x − 3) 4 (x − 3)2 9 7(x + 2) 63(3x − 1) 9 (3x − 1)2

24 −4 x + 3 6
7. (i) 5 − (ii) 1 + −
+5 x2 2
5(x + 1) 5(x + 2)
3 42 x − 75 2 3 −12 x − 29 22
(iii) + + (iv) + −
2 10(2x2 + 5) 5 x 2 14(2x2 + 3) 7(2x + 1)

1 1 5 75 2 125 3 4
8. (i) 1 − 2 x + 4 x2 − 8 x3 ; |x| < (ii) + x+ x + x ; |x| <
2 4 8 64 64 5
3 9 27 3 1 1 5 25 2 875 3 8
(iii) 1 + x − x2 + x ; |x| < (iv) + x+ x + x ; |x| <
2 8 16 3 2 48 576 41472 5
2 2 4 3 3
(v) 1 + 2 x + x − x ; |x| < (vi) 2 + 3x + 3x2 + 3x3 ; |x| < 1
3 27 4
3 5 93 2 189 3 2 1
(vii) + x + x + x ; |x| < (viii) 5 + 9 x − 12 x2 + 22 x3 ; |x| <
4 2 16 16 3 4
7 2 5 3
(ix) 2 + 4 x − x + x ; |x| < 1
4 4

356
8
9. (i) a = −1 (ii)
3

1 1 3 3 9 15 3
10. (i) − + ; − + x − x2 + x
x+1 x−2 2 4 8 16
16 8 4 4 32 172 2 776 3
(ii) − + 2 ; − x+ x − x
2 x + 3 x + 1 (x + 1) 3 9 27 81
−2(x + 1) 1
(iii) + ; −1 − x + x2 + x3
3(2x2 + 1) 3(x − 1)
9 16 1 2 7 3 37 4 175 5
(iv) 1 − + ; x + x + x + x
x − 3 x − 4 12 144 1728 20736
125 44 8 1 3 3 4 99 5 349 6
(v) 1 + − − ;− x − x − x − x
9(x − 5) 9(x − 2) 3 (x − 2)2 20 50 2000 10000
1 11(−3x + 1) 2 3 3 3 2 1 3
(vi) + + ; − x+ x + x
3 21(3x2 + 4) 7(x + 1) 4 4 16 16

Miscellaneous Exercise 10

4 3 a
1. (i) − , 0 (ii) x < −11, or, x > 1 (iii) −1 < x < 2. (i) ± a (ii) − , 3a 3. (i) a = 0, b = −3
5 7 3

3. (ii) x < 1 4. (i) a = −2, b = −13, c = −10 (ii) −2, −1, 5 5. (i) a = 1 (ii) x2 − x + 1

3 3 86 13 11 4961
6. (i) a = 2 (ii) − 7. a = , b = −2, k = − ; a = − , b = ,k= 10. (ii) a = 2
2 2 81 10 5 4050

Exercise 11.1

125 1
1. (i) 16 (ii) (iii) (iv) 1 2. (i) log2 8 = 3 (ii) log10 0.01 = −2 (iii) loge 5 = x
27 4

1 1 2
3. (i) 43 = 64 (ii) 36 2 = 6 (iii) e2 = p 4. (i) 2 (ii) − 4 (iii) (iv)
2 3
   √ 
1 4p2 q2 p
5. (i) 5q (ii) − 2q (iii) +q 6. (i) log5 (p5 q 4 ) (ii) log2 (iii) lg
2 q3 r
 
10
7. (i) f −1
(x) = 2 ln x, x > 0 (ii) f −1
(x) = ln (x − 1), x > 1 (iii) f −1
(x) = ln , x < 10
10 − x

Exercise 11.2

1. (i) 0.0524 (ii) 1.08 (iii) 2.95 (iv) − 0.958 2. (i) 1.16 (ii) 8.05 (iii) 0.631 (iv) − 1, 0.431

5 5
3. (i) 0.896 (ii) − 0.129 (iii) 1.61 (iv) 1.10, 1.39 4. (i) (ii) 4 (iii) (iv) 6
8 2
 
2 ln 2 ln 5
5. (i) x < ln (ii) x > − (iii) ln 2 < x < ln 3 (iv) 0 < x <
3 ln 3 ln 2

357
Exercise 11.3

1. (i) lg y = k lg x + lg a (ii) lg y = (lg b)x + lg a (iii) y = (logb a)x (iv) ln y = bx + ln a

x+4
2
2. a = 3 3. y = 10x 4. y = e 4 5. a = 31.6, b = 0.316 6. a = 1.21, k = 279 7. a = 12.3, b = 1.48

Miscellaneous Exercise 11

1. (i) 6.68 (ii) − 2.71 (iii) 0.904 2. (i) 1.77 (ii) 0.693, 1.95 (iii) 1.27

3. (i) 4 (ii) 1 (iii) 3 4. (i) 0, 2.20 (ii) − 0.431 (iii) − 1, 1.46 5. (i) y = (log5 6)x

ex − 3
5. (ii) y = x + ln 8 (iii) y = (− log3 4)x + log3 12 6. f −1 (x) = ,x∈R 7. (i) 6
2

7. (ii) 1.58 8. (i) 19 (ii) 1.18 (iii) 10 9. a = 7.40, b = 0.189 10. (i) m = 3, n = 5 (ii) x = 1.47

Exercise 12.1

0 0
√ √ 2 3
1. (i) π/3, 5π/3 (ii) 240 , 300 (iii) − π/2, π/2 2. (i) − 3 (ii) 2 (iii) −
3

4. (i) 126.90, 306.90 (ii) 199.50 , 340.50 (iii) 66.40 , 293.60

4. (iv) 63.40 , 1350 , 243.40, 3150 (v) 300 , 900 , 1500 (vi) 600 , 109.50 , 250.50, 3000

4. (vii) 63.40, 111.80, 243.40 , 291.80 5. (ii) 1.23, 5.05

Exercise 12.2
63 16 63 33 56 33
1. (a) (i) (ii) − (iii) − (iv) − (v) (vi) −
65 65 16 65 65 56
297 304 297 87 416 87
(b) (i) (ii) (iii) (iv) (v) (vi)
425 425 304 425 425 416
143 24 143 17 144 17
(c) (i) (ii) − (iii) − (iv) − (v) (vi) −
145 145 24 145 145 144
533 756 533 43 924 43
(d) (i) (ii) − (iii) − (iv) (v) (vi)
925 925 756 925 925 924
√ √ √ √ √ √ √ √
6− 2 6+ 2 √ 6+ 2 6− 2 √
2. (i) (ii) (iii) 2 − 3 3. (i) (ii) (iii) 2 + 3
4 4 4 4
√ √ √ √ √
24 + 7 3 7 − 24 3 −672 − 625 3 24 − 7 3 7 + 24 3
4. (i) (ii) (iii) (iv) (v)
50 50 1679 50 50

−672 + 625 3
4. (vi) 6. (i) 26.60, 206.60 (ii) 71.60 , 251.60 (iii) 124.70, 304.70
1679

6. (iv) 1050 , 1650 , 243.40, 2850 , 3450 7. (ii) 22.50 , 202.50 8. (ii) 750 , 2550

358
Exercise 12.3

24 7 24 120 119 120 336 527 336


1. (a) (i) (ii) (iii) (b) (i) (ii) (iii) (c) (i) (ii) − (iii) −
25 25 7 169 169 119 625 625 527

3. (i) 11.80 , 78.10 , 191.80, 258.10 (ii) 00 , 450 , 1350, 1800 , 2250, 3150 , 3600 (iii) 600 , 1800 , 3000
(iv) 900 , 2100 , 3300 (v) 00 , 300 , 1500, 1800 , 3600 (vi) 2100 , 2700 , 3300
(vii) 41.40 , 1800 , 318.60 (viii) 48.20 , 104.50, 255.50, 311.80

4. (ii) π/12, 5π/12, 7π/12, 11π/12 5. (ii) − 8 3 (iii) 0.464, 2.03 6. (iii) 1350

Exercise 12.4

1. (i) 13 cos (θ − 22.60) (ii) 2 sin (θ − 600 ) (iii) 25 sin (θ + 16.30 ) (iv) 5 cos (θ + 63.40)
√ √ √
2. (i) 13 cos (θ − 56.30 ) (ii) greatest = 13, θ = 56.30 ; least = − 13, θ = 236.30
√ √ √
3. (i) 2 sin (θ − π/4) (ii) greatest = 2, θ = 3π/4; least = − 2, θ = 7π/4

4. (i) 17 cos (θ + 28.10) (ii) greatest = 17, θ = 331.90; least = −17, θ = 151.90

5. (i) 29 sin (θ + 0.810) (ii) greatest = 29, θ = 0.76; least = −29, θ = 3.90

Miscellaneous Exercise 12

1−p p2 + 2p − 1
1. (ii) 0.615, 2.53 2. (ii) 108.40 , 288.40 3. (i) (ii)
1+p −p2 + 2p + 1

4. (ii) 14.50 , 165.50, 199.50, 340.50 5. (ii) 27.70 , 92.30 6. (ii) 16 (iii) 0.365, 1.21, 1.94, 2.78
√ 
7. (i) 2 sin x + 450 (ii) 155.70 , 294.30 8. (i) − 1, 1.207, −0.207 (iii) 191.90 , 2700 , 348.10
 π
9. (i) 2 sin x + (ii) greatest = 2, θ = π/6; least = −2, θ = 7π/6 (iii) π/4, 11π/12
3

15 8 3 4 36 77
10. (i) (ii) (iii) (iv) (v) (vi)
17 17 5 5 85 85

Exercise 13.1

2 x
1. (i) 2 cos (2x + 1) (ii) − 9 sin (3x + π/6) (iii) sec2
5 5
2
(iv) 20e5x+4 (v) (vi) cot x
(2x + 3)
(vii) − sin (2x) (viii) 9 sin2 (3x) cos (3x) (ix) 2 tan x sec2 x

(x) 4 cos (2x) e2 sin (2x) (xi) − 6 sec2 (3x) e1−2 tan (3x) (xii) e2 sin x+5 cos x (2 cos x − 5 sin x)

359
2. (i) x cos x + sin x (ii) xe3x (3x + 2) (iii) x2 (3 ln x + 1)

x (5 x − 4) − cos x − x sin x
(iv) e2x (2 tan x + sec2 x) (v) 2 (vi)
(5 x − 2) x2
(vii) sec x tan x (viii) − cot x cosec(x) (ix) − cosec2 (x)

sin x(2x cos x − sin x)


(x) e−x (cos x − sin x) (xi) (xii) ln (cos x) − x tan x
x2

1
3. (i) 0.73 (ii) 1.05 (iii) 0.54 4. (i) 0.203 (ii) 1.22 (iii)
8
π π
5. (i) m = −1; tangent: x + y = + 1; normal:y = x + 1 −
2 2
(ii) m = 2; tangent: y = 2x + 1; normal: x + 2y = 2

(iii) m = 3; tangent: y = 3x; normal: x + 3y = 10

(iv) m = 1; tangent: y = x; normal: x + y = π


 
π 2π √
6. , − 3 7. (6, 3 ln 6 + 6) 8. (−2, e2 ), maximum 9. (−1.11, 0.244) 10. (2, e2 ), minimum
6 3

Exercise 13.2

dy x dy 2 x−y dy 3
1. (i) =− (ii) = (iii) =
dx y dx x−2y dx 2y
dy y2 dy −ey + 1 dy 3 cos (x)
(iv) =− 2 2 y (v) = (vi) =−
dx x (y e + 1) dx ey x dx 2 cos (y)

dy dy dy
2. (i) = (ln 2)2x (ii) = −(ln 3)3−x (iii) = (x ln 5 + 1)5x
dx dx dx
dy dy x−y dy y ln y
(iv) = (ln x + 1)xx (v) = (vi) =
dx dx x ln x dx y ey − x

5
3. (i) (ii) − 4 (iii) 3 (iv) − 1 4. (i) (0, 1) (0, −1) (ii) (2, −4), (−2, 4) (iii) (0, −1), (−2, −5)
4

5. 4y = x − 5; 3x + 12y = 7 6. 3x + 4y = 1 7. (1, −2) 8. (ii) 4y = 27x − 14

Exercise 13.3

1 9
1. (i) (ii) −1 (iii) 2(t − 1) (iv) − cos t (v) −4 sin t (vi) 2t − t2
t 4

2t
2. (i) (ii) 2y + 3e3 x = 3e6 + 2 3. tangent: x + 2y = −4; normal: y = 2x − 12
3e3t

−2 4 4
4. (i) (ii) 16x + 12y = 11 5. (1, −2), (−1, 2) 6. ,
3 cos t 3 5

360
Miscellaneous Exercise 13

 
1
1. x = π/4, maximum 2. 0.464 3. (ii) 2y = −x 4. , −1 5. (ii) (−1, 3), (1, −3) (iii) (−3, 1), (3, −1)
2
   
1 15 9 cos t
7. (ii) y = −2x + 1 8. (ii) √ ,0 9. (i) y = 2x + 3 (ii) − ,− 10. (i) −
2 4 2 sin t

Exercise 14.1

e3x e5x+3 4 −3x x


1. (i) +c (ii) −3 e−x + c (iii) +c (iv) − 2e3−5x + c (v) e 2 + c (vi) 6e 2 −5 + c
3 5 3

e2 − 1 e−1
2. (i) (ii) 3(e2 − 1) (iii) (iv) 3e(e − 1)
2 e
1 1
3. (i) ln |x| + c (ii) 2 ln |x| + c ln |2x + 1| + c (iii)
3 2
2 5
(iv) − ln |1 − 3x| + c (v) 3 ln |3 + 2x| + c (vi) ln |7 − 4x| + c
3 4
 
1 5
4. (i) 2 ln 3 (ii) ln 2 (iii) 3 ln 2 (iv) ln
3 3

2 3 2
5. (i) − cos (3x) + c (ii) − sin (7x) + c (iii) cos (5x − π/6) + c (iv) sin (3x + π/3) + c
3 7 5
 x sin (2x) 3x 3
(v) 8 tan x2 + c (vi) tan (x + π/4) − x + c (vii) + +c (viii) − sin (4x) + c
2 4 2 8

√ √
1 1 3 π π 3 π 1
6. (i) (ii) (iii) (iv) 2 − (v) − (vi) + 7. (i) 1 + e (ii) 1
2 4 2 2 6 8 16 8
 √
π 3 5 3 1
8. (i) 2 cos x + (ii) 9. (ii) 10. (ii) π+
6 6 24 32 4

Exercise 14.2

1  1
1. (i) ln x4 + 1 + c (ii) ln | sin x| + c (iii) − ln |cos (2 x)| + c
4 2
1 1
(iv) ln (2 ex + 1) + c (v) ln (2 − cos x) (vi)
ln |3 + 4 sin x| + c
2 4
     
1 1 e+1 4 1 32
2. (i) ln 2 (ii) ln 4 (iii) ln (iv) ln 2 3. (i) ln (ii) ln 20 (iii) ln
3 2 2 3 2 27
       
1 1 16 1 4 1 5 1 1 4
4. (i) ln 6 − (ii) ln + (iii) ln − 5. (i) ln (ii) ln 2 (iii) ln
2 8 3 4 3 6 2 2 2 15
     
4 1 3 1 8192
6. (i) ln + 2 (ii) ln + (iii) ln +1
3 2 4 2 81

361
   
1 5 1 315 1 232 2
7. (i) ln 24 + (ii) ln (iii) ln +
4 4 4 220 36 327 9
     
1 9 4 1 1 3
8. (i) ln +1 (ii) ln + (iii) ln +1
2 2 27 2 4 4096

Exercise 14.3
5
1. (i) (3x − 1)e3x + c (ii) − (x + 1)e−x + c (iii) (x2 − 2x + 2)ex + c
9
1 1
(iv) 3(sin x − x cos x) + c (v) x sin (2x) + cos (2x) + c (vi) (x2 − 2) sin x + 2x cos x + c
2 4
1 2 x2 2 √ 4 √
(vii) x(ln (2x) − 1) + c (viii) x ln x − +c (ix) x x ln x − x x + c
2 4 3 9
π π 1 e−2
2. (i) −1 (ii) − (iii) 2e − 1 (iv)
2 8 4 e
9 5 8 7
(v) 2 ln 2 − 1 (vi) ln 3 − 2 ln 2 − (vii)ln 2 − (viii) 6 ln 9 − 8
2 4 3 9
   
1 1 1 e2 + 1 9
3. (i) 1, , maximum (ii) 1 4. (i) , − , minimum (ii) 5. (i) 6 (ii)
e e e 4 e

Exercise 14.4

1 √ √ 1 5 π 3 1 π π
1. (i) (ii) 2( e + 1 − 2) (iii) (iv) (v) − (vi) (vii) (viii)
4 3 4 48 64 ln 4 4 2
1 5 2 π 2 π π 1
2. (i) − cos3 (x) + c (ii) 3. (i) (x − 1) + 1 (ii) 4. (i) 4 − (x + 1) (ii) 5. −
3 24 4 3 4 2
Exercise 14.5

1. (i) 0.2877 (ii) (a) 0.2917 (ii) (b) 0.2887 (ii) (c) 0.2881 (iii) overestimate 2. 1.0947, underestimate

3. (i) (a) 80 (b) 71.1111 (ii) (a) 0.8056 (b) 0.7328 (iii) (a) 0.9342 (b) 0.9418 (iv) (a) 1.0839 (b) 1.0549

4. (i) 2.1383 (ii) ln x is concave (iii) ln (64) − 2 5. 3.07

Miscellaneous Exercise 14
 
3 2x tan (3x) e3x−2
1. (i) − cos (2x) + c (ii) 6 sin +c (iii) +c (iv) tan x − x + c (v) +c
2 3 3 3
  π √ 
1 1 π2 1 1
1. (vi) ln |2x − 1| + c 2. (i) (ii) 3. (i) ,2 (ii) + 2 ln 2 4. (i) , 2 (ii) 2
2 2 8 2 3 4
 
π(π + 2) 3 e4 − 13
4. (iii) 6. (ii) ln 7. P = 1, Q = −2, R = 1, S = 3 8. (i) 1 (iii)
2 2 4 e4
    p
1 1 1 3 π 4
9. (i) 2, , −2, − (ii) ln |x2 + 4| + c (iii) 2 e4 − 1 10. (i) − (ii) 4.3068 (iii) (e − 5)
4 4 2 2 4

362
Exercise 15.1

1. (i) 2 (ii) 1 (iii) 2 (iv) 1 (v) 2 (vi) 1 (vii) 1 (viii) 1 (ix) 1 (x) 1

3. (i) 5.54 (ii) 1.32 (iii) 1.90 (iv) 0.35 (v) 0.34 (vi) 1.32 (vii) 0.37 (viii) 1.93 (ix) 0.82 (x) 1.31

Exercise 15.2

1. (ii) − 2.69 2. (ii) 1.32 3. (ii) 2.0021 4. (ii) 0.62 5. (ii) y = 5 sin x, y = 4x (iv) 1.13

6. (ii) y = sin (2x), y = 2x − 1 (iii) 0.97

1
7. (ii) y = tan x, y = (iv) 0.86 8. (ii) y = cos x, y = 1.5 − x (iii) 0.81
x
 2  
33 5 5 33
9. (i) y = −2 x+ , maximum pt − , (ii) y = 1 − 5x − 2x2 , y = x3 (iv) 0.185
8 4 4 8

10. (i) 1 (ii) 1.41

Miscellaneous Exercise 15

1. (i) 1.73 (ii) 3 2. (ii) y = 25 sin x, y = 52x2 (iv) 0.46 3. (ii) 0.315 4. (iii) 1.33
 
1 23
5. (i) a = 5, b = 2 (ii) 1.24 6. (i) Maximum pt (−1, 1) Minimum point − , (v) − 1.755
3 27

1
7. (ii) y = tan x, y = 2x (iv) 1.17 8. (ii) y = ln x − 1, y = (iv) 3.59
x

9. (i) 2 sin (x − π/4) (iii) 0.87 10. (iv) 1.42

Exercise 16.1

1. (i) i + 2j + 3k + λ (2i − j) (ii) − i + 2j + 6k + λ (i − 4j − 4k) 3. λ = 2, a = 6, b = 3

4. (i) parallel (ii) parallel (iii) skew (iv) skew (v) intersect, 5i − 4j + k

4. (vi) intersect, −13i + 20j − k 5. λ = 3, µ = −1, a = 11, −i + 5j − 2k

Exercise 16.2

1
1. (i) 15.20 (ii) 120.30 (iii) 127.30 (iv) 71.70 (v) 900 2. (i) (11i + 11j − k), 1
3
√ √
3 3 1162 1 13 10 √
2. (ii) (i − 11j + 2k) , (iii) (−9i + 2j − 10k) , (iv) i + j + k, 5
14 14 5 5
√ √
86 461 6521 √
3. (i) (ii) (iii) (iv) 73
13 3 9

363
Exercise 16.3

1. (i) 7i − 2j − 5k (ii) − 2i − 18j − 12k (iii) 15i + 2j + 5k (iv) − 27i + 26j − 19k (v) i + 3j + 4k

1
2. A: No, B: Yes 4. (i) contained (ii) parallel (iii) intersect, (29i + 92j − 9k)
21

2 1 1
5. (i) (9i + 8j + 7k) (ii) (22i + 4j + 9k) (iii) − (11i + 2j + 24k)
15 5 3

6. (i) 2x − y − z = 1 (ii) 5x + y + 7z = 0 (iii) x + y + z = −3 (iv) 11x + 2y + 5z = 30


(v) 5x + 13y + 7z = 21 (vi) 5x − 2y − z = 8 (vii) 10x − 11y + 6z = 13

Exercise 16.4

1. (i) 57.70 (ii) 80.80 (iii) 70.90 2. (i) 25.70 (ii) 800 (iii) 72.20 3. (i) 0i + 0j + 0k, 7
√ √
1 1 25 6 16 14
3. (ii) (7i + 2j − 4k), 1 (iii) (79i + 158j − 13k), 4. (i) (ii) 4 (iii)
3 54 18 7 2

4 1 10 8 5 2
5. (i) i − j + λ(i + 7j + 11k) (ii) i + j + λ(2i + 7j + 3k) (iii) i − j + λ(3i + j − k)
11 11 3 3 3 3

5. (iv) −2i + 4j + λ(i − 3j − k) 6. (i) ABC : 2x + y = 4, OAB : z = 0 (ii) 900 (iii) 2i + λ(i − 2j)

Miscellaneous Exercise 16


0 1 1 3 22 1 1
1. (i) 73.9 (ii) i + j + λ(3i + 16j + 18k) (iii) 2. (i) (13i + 11j − 2k) (ii) (7i + 17j + 10k)
2 3 22 14 2

2. (iii) 2y − 3z = 2 3. 9i − j + 4k 4. i − 3j + 2k + λ(i − 2j + 4k) 5. (ii) 4i + j + 2k

7
6. − 2j − 2k 7. (i) 4i + j + 2k (ii) (iii) 11.10 8. 3x − 2y − z = 16
3

1
9. (i) − 6i + 5j + 13k (ii) i + 2j − 3k + λ(−6i + 5j + 13k) 10. (i) 68.20 (ii) (39i + 10j + 26k)
11

Exercise 17.1

x2 − 2x + 2A 1 1
2. (i) y = (ii) y = Aex − 1 (iii) y = (iv) y =
2 x−A A − ln x
x
(v) y = (vi) y = tan−1 (A − ln x) (vii) y = ln (ex + A) (viii) y = A(x − 2) − 3
Ax + 1
r  2
4 4x3 + 135 3(2 − x) 7 ex − 1 √
3. (i) y = e3x−6 (ii) y = (iii) y = (iv) y = (v) y = 2 sin x − 1
3 2+x 2 ex

1  √
4. (i) ln u2 + p + c (ii) y = x2 + 4 5. (−0.203, 0)
2

364

1 1 7 x2 − 2 x − 1
6. (i) − (ii) y = 8. (i) ln (sin u) + c (ii) y = 2 sin x − 1
x x+1 (x + 1)
 
1 1 1 y 1
9. (i) − ln (cos u) + c (ii) x2 cos (2 y) = 1 10. (i) − + (ii) x = ln −
y y − 1 (y − 1)2 y−1 y−1

Exercise 17.2

dr dv dn dv k
1. (i) = −k (ii) = −kv (iii) = kn (iv) =
dt dt dt dt v
dT dy √ dN N
(v) = −kT 2 (vi) =k x (vii) = −R
dt dx dt 80

3t + 10 dN
3. r2 = , t = 10 4. (i) = −kN, N = N0 e−kt 8. (ii) 0.203 (iii) 5.41 hour
10 dt

9. (iii) 8 (iv) 4.95 (v) x → 10 10. (ii) 0.0693 (iii) 37.5

Miscellaneous Exercise 17

1 2 1 6x2 ln x − 3x2 + 35 1
2. (i) x ln (x) − x2 + c (ii) y 3 = 3. (i) +c
2 4 4 2 cos2 x
   
1 1 y+1 1 2y 2
4. (i) −2 + − (ii) x = ln − 2y 5. (ii) x = ln
y+1 y−1 3(y − 1) 2 y2 + 1

3(2x + 1) dN 1
6. y = 7. (ii) = kN (iii) 60 8. (ii) t = 30 h 3 (iii) 18 s
x+1 dt

dp 10 e0.282t
9. (ii) x = 100(1 − e−0.01t ) (iii) 39.4 (iv) 100 10. (i) = kp(10 − p) (ii) p = , q = 1.96
dt 1 + e0.282t

Exercise 18.1

11 2 11 2
1. (a) (i) 4 + 6i (ii) − 2 − 2i (iii) − 5 + 10i (iv) 5 (v) 25 (vi) + i (vii) − i
25 25 5 5
8 69 8 69
(b) (i) 11 + 9i (ii) 3 + 15i (iii) 64 + 27i (iv) 193 (v) 25 (vi) − + i (vii) − − i
25 25 193 193
1 1
(c) (i) − 1 − 12i (ii) 5 + 2i (iii) − 41 + i (iv) 29 (v) 58 (vi) + i (vii) 1 − i
2 2 √ √
√ √ √ 2 5 2 4 5 2
(d) (i) − 1 + 4 2 (ii) 3 − 2 2 (iii) − 8 + 2 (iv) 3 (v) 22 (vi) − i (vii) + i
11 22 3 3

2. (i) 5 + 12i (ii) − 2 + 2i (iii) − 8 − 10i (iv) 1 − i (v) − 76i

Exercise 18.2
√ √
√ 1 7 √ 2 11
1. (i) − 1 ± 2i (ii) 2 ± i (iii) − ± i (iv) ± 1 − 3i (v) 1, 1 ± i (vi) − 2, − ± i
4 4 3 3

365
2. (i) 1 − i, x2 − 2x + 2 = 0 (ii) 7 + 2i, x2 − 14 x + 53 = 0 (iii) − 3 + 2i x2 + 6x + 13 = 0

√ 3 7
2
(iv) − 5 − 2i, x + 10x + 27 = 0 (v) − i, 2x2 − 3x + 2 = 0
4 4
3. (i) − 2i, x3 − 3 x2 + 4 x − 12 = 0 (ii) 4 + 5i, x3 − 7 x2 + 33 x + 41 = 0
1 4
3. (iii) + i, 9x3 − 6x2 + 17x = 0 4. 1 − 3i, −2
3 3

5. (i) 1 − 2i (ii) − 1 ± 2 i 6. u = 3 − i, v = 2 + i 7. (i) ± (3 + 2i) (ii) ± (3 − 2i)
√ √ ! √ √ !
3 2 2 6 2
7. (iii) ± (4 − 3i) (iv) ± (4 + i) (v) ± + i (vi) ± − i
2 2 2 2

Exercise 18.3
√ √ √ √
1. (i) 2 (cos (π/4) + i sin (π/4)) , 2e(π/4)i (ii) 2 (cos (−π/4) + i sin (−π/4)) , 2e(−π/4)i
√ √ √ √
(iii) 2 (cos (3π/4) + i sin (3π/4)) , 2e(3π/4)i (iv) 2 (cos (−3π/4) + i sin (−3π/4)) , 2e(−3π/4)i
(v) 2 (cos (π/6) + i sin (π/6)) , 2e(π/6)i (vi) 2 (cos (−π/6) + i sin (−π/6)) , 2e(−π/6)i
(vii) 2 (cos (5π/6) + i sin (5π/6)) , 2e(5π/6)i (viii) 2 (cos (−5π/6) + i sin (−5π/6)) , 2e(−5π/6)i
√ √ √ √
(ix) 8 (cos (π/3) + i sin (π/3)) , 8e(π/3)i (x) 8 (cos (−π/3) + i sin (−π/3)) , 8e(−π/3)i
√ √ √ √
(xi) 8 (cos (2π/3) + i sin (2π/3)) , 8e(2π/3)i (xii) 8 (cos (−2π/3) + i sin (−2π/3)) , 8e(−2π/3)i
(xiii) 13 (cos (−0.395) + i sin (−0.395)) , 13e−0.395i (xiv) 17 (cos (−2.652) + i sin (−2.652)) , 17e−2.652i
√ √ √ √
(xv) 41 (cos (2.246) + i sin (2.246)) , 41e2.246i (xvi) 3 (cos (0.955) + i sin (0.955)) , 3e0.955i
b = 2π/3
4. (i) π/12 (ii) π/12 (iii) 11π/12 (iv) 5π/12 5. (i) 2, −π/3, π/3 (ii) OP = OQ, P OQ
√ √ √ √
√ 1−3 3 3+ 3 1+3 3 3− 3
6. 2, ±π/6, 3 ± i 7. a = , b= ; a= , b=
2 2 2 2
8. (i) 3 + i, 2 + 14i, −1 − i (ii) − 0.464, 1.89, 1.43 (iii) parallel (iv) − 3π/4
Exercise 18.4
5. (ii) 4, 6 (iii) 2.30, 2.70 6. 2.80
Miscellaneous Exercise 18
18 1
1. (i) 6 − 10i (ii) − 6 + 17i (iii) 7 − 24i (iv) − + i
13 13
2. (i) z1 = (3 − 5i), z2 = −3 + 5i (ii) − 1, π 3. 2 + i, a = −4, b = 5 4. (i) − i, −π/2 (ii) 5 − i

5. (i) 2(cos (π/3) + i sin (π/3)), 4, 2π/3, 8, π (ii) 1 − 3i 6. (i) 6 (ii) 1 + i, 3
√ √
√ √ √ (1 + 2 3) (2 − 3)
7. (i) 2 + 2 3i (ii) (2 − 4 3) + (4 + 2 3i) (iii) + i
8 8
8. (i) ± (5 − 4i) (ii) 5 − 2i, 2i (iii) 1.95 rad
√ √
1 1 2 π 2 1
9. (i) + i (ii) , (iii) + , 1.15
2 2 2 4 2 4
√ √ √
10. (i) 1, 2π/3, 2, π/4 (ii)
2, 11π/12, 2, −5π/12
√ √ √ √
b b b − 3−1 3−1 3−1 − 3−1
(iii) P Q = QR = P R, P QR = QP R = P RQ = π/3 (iv) + i, + i
2 2 2 2

366

Das könnte Ihnen auch gefallen